Nothing Special   »   [go: up one dir, main page]

Mcqs For Frcophth Part 2

Download as pdf or txt
Download as pdf or txt
You are on page 1of 253

MCQs for 

FRCOphth Part 2
MCQs for FRCOphth Part 2

Edited by
Darren S. J. Ting, MBChB, PgCertHPE, DRCOphth, FRCOphth
(Recipient of FRCOphth Crombie Medal and DRCOphth Cornwall Prize)
Fight for Sight/​John Lee, Royal College of Ophthalmologists (RCOphth)
Primer Fellow
Academic Ophthalmology
University of Nottingham
Nottingham, UK

David H. W. Steel, MBBS, MD, FRCOphth


Consultant Ophthalmologist,
Sunderland Eye Infirmary
Sunderland, UK; and
Honorary Professor of Retinal Surgery,
Newcastle University
Newcastle upon Tyne, UK

1
3
Great Clarendon Street, Oxford, OX2 6DP,
United Kingdom
Oxford University Press is a department of the University of Oxford.
It furthers the University’s objective of excellence in research, scholarship,
and education by publishing worldwide. Oxford is a registered trade mark of
Oxford University Press in the UK and in certain other countries
© Oxford University Press 2020
The moral rights of the authors have been asserted
First Edition published in 2020
Impression: 
All rights reserved. No part of this publication may be reproduced, stored in
a retrieval system, or transmitted, in any form or by any means, without the
prior permission in writing of Oxford University Press, or as expressly permitted
by law, by licence or under terms agreed with the appropriate reprographics
rights organization. Enquiries concerning reproduction outside the scope of the
above should be sent to the Rights Department, Oxford University Press, at the
address above
You must not circulate this work in any other form
and you must impose this same condition on any acquirer
Published in the United States of America by Oxford University Press
98 Madison Avenue, New York, NY 006, United States of America
British Library Cataloguing in Publication Data
Data available
Library of Congress Control Number: 20995720
ISBN 978–​0–​9–​882576–​0
Printed in Great Britain by
Ashford Colour Press Ltd, Gosport, Hampshire
Oxford University Press makes no representation, express or implied, that the
drug dosages in this book are correct. Readers must therefore always check
the product information and clinical procedures with the most up-​to-​date
published product information and data sheets provided by the manufacturers
and the most recent codes of conduct and safety regulations. The authors and
the publishers do not accept responsibility or legal liability for any errors in the
text or for the misuse or misapplication of material in this work. Except where
otherwise stated, drug dosages and recommendations are for the non-​pregnant
adult who is not breast-​feeding
Links to third party websites are provided by Oxford in good faith and
for information only. Oxford disclaims any responsibility for the materials
contained in any third party website referenced in this work.
DEDICATION

‘To my wife, Jiani, and my family for their unconditional support and love;
and to my trainers, colleagues, and friends in the North East Deanery for the amazing time
and memories during my ophthalmology training.’
Darren S. J. Ting

‘To the many ophthalmic trainees at Sunderland Eye Infirmary and the North East Deanery
that I have had the pleasure of working with.’
David H. W. Steel
FOREWORD

Test yourself and teach yourself. To do both at the same time: read this book, study it, and learn
from it. The format and content are unique compared to other MCQ books that I read in my long
gone days as a trainee, or have seen in recent times. The questions are standard, testing factual
knowledge and the application of knowledge in clinical settings. The big selling point of this book
is in the answer section. The authors have provided elaborate explanations for the answers, which
enhance the learning experience and the retention of knowledge. At times, getting an answer
correct can still leave a gap in one’s knowledge as MCQs do not test the depth of understanding an
individual may or may not have of the subject. But for this book, the answers and the explanations
go that extra step to fill any gaps that might have existed. It actually makes the test an enjoyable
experience and by the time the fun is over, you will be well prepared for the actual examination,
which hopefully will no longer be a daunting experience.
Reading this book is also a great way for those who have passed all their examinations to refresh
their knowledge.
Harminder S Dua, CBE
MBBS, DO, DO (London), MS, MNAMS, FRCS (Edinburgh), FEBO (EU), FRCOphth,
FRCP (Edinburgh, Honorary), FCOptom. (UK, Honorary), FRCOphth (UK, Honorary), MD, PhD
(Former President of Royal College of Ophthalmologists 20–204)
Chair and Professor of Ophthalmology
University of Nottingham
Queen’s Medical Centre
Nottingham
United Kingdom
PREFACE

‘Too much to do, too little time’—​a common notion that probably describes the life of most
trainees, including me.
As a clinical and research fellow who had just recently passed the Fellowship of the Royal College
of Ophthalmologists (FRCOphth) final exam, I realize that we, as trainees, often have limited
time to prepare and revise for the ‘test’ while trying to juggle so many things in our work and
home lives simultaneously. For the preparation of my FRCOphth written exam (see Table P.),
I relied on several different revision materials and question banks; however, many of these were
either outdated, associated with expensive subscription fees, or orientated to US practice, which
often differs from that in the UK. In addition, few questions covered recent landmark studies and
the Royal College of Ophthalmologists (RCOphth) curriculum, including relevant GMC/​NICE/​
RCOphth/​Good Medical Practice guidelines, all of which are frequently tested in both the written
and oral parts of the exam.
This book aims to provide up-​to-​date revision material specifically targeting the FRCOphth
Part 2 multiple choice questions (MCQs) exam. It places more emphasis on areas that are not
well covered in other revision materials, including the GMC/​RCOphth guidelines, professional
guidance, and the choice and rationale for clinical investigations and management. The questions
are organised into five sections reflecting the subdivisions of the MCQ exam, and one mock exam
containing an additional 90 questions. All 360 MCQs are provided with a detailed explanation
and referenced. The questions are accurately mapped against the RCOphth curriculum and we
therefore hope it will help not only in the MCQs, but also in the other parts of the exam.
We are extremely grateful to the expert clinicians who have contributed questions from their own
subspecialist areas. The book could not have been completed without their expertise and help. As
editors we have tried to ensure a consistent style and approach throughout the book to ease the
learning process. We hope that this book will help you on your path to success in the FRCOphth
Part 2 exam.
Good luck!

Darren S. J. Ting and David H. W. Steel


CONTENTS

Abbreviations xiii
Contributors xix
Structure of the FRCOphth Part 2 Written Exam xxi

 Clinical Ophthalmology 
Questions 
Answers 5

2 Clinical Ophthalmology 2
Questions 35
Answers 54

3 Clinical Ophthalmology 3
Questions 77
Answers 92

4 Pharmacology, Therapeutics, and Investigations


Questions 5
Answers 25

5 Basic Science and Miscellaneous


Questions 37
Answers 45
xii Contents

6 Mock Exam
Questions 57
Answers 77

Index 205
ABBREVIATIONS

A&E accident and emergency


AC anterior chamber
ACE angiotensin-​converting  enzyme
AChR antiacetylcholine receptors
AF atrial fibrillation
AGIS Advanced Glaucoma Intervention Study
AIDS acquired immune deficiency syndrome
AK acanthamoeba keratitis
AMD age-​related macular degeneration
ANA antinuclear antibody
ANCA antineutrophilic cytoplasmic antibody
APMPPE acute posterior multifocal placoid pigment epitheliopathy
ARC abnormal retinal correspondence
AREDS Age-​related Eye Disease Study
ARN acute retinal necrosis
ASCRS American Society of Cataract and Refractive Surgery
ASRS American Society of Retina Specialists
AU anterior uveitis
BCC basal cell carcinoma
BCG bacillus Calmette–​Guérin
BCVA best corrected visual acuity
BD twice a day
BE Basic exotropia
BETT Birmingham Eye Trauma Terminology
BHL bilateral hilar lymphadenopathy
BM Bruch’s membrane
BMI body mass index
BNF British National Formulary
BPES blepharophimosis, ptosis, and epicanthus inversus syndrome
BSV binocular single vision
CAI carbonic anhydrase inhibitor
xiv Abbreviations

CAR cancer-​associated retinopathy


CCF carotico-​cavernous fistula
CCTS Collaborative Corneal Transplant Studies
CFEOM congenital fibrosis of extraocular muscles
CH cluster headache
CI convergence insufficiency
CJD Creutzfeldt–​Jakob disease
CL contact lens
CMO cystoid macular oedema
CMT central macular thickness
CNS central nervous system
CNTGS Collaborative Normal Tension Glaucoma Study
CNVM choroidal neovascular membrane
COAG chronic open angle glaucoma
COG Children’s Oncology Group
COMS Collaborative Ocular Melanoma Study
COPD chronic obstructive pulmonary disease
CPEO chronic progressive external ophthalmoplegia
CRAO central retinal artery occlusion
CRP C-​reactive protein
CRT central retinal thickness
CSCR central serous chorioretinopathy
CSF cerebrospinal fluid
CSS Churg–​Strauss syndrome
CT computed tomography
CTA computed tomography angiogram
CTG cytosine-​thymine-​guanine
CZS congenital Zika syndrome
DALK deep anterior lamellar keratoplasty
DCCT Diabetes Control and Complications Trial
DE distance exotropia
DEXA dual energy X-​ray absorptiometry
DMARD disease-​modifying antirheumatic drug
DMO diabetic macular oedema
DON dysthyroid optic neuropathy
DREAM DRy Eye Amniotic Membrane
DRS Diabetic Retinopathy Study
DRVS Diabetic Retinopathy Vitrectomy Study
DS Duane syndrome
Abbreviations xv

DSG dacryoscintigram
DVLA Driver and Vehicle Licensing Agency
EGF epidermal growth factor
EGFR epidermal growth factor receptor
EMGS Early Manifest Glaucoma Study
ENT ear, nose, and throat
EON ethambutol optic neuropathy
ESCRS European Society of Cataract & Refractive Surgeons
ESR erythrocyte sedimentation rate
ETDRS Early Treatment for Diabetic Retinopathy Study
ETROP Early Treatment of Retinopathy of Prematurity
EUGOGO European Group on Graves’ orbitopathy
EVS Endophthalmitis Vitrectomy Study
FA fluocinolone acetonide
FAF fundus autofluorescence
FBC full blood count
FDA Food and Drug Administration
FDT forced duction test
FFA fundus fluorescein angiography
FHC Fuchs’ heterochromic cyclitis
FLACS femtosecond laser-​assisted cataract surgery
FLAIR fluid attenuated inversion recovery
G6PD glucose-​6-​phosphate dehydrogenase
GA gyrate atrophy
GABA gamma-​aminobutyric  acid
GCA giant cell arteritis
GMC General Medical Council
GON glaucomatous optic neuropathy
GP general practitioner
H&E haematoxylin & eosin
HEDS Herpetic Eye Disease Study
HIV human immunodeficiency virus
HLA human leukocyte antigen
HORV haemorrhagic occlusive retinal vasculitis
HSK herpes simplex keratitis
HSV herpes simplex virus
HZO herpes zoster ophthalmicus
ICG indocyanine green
ICGA indocyanine green angiography
xvi Abbreviations

IDEX Intermittent distance exotropia


IFIS intraoperative floppy iris syndrome
IIH idiopathic intracranial hypertension
IIHTT IIH treatment trial
ILM internal limiting membrane
INO internuclear ophthalmoplegia
IOP intraocular pressure
IPD interpupillary distance
IR inferior rectus
ISCEV International Society for Clinical Electrophysiology of Vision
ITC iridotrabecular contact
JIA juvenile idiopathic arthritis
KF Kayser–​Fleischer
KSS Kearns–​Sayers syndrome
LFT liver function test
LHON Leber’s hereditary optic neuropathy
LIO left inferior oblique
LR lateral rectus
LSCD limbal stem cell deficiency
LSR left superior rectus
MAR melanoma-​associated retinopathy
MCQ multiple-​choice question
MD myotonic dystrophy
MED monocular elevation deficiency
MG myasthenia gravis
MGJWS Marcus-​Gunn jaw winking syndrome
MLD maximum linear diameter
MLF medial longitudinal fasciculus
MOA mechanisms of  action
MR medial rectus
MRA magnetic resonance angiography
MRI magnetic resonance imaging
MRSE mean refractive spherical equivalent
MuSK muscle-​specific  kinase
NASCET North American Symptomatic Carotid Endarterectomy Trial
NDESP NHS Diabetic Eye Screening Programme
NFAT nuclear factor of activated T cells
NICE National Institute for Health and Care Excellence
NNT numbers need to treat
Abbreviations xvii

NOG National Osteoporosis Guideline Group


NPL non-​perceptive to light
OCP ocular cicatricial pemphigoid
OCT optical coherence tomography
OD left eye
OHTS Ocular Hypertension Study
OID orbital inflammatory disease
ONTT Optic Neuritis Treatment Trial
OS right eye
OSSN ocular surface squamous neoplasia
OTS ocular trauma score
PACG primary angle closure glaucoma
PACS primary angle closure suspect
PAM primary acquired melanosis
PAS peripheral anterior synechiae
PAT prism adaptation test
PCG primary congenital glaucoma
PCMO post-​cataract surgery macular oedema
PCO posterior capsular opacification
PD prism dioptres
PDR proliferative diabetic retinopathy
PDS pigment dispersion syndrome
PED pigment epithelial detachments
PEDF Pigment epithelium derived factor
PGA prostaglandin analogues
PHN post-​herpetic neuralgia
PIC punctate inner choroidopathy
PK penetrating keratoplasty
PlGF placental growth factor
PMD pellucid marginal degeneration
POGS Parinaud’s oculo-​glandular syndrome
POHS presumed ocular histoplasmosis syndrome
POT parietal-​occipital-​temporal
PPCD posterior polymorphous corneal dystrophy
PPD plasma protein derivative
PPRF paramedian pontine reticular formation
PRN pro re nata
PSS primary Sjogren’s syndrome
PUK peripheral ulcerative keratitis
xviii Abbreviations

QALY quality-​adjusted life year or quality-​adjusted-​life-​year


QID four times a day
RAM retinal arterial macroaneurysm
RAPD relative afferent pupillary defect
RD retinal detachment
RIO right inferior oblique
RLR right lateral rectus
RP retinitis pigmentosa
RPE retinal pigment epithelium
RSR right superior rectus
RVO retinal vein occlusions
SCC squamous cell carcinoma
SJS Steven–​Johnson syndrome
SLT selective laser trabeculoplasty
SO superior oblique (also Sympathetic ophthalmia)
SOD septo-​optic dysplasia
SOF superior orbital fissure
SOM superior oblique myokymia
SR superior rectus
SSS silent sinus syndrome
SUN Standardization of Uveitis Nomenclature
TED thyroid eye disease
TIA transient ischaemic attack
TON traumatic optic neuropathy
TPMT thiopurine methyltransferase
TVT Tube Versus Trabeculectomy
U&E urea and electrolytes test
UBM ultrasound biomicroscopy
UC ulcerative colitis
UZS Urrets–​Zavalia syndrome
VEGF vascular endothelial growth factor
VEP visual evoked potential
VHL Von Hippel Lindau
VKH Vogt–​Koyanagi–​Harada
VR vitreo-​retinal
WD Wilson’s disease
WHO World Health Organization
XLOA X-​linked ocular albinism
XLRS X-​linked recessive juvenile retinoschisis
CONTRIBUTORS

Wendy Adams  Consultant Ophthalmologist (Neuro-Ophthalmology), Sunderland Eye Infirmary,


Sunderland, UK 
Lucy Clarke  Consultant Ophthalmologist (Oculoplastic, Orbit and Emergency Eye Care),
Newcastle Eye Centre, Royal Victoria Infirmary, Newcastle upon Tyne, UK 
Lawrence Gnanaraj  Consultant Ophthalmologist (Paediatric Ophthalmology and Strabismus),
Sunderland Eye Infirmary, Sunderland, UK 
William Innes  Consultant Medical Ophthalmologist, Newcastle Eye Centre, Royal Victoria
Infirmary, Newcastle upon Tyne, UK 
David L. Lunt  Consultant Ophthalmologist (Glaucoma), Department of Ophthalmology, South
Tees Hospitals NHS Foundation Trust, Middlesbrough, UK 
Philip Severn  Consultant Ophthalmologist (Medical Retina) and Clinical Director, Department of
Ophthalmology, South Tees Hospital NHS Foundation Trust, Middlesbrough, UK 
Jonathan Smith  Consultant Ophthalmologist (Medical and Surgical Retina), Sunderland Eye
Infirmary, Sunderland, UK 
Laura Steeples  Consultant Ophthalmologist (Uveitis), Manchester Royal Eye Hospital,
Manchester, UK 
Shi Zhuan Tan  Vitreoretinal Fellow, Guys’ and St Thomas Hospital, London, UK 
Darren S. J. Ting  Fight for Sight/​John Lee, Royal College of Ophthalmologists (RCOphth)Primer
Fellow, Academic Ophthalmology, University of Nottingham, Nottingham, UK 
STRUCTURE OF THE FRCOphth
PART 2 WRITTEN EXAM

Table P.  Structure of the FRCOphth part 2 written exam


Clinical ophthalmology 28
Cornea and external eye 20
Glaucoma 0
Cataract and lens 0
Retina 0
Neurology and pupil 6
Medicine 0
Strabismus and paediatrics 6
Trauma 4
Oculoplastic and orbit 0
Uveitis and oncology 2
Pharmacology and therapeutics 2
Investigations 6
Ophthalmic 9
Orthoptic 3
Neuroimaging 3
Other 
Basic science 8
Anatomy/​Physiology 2
Pathology 2
Genetics 3
Optics 
Miscellaneous 6
Statistics/​Epidemiology 3
Research/​EBM 3
Nutrition 
Ethics/​Driving 4
Economics 
Guidelines/​Standards 4

Reproduced with the kind permission of The Royal College of Ophthalmologists.


https://www.rcophth.ac.uk/wp-content/uploads/209/03/Part-2-FRCOphth-
Written-Notes-for-Candidates-Updated-March-209.pdf
chapter CLINICAL OPHTHALMOLOGY 

 QUESTIONS

. A 35-​year-​old man presented with a 2-​day history of right painful


red eye. Examination revealed dendritic ulcer with no stromal
vascularization/​oedema or keratic precipitates. Anterior chamber is
deep and quiet. He had two similar episodes of dendritic keratitis in the
past year. What is the best management plan for this patient?
. Topical aciclovir 3% 5×/​day for 2 weeks +/​–​epithelial debridement
A
B. Topical aciclovir 3% 5×/​day for 2 weeks + topical prednisolone % QID for 2 weeks
C. Topical aciclovir 3% 5×/​day for 2 weeks + oral aciclovir 400 mg BD for at least 6 months
D. Topical aciclovir 3% 5×/​day for 2 weeks + oral aciclovir 400 mg 5×/​day for 2 weeks then
BD for at least 6 months

2. Which of the following statements about ocular cicatricial pemphigoid


(OCP) is correct?
A. Identification of linear deposition of immunoglobulin G (IgG), IgE, and C3 complement
factors at the epithelial basement membrane zone using direct immunofluorescence (DIF)
test is diagnostic of  OCP
B. Negative immunofluorescent test of conjunctival biopsy excludes the diagnosis of OCP
C. To achieve best diagnostic yield of OCP, it is best to obtain the conjunctival biopsy from
less inflamed areas
D. Conjunctival sample should be transported in a sterile pot containing formalin solution for
immunohistochemical analysis

3. Which of the following histopathological stains is MOST useful in the


diagnosis of Avellino corneal dystrophy?
. Alcian blue stain
A
B. Congo red stain
C. Haematoxylin & eosin (H&E stain)
D. Oil-​red-​O  stain

4. In addition to limbal ischaemia, which of the following signs has been


shown to have a poor prognostic value in chemical eye injury?
. More than 2+ cells in anterior chamber
A
B. Central epithelial defect of  > 80%
C. Raised intraocular pressure (IOP) of 28 mmHg or more
D. Severe conjunctival involvement of  > 80%
2 Clinical Ophthalmology  | QUESTIONS

5. Which of the following statements regarding ocular surface squamous


neoplasia (OSSN) is correct?
A. Patients presenting with OSSN should always be screened for human immunodeficiency
virus (HIV) and acquired immune deficiency syndrome (AIDS)
B. The incidence of OSSN has been estimated at around –​2 per 0 000/​year in Caucasian
populations
C. OSSN usually presents as a unilateral vascularized mass at the inferior fornix area
D. Bowen’s disease is a type of OSSN that affects the conjunctival basement membrane

6. Which of the following organisms has the potential to penetrate an


intact corneal epithelium?
A. Corynebacterium diphtheriae
B. Group A streptococcus
C. Moraxella lacunata
D. Pseudomonas aeruginosa

7. A 24-​year-​old patient presents to the eye clinic with a 6-​month history


of photophobia and decreased vision in the right eye. His past ocular
history is significant for a right chemical eye injury, which was treated
elsewhere. His presenting vision is 6/​36. Examination shows 3 clock-​
hours of peripheral conjunctivalization of the cornea, extending to the
visual axis, which is also associated with stippled fluorescein staining.
Which of the following is the best initial treatment option for this
patient?
. Amniotic membrane transplant
A
B. Anterior lamellar keratoplasty
C. Autologous limbal stem cell transplant
D. Sequential sector conjunctival epitheliectomy

8. Which of the following findings regarding corneal impression cytology is


suggestive of limbal stem cell deficiency?
. Absence of goblet cell
A
B. Presence of cytokeratin (CK)-​3
C. Presence of   CK-​2
D. Presence of   CK-​3

9. Which of the following corneal dystrophies has a different gene


mutation from the others?
. Epithelial basement membrane dystrophy (or map-​dot-​fingerprint dystrophy)
A
B. Granular dystrophy
C. Meesmann dystrophy
D. Thiel–​Behnke dystrophy
Clinical Ophthalmology  | QUESTIONS 3

0. A 25-​year-​old patient with keratoconus underwent an uncomplicated


penetrating keratoplasty for visual rehabilitation. On the next day,
the patient complains of a fixed dilated pupil with some headache.
Examination of the eye shows a fixed and dilated pupil with a deep and
quiet anterior chamber. The dilated pupil is not responsive to topical
pilocarpine 2% drops. Otherwise there is no sign of ptosis. What should
be the next step in your management?
. Check the patient’s ocular motility
A
B. Check the patient’s IOP
C. Organize a computed tomography angiogram of the head
D. Organize an MRI head with contrast

. A 75-​year-​old patient presents with vesicular rashes affecting the right
side of the forehead and scalp and the base of the nose, respecting
the midline of the face. On examination there are pseudodendritic
changes on the cornea with mild intraocular inflammation. Which of the
following statements concerning this scenario is true?
. This patient has a high risk of developing corneal denervation
A
B. Oral aciclovir is not useful if it is started 72 hours after the onset of rashes
C. Topical aciclovir should be used in this case
D. Oral antibiotic should be started if there is evidence of bilateral periorbital swelling

2. A 53-​year-​old man, with a background of right penetrating keratoplasty


performed  year ago, presents with a 3-​day history of right eye
photophobia and mildly reduced vision. He has recently recovered from
a viral illness. Slit-​lamp examination reveals a minimally red eye with
mild papillary conjunctival changes and multiple subepithelial opacities
within the corneal graft. Otherwise there is no sign of epithelial defect,
stromal oedema, keratic precipitates, or anterior chamber activity.
Which of the following options is the most appropriate management for
this patient?
. Start on regular topical lubricants and reassure the patient
A
B. Start on topical chloramphenicol 0.5% four times a day for  week and review the patient
in a few weeks’ time
C. Start on topical prednisolone 0.5% twice a day and review in a few weeks’ time
D. Start on topical dexamethasone 0.% four to six times a day and review the patient in a
few days’ time

3. Which of the following options regarding corneal arcus is correct?


. It may be associated with peripheral corneal thinning
A
B. The lipid deposition is restricted to the stromal level
C. It usually starts at 3 and 9 o’clock
D. Ipsilateral corneal arcus may be associated with ipsilateral carotid artery disease
4 Clinical Ophthalmology  | QUESTIONS

4. Which layer of the cornea is affected by the copper deposition in


Wilson’s disease?
A. Epithelium
B. Bowman’s  layer
C. Stroma
D. Descemet’s membrane

5. Which of the following conditions is associated with ‘curly fibres’ in


Bowman’s layer on histopathologic examination?
. Meesmann epithelial dystrophy
A
B. Reis–​Buckler dystrophy
C. Schnyder central crystalline dystrophy
D. Thiel–​Behnke dystrophy

6. Which of the following options regarding keratoconus is NOT correct?


. Breaks in the Bowman’s layer is a diagnostic feature of keratoconus
A
B. Posterior keratoconus is usually congenital and unilateral
C. Posterior corneal curvature is usually not affected in keratoconus
D. Belin Ambrosio Enhanced Ectasia Display is useful in detecting early keratoconus

7. Which of the following statements regarding Peter’s anomaly is correct?


A. It may be associated with systemic abnormalities such as cardiac and central nervous
system malformations
B. It is caused by a mutation of the TIMP3 gene
C. Approximately 20% of the patients develop glaucoma
D. It is caused by developmental abnormalities of the corneal stroma

8. A 4-​year-​old patient presents to the eye clinic with her mum,
concerning about a recent increase in size of a ‘lesion’ on the white part
of the eye. On examination, there is a minimally elevated, pigmented
lesion on the conjunctiva near the limbus area with cystic changes and
without vascularization or epithelial defect. She mentions that the
pigmented lesion has been present for many years. What is the most
likely diagnosis?
A. Conjunctival melanoma
B. Conjunctival melanosis
C. Conjunctival  naevus
D. Conjunctival papilloma
Clinical Ophthalmology  | QUESTIONS 5

9. Based on the evidence up to 208, which of the following statements


regarding the role of topical steroids in bacterial keratitis is true?
A. Patients with worse visual acuity at initial presentation benefit more with additional topical
steroids than those with better presenting visual acuity
B. Topical steroids hasten the healing time of bacterial keratitis
C. Topical steroids increase the risk of corneal perforation compared to those without topical
steroids
D. Patients with Nocardia keratitis have a better visual outcome when additional topical
steroids are given along with topical antibiotics

20. Which of the following statements regarding Mooren’s ulcer is correct?


. It is a type of peripheral ulcerative keratitis associated with scleritis
A
B. It may be associated with hepatitis C infection
C. The ulcer is usually painless, which is a distinguishing feature from other types of peripheral
ulcerative keratitis
D. Bilateral disease usually has a less painful presentation, with better responsiveness to
therapy compared to the unilateral disease

2. A 30-​year-​old patient presents to the eye clinic with reduced vision in
the right eye. Slit-​lamp examination shows a beaten bronze appearance
of the corneal endothelium with iris atrophy and peripheral anterior
synechiae on gonioscopy. Examination of the left eye is entirely normal.
Which of the following statements concerning this condition is correct?
. This condition normally has an autosomal dominant inheritance
A
B. It is more commonly found in men than in women
C. Around 20% of the patients are affected by glaucoma
D. PCR testing of the affected corneal endothelium may reveal HSV DNA

22. Which of the following statements regarding primary acquired


melanosis (PAM) is true?
. Most conjunctival melanomas arise from PAM
A
B. It is more commonly found in patients with darker skin
C. Limbal conjunctiva is the most commonly affected area
D. PAM with any degree of atypia is associated with an increased risk of developing
melanoma

23. Which of the following statements regarding pellucid marginal


degeneration (PMD) is correct?
. It affects both males and females equally
A
B. The steepest area of cornea corresponds with the thinnest area of the cornea
C. It is associated with against-​the-​rule astigmatism
D. ‘Crab-​claw’ pattern on corneal topography is a pathognomonic finding of PMD
6 Clinical Ophthalmology  | QUESTIONS

24. Which of the following statements regarding amniotic membrane is


correct?
A. The use of amniotic membrane is contraindicated in patients with active herpes simplex
viral keratitis
B. Cryopreserved amniotic membrane has been shown to improve signs and symptoms of
dry eye disease
C. It has anti-​inflammatory, antifibrotic, and wound healing properties but no antimicrobial
properties
D. Cryopreservation is currently the only technique to preserve amniotic membrane for
clinical use

25. A 70-​year-​old woman presents to the eye clinic with a 4-​day history
of right-​sided conjunctival redness, jaw swelling, and fever. He has no
previous medical or ocular history. She has two pet rabbits at home
and denied any history of cat scratch or exposure. Examination reveals
severe right conjunctival hyperaemia and multiple conjunctival nodules
and ulceration. There is also associated pre-​auricular and submandibular
lymphadenopathy. Which of the following organisms is the most likely
culprit in this scenario?
A. Bartonella henselae
B. Francisella tularensis
C. Sporotrichum schenckii
D. Treponema pallidum

26. A 25-​year-​old patient with advanced keratoconus underwent a deep


anterior lamellar keratoplasty for visual rehabilitation. During the
operation, there was an inadvertent perforation of the Descemet’s
membrane, necessitating a conversion of deep anterior lamellar
keratoplasty (DALK) to penetrating keratoplasty (PKP). Which of the
following statements is correct?
A. There is no significant difference in the graft rejection rate between PKP and DALK for
keratoconus
B. There is a significant increased risk of keratometric astigmatism after PKP compared
to DALK
C. The patient is likely to have a worse uncorrected visual acuity at 6 months with PKP
compared to DALK
D. The patient is likely to have a better corrected visual acuity at 6 months with PKP
compared to DALK

27. Which of the following statements regarding herpes zoster or shingles


vaccination in the United Kingdom is true?
. Shingles vaccination consists of inactivated/​killed varicella-​zoster virus
A
B. It is available for all people of more than 70 years old
C. Shingles vaccination could result in reactivation of herpes zoster keratitis
D. People who had shingles previously do not benefit from shingles vaccination
Clinical Ophthalmology  | QUESTIONS 7

28. According to the Collaborative Corneal Transplant Studies (CCTS),


which of the following statements is correct?
A. Human leukocyte antigen-​A (HLA-​A) antigen matching significantly reduces the risk of
corneal graft rejection
B. HLA-​DR antigen matching significantly reduces the risk of corneal graft rejection
C. ABO blood group matching may reduce the risk of corneal graft rejection
D. Positive donor-​recipient cross-​match significant increases the risk of corneal graft rejection

29. A 3-​year-​old boy was brought in by his mum to the eye clinic with 2
weeks’ history of itchy and painful eyes. He was treated by his general
physician with topical chloramphenicol during the preceding week
with no improvement. The boy was very uncooperative during the
examination and you could only obtain a quick glance from slit-​lamp
examination, which revealed inflamed eyes with moderate papillary
conjunctivitis. There was also a moderate size corneal plaque with
epithelial defect at the superior paracentral cornea in the right
eye. Measurement of IOP was not possible. The boy was started on
topical olopatadine and topical prednisolone 0.5% four times a day. On
examination during the following week, the corneal changes remained
the same. Which of the following is the most appropriate next step for
this child?
. Change topical prednisolone 0.5% to topical dexamethasone 0.% six times a day
A
B. Change topical chloramphenicol four times a day to topical levofloxacin six times a day
C. Start the child on topical ciclosporin 0.% once at night
D. Arrange for the child to have removal of corneal plaque under general anaesthesia soon

30. Which of the following statements regarding the mechanism of action


of Lifitegrast (recently approved for treatment of dry eye disease) is
correct?
A. Inhibits CD-​20 antibody
B. Inhibits cytotoxic T cells (CD8)
C. Inhibits lymphocyte function-​associated antigen 
D. Inhibits calcineurin/​nuclear factor of activated T cells

3. Which of the following types of anterior segment dysgenesis is strongly


associated with corneal leukoma, craniofacial, and central nervous
system abnormalities?
A. Aniridia
B. Axenfeld-​Rieger syndrome
C. Peter’s anomaly
D. Trabeculodysgenesis
8 Clinical Ophthalmology  | QUESTIONS

32. A 59-​year-​old hypermetropic female patient with a best corrected


visual acuity of 6/​5 in both eyes (OU) attends your clinic due to elevated
IOP at her optician. She has gonioscopically occludable iridocorneal
angles, IOP of 27 mmHg OD, and 32 mmHg OS. She has a notch in
the neuroretinal rim in both optic discs, and evidence of early arcuate
defects on perimetry. Which intervention is superior in terms of efficacy
and cost-​effectivity?
. Clear lens extraction
A
B. Peripheral iridoplasty
C. Peripheral iridotomy
D. Selective laser trabeculoplasty

33. Which of the following statements regarding the landmark glaucoma


studies is correct?
A. Advanced Glaucoma Intervention Study (AGIS) found that Caucasian patients had better
IOP control if they had primary ALT
B. Collaborative Normal Tension Glaucoma Study (CNTGS) found that over the study period
almost two-​thirds of patients did not progress even without treatment
C. Early Manifest Glaucoma Study (EMGS) found that age, race, and optic disc haemorrhages
were risk factors for progression
D. Ocular Hypertension Study (OHTS) found that age, race, and optic disc haemorrhages
were risk factors for progression

34. A patient attends clinic one day after a trabeculectomy operation on his
right eye. His Snellen best corrected visual acuity is 6/​36 OD and 6/​9 OS.
Which of the following is most likely to be TRUE?
A. His IOP is 4 mmHg. His anterior chamber is shallow. You suspect overdrainage and a
conjunctival leak and so check with topical fluorescein 2%
B. His IOP is 4 mmHg. His anterior chamber is shallow. You suspect a partial/​complete
blockage of the sclerostomy may be the cause
C. His IOP is 35 mmHg. His anterior chamber is shallow. You suspect retained viscoelastic
may be the cause
D. His IOP is 35 mmHg. His anterior chamber is shallow. You suspect aqueous misdirection
may be the cause and start him on topical pilocarpine 2% immediately pending further
management

35. A patient who has had a trabeculectomy within the last 4 weeks has the
clinical appearance of a conjunctival wound leak. All of the following is
most likely to be correct, EXCEPT:
. This can be associated with a risk of blebitis and endophthalmitis
A
B. Any wound leak requires further surgery to ensure adequate closure to reduce the risk of
postoperative infection
C. Slow flow wound leak can be managed conservatively, including contact lens
D. Is more likely in patients who have been on long-​term ocular antihypertensive medication
Clinical Ophthalmology  | QUESTIONS 9

36. Which of the following statements about the side effects of


acetazolamide is correct?
. May increase the level of potassium in the blood
A
B. Is safe to be used in patients with liver impairment
C. Is associated with an elevation in blood pressure
D. Is associated with Stevens–​Johnson syndrome

37. A general practitioner (GP) contacted you about a 55-​year-​old male


patient who has presented to his practice with symptoms consistent
with impotence. He has primary open angle glaucoma and the GP
wondered about the possible side effect from his glaucoma medication.
Which medication is MOST likely to be responsible?
A. Apraclonidine
B. Bimatoprost
C. Dorzolamide
D. Levobunolol

38. A 38-​year-​old female patient is referred into the clinic following a


high IOP reading. On examination she has an IOP of 7 mmHg OD
and 35 mmHg OS. The left iris appears paler and she has stellate
keratic precipitates across the whole corneal endothelium. She has
some circulating cells in the anterior chamber. Which of the following
statements about this condition is TRUE?
. Anterior chamber activity always requires aggressive treatment with topical steroids
A
B. There are normally no associated posterior segment findings
C. Trabeculectomy is known to be ineffective
D. Cataract surgery in this condition is associated with an increased risk of intraoperative
bleeding

39. A 45-​year-​old, type  diabetic, male patient attends a follow-​up


appointment  month following a fresh vitreous haemorrhage in his
right eye. He has had a previous vitrectomy due to the same condition
in the same eye. On examination he has developed diffuse corneal
oedema, anterior chamber activity, and an elevated IOP of 45 mmHg
OD (5 mmHg OS). No hyphema is noted. The view is limited but the
vitreous haemorrhage does not seem to have altered. Which of the
following is MOST LIKELY to describe his problem?
. Ghost cell glaucoma
A
B. Posner-​Schlossman syndrome
C. Red cell glaucoma
D. Schwartz–​Matsuo syndrome
10 Clinical Ophthalmology  | QUESTIONS

40. A 54-​year-​old female is referred to your clinic with suspected


narrow angles. Her refraction is +4.75 D OD and +4.25 D OS. She
is asymptomatic. Her IOP is 28 mmHg OD and 30 mmHg OS. On
gonioscopic examination, you are able to visualize the trabecular
meshwork for approximately 90° in each eye. Optic disc assessment,
visual field testing, and optical coherence tomography of the optic disc
and macula are all normal. Which of the following correctly summarizes
your findings?
A. Occludable  angles
B. Primary angle closure
C. Primary angle closure glaucoma
D. Primary angle closure suspect

4. Which category of IOP lowering medication suppresses aqueous


production and increases uveoscleral outflow?
A. α-​2-​adrenergic agonists
B. β-​blockers
C. Carbonic anhydrase inhibitors
D. Prostaglandin analogues

42. You see a 67-​year-​old male patient with ocular hypertension who takes
topical latanoprost at night in both eyes. His IOP is 20 mmHg. After
completing all your investigations, you are satisfied there is no evidence
of conversion to open angle glaucoma. Which of the following best
describes the timing of the next review appointment?
A. 4–​6 months
B. 6–​9 months
C. 9–​2 months
D. 2–​8 months

43. In patients with high hypermetropia and/​or very short axial length,
which of the following procedures is known to cause aqueous
misdirection (also known as malignant glaucoma)?
A. Cataract surgery
B. Peripheral iridotomy
C. Trabeculectomy
D. All of the above

44. When is it most appropriate to perform laser trabeculoplasty on a


patient with glaucoma?
A. Patient with chronic open angle glaucoma taking ≥ medication and inadequate IOP
control
B. First-​line therapy in a patient with ocular hypertension or chronic open angle glaucoma
C. Following laser peripheral iridotomy in primary angle closure glaucoma
D. First-​line therapy in a patient with primary or secondary angle closure glaucoma
Clinical Ophthalmology  | QUESTIONS 11

45. A 75-​year-​old patient presents to the clinic with a -​week history of


right eye pain. Examination revealed a hypermature cataract with raised
IOP and deep anterior chamber with cells and flare. What is the likely
diagnosis of this case?
. Lens particle glaucoma
A
B. Phacoanaphylatic glaucoma
C. Phacolytic glaucoma
D. Phacomorphic glaucoma

46. During routine cataract surgery you become aware that there is a
posterior capsule rupture. A quadrant of the lens nucleus drops into the
anterior vitreous. Which of the following statements is most likely to be
correct?
A. Stop phacoemulsification immediately. Introduce a dispersive viscoelastic. If the lens
fragment is within reach proceed with the phaco probe with caution to remove from the
anterior vitreous
B. Proceed with caution to remove the remainder of the lens fragments from the anterior
segment. You can then decide how to proceed
C. Stop phacoemulsification immediately. Introduce a dispersive viscoelastic. Do not attempt
to remove the lens fragment with the phaco probe. Safely close the operative site and
arrange discussion with vitreo-​retinal (VR) service to arrange for completion of surgery
+/​–​intraocular lens (IOL) as a secondary procedure
D. Stop phacoemulsification immediately. Take appropriate steps, and remove lens fragment
with vitrector

47. The following statements about post-​phacoemulsification cystoid


macular oedema (CMO, or Irvine–​Gass syndrome) are true, EXCEPT:
A. If investigated with optical coherence tomography (OCT), many more patients will have
CMO than observed clinically
B. There is strong evidence to suggest that prostaglandin analogues should be stopped in
those patients with pre-​existing glaucoma of any subtype
C. Diabetes mellitus is a risk factor
D. Uveitis is a risk factor

48. During phacoemulsification you rupture the posterior capsule. Following


successful anterior vitrectomy and removal of all lens matter, you decide
a sulcus IOL will be needed. The initial lens selected was +8.00 D three-​
piece IOL. Which of the following lenses would most closely match this
refractive plan, assuming no optic capture is possible?
A. +6.00 D
B. +7.00 D
C. +8.00 D
D. +9.00 D
12 Clinical Ophthalmology  | QUESTIONS

49. Which of the following statements is correct with regard to an outbreak


of acute postoperative endophthalmitis?
. Cease all bilateral cataract cases and consider cessation of all surgery
A
B. The responsible organisms are mostly contaminants
C. It is defined as an outbreak if more than two cases are traced back to one operating list
D. Only cases that comprised a proven outbreak need to be reported as clinical incidents

50. Which of the following is a potential cause for myopic refractive


surprise?
. Previous laser refractive surgery for hypermetropia
A
B. Lower A constant
C. Undiagnosed staphyloma
D. All of the above

5. You see a patient in clinic with bilateral inferonasal lens subluxation.
There is no history of trauma. The patient is tall with thin limbs and
extremities (‘marfanoid’). Which of the following statements is correct?
. The gene defect is most likely to be fibrillin
A
B. Cataract surgery should be performed under general anaesthesia due to increased risk of
intraoperative complications
C. The metabolic abnormality results in increased bone density
D. A blood or urine test could confirm the diagnosis of this autosomal recessive condition

52. Which of the following statements should be read aloud as part of the
‘Sign in’ procedure according to the World Health Organization (WHO)
surgical safety checklist for cataract?
. Have all team members introduced themselves by name and role?
A
B. What lens model and power is to be used?
C. Is there any anticipated difficulty with patient positioning?
D. Is the correct lens implant present?

53. Which of the following statements regarding pseudoexfoliation


syndrome (PXF) is true?
. These patients have deeper iridocorneal angles
A
B. The degree of PXF material correlates with the severity of zonular dehiscence
C. Lenticular instability can always be managed with a capsular tension ring intraoperatively
D. Is associated with Eastern European and Scandinavian population
Clinical Ophthalmology  | QUESTIONS 13

54. As a cataract surgeon, you may encounter a variety of problems during


surgery. Which of the following statements is true?
. Reverse pupil block is more common in myopes
A
B. Bimanual irrigation/​aspiration has been proven to be safer and more efficient than coaxial
C. Shorter eyes are less likely to be associated with intraoperative aqueous misdirection
D. Intraoperative floppy iris syndrome can be avoided if you ask patients to stop their
α-​blocker 2 weeks prior to surgery

55. A patient calls the eye department 3 days following cataract surgery.
They feel that the vision is reduced and the eye is red and light sensitive.
Please select the most appropriate answer from the following:
A. Advise the patient to increase the topical steroid drops but have a low threshold for
attending the eye casualty
B. The most common causative organisms include Staphylococcus aureus, Staphylococcus
epidermis, and Streptococcus spp.
C. Consider early vitrectomy if the vision is better than perception of light
D. In addition to intravitreal antibiotics, topical or intravitreal corticosteroids should also
always be administered

56. Which of the following does NOT reduce the risk of posterior capsular
opacification?
. Square edge design of the IOL optic
A
B. Haptics with flexible arms and posterior flexion
C. Complete overlapping of anterior capsulorrhexis and anterior surface of IOL optic
D. Polishing of the posterior capsular after cleaning of lens cortex

57. What effect does intravitreal silicone oil have in a pseudophakic eye?


. It makes the refraction less hypermetropic
A
B. It makes the refraction more hypermetropic
C. No effect on refraction
D. Variable effect on refraction

58. Which of the following statements on toxic anterior segment syndrome


is correct?
. Examination usually reveals diffuse corneal oedema with presence of hypopyon
A
B. It usually acts as a precursor of infective endophthalmitis
C. It is caused by inadvertent introduction of infectious substances into the anterior segment
D. It can be distinguished from postoperative inflammation based on its unresponsiveness to
topical steroids
14 Clinical Ophthalmology  | QUESTIONS

59. Which of the following diseases is most commonly associated with


posterior subcapsular cataract?
A. Alport syndrome
B. Fabry’s disease
C. Lowe’s syndrome
D. Waardenburg syndrome

60. Compared to the standard phacoemulsification cataract surgery, which


of the following is true regarding femtosecond laser-​assisted cataract
surgery (FLACS)?
. FLACS is associated with a lower risk of postoperative CMO
A
B. FLACS is associated with a lower risk of posterior capsular rupture
C. The risk of anterior capsular tear is similar between FLACS and standard
phacoemulsification
D. FLACS is more cost-​effective
chapter CLINICAL OPHTHALMOLOGY 

 ANSWERS

. Answer: A
Management of herpetic simplex keratitis (HSK) is a popular exam topic. This question tests the
knowledge on the landmark study—​Herpetic Eye Disease Study (HEDS), which consists of six
arms (three therapeutic arms, two prophylactic arms, and one observational arm). For epithelial
HSK the treatment is purely with topical antiviral (commonly aciclovir/​ganciclovir in the United
Kingdom); the use of oral aciclovir is only indicated if patient could not tolerate topical antiviral.
There is no evidence of reducing recurrence of HSK with the use of oral antiviral in patients with
strictly epithelial disease. Mechanical debridement may provide additional benefit in epithelial HSK,
according to a Cochrane review.
The Herpetic Eye Disease Study Group. A controlled trial of oral acyclovir for the prevention of
stromal keratitis or iritis in patients with herpes simplex virus epithelial keratitis. The Epithelial Keratitis
Trial. Arch Ophthalmol 997;5(6):703–​2.
Wilhelmus KR. Antiviral treatment and other therapeutic interventions for herpes simplex virus
epithelial keratitis. Cochrane Database Syst Rev 205;:CD002898.

2. Answer: C
OCP is an uncommon cicatricial conjunctival disease that is diagnosed on clinical ground +/​–​
immunohistopathologic evidence. A positive DIF test is diagnostic of OCP but a negative result
does not exclude the diagnosis. Severely inflamed conjunctiva can result in extensive scarring,
which may reduce the diagnostic yield of the conjunctival biopsy; therefore, biopsy should be
obtained from tissue adjacent to an inflamed site. Identification of linear deposition of IgG, IgA
(not IgE), and C3 complement factors at the epithelial basement membrane zone using DIF test
is diagnostic of OCP. Conjunctival biopsy sample should be transported in Michel medium for
immunohistochemical analysis, whereas formalin solution is used for histopathologic analysis for any
evidence of neoplastic changes.
Ahmed M, et al. Ocular cicatricial pemphigoid: pathogenesis, diagnosis and treatment. Prog Retin Eye
Res 2004;23(6):579–​92.

3. Answer: B
Avellino corneal dystrophy is a rare corneal dystrophy that consists of lattice dystrophy and
granular dystrophy. Therefore, Congo red stain is useful in detecting the amyloid deposits in
Avellino corneal dystrophy. The corneal stains can be remembered by the following mnemonic:
Marilyn Monroe Always Gets Her Man in LA City, She Likes Obese man.
Macular dystrophy –​ Mucopolysaccharide –​ Alcian blue stain;
Granular dystrophy –​ Hyaline –​ Masson trichrome stain;
16 Clinical Ophthalmology  | ANSWERS

Lattice dystrophy –​ Amyloid –​ Congo red stain;


Schnyder dystrophy –​ Lipid –​ Oil-​red-​O  stain.
Kanski J, Bowling B. Clinical Ophthalmology: A Systematic Approach, 7th edition. Chapter 6: Cornea,
pp. 26–​2. Edinburgh/​New York: Elsevier/​Saunders,  20.

4. Answer: D
In 200 Dua et al. published a new classification on chemical eye injury and provided additional
prognostic factors to predict the outcome of the injury. The main differences between Dua’s
classification and Roper-​Hall classification are that the former includes limbal involvement (instead
of limbal ischaemia) and conjunctival involvement due to its ability to re-​epithelialize the cornea
if there is limbal stem cell deficiency. The reasons for modifications are that limbal involvement
(fluorescein staining of the limbus) is more objective and consistent than limbal ischaemia,
which can vary considerably among clinicians, and conjunctival involvement predicts the extent
of conjunctival stem cell loss, which plays a vital role when there is significant limbal stem cell
deficiency. There is a saying—​‘corneal epithelium is better than conjunctival epithelium; conjunctival
epithelium is better than no epithelium’ (see also Table .).

Table .  Classification system for chemical eye injury


Grading Limbal involvement Conjunctival Prognosis
(clock-​hour) involvement (%)
 0 0 Very good
2 ≤3 ≤30 Good
3 >3–​6 >30–​50 Good
4 >6–​9 >50–​75 Good—​guarded
5 >9–​2 >75–​<00 Guarded—​poor
6 2 (total) 00 (total) Very poor

Reproduced from British Journal of Ophthalmology, Dua, H. et al. A new classification of ocular surface burns. 85():
379–83. http://dx.doi.org/0.36/bjo.85..379. Copyright © 200, British Medical Journal. With permission from BMJ
Publishing Group Ltd.

5. Answer: A
OSSN is the most common non-​pigmented malignancy of the ocular surface. It encompasses a
spectrum of diseases, ranging from conjunctival intraepithelial neoplasia (also known as Bowen’s
disease), which basement membrane is not affected, to squamous cell carcinoma. It usually presents
as a unilateral vascularized limbal mass located at the interpalpebral fissure due to ultraviolet (UV)
light exposure.
The incidence rate ranged between 0.03 and .9 per 00 000/​year in the Caucasian population
to around 3 per 00 000/​year in the African population. The main risk factors for OSSN include
immunosuppression (e.g. AIDS, post-​organ transplantation, lymphoma, xeroderma pigmentosa),
ultraviolet B radiation, smoking, and infectious diseases such as HIV-​ and -​2, human papillomavirus
(HPV), hepatitis B and C virus. More importantly, OSSN may manifest as the first sign of HIV in
some cases; therefore, all patients presenting with OSSN should be checked for HIV/​AIDS and
other infectious diseases. Moreover, OSSN in HIV-​positive patients are usually associated with
larger and thicker tumour and a higher incidence of corneal, scleral, and orbital involvement.
Management of OSSN includes surgical resection and medical treatment, including topical
mitomycin C, 5-​fluorouracil, interferon α-​2b, and cidofovir. The risk of recurrence following surgical
Clinical Ophthalmology  | ANSWERS 17

treatment or topical medical treatment alone may be as high as 43%, highlighting the need of long-​
term post-​treatment surveillance.
Cicinelli MV, et al. Clinical management of ocular surface squamous neoplasia: a review of the current
evidence. Ophthalmol Ther 208;7(2):247–​62.
Kamal S, et al. Ocular surface squamous neoplasia in 200 patients: a case-​control study of
immunosuppression resulting from human immunodeficiency virus versus immunocompetency.
Ophthalmology 205;22(688):94.

6. Answer: A
Several organisms have the ability to penetrate intact corneal epithelium and this group of
organisms can be remembered using the following mnemonic ‘CHANeLS’.
C—​Corynebacterium
H—​Haemophilus influenza
A—​Acanthamoeba
Ne—​Neisseria gonorrhoea/​meningitidis
L—​Listeria
S—​Shigella
Microbial keratitis caused by this group of organisms need to be treated aggressively to prevent
permanent visual loss.
ASCRS. Special Report: Acanthamoeba Keratitis. July 2007. Available at: http://​www.ascrs.org/​sites/​
default/​files/​resources/​Acanthamoeba%20Keratitis.pdf
Tjia KF, et al. The interaction between Neisseria gonorrhoeae and the human cornea in organ culture.
An electron microscopic study. Graefes Arch Clin Exp Ophthalmol 988;226:34–​5.

7. Answer: D
The clinical scenario describes a patient with partial limbal stem cell deficiency (LSCD) following
chemical eye injury. Signs of LSCD include epithelial opacity (due to a mixture of metaplastic
corneal and conjunctival epithelial cells), stippled/​delayed fluorescein staining, loss of palisades of
Vogt, vortex/​columnar keratopathy, superficial vascularization, and recurrent/​persistent corneal
epithelial defect.
The management of LSCD depends on the patient’s symptoms, laterality of the affected eyes
(e.g. unilateral vs. bilateral), the severity of the disease (e.g. partial vs. total), and the state of the
ocular surface. In this case, this patient suffers from a partial LSCD and therefore sequential sector
conjunctiva epitheliectomy should be attempted first before undergoing autologous limbal stem
cell transplant. Anterior lamellar keratoplasty is much more invasive and does not address the
underlying LSCD problem. Amniotic membrane transplant will not work in this case because it is
mainly used to promote healing of corneal epithelial defect, which is absent in this case.
Dua HS, et al. Contemporary limbal stem cell transplantation—​a review. Clin Exp Ophthalmol
200;38:04–​7.
Le Q, et al. The diagnosis of limbal stem cell deficiency. Ocul Surf 208;6:58–​69.

8. Answer: D
Impression cytology serves as a useful clinical tool for investigating LSCD. It is performed by applying
a cellulose acetate filter paper to the ocular surface to remove the very superficial layers of ocular
surface epithelium. The presence of goblet cells on the cornea indicates the invasion of conjunctival
18 Clinical Ophthalmology  | ANSWERS

cells, which is a hallmark of LSCD. However, the absence of goblet cells on cornea does not exclude
the diagnosis of LSCD. CK-​3 and CK-​9 are important conjunctival surface markers and the presence
of these factors are suggestive of LSCD whereas CK 3 and CK 2 are corneal surface markers.
Le Q, et al. The diagnosis of limbal stem cell deficiency. Ocul Surf 208;6:58–​69.
Ramirez-​Miranda A, et al. Keratin 3 is a more specific marker of conjunctival epithelium than keratin
9. Mol Vis 20;7:652–​6.

9. Answer: C
Meesman’s dystrophy is a rare epithelial corneal dystrophy with an autosomal dominant inheritance.
It is caused by gene mutations at CK-​3 and CK-​2, which are both important cytokeratins for
corneal epithelium. The rest of the corneal dystrophies listed in the answer options are all caused
by mutations in the BIGH3 gene, or known as transforming growth factor β-​induced (TGFBI), on
chromosome 5q3. The list of corneal dystrophies caused by TGFBI/​BIGH3 mutation can be
memorized as ‘LARGE’ (BIG = LARGE).
L—​Lattice dystrophy (Type  and Type 3A)
A—​Avellino dystrophy (combination of granular and lattice dystrophy)
R—​Reis–​Buckler dystrophy/​Theil-​Behnke dystrophy (milder form of Reis–​Buckler)
G—​Granular dystrophy
E—​Epithelial basement membrane dystrophy (in some cases)
Boutboul S, et al. A subset of patients with epithelial basement membrane corneal dystrophy have
mutations in TGFBI/​BIGH3. Hum Mutat 2006;27:553–​7.
Han KE, et al. Pathogenesis and treatments of TGFBI corneal dystrophies. Prog Retin Eye Res 206;50:67–​88.

0. Answer: B
This scenario describes a patient who has developed a condition called Urrets-​Zavalia syndrome
(UZS), which is characterized by a fixed and dilated pupil usually following penetrating keratoplasty
or other types of intraocular surgeries such as DALK, Descemet stripping endothelial keratoplasty,
and cataract surgery, among others. The incidence of UZS following penetrating keratoplasty is
estimated at 0–​7.7%. The main risk factors for UZS are intraoperative iris injury, intraoperative
or postoperative raised IOP (usually within 24 hours postoperative), leading to iris ischaemia.
Therefore, checking the IOP should be the next step of the management plan in this patient. It
is unlikely that the patient has suddenly developed a dilated pupil secondary to third nerve palsy,
especially without any sign of ptosis. Therefore neuroimaging is not required at this stage. Checking
the ocular motility is reasonable but should come after the measurement of IOP.
Isac MMS, Ting DSJ, et al. Spontaneous pupillary recovery in a patient with Urrets-Zavalia syndrome
following Descemet’s membrane endothelial keratoplasty. Med Hypothesis Discov Innov Ophthalmol
209;8:7–0.
Spierer O, Lazar M. Urrets-​Zavalia syndrome (fixed and dilated pupil following penetrating
keratoplasty for keratoconus) and its variants. Surv Ophthalmol 204;59:304–​0.

. Answer: A
This is a typical scenario of a patient presenting with herpes zoster ophthalmicus (HZO) with
ocular involvement. The standard treatment of HZO is systemic antiviral treatment; for instance,
oral aciclovir 800 mg five times a day for 7–​0 days, which is the common first-​line treatment in the
United Kingdom. Topical aciclovir is commonly and inappropriately used by many clinicians when
there are corneal changes; however, studies did not show any beneficial effect of topical aciclovir
Clinical Ophthalmology  | ANSWERS 19

in early HZO. Starting oral aciclovir within 72 hours of the onset of rash significantly reduces the
risk of post-​herpetic neuralgia and ocular complications; however, it should still be considered in
patients who present after 72 hours of the onset of rash, especially when there are still new lesions
forming. Involvement of the base, side, or tip of the nose (Hutchinson’s sign) increases the risk of
ocular inflammation and corneal denervation by 3–​4 times. Bilateral periorbital swelling is caused by
gravitational oedema instead of spreading of infection; therefore, antibiotic is not warranted.
Neoh C, et al. Comparison of topical and oral acyclovir in early herpes zoster ophthalmicus. Eye (Lond)
994;8:688–​9.
Ting DSJ, et al. Herpes zoster ophthalmicus. BMJ 209;364:k5234.

2. Answer: D
This case scenario describes an uncommon presentation of subepithelial/​stromal corneal graft
rejection, which is characterized by numerous subepithelial opacities resembling adenoviral keratitis
(Krachmer’s spots). The incidence of this type of stromal rejection ranges between 2% and
5%. Although this entity represents a low-​grade rejection process, it may be associated with or
heralds the onset of other types of graft rejection; therefore, frequent topical steroids with close
monitoring are advisable. The absence of follicular conjunctivitis and presence of subepithelial
opacities within the corneal graft only go against the diagnosis of adenoviral keratitis. Erring on the
cautious side, the patient should be treated for possible stromal rejection in this case. Corneal graft
rejection can take place in various layer of cornea (Table .2).

Table .2  Types of corneal graft rejection


Types of rejection Clinical features
Epithelial Epithelial rejection line
Stromal (low grade) Subepithelial opacities (Krachmer’s spots)
Stromal (high grade) Stromal haze, oedema, and vascularization
Endothelial Most common and debilitating type; corneal oedema, endothelial line
(Khodadoust line), keratic precipitates, anterior chamber activity

Krachmer JH, Alldredge OC. Subepithelial infiltrates: a probable sign of corneal transplant rejection.
Arch Ophthalmol 978;96:2234–​7.
Panda A, et al. Corneal graft rejection. Surv Ophthalmol 2007;52:375–​96.

3. Answer: A
Corneal arcus is a common degenerative corneal disease caused by lipid deposition in
the peripheral stroma. The change normally starts at 2 and 6 o’clock and slowly spreads
circumferentially to cover the entire peripheral stroma. In advanced cases, the Bowman’s layer
and Descemet’s membrane may also be affected. Corneal arcus is frequently associated with
hypercholesterolaemia and the presence of corneal arcus in young patients should prompt the
investigation for familial hypercholesterolaemia and hyperbetalipoproteinaemia. The central edge
is usually blurred with a distinct peripheral margin, which may be sometimes associated with
peripheral thinning, named ‘senile furrow degeneration’. Ipsilateral corneal arcus is rare and has
been reported to be associated with ipsilateral ocular hypotony (due to increased blood flow to the
anterior segment) and contralateral carotid artery disease (due to reduced blood flow).
Barchiesi BJ, et al. The cornea and disorders of lipid metabolism. Surv Ophthalmol 99;36:–​22.
20 Clinical Ophthalmology  | ANSWERS

4. Answer: D
Wilson’s disease (WD) is a rare autosomal recessive systemic condition characterized by abnormal
accumulation of copper in various parts of the body, notably the basal ganglia, liver, and eye.
It is caused by a mutation of the gene that regulates copper transport protein (ATP7B). This
transporting protein supplies copper to a glycoprotein called caeruloplasmin, which transports
copper to other parts of the body via the blood. Defective ATP7B results in increased accumulation
of copper in the liver and apocaeruloplasmin (non-​copper binding form of caeruloplasmin), which
is rapidly degraded in the blood stream. The diagnosis of WD can be made by increased serum
and urinary copper levels, reduced serum caeruloplasmin, MRI brain of the basal ganglia showing
increased intensity on T2 scan (‘face of giant panda’), and liver biopsy (gold standard). The ocular
signs include Kayser-​Fleischer (KF) ring (brownish ring at the peripheral cornea at the Descemet’s
membrane) and sunflower cataract. KF ring is present in 50–​60% of patients with isolated hepatic
WD and in more than 90% of patients with neurologic involvement. Interestingly KF ring may
disappear on D-​penicillamine, the systemic treatment for WD that promotes urinary excretion of
copper.
Kelly C, Pericleous M. Wilson disease: more than meets the eye. Postgrad Med J 208;94:335–​47.

5. Answer: D
Meesmann dystrophy is a rare, autosomal dominant, epithelial corneal dystrophy that is
characterized by irregular thickening of the epithelial basement membrane and intraepithelial cysts.
Reis–​Buckler dystrophy is an autosomal dominant, corneal dystrophy that affects the Bowman’s
layer. Histopathologic examination normally reveals replacement of Bowman’s layer and epithelial
basement membrane with fibrous tissues. Thiel–​Behnke dystrophy has similar features to Reis–​
Buckler dystrophy but with additional features of ‘curly fibres’ in Bowman layer on electron
microscopy. Schnyder central crystalline dystrophy is associated with abnormal metabolism of lipid
and around 50% of the affected patients have hypercholesterolaemia. Histopathologic examination
typically shows deposition of cholesterol and phospholipids.
Kanski J, Bowling B. Clinical Ophthalmology: A Systematic Approach, 7th edition. Chapter 6: Cornea,
pp. 22–​23. Edinburgh/​New York: Elsevier/​Saunders,  20.

6. Answer: C
Keratoconus is the most common corneal ectatic disorder, with an estimated prevalence of :2000.
It is associated with eye rubbing and connective tissue diseases, including Marfan’s syndrome,
Ehlers–​Danlos syndrome, osteogenesis imperfecta, and others. Both anterior and posterior corneal
surfaces are affected in keratoconus. In fact, posterior corneal changes are often the first clinically
detectable structural changes because epithelial remodelling may mask the early anterior surface
changes. Belin Ambrosio Enhance Ectasia Display is a useful analytic tool (embedded in Pentacam
software) to detect early keratoconus. It displays the anterior and posterior elevation data relative
to the best-​fit-​sphere, which is calculated with a fixed optical zone of 8 mm, omitting the 4 mm
zone around the elevated cone. Breaks in Bowman’s layer is a diagnostic feature of keratoconus
on histopathologic examination. Sometimes they may be seen as subepithelial reticular opacities on
clinical examination if the breaks are filled with scar tissue. Posterior keratoconus is a rare, typically
non-​progressive ectatic disorder characterized by an increased curvature of the posterior corneal
surface. It is usually congenital, unilateral, and sporadic in nature.
Mas Tur V, et al. A review of keratoconus: diagnosis, pathophysiology and genetics. Surv Ophthalmol
207;62:770–​83.
Silas MR, et al. Posterior keratoconus. Br J Ophthalmol 208;02:863–​7.
Clinical Ophthalmology  | ANSWERS 21

7. Answer: A
Peter’s anomaly is a rare type of anterior segment dysgenesis syndrome characterized by central
opaque cornea (leukoma). The pathogenesis is unclear but it is postulated that there is failure of
separation of cornea and lens during embryogenesis. The corneal endothelium and Descemet’s
membrane does not develop properly, with opacity overlying the defected area. Some cases may
be associated with corneal-​lenticular adhesion or cataract (type 2). Most cases are sporadic but
autosomal recessive and dominant inheritance have been reported. It is associated with mutations
of several genes, including PAX6 (associated with aniridia), PITX2, and FOXC (associated with
anterior segment dysgenesis/​Axenfeld-​Rieger syndrome). TIMP3 gene mutation is linked to Sorsby
fundus dystrophy. Approximately 50–​70% patients develop glaucoma. Peter’s anomaly may be
associated with other ocular abnormalities such as persistent hyperplastic primary vitreous and
microphthalmia, and systemic abnormalities affecting the heart, ear, CNS, and genitourinary
systems. Peters-​plus syndrome refers to patients with Peter’s anomaly associated with cleft lip and
palate, abnormal ears, short stature, and mental retardation.
Shigeyasu C, et al. Clinical features of anterior segment dysgenesis associated with congenital corneal
opacities. Cornea 202;3:293–​8.

8. Answer: C
This scenario describes a typical presentation of conjunctival naevus, which is a benign melanocytic
tumour of the conjunctiva. It is the most common conjunctival pigmented tumours with no gender
predilection. Features suggestive of conjunctival naevus include unilaterality, focality of the lesion,
chronicity, and the presence of cysts (in around 60%). Interestingly, conjunctival naevus expresses
progesterone receptors, which might explain the changes during hormonal alternations such
as puberty or pregnancy. These changes often raise clinical concern of conjunctival melanoma,
increasing the number of unnecessary surgical excision of the lesion. Features suggestive of
conjunctival melanoma include elevation of the lesion, immobility, and vascularity. Conjunctival
melanosis is caused by excessive melanin production and retention of pigment by epithelial
melanocytes. In contrast to conjunctival naevus, it does not elevate the surface of conjunctiva and
not associated with cystic changes. It may be associated with periocular skin changes called naevus
of Ota (oculodermal melanocytosis). Conjunctival papilloma is a benign conjunctival epithelial
tumour characterized by lobulated changes with a central vascular core. They tend to be larger and
in multiple numbers in children and adolescents than in adults.
Kaliki S, et al. Conjunctival papilloma: features and outcomes based on age at initial examination. JAMA
Ophthalmol 203;3:585–​93.
Oellers P, Karp CL. Management of pigmented conjunctival lesions. Ocul Surf 202;0:25–​63.

9. Answer: A
The use of topical steroids in bacterial keratitis has always been a controversial issue. According
to a recent Cochrane review of four randomized controlled trials, there is currently inadequate
evidence showing that topical steroids improve visual acuity, infiltrate, scar size, corneal perforation
rate, and healing time on bacterial keratitis when compared to topical antibiotics alone. However,
on subgroup analysis, patients with low vision (counting fingers or worse) at baseline had .7 lines
better vision at 3 months in the topical steroids group compared with placebo group. Central ulcers
that were treated with topical steroids also had a better 3-​month corrected-​distance-​visual-​acuity
(around two lines better) at 3 months compared to placebo. In addition, non-​Nocardia ulcers have
one-​line visual improvement with additional topical steroids, whereas Nocardia ulcers have worse
outcome with additional topical steroids when compared to placebo.
Austin A, et al. Update on the management of infectious keratitis. Ophthalmology 207;24:678–​89.
22 Clinical Ophthalmology  | ANSWERS

Herretes S, et al. Topical corticosteroids as adjunctive therapy for bacterial keratitis. Cochrane Database
Syst Rev 204;0:CD005430.

20. Answer: B
Mooren’s ulcer is an idiopathic peripheral ulcerative keratitis with complete absence of systemic
disorder that is responsible for the progressive destruction of the cornea. It is usually painful
and progressive, typically starting from the peripheral cornea and progresses circumferentially
and centrally. The ulcer is concentric to the limbus, with the leading edges being undermined,
infiltrated, and de-​epithelialized. This also creates an overhanging edge at its central border. It is not
associated with scleritis. Hepatitis C infection and hookworm infestation have been reported to
be associated with Mooren’s ulcer. Wood and Kaufman classified Mooren’s ulcer into two types;
type  ulcer usually affects older patients and mild in symptoms with good response to therapy,
whereas the type 2 ulcer usually affects younger patients with a more aggressive clinical course and
poorer response to therapy. However further studies have shown that bilateral disease has a more
aggressive presentation with poorer response to treatment compared to unilateral disease. The
treatment includes intensive topical steroids, conjunctival resection, systemic immunosuppression,
and other additional surgeries such as lamellar keratoplasty and keratoepithelioplasty.
Chen J, et al. Mooren’s ulcer in China: a study of clinical characteristics and treatment. Br J Ophthalmol
2000;84:244–​9.
Garg P, Sangwan VS. Mooren’s ulcer. In: Krachmer JH, Mannis MJ, Holland EJ (eds). Cornea, 3rd edition.
New York, NY: Elsevier, 20.

2. Answer: D
This is a clinical scenario of a patient presenting with iridocorneal corneal (ICE) syndrome. ICE
syndrome is a unique unilateral ocular disease characterized by irregular corneal endothelium
with varying degrees of corneal oedema, iris atrophy, and peripheral anterior synechiae. It occurs
sporadically and does not have a specific inheritance pattern. It consists of three clinical variants,
which can be easily remembered as ‘ICE’: (a) Iris naevus/​Cogan-​Reese syndrome; (b) Chandler
syndrome (most common subtype (50%), typically presents with most significant extent of corneal
oedema, with less iris findings); and (c) Essential iris atrophy (usually has greater extent of iris
atrophy, as the name suggested, with polycoria, ectropion uveae, and corectopia). High peripheral
anterior synechiae extending above the Schwalbe’s line is considered a pathognomonic feature
of ICE syndrome. It is more commonly found in women than in men, between the age of 20 and
50 years. Around 50% of the patients with ICE also develop glaucoma.
Silva L, et al. The iridocorneal endothelial syndrome. Surv Ophthalmol 208;63:665–​76.

22. Answer: A
PAM is a potentially serious melanocytic lesion that affects the conjunctival epithelium and may
progress to conjunctival melanoma. It has been estimated that 75% of the conjunctival melanoma
arise from PAM. Bulbar conjunctiva (9%) is the most commonly affected area, followed by limbal
conjunctiva (55%), cornea (23%), and forniceal conjunctiva. According to one of the largest studies
in the literature, it found that 96% of patients with PAM were Caucasian. The most significant risk
factor for both recurrence and progression to melanoma is the extent of PAM in clock-​hours. PAM
without or with mild atypia shows 0% progression to melanoma, whereas PAM with severe atypia
shows progression to melanoma in 3% at 0-​year follow-​up.
Shields JA, Primary acquired melanosis of the conjunctiva: risks for progression to melanoma in 3
eyes. The 2006 Lorenz E. Zimmerman lecture. Ophthalmology 2008;5:5–​9.
Clinical Ophthalmology  | ANSWERS 23

23. Answer: C
PMD is a rare, idiopathic, corneal ectatic disorder that affects the peripheral cornea, usually the
inferior quadrant in a crescentic fashion. It more commonly affects males and the onset is usually
between the second and fifth decades. It is usually associated with against-​the-​rule astigmatism and
has a ‘crab-​claw’ or ‘kissing-​dove’ appearance on the corneal topography. However, studies have
shown that keratoconus may also have similar appearance on corneal topography, highlighting the
importance of interpreting the sign along with the pachymetry maps. In keratoconus, the steepest
area of cornea corresponds with the thinnest area of cornea whereas in PMD, the steepest area of
cornea is usually superior to the thinned area.
Jinabhai A, et al. Pellucid corneal marginal degeneration: a review. Cont Lens Anterior Eye
20;34:56–​63.

24. Answer: B
The use of amniotic membrane is becoming increasingly common in ophthalmology. It promotes
epithelialization and exhibits anti-​inflammatory, antifibrotic, antiangiogenic, and antimicrobial
properties. It can be used for treatment of persistent epithelial defects, non-​healing corneal ulcers,
chemical eye injury, corneal perforation, bullous keratopathy, LSCD, conjunctival reconstruction,
and dry eyes. It can also be used as an adjuvant therapy for herpetic epithelial keratitis to improve
corneal epithelialization and reduce ocular surface inflammation. Amniotic membrane can be
preserved using cryopreservation, lyophilization (freeze-​drying), and air-​drying techniques.
Cheng AMS, Tseng SCG. Self-​retained amniotic membrane combined with antiviral therapy for
herpetic epithelial keratitis. Cornea 207;36:383–​6.
Jirsova K, Jones GLA. Amniotic membrane in ophthalmology: properties, preparation, storage and
indications for grafting—​a review. Cell Tissue Bank 207;8:93–​204.
McDonald MB, et al. Treatment outcomes in the DRy Eye Amniotic Membrane (DREAM) study. Clin
Ophthalmol 208;2:677–​8.

25. Answer: B
This is a clinical scenario of Parinaud’s oculo-​glandular syndrome (POGS). POGS was first described
in 889 by Henri Parinaud on two patients with unilateral nodular or ulcerative conjunctivitis
associated with regional lymphadenopathy. All the organisms listed in the answer options have
been implicated in POGS. Bartonella henselae—​the causative organism of cat scratch disease—​is
the most common cause of POGS; however, the clinical history specifies that the patient has no
contact with cat, rendering the diagnosis unlikely. Francisella tularensis causes tularemia and it is
also one of the more frequent causes of POGS. Most patients with tularemia contract the infection
through contact with rabbits, ticks, and squirrels. Sporotrichum schenckii—​another common cause
of POGS—​is an organism that causes ocular sporotrichosis, which is usually caused by trauma from
contaminated vegetable matter or dirt. Treponema pallidum is a motile spirochete responsible for
syphilis. The patient’s age makes this diagnosis extremely unlikely in this scenario.
Altuntas EE, et al. Tularemia and the oculoglandular syndrome of Parinaud. Braz J Infect Dis
202;6:90–​.

26. Answer: D
This is not an uncommon surgical scenario and having a good knowledge helps to counsel the
patient better preoperatively. According to systematic reviews, there is strong evidence to
suggest that best corrected visual acuity and uncorrected visual acuity are better with penetrating
24 Clinical Ophthalmology  | ANSWERS

keratoplasty (PK) than DALK at 6 months or more, that refractive astigmatism and graft rejection
are less with DALK, and with no difference in spherical equivalent and keratometric astigmatism.
Henein C, Nanavaty MA. Systematic review comparing penetrating keratoplasty and deep anterior
lamellar keratoplasty for management of keratoconus. Cont Lens Anterior Eye 207;40:3–​4.

27. Answer: C
Shingles vaccination is available in the United Kingdom for people who are older than 70 but not
beyond 80 years of age. It is still beneficial for people who had previous shingles to receive the
vaccination to boost the immunity against further attack. It has been shown to reduce the incidence
rate of shingles (by around 2-​fold) and post-​herpetic neuralgia (by around 2–​3-​fold). As the vaccine
contains live-​attenuated varicella-​zoster virus, it may rarely result in reactivation of herpes zoster
infection such as keratitis and HZO.
Jastrzebski A, et al. Reactivation of herpes zoster keratitis with corneal perforation after zoster
vaccination. Cornea 207;36:740–​2.
Matthews I, et al. Assessing the effectiveness of zoster vaccine live: a retrospective cohort study using
primary care data in the United Kingdom. Vaccine 208; pii: S0264-​40X(8):366–​6.
NHS. Shingles Vaccine Overview. Available at: https://​www.nhs.uk/​conditions/​vaccinations/​
shingles-​vaccination/​

28. Answer: C
CCTS represents one of the landmark studies that was designed to evaluate the effect of donor-​
recipient histocompatibility matching and cross-​matching on the survival of corneal transplants in
high-​risk patients. It was found that neither HLA-​A, -​B, nor –​DR significantly reduces the risk of
graft failure or incidence of rejection. Positive donor-​recipient cross-​match does not substantially
increase the risk of corneal graft rejection. However, ABO blood group matching may reduce the
risk of corneal graft failure and rejection.
The Collaborative Corneal Transplantation Studies (CCTS). Effectiveness of histocompatibility
matching in high-​risk corneal transplantation. The Collaborative Corneal Transplantation Studies
Research Group. Arch Ophthalmol 992;0:392–​403.
Van Essen TH, et al. Matching for human leukocyte antigens (HLA) in corneal transplantation—​to do
or not to do. Prog Retin Eye Res 205;46:84–​0.

29. Answer: D
This is a clinical vignette of vernal keratoconjunctivitis (VKC) with persistent non-​healing corneal
ulcer/​plaque (also known as shield ulcer). The only way to treat this persistent complication of
VKC is to remove the plaque to allow for corneal re-​epithelialization. Unfortunately, the child could
not cooperate during slit-​lamp examination, otherwise corneal scrapping of the plaque should be
attempted in the clinic before doing it under general anaesthesia. Changing the topical antibiotic and
topical steroids will not improve the shield ulcer.
Addis H, Jeng BH. Vernal keratoconjunctivitis. Clin Ophthalmol 208;2:9–​23.

30. Answer: C
Lifitegrast is a recently Food and Drug Administration (FDA)-​approved treatment for dry eye
disease. It is a novel small molecule integrin that inhibits T-​cell-​mediated inflammation by blocking
the binding of two important cell surface proteins, namely the lymphocyte function-​associated
antigen  and intercellular adhesion molecule . Rituximab is a monoclonal antibody that inhibits
CD-​20 whereas ciclosporin inhibits calcineurin/​nuclear factor of activated T cells.
Clinical Ophthalmology  | ANSWERS 25

Perez VL, et al. Lifitegrast, a novel integrin antagonist for treatment of dry eye disease. Ocul Surf
206;4:207–​5.

3. Answer: C
In aniridia, other than iris hypoplasia, the most common other ocular findings to be aware of are
peripheral corneal opacity (due to LSCD but not leukoma), cataract, optic nerve hypoplasia, foveal
hypoplasia, and nystagmus. Trabeculodysgenesis is another word for primary congenital glaucoma.
Axenfeld-​Rieger anomaly is associated with iris and angle dysgenesis, but not typically leukoma.
Gould DB, John SW. Anterior segment dysgenesis and the developmental glaucomas are complex
traits. Hum Mol Genet 2002;:85–​93.

32. Answer: A
The vignette describes primary angle closure glaucoma (i.e. not just gonioscopic findings, but
elevated IOP and evidence of glaucomatous optic neuropathy). The ‘Effectiveness of early lens
extraction for the treatment of primary angle closure glaucoma (EAGLE)’ study is a landmark study
evaluating the role of early lens extraction for the treatment of primary angle closure with IOP of
30 mmHg or greater, or primary angle closure glaucoma in 49 patients. The EAGLE study found
that clear lens extraction showed greater efficacy, in terms of lower mean IOP and higher mean
health status score, and cost-​effectiveness as compared to laser peripheral iridotomy.
Azuara-​Blanco A, et al. Effectiveness of early lens extraction for the treatment of primary angle-​
closure glaucoma (EAGLE): a randomised controlled trial. Lancet 206;388:389–​97.

33. Answer: B
This is a challenging question that requires knowledge of the key findings of these landmark
glaucoma trials. This should be considered a minimum level for the exam. For studies of any
subspecialty, it is worth noting the study design, patient cohort/​control/​comparator groups, and
key findings.
AGIS found that Caucasian patients had better IOP control with primary trabeculectomy,
and black patients with primary ALT. EMGS found age, optic disc haemorrhages, intraocular
pressure, pseudoexfoliation and bilateral findings were risk factors, but not race. OHTS
found age, high IOP, high cup-​to-​disc ratio, high PSD, low CCT were risk factors, but not
optic disc haemorrhages or race.
Collaborative Normal Tension Glaucoma Study Group. The effectiveness of intraocular pressure
reduction in the treatment of normal tension glaucoma. Am J Ophthalmol 998;26:498–​505.
Heijl A, et al. Reduction of intraocular pressure and glaucoma progression: results from the Early
Manifest Glaucoma Trial. Arch Ophthalmol 2002;20:268–​79.
Kass MA, et al. The Ocular Hypertension Treatment Study: a randomized trial determines that topical
ocular hypotensive medication delays or prevents the onset of primary open-​angle glaucoma. Arch
Ophthalmol 2002;20:70–​3.
The AGIS Investigators. The Advanced Glaucoma Intervention Study (AGIS): 7. The relationship
between control of intraocular pressure and visual field deterioration. Am J Ophthalmol
2000;30:429–​40.

34. Answer: A
Overdrainage and conjunctival leaks do not go hand in hand, but the purpose of this answer is
that if you suspect a leak, you should check with fluorescein 2% (Seidel test). Option B is incorrect
26 Clinical Ophthalmology  | ANSWERS

because a blocked sclerostomy would typically be associated with a high postoperative pressure.
Option C is incorrect as retained viscoelastic in the anterior chamber would typically result in a
deep anterior chamber. Option D is incorrect as topical pilocarpine can cause further anterior
displacement of the lens-​iris diaphragm, worsening aqueous misdirection syndrome.
Murdoch I. Post-​operative management of trabeculectomy in the first three months. Community Eye
Health 202; 25:73–​5.

35. Answer: B
Options A, C, and D are all true. Option B is false because a conjunctival wound leak does not
always necessitate surgery. It depends on the individual clinical picture. Observation may be
reasonable if the leak is not deemed excessive. Parameters that may require surgery include: (a)
profuse, constant leak; (b) conjunctival retraction; or (c) the location of the leak is at limbal edge.
With regard to Option D, this is thought to be secondary to the deleterious effect of long-​term
eye drops on the health of the conjunctiva, increasing the risk of conjunctival wound leak. Take
time to read the phrasing of the question carefully as this is a negatively phrased question, which
is a common multiple-​choice question (MCQ) style in the Fellowship of the Royal College of
Ophthalmologists (FRCOphth) part 2 written exam.
Henderson HW, et al. Early postoperative trabeculectomy leakage: the incidence, time course and
severity and its impact on surgical outcome. Br J Ophthalmol 2004;88:626–​9.

36. Answer: D
Acetazolamide can cause a range of side effects, which all ophthalmologists should be familiarized
with. These include metabolic acidosis, renal stone, renal impairment, hepatic impairment,
Steven–​Johnson syndrome, dizziness, electrolyte imbalance (e.g. hypokalaemia) and rarely hypo-​
or hyperglycaemia. It is also important to note that its use is contraindicated in patients with
sulfonamide hypersensitivity.
National Institute for Health and Care Excellence (NICE). Acetazolamide. Available at: https://​bnf.nice.
org.uk/​drug/​acetazolamide.html#indicationsAndDoses

37. Answer: D
Impotence is a rare, but underreported side effect of topical β-​blocker medication. Other side
effects of topical β-​blocker include bronchospasm, bradycardia, fatigue, peripheral coldness, and
sleep disturbance with nightmares. It may also affect carbohydrate metabolism, causing hypo-​or
hyperglycaemia, and interfere with metabolic and autonomic responses to hypoglycaemia, masking
the symptoms such as tachycardia. A cardioselective β-​blocker, which has more selectivity to β
receptors than β2 (bronchial) receptors, is preferred in patients with diabetes and asthma/​chronic
obstructive pulmonary disease (if no other group of medication is suitable).
National Institute for Health and Care Excellence (NICE). Beta-​Adrenoceptor Blocking Drugs. Available
at: https://​bnf.nice.org.uk/​treatment-​summary/​beta-​adrenoceptor-​blocking-​drugs.html

38. Answer: D
The vignette describes a Fuchs’ heterochromic cyclitis (FHC). ‘Twig-​like’ vessels often form across
the angle that can bleed during lens extraction, which is known as Amsler’s sign. Some level of
anterior chamber activity can be seen long term, which does not require management. Involvement
of the posterior segment such as vitritis is often seen. Trabeculectomy is usually the first-​line
surgical option for FHC.
Clinical Ophthalmology  | ANSWERS 27

Jones NP. Glaucoma in Fuchs’ heterochromic uveitis: aetiology, management and outcome. Eye
99;5:662–​7.
La Hey E, et al. Treatment and prognosis of secondary glaucoma in Fuchs’ heterochromic iridocyclitis.
Am J Ophthalmol 993;6:327–​40.

39. Answer: A
This is a rare secondary open angle glaucoma caused by degenerated red blood cells (ghost cells)
from a previous vitreous haemorrhage, blocking the trabecular meshwork. To access the anterior
chamber (AC) there must be some communication between the anterior and posterior segments,
either from previous vitrectomy, trauma, capsulotomy, or zonular damage from another aetiology.
These tan-​coloured cells may be visible circulating in the AC or as a layer overlying the iridocorneal
angle on gonioscopy. Aqueous suppressants are generally the first-​line therapy, but patients may
require AC washout, vitrectomy, or even trabeculectomy.
Posner-​Schlossman syndrome is characterized by unilateral painless, cyclical, acute rises in IOP
(40–​80 mmHg) that typically affect young males. It is associated with AC inflammation, but the eye
remains white. Red cell glaucoma describes the IOP elevation caused by hyphema and blockage
of the trabecular meshwork. Schwartz–​Matsuo syndrome may occur if this uncommon secondary
open angle glaucoma follows a similar mechanical blockage of the trabecular meshwork, but is
caused by photoreceptor outer segments released following a rhegmatogenous retinal detachment.
Shields MB. Shields’ Textbook of Glaucoma, 5th edition. Philadelphia, PA: Lippincott Williams &
Wilkins, 2005.

40. Answer: B
It is important to be familiar with the correct terminologies of primary angle closure. The following
table (Table .3) is useful to show the spectrum of the disorder.

Table .3  Classifications of the spectrum of primary angle closure


≥2 quadrants of ITC Elevated IOP +/​− PAS GON
PACS Present Absent Absent
PAC Present Present Absent
PACG Present Present Present

PACS, primary angle closure suspect; PACG, primary angle closure glaucoma; ITC, iridotrabecular contact; PAS,
peripheral anterior synechiae; GON, glaucomatous optic neuropathy.
Data from Panda A, et al. Corneal graft rejection. Surv Ophthalmol 2007;52:375–96; and Krachmer JH, Alldredge OC.
Subepithelial infiltrates: a probable sign of corneal transplant rejection. Arch Ophthalmol 978;96:2234–7.

4. Answer: A
This is a common FRCOphth question. The following table (Table .4) is a useful summary:
28 Clinical Ophthalmology  | ANSWERS

Table .4  The effect of certain medications on intraocular pressure


Uveoscleral Aqueous production Conventional outflow
PGA i *
β-​blocker d
CAI d
α2-​agonist i d
Miotic (d) i
PGA, prostaglandin analogues; CAI, carbonic anhydrase inhibitor.
*Note also that prostamides (such as bimatoprost) also increase conventional outflow but PGAs do not. Interestingly, bimatoprost
and pilocarpine also actually increase aqueous production, but at a rate that is not significant.

Tataru CP, Purcarea VL. Antiglaucoma pharmacotherapy. J Med Life 202;5(3):247–​5.

42. Answer: D
Anecdotal evidence across clinics in the United Kingdom shows that the review time for treated and
untreated ocular hypertension (OHT) is often shorter than is clinically indicated, and NICE has tried
to tackle this in their latest guidelines. Note that the following table (Table .5) is identical for either
OHT patients on treatment, or patients with suspected chronic open angle glaucoma (COAG).

Table .5  Recommended review times for treated and untreated ocular hypertension (OHT)


Conversion to COAG Control of IOP Time to next assessment
Not detected, or uncertain No Review management plan and review
–​4 months
Uncertain Yes 6–​2 months
No conversion Yes 2–​8 months
Conversion No or Yes See COAG recommendations

© NICE (207) NG8 Glaucoma: diagnosis and management. Available from www.nice.org.uk/guidance/ng8 All rights reserved.
Subject to Notice of rights NICE guidance is prepared for the National Health Service in England. All NICE guidance is subject to
regular review and may be updated or withdrawn. NICE accepts no responsibility for the use of its content in this product/publication.

43. Answer: D
This question highlights just how prone patients with small eyes can be to this acute iatrogenic
secondary angle closure glaucoma. It is caused by aqueous drainage into the vitreous cavity resulting
in anterior displacement of the vitreous, ciliary body, and lens, with subsequent secondary angle
closure. Other procedures known to have resulted in aqueous misdirection include insertion of an
aqueous shunt, or even initiation of miotic therapy.
Clinical features include myopic shift, raised IOP (40–​80 mmHg) and a shallow or flat AC in the
absence of pupil block. Anterior segment imaging findings include anterior displacement of the iris-​
lens diaphragm (which includes the ciliary body). Management should avoid miotics; therefore, it is
imperative to differentiate from the clinically similar acute angle closure. In fact, the use of topical
atropine can encourage posterior rotation of the ciliary body. A variety of treatments, including
YAG anterior hyaloidotomy through an existing peripheral iridotomy, surgical peripheral irido-​
zonulo-​hyaloidectomy (with or without anterior/​core vitrectomy), have been described.
Shahid H, Salmon JF. Malignant glaucoma: a review of the modern literature. J Ophthalmol
202;202:852659.
Clinical Ophthalmology  | ANSWERS 29

44. Answer: A
Due to the relative very low side effect or complication profile, selective laser trabeculoplasty (SLT)
may be successful in a number of different scenarios. The only national guidance available is for
use in COAG. Importantly, again despite varied local practices, the guidance does not include an
indication for ocular hypertension (OHT).
As the evidence is not yet established, and the current national guidance does not cover OHT,
Option B is incorrect. Primary SLT is practised in glaucoma clinics throughout the United Kingdom,
and may be more effective than eye drops in patient who have been taking various topical therapies
for many years. The Cochrane collaboration reported that there is no evidence to determine
the effectiveness of laser trabeculoplasty compared to contemporary medication in open angle
glaucoma or ocular hypertension, but the LiGHT trial is likely to guide us further in this (results
not yet published). Option C is incorrect; however, if there is angle deepening following peripheral
iridotomy or cataract surgery, this may be an option. It appears safe in this subgroup of patients,
and as effective as a prostaglandin analogue in the short term in reducing IOP, however, there is
little current evidence on its long-​term effectiveness. Option D is incorrect as there is currently
little evidence in this area and eyes with narrow/​occludable iridocorneal angles may be difficult or
impossible for this modality of laser. These patients should have laser iridotomy or lens extraction
instead.
De Moura CR, et al. Laser trabeculoplasty for open angle glaucoma. Cochrane Database Syst Rev
2007;(4):CD00399.
Garg A, Gazzard G. Selective laser trabeculoplasty: past, present, and future. Eye (Lond)
208;32:863–​76.
Vickerstaff V, et al. Statistical analysis plan for the Laser-​st versus Drops-​st for Glaucoma and Ocular
Hypertension Trial (LiGHT): a multi-​centre randomised controlled trial. Trials 205;6:57.

45. Answer: C
Following is the summary of types of lens-​induced glaucoma:
(a) Lens particle glaucoma—​a type of secondary open angle glaucoma caused by inflammation of
the lens particle, after a breach in the lens capsule, either from surgery or trauma.
(b) Phacomorphic glaucoma—​a type of secondary angle closure glaucoma caused by large
cataractous lens with resultant narrowing and blockage of the AC angle.
(c) Phacolytic glaucoma—​a type of secondary open angle glaucoma caused by leakage of the
soluble lens protein of hypermature cataract into the AC, causing trabecular obstruction.
There is no history of trauma or surgery.
(d) Phacoanaphylatic glaucoma—​a type of secondary open angle glaucoma caused by
granulomatous inflammatory reaction to the lens antigen, usually after trauma or
postoperative lens retention.
American Academy of Ophthalmology. Lens Induced Glaucomas. Available at: http://​eyewiki.aao.org/​
Lens_​Induced_​Glaucomas

46. Answer: C
Option A is incorrect as using the phaco probe in the vitreous will cause greater traction on the
vitreous, pulling more vitreous to the probe or out of the vitreous cavity. This is associated with
greater risk of retinal damage. The phaco probe has no ability to ‘cut’ tissue like a vitrector has.
Option B is incorrect as if there is already vitreous present, the outcome can be similar to answer
A. Option D is incorrect as the College generally want you to take the most conservative steps,
30 Clinical Ophthalmology  | ANSWERS

working as a general ophthalmologist. Posterior vitrectomy is a subspecialist skill, and should be


carried out by a specialist VR surgeon.
Hong AR, Sheybani A, Huang AJ. Intraoperative management of posterior capsular rupture. Curr Opin
Ophthalmol 205;26:6–​2.

47. Answer: B
Post-​cataract surgery macular oedema (PCMO) or Irvine–​Gass syndrome is a common cause
for reduced vision after cataract surgery. Diabetes, retinal vein occlusion, epiretinal membrane,
macular hole, and uveitis are the most important risk factors for PCMO. A recent systematic review
of 3 articles did not show any evidence of prostaglandin analogue increasing the risk of PCMO
regardless of the time point; therefore, there is no evidence to support stopping prostaglandin
analogue before or during the course of cataract surgery.
Hernstadt DJ, Husain R. Effect of prostaglandin analogue use on the development of cystoid macular
edema after phacoemulsification using STROBE statement methodology. J Cataract Refract Surg
207;43:564–​9.
Wielders LHP, et al. Prevention of macular edema after cataract surgery. Curr Opin Ophthalmol
208;29:48–​53.

48. Answer: B
When deciding to place an IOL into the sulcus, you need to be aware of the altered effective lens
position (i.e. it will be more anterior). If the refractive power is unchanged, this lens would focus
the image slightly in front of the retina. This would result in a more myopic result than initially
intended. You therefore need to reduce the power. Although somewhat arbitrary, removing 0.5–​
.0 D (depending of the original lens power) is an effective way of doing this with some degree of
accuracy. There is a useful website for calculation of the lens power difference between in-​the-​bag
and sulcus implant.
If optic capture were possible (due to an intact anterior rhexis which had been sized appropriately
for capturing the IOL optic), you would not need to change the IOL power as the relative IOL
position would be the same (or very close to) the IOL position if it was inserted into the bag.
Doctor Hill. Calculating Bag vs. Sulcus IOL Power. Available at: http://​www.doctor-​hill.com/​iol-​main/​bag-​
sulcus.htm
Millar ER, et al. Effect of anterior capsulorhexis optic capture of a sulcus-​fixated intraocular lens on
refractive outcomes. J Cataract Refract Surg 203;39:84–​4.

49. Answer: A
The majority (60–​80%) of the postoperative endophthalmitis (POE) cases are caused by
commensal organisms, mainly Staphylococcus and Streptococcus spp. More than two cases traced
back to one operating list should certainly raise suspicion, as should a rate higher than 0.4%,
or a number of cases within a few days/​weeks. However, this does not define an outbreak, as
clusters of cases can mimic an outbreak. The statistical method should be discussed with the local
microbiology team. All cases of POE should be reported as clinical incidents.
Royal College of Ophthalmologists. Ophthalmic Services Guidance: Managing an Outbreak of
Postoperative Endophthalmitis. July 206. Available at: https://​www.rcophth.ac.uk/​wp-​content/​uploads/​
206/​07/​Managing-​an-​outbreak-​of-​postoperative-​endophthalmitis.pdf

50. Answer: A
Common causes of refractive surprises are summarized in Table .6:
Clinical Ophthalmology  | ANSWERS 31

Table .6  Potential causes for myopic refractive surprise


Myopic surprise Hypermetropic surprise
Previous hyperopic laser Previous myopic laser
Higher A constant Lower A constant
Retained viscoelastic behind IOL Undiagnosed staphyloma
Poor biometry, incorrect IOL calculation, poor surgical wound construction, incorrect IOL positioning

In terms of management of refractive surprise, the most important factor is to take time to fully
assess the patient:
(a) Was the biometry accurate with the correct formula chosen?
(b) Was the correct IOL put in?
(c) On clinical examination, is there any evidence of previously unrecognized or new
pathology: keratoconus, oedema, poor wound construction, previous refractive surgery?
(d) Is the IOL positioned correctly?
It is worth repeating the biometry, keratometry, and corneal topography. It is only then you can
begin to consider the most appropriate management plan, which may require use of contact lenses,
laser treatments, or returning to theatre for IOL exchange.
Alio JL, et al. Management of residual refractive error after cataract surgery. Curr Opin Ophthalmol
204;25:29–​7.

5. Answer: D
The diagnosis is homocystinuria, which is an autosomal recessive metabolic disorder of methionine,
leading to an abnormal accumulation of homocysteine and its metabolites in blood and urine. It is
caused by deficiency of cystathionin β-​synthase enzyme. Be aware that marfanoid is a description
of their stature and not a diagnosis of Marfan’s syndrome. Option B is incorrect as this suggests a
diagnosis of Marfan’s syndrome, which the typical ectopic position of the lens is superotemporally.
General anaesthesia is associated with increased risk of thromboses in those with homocystinuria
and should be avoided if possible. In addition, osteoporosis is common in these patients depending
on their genotype.
Kumar T, et al. Homocystinuria: therapeutic approach. Clin Chim Acta 206;458:55–​62.

52. Answer: C
The WHO checklist provides a list of checks that needs to be performed before giving anaesthesia
(‘Sign in’), before start of cataract surgery (‘Time out’), and upon completion of cataract surgery
(‘Sign out’). Checking for any difficulty in positioning should form part of the checks during ‘Sign in’
stage. All the other options should be checked during the ‘Time out’ stage. In most hospitals, special
requirement for positioning would have been assessed as part of the preoperative assessment rather
than at the time the patient is brought into the operating theatre. All other answers are correct.
NHS and National Patient Safety Agency. Surgical Safety Checklist: For Cataract Surgery Only, 200.
Available at: https://​www.rcophth.ac.uk/​wp-​content/​uploads/​204/​2/​200_​PROF_​062_​Cataract_​
Surgery_​Checklist.pdf
32 Clinical Ophthalmology  | ANSWERS

53. Answer: D
In pseudoexfoliative (PXF) eyes, the angles are typically narrower. There is a limit to which a
capsular tension ring will be effective, usually less than 3 clock-​hour involvement. Also note that
vitreous can come forward via the area of zonular dehiscence, even the posterior capsule is intact.
Interestingly the degree of PXF material does not correlate with the severity of zonular dehiscence
or the prevalence of PXF glaucoma.
PXF can cause problems intraoperatively during cataract surgery due to poor pupil dilation and
zonular instability. Steps to help combat difficulties include:
• Extensive discussion with the patient preoperatively so they are aware of the potential
problems
• Pupil manipulation (e.g. iris hooks, Malyugin ring)
• Adequate hydrodissection (minimizing torsional forces on the zonules during lens rotation)
• Minimal phaco energy
• Capsule tension ring if zonular dehiscence is observed. If phacodonesis is visualized at the slit
lamp, it may be appropriate to refer to VR for cataract surgery
• Gentle, slow movement of instruments
Shingleton BJ, et al. Pseudoexfoliation and the cataract surgeon: preoperative, intraoperative, and
postoperative issues related to intraocular pressure, cataract, and intraocular lenses. J Cataract Refract
Surg 2009;35:0–​20.

54. Answer: A
Reverse pupil block is more common in myopes. It can cause acute rise in IOP and can be
painful for the patient. The simplest way to resolve this is to use a second instrument to lift the
iris anteriorly, relieving the pupil block. There is no evidence to prove that bimanual irrigation/​
aspiration is safer than coaxial method. Shorter eyes are more likely to be associated with
intraoperative aqueous misdirection. This highest risk group is patients with nanophthalmia. Even
relatively short-​term use of α-​antagonists can result in lifelong risk of intraoperative floppy iris
syndrome (IFIS) and stopping the medication preoperatively does not reduce the risk. However,
alfuzosin may have a lower risk of IFIS than tamsulosin. Intracameral phenylephrine and/​or iris
manipulation (e.g. iris hooks, Malyugin ring) may help.
Chang DF, et al. Prospective masked comparison of intraoperative floppy iris syndrome severity with
tamsulosin versus alfuzosin. Ophthalmology 204;2:829–​34.

55. Answer: B
Although Option A sounds like a reasonable advice, the vignette should raise your concern
about a possible diagnosis of postoperative endophthalmitis, so you are obliged to review the
patient as soon as is possible. The Endophthalmitis Vitrectomy Study (EVS) showed a threefold
improvement in attaining 6/​2 for those with a vision of perception of light (N.B. patients
with vision of non-​perception of light were excluded from the study). Although topical or
intravitreal corticosteroids are frequently administered as part of the treatment of postoperative
endophthalmitis, there is little evidence to confirm the effect and therefore such practice cannot
be enforced in all cases.
Endophthalmitis Vitrectomy Study Group. Results of the Endophthalmitis Vitrectomy Study.
A randomized trial of immediate vitrectomy and of intravenous antibiotics for the treatment of
postoperative bacterial endophthalmitis. Arch Ophthalmol 995;3:479–​96.
Clinical Ophthalmology  | ANSWERS 33

56. Answer: D
Studies have shown that polishing the posterior capsule after lens cortex cleaning had no significant
role in delaying or preventing posterior capsular opacification (PCO). Other factors listed in the
options and acrylic hydrophobic lens have been shown to reduce the risk of PCO.
Khalifa MA. Polishing the posterior capsule after extracapsular extraction of senile cataract. J Cataract
Refract Surg 992;8:70–​3.
Pandey SK, et al. Posterior capsule opacification: a review of the aetiopathogenesis, experimental and
clinical studies and factors for prevention. Indian J Ophthalmol 2004;52:99–​2.

57. Answer: B
Intravitreal silicone oil can result in significant refractive changes in phakic, pseudophakic, and
aphakic eyes. In phakic and pseudophakic eyes, as the silicone oil fills up the vitreous cavity, the
concavity of the anterior part of the silicone oil at the interface of the posterior lens renders the
eye more hypermetropic. Effectively it acts like a concave lens. Conversely, the convexity of the
anterior part of the silicone oil acts as a convex lens, rendering the eye less hypermetropic.
Hotta K, Sugitani A. Refractive changes in silicone oil-​filled pseudophakic eyes. Retina 2005;25:67–​70.

58. Answer: A
Toxic anterior segment syndrome (TASS) is a sterile postoperative inflammatory reaction caused
by non-​infectious substances that enter the anterior segment, resulting in inflammation and toxic
damage to intraocular tissues. It typically starts within 24 hours of cataract surgery or other
anterior segment surgeries compared to 3–​7 days in postoperative endophthalmitis—​the main
differential diagnosis of TASS. The clinical characteristics of TASS include severe pain, eye redness,
severe anterior segment inflammation with fibrins and hypopyon, diffuse limbal-​to-​limbal corneal
oedema, negative Gram stain and culture results, and improvement with topical steroids.
Mamalis N, et al. Toxic anterior segment syndrome. J Cataract Refract Surg 2006;32:324–​3.

59. Answer: B
Fabry’s disease is a rare X-​linked lysosomal storage disorder caused by deficiency in α-​galactosidase
A enzyme. Ocular abnormalities include vortex keratopathy, congenital cataract, and tortuous
conjunctival and retinal vessels. The severity of vascular tortuosity may predict the level of impairment
of cardiac and renal functions. Anterior lenticonus is most commonly associated with Alport’s
syndrome (characterized by nephritic haematuria and deafness), Lowe’s syndrome, and Waardenburg’s
syndrome. The following article provides a good summary of all types of childhood cataract.
Amaya L, et al. The morphology and natural history of childhood cataracts. Surv Ophthalmol
2003;48:25–​44.
Sodi A, et al. Ocular manifestations of Fabry’s disease: data from the Fabry Outcome Survey. Br J
Ophthalmol 2007;9:20–​4.

60. Answer: C
Based on a Cochrane review of 6 randomized controlled trials, there is insufficient evidence
to suggest that FLACS is more superior than standard cataract surgery, in terms of visual acuity,
anterior and posterior capsular tears, postoperative CMO and elevated IOP, patient-​reported
outcomes, and cost-​effectiveness.
Day AC, et al. Laser-​assisted cataract surgery versus standard ultrasound phacoemulsification cataract
surgery. Cochrane Database Syst Rev 206;7:CD00735.
chapter CLINICAL OPHTHALMOLOGY 2

2 QUESTIONS

. A 23-​year-​old male presents with an acute inferior macular on


retinal detachment following blunt trauma to the right eye. Slit-​
lamp examination shows a clear lens, and a retinal dialysis within the
inferonasal quadrant with no posterior vitreous detachment. The
subretinal fluid encroaches on the inferior arcades. Which would
be the most likely primary surgical option used to repair the retinal
detachment?
A. Pneumatic retinopexy
B. Scleral buckle and cryotherapy
C. Suprachoroidal  buckle
D. Vitrectomy, endolaser, and gas endotamponade

2. A patient presents with a longstanding total retinal detachment. Retinal


examination shows fixed retinal folds, with a preretinal membrane
extending across 4 clock hours. The membrane is anterior to the
equator. What grade of proliferative retinopathy (PVR) does the
patient have?
A. PVR  A
B. PVR  B
C. PVR  CA-​4
D. PVR  CP-​4

3. Which of the following microorganisms are most commonly isolated in


cases of postphacoemulsification bacterial endophthalmitis in Europe?
A. β-​haemolytic streptococci
B. Coagulase-​negative staphylococcus
C. Gram-​negative bacteria (including Pseudomonas species)
D. Staphylococcus aureus
36 Clinical Ophthalmology 2 | QUESTIONS

4. According to the European Society of Cataract & Refractive Surgeons


(ESCRS) study of prophylaxis of postoperative endophthalmitis after
cataract surgery, which of the following risk factors was associated with
an increased risk of postoperative endophthalmitis?
A. Immunosuppressed patient
B. Not giving an intracameral injection of cefuroxime
C. Reusable equipment
D. Sutureless wound closure

5. Which of the following statements describes a stage 2 full-​thickness


macular hole (Gass classification)?
A. A central round retinal defect <400 microns diameter, no Weiss’s ring, rim of
elevated retina
B. A central yellow spot, loss of foveolar depression, no vitreofoveolar separation
C. A central round retinal defect ≥400 microns diameter, no Weiss’s ring, rim of elevated
retina, with or without prefoveolar opacity
D. A central round retinal defect, rim of elevated retina, Weiss’s ring, with or without
prefoveolar opacity

6. Which of the following complications of diabetic retinopathy are an


indication for three port pars plana vitrectomy?
A. Non-​clearing vitreous haemorrhage
B. Taut posterior hyaloid face
C. Tractional retinal detachment involving the macula
D. All of the above

7. Which of the following conditions is ocriplasmin licensed and approved


by NICE to treat?
A. Vitreomacular adhesion
B. Full-​thickness macular hole with a minimum linear diameter (MLD) of 450 microns
C. Vitreomacular traction with epiretinal membrane
D. Full-​thickness macular hole (with an MLD of 250 microns) with vitreomacular traction
present

8. Which of the following about optic disc pit maculopathy is true?


. It affects both sexes equally
A
B. It commonly manifests during childhood
C. The commonest presentation is with subretinal fluid
D. It is usually associated with a choroidal coloboma
Clinical Ophthalmology 2 | QUESTIONS 37

9. Where is the mostly likely position of the retinal tear in Figure 2.
(shaded area represents the area of detached retina)?
A. Inferonasal
B. Inferotemporal
C. Superonasal
D. Superotemporal

Figure 2.  

0. Which of the following is NOT an associated risk factor for central
retinal vein occlusion?
A. Diabetes mellitus
B. High plasma viscosity due to blood dyscrasia
C. Hypertension
D. Longer axial length

. Which of the following conditions is associated with angioid streaks?


A. Diabetes insipidus
B. Mercury toxicity
C. Paget’s disease
D. Sarcoidosis

2. A 34-​year-​old myopic (–​9.00) female presents with worsening vision and
distortion in her left eye. She has no past ocular history. Visual acuity
is 82 and 49 EDTRS letters in the right eye (OD) and left eye (OS),
respectively. An optical coherence tomography angiography (Figure 2.2)
scan of the left eye shows the following appearance. Which treatment
option would you recommend?
. Focal argon laser
A
B. Intravitreal aflibercept 40 mg/​ml
C. Observation
D. Triamcinolone acetate
38 Clinical Ophthalmology 2 | QUESTIONS

Figure 2.2  

3. Which of the following conditions can cause an electronegative


electroretinogram (ERG)—​full field, bright flash in scotopic conditions?
. Acute macular neuroretinopathy
A
B. Best’s disease
C. Retinitis pigmentosa
D. X-​linked retinoschisis

4. An asymptomatic 9-​year-​old female is referred to the eye clinic with


an abnormal appearance to the optic nerve margins. An ocular B-​scan
confirms hyper-​reflective deposits within the nerve head consistent with
optic disc drusen. Which of the following conditions are associated with
optic disc drusen?
A. Alport’s syndrome
B. Gronblad–​Strandberg syndrome
C. Retinitis pigmentosa
D. All of the above
Clinical Ophthalmology 2 | QUESTIONS 39

5. A 7-​year-​old male presents with bilateral retinal detachments due to


giant retinal tears. He is known to have hearing difficulties and his older
brother and father have also previously undergone treatment for retinal
detachments. Which of the following is the most likely underlying
diagnosis?
. Familial exudative vitreoretinopathy
A
B. Knobloch syndrome
C. Stickler syndrome
D. X-​linked retinoschisis

6. Which of the following studies reported that intensive blood sugar
control reduced the mean risk of developing diabetic retinopathy by 76%
in type  diabetes?
. Diabetes Control and Complications Trial (DCCT)
A
B. Diabetic Retinopathy Study (DRS)
C. United Kingdom Prospective Diabetes Study (UKPDS)
D. Wisconsin Epidemiological Study of Diabetic Retinopathy (WESDR)

7. The Early Treatment for Diabetic Retinopathy Study (ETDRS) is


a landmark trial. It reported results on the timing of panretinal
photocoagulation (PRP) in proliferative diabetic retinopathy (PDR) and
the outcome of focal laser treatment in the management of diabetic
maculopathy. All of the following statements were used to define
clinically significant diabetic maculopathy (CSMO), EXCEPT:
. Retinal thickening at or within 500 μm of the fovea
A
B. Any microaneurysm or haemorrhage within one-​disc diameter of the centre of fovea with
associated best corrected visual acuity of 6/​2 or worse
C. Hard exudates at or within 500 μm of the fovea if associated with adjacent retinal
thickening
D. An area or areas of retinal thickening one-​disc area in size, at least part of which is within
one-​disc diameter of the fovea

8. The Diabetic Retinopathy Study (DRS) demonstrated a 60% reduction


in severe visual loss (defined as vision less than 5/​200 at two or
more consecutive follow-​up visits) in eyes treated with panretinal
photocoagulation compared with controls. Which of the following
examination findings indicated high-​risk PDR?
. New vessels at the optic disc (NVD) >/​4 of disc diameter
A
B. NVD without vitreous haemorrhage
C. New vessels elsewhere (NVE) ≥/​2 of disc diameter with vitreous haemorrhage
D. Tractional retinal detachment
40 Clinical Ophthalmology 2 | QUESTIONS

9. Which of the following trial results provide evidence for the use of
fluocinolone acetonide intravitreal implant in the management of
chronic diabetic macular oedema?
A. BEVORDEX
B. FAME A and B
C. Protocol S (Diabetic Retinopathy Clinical Research Network—​DRCR net)
D. VIVID and VISTA

20. A pseudophakic, type 2 diabetic patient presents with worsening vision


due to a centre involving macular oedema, central macular thickness
(CMT) 370 microns. He has no previous ocular treatment. Which of the
following treatments is the most appropriate choice for management in
this case?
A. Aflibercept
B. Dexamethasone intravitreal implant
C. Focal argon laser
D. Ranibizumab

2. A 63-​year-​old male is referred via his optometrist with worsening vision
in both eyes. He reports a steady decline over a number of years with
some metamorphopsia. Retinal examination was normal other than
bilateral capillary abnormalities in the perifoveal region of each eye.
Optical coherence tomography (Figure 2.3A) and blue reflectance
(Figure 2.3B) showed the following characteristic appearances. What is
the most likely diagnosis?
. Bilateral retinal vein occlusions
A
B. Diabetic maculopathy
C. Macular telangiectasia Type 
D. Macular telangiectasia Type 2

(a)

Figure 2.3 
Clinical Ophthalmology 2 | QUESTIONS 41

(b)

Figure 2.3 Continued

22. A 34-​year-​old female patient presents with worsening vision in her left
eye. Fundal exam shows epiretinal membrane causing macular pucker,
a pinkish mass with surrounding subretinal exudation is seen within the
inferotemporal quadrant (Figure 2.4). Which of the following is the most
likely diagnosis?
. Amelanotic choroidal melanoma
A
B. Intermediate uveitis
C. Retinal cavernous haemangioma
D. Vasoproliferative  tumour

Figure 2.4 
Reprinted by permission from Springer Nature: Nature, Eye, 24(3): 468–47, Retinal vasoproliferative tumours. Rennie, I.
https://doi.org/0.038/eye.2009.305. Copyright © 200, Springer Nature.
42 Clinical Ophthalmology 2 | QUESTIONS

23. Which of the following medications could be best considered for the
treatment of a non-​resolving central serious chorioretinopathy (CSCR)
with worsening vision?
A. Eplerenone
B. Isoniazid
C. Prednisolone
D. Rifampicin

24. Which of the following vitamins is associated with cystic changes at the
macula?
A. Vitamin  B
B. Vitamin  B2
C. Vitamin  B3
D. Vitamin  B6

25. Photodynamic therapy (using verteporfin) is a treatment option in all of


the following, EXCEPT:
. Central serous chorioretinopathy
A
B. Choroidal haemangioma
C. Macular telangiectasia type 2
D. Polypoidal choroidal vasculopathy

26. A 35-​year-​old female presents to the eye clinic with a 2-​week history of
right eye metamorphopsia. She is highly myopic (–​8 D). Her corrected
distance visual acuity is 6/​8 OD and 6/​6 OS. Slit-​lamp examination
revealed a small area of retinal pigment epithelial disturbance with
subretinal fluid at the macula of the right eye. Which of the following
treatment regimens is most appropriate for this patient?
A. A single intravitreal ranibizumab injection followed by monthly clinical observation with
further top-​up injections as required
B. A loading dose of 3 monthly intravitreal ranibizumab injections followed by monthly
observation with further top-​up injection as required
C. A loading dose of 3 monthly intravitreal aflibercept injections followed by monthly follow-​
up with further top-​up injection as required
D. A loading dose of 3 monthly intravitreal aflibercept injections followed by 2-​monthly
injections up to a -​year time point

27. Which of the following statements regarding intravitreal lampalizumab


injection for dry age-​related macular degeneration is correct?
. Lampalizumab inhibits complement factor H
A
B. Lampalizumab does not reduce the enlargement of geographic atrophy
C. The benefit of lampalizumab is influenced by the complement factor I-​profile biomarker
D. It is associated with higher risk of postintravitreal injection endophthalmitis
Clinical Ophthalmology 2 | QUESTIONS 43

28. A 59-​year-​old female patient presents with gradual deterioration of the


central vision. She had previously undergone mastectomy for breast
cancer and has been taking tamoxifen medication for past 3 years. What
is the most likely changes you will see on fundus examination?
A. White-​yellow intraretinal crystals in the macular or paramacular area with or
without oedema
B. Ring-​shaped yellow-​orange crystals in the superficial retina
C. Narrowing of the retinal arterioles with wrinkling of the retinal surface
D. Retinal microangiopathy consisting of cotton wool spots and retinal haemorrhages

29. Which of the following fits with the description of ‘multiple intraretinal
blood-​filled saccules with bunch-​of-​grapes appearance’?
A. Capillary haemangioma
B. Cavernous haemangioma
C. Racemose haemangioma
D. Vasoproliferative  tumour

30. Which of the following statements regarding cancer-​associated


retinopathy (CAR)/​melanoma-​associated retinopathy (MAR) is NOT
correct?
A. optical coherence tomography (OCT) of the macula usually shows thinning of the
inner retina
B. The late stage of the disease may resemble the appearance of retinitis pigmentosa
C. Antirecoverin antibody has been associated with CAR
D. CAR usually affects rods more than cones

3. A 45-​year-​old woman had recently undergone a thyroidectomy for


a benign, large-​sized multinodular goitre. She presents to the eye
emergency department  week later with a right-​sided mild droopy
eyelid. The findings of the pupil examination are detailed in Table 2..
What pharmacological tests can be performed that day to help confirm
the diagnosis?
. Topical tropicamide %
A
B. Topical pilocarpine 0% followed by hydroxyamphetamine %
C. Topical apraclonidine %
D. Topical phenylephrine 0%

Table 2.  Pupil examination findings


Right pupil Findings Left pupil
2 mm Bright 2.5 mm
3 mm Dim 4 mm
Brisk Light reflex Brisk
Normal Near response Normal
44 Clinical Ophthalmology 2 | QUESTIONS

32. A 26-​year-​old female presents with one pupil larger than the other
which she noticed in the mirror. There is no pain associated but she had
fallen over last week with no head injury. On examination there was no
ptosis, no diplopia. In bright condition, pupils measured 3.5 mm (right)
and 6.5 mm (left); in dim condition, the pupils measured 6.5 mm (right)
and 7 mm (left). There was no relative afferent pupillary defect (RAPD).
What would be the most likely slit-​lamp findings?
. Heterochromia of the irides
A
B. Iris transillumination
C. Posterior synechiae
D. Sectoral palsy of iris sphincter

33. Which of the following about the prognosis of non-​arteritic anterior


ischaemic optic neuropathy is correct?
. The involvement of the fellow eye is estimated at about 5–​25% over 5 years
A
B. About 50% of the affected patients have a final visual outcome of 6/​60 or worse
C. The recurrence rate in the affected eye is around 25%
D. Around 5% of the affected eyes will recover at least 3 Snellen visual acuity over time

34. Which of the following statements about idiopathic intracranial


hypertension (IIH) is correct?
. Long-​term use of systemic corticosteroids is a recognized risk factor
A
B. The gender predilection is not influenced by puberty
C. Normal neuroimaging with normal cerebrospinal fluid (CSF) constituents is essential to
make the diagnosis of  IIH
D. The recent IIH treatment trial (IIHTT) showed that acetazolamide was effective in
improving the visual field defect in IIH patients with severe visual field loss (mean deviation
worse than –​2 dB)

35. An 87-​year-​old female presented to the eye emergency department


with sudden loss of vision in her right eye. This had come on 2 hours
before attending the hospital. She has been losing weight over the past
3 months and had a chronic headache. She had been surviving on soup
and hot drinks as she found it painful to eat. On examination her visual
acuity was perception of light OD and 6/​9 OS. She complained of slight
blurring of vision in her left eye while in the emergency department.
The erythrocyte sedimentation rate (ESR) was 26 mm/​h and C-​
reactive protein (CRP) was 42 mg/​L. What would be the most likely
finding on examination of her right eye?
. Normal optic nerve
A
B. Optic atrophy
C. Optic disc oedema with retinal cotton wool spots
D. Telangiectasia of optic nerve head
Clinical Ophthalmology 2 | QUESTIONS 45

36. A 47-​year-​old man presents to eye casualty with sudden onset vertical
diplopia. He had noticed the diplopia developing over a 2-​week period
changing from being intermittent to constant. He felt that the diplopia
was not present when he woke in the morning but came on as soon
as he got up and went downstairs. On examination he had normal
visual acuity of 6/​9 OD and 6/​6 OS. See Table 2.2 for the ocular motility
measurements. What additional test would you do next?
. Bielchowsky head tilt test
A
B. Measurement of  pupils
C. Magnetic resonance imaging (MRI) scan of  head
D. Thyroid function tests

Table 2.2  Ocular motility measurements


^R/​L
2^R/​L 0^ R/​L 4^R/​L
6^R/​L

Courtesy of the European Society of Cataract and Refractive Surgeons


(ESCRS). Reprinted from ESCRS Guidelines for Prevention and Treatment of
Endophthalmitis Following Cataract Surgery: Data, Dilemmas and Conclusions
203. Peter Barry, Luis Cordoves, Susanne Gardner. https://education.escrs.org/
wp-content/uploads/208/08/ENGLISH_208_updated.pdf

37. In a patient with suspected tonic pupil, which of the following


pharmacological tests is useful in establishing the diagnosis?
. Topical apraclonidine 0.5%
A
B. Topical cocaine 4%
C. Topical pilocarpine 0.%
D. Topical pilocarpine %

38. In a patient with suspected ocular myasthenia, which of the following


tests would you perform next?
A. Chest  X-​ray
B. Computed tomography (CT) scan of  head
C. Ice pack test
D. Tensilon  test
46 Clinical Ophthalmology 2 | QUESTIONS

39. A 7-​year-​old male presented to the eye department with a 2-​week


history of painless reduction in vision in his right eye. He was fit and
well and had just completed his basic army training. He had no history
of injury and was on no medication. He had no peripheral neurological
signs. His best corrected visual acuity was recorded as 6/​8 OD and 6/​
6 OS. He had no RAPD but his colour perception on Ishihara plates was
reduced to 3/​5 OD and 4/​5 OS. He had an MRI head and orbits which
was normal. On examination of his fundus, he had a crowded small optic
nerve head and apparent swelling of the nerve fibre layer around the
optic nerve. Two weeks later, his vision had deteriorated to 6/​60 OD and
6/​8 OS. What would you expect to find on visual field testing?
A. Bitemporal hemianopia
B. Centrocaecal scotoma
C. Homonymous hemianopia
D. Peripheral visual field defect with tunnel vision

40. Which features of a third nerve palsy would indicate an ischaemic


aetiology?
A. Aberrant regeneration
B. Improvement towards the end of the day
C. Pupil-​sparing with inferior rectus weakness
D. Recovery after 3 months

4. A 58-​year-​old man presents with sudden change in vision in his


right eye. He had started to notice that the TV was not so clear the
evening before but thought it was tiredness. He has been treated for
hypertension for past 6 years. He is otherwise well. His blood pressure
on examination is 33/​78. Visual acuity on examination is 6/​7.5 OD and
6/​6 OS. He felt his vision was blurred when he looked down. There is a
RAPD in the right eye. What features would you expect to see on fundal
examination?
. Inferior retinal detachment
A
B. Large cup-​to-​disc  ratio
C. Macular haemorrhage
D. Optic nerve oedema
Clinical Ophthalmology 2 | QUESTIONS 47

42. A 60-​year-​old patient, type 2 diabetic, presented to the eye emergency


department with a sudden drooping of the right upper eyelid with
diagonal diplopia. On examination there was a 3 mm ptosis with
restriction of eye movement on adduction of the right eye. Both pupils
were equal and reactive to light. He underwent an urgent CT brain scan
which is normal. The patient presents to you a week later in the clinic.
He complains of ongoing symptoms and examination findings remains
the same. What is the most appropriate next step?
. CT angiography of the circle of  Willis
A
B. Daily pupil check
C. Follow-​up in 2 months’ time and advise the patient to return if there is any sudden
deterioration
D. MRI head with no contrast

43. A 53-​year-​old male presented to the eye department with a 6-​month


history of reduction in visual acuity in both eyes. He was very thin
in appearance and smelled strongly of cigarette smoke and looked
unkempt. Visual function when he attended was 6/​24 OD and 6/​36 OS.
He had reduced colour vision in both eyes, 2/​5 OD and /​5 OS. Both
optic nerves appeared pale on fundal examination. What other features
would you want to explore in his history?
. Any current treatment for tuberculosis
A
B. Any recent visit to optometrist
C. Any recent bariatric surgery
D. Any recent eye injury

44. A 35-​year-​old healthy man presents to the eye clinic with intermittent
visual disturbance, described as ‘vision jumping up and down’ in the
right eye for 2 weeks. He mentions that he has been undergoing quite
a lot of stress recently. Slit-​lamp examination revealed a low amplitude,
high-​frequency torsional nystagmoid movement in the right eye only.
Examination of the left eye is otherwise unremarkable. Which of the
following topical medication has been tried for this clinical condition
with reported success?
A. Apraclonidine
B. Dexamethasone
C. Dorzolamide
D. Timolol

45. Which of the following risk factors is associated with a higher risk of
multiple sclerosis in patients with optic neuritis?
. Pain on eye movement
A
B. Severe optic disc swelling
C. Non-​perceptive of light (NPL) vision
D. Male  sex
48 Clinical Ophthalmology 2 | QUESTIONS

46. Which of the following features is associated with right-​sided, non-​


dominant, parietal lobe infarction?
A. Dysgraphia
B. Left-​right disorientation
C. ‘Pie in the sky’ visual field defect
D. Visual neglect

47. See-​saw nystagmus is most likely to be seen in which of the following


conditions?
. Cerebellar arteriovenous malformation
A
B. Craniopharyngioma
C. Parietal lobe infarction
D. Temporal lobe tumour

48. A 72-​year-​old patient, with a background of hypertension, presents to


the eye clinic with a sudden onset of right droopy eyelid and double
vision. He also appears to be losing balance towards the right side.
Otherwise there is no associated tremor or weakness of the limbs.
Ocular examination of the right eye revealed a 3 mm ptosis and
weakness in adduction and elevation on abduction. What is the most
likely diagnosis in this case?
A. Benedikt syndrome
B. Claude’s syndrome
C. Nothnagel syndrome
D. Weber syndrome

49. A 73-​year-​old diabetic man presented to eye casualty with a 2-​day


history of double vison and a unilateral partial ptosis of his left upper
eyelid. He felt that images were separated vertically and twisted. He
could move his eye upwards a little. He had slight discomfort around
his left eye. His vision was equal in both eyes at 6/​6 Snellen and on
examination he had no anisocoria, both pupils were equal in light and
dark. What is the most likely diagnosis in a man of his age?
A. Aneurysm
B. Cavernous sinus meningioma
C. Ischaemia
D. Myasthenia  gravis

50. Which of the following features is most likely seen in Parinaud dorsal
midbrain syndrome?
A. Convergence insufficiency
B. Light-​near dissociation
C. Pursuit movement is affected earlier than saccadic movement
D. Upward gaze palsy
Clinical Ophthalmology 2 | QUESTIONS 49

5. A 52-​year-​old healthy male patient presents to the eye department


with a 2-​week history of intermittent visual disturbance, described as
a persisting after-​image lasting for 0–​5 min after looking at certain
objects. He was started on a medication a few weeks prior to the onset
of this symptom. Which of the following drugs is most likely implicated
in this clinical scenario?
A. Carbamazepine
B. Vigabatrin
C. Quinine
D. Topiramate

52. Which of the following genetic mutations carries the best prognosis for
Leber’s hereditary optic neuropathy?
A. 669
B. 3460
C. 778
D. 4484

53. Which of the following features is suggestive of bilateral superior oblique


palsies?
A. Chin-​up head posture
B. Excyclotorsion  >0°
C. Prominent A pattern
D. Bilateral failure of abduction in depression

54. A 34-​year-​old female presented to A&E with sudden loss of vision in


her left eye. Visual acuity was recorded as 6/​2 OD and no perception
of light OS. She described an ache across her forehead. There was no
history of trauma and her general health was good. On examination she
had normal eye movements, her eyes were white and quiet, and she had
normal pupil reactions. What is the most likely diagnosis?
. Central retinal artery occlusion (CRAO)
A
B. Leber’s hereditary optic neuropathy (LHON)
C. Optic neuritis
D. Unexplained visual loss
50 Clinical Ophthalmology 2 | QUESTIONS

55. A lung-​transplant patient on immunosuppression presents with a large


focus of retinitis in one eye associated with patchy haemorrhages and
vascular occlusion. Aqueous sampling is positive for cytomegalovirus
(CMV) and negative for herpes simplex types  and 2 and varicella
zoster virus. Peripheral blood is positive for CMV with high viral load.
What is the most appropriate therapeutic strategy for this patient?
. Intravitreal foscarnet injection alone
A
B. Intravitreal foscarnet plus oral valaciclovir
C. Intravitreal foscarnet plus oral valganciclovir
D. Intravitreal foscarnet plus intravenous aciclovir

56. A patient with a history of type 2 diabetes (diet controlled) and


hypertension is attending for an assessment of diabetic retinopathy.
A recent HbAC is 54 mmol/​mol. What is the correct interpretation of
this value?
. The HbAC value demonstrates excellent control
A
B. A target HbAc is <48 mmol/​mol
C. A HbAC <58 mmol/​mol indicates excellent control
D. The HbAC value is satisfactory control

57. A dual energy X-​ray absorptiometry (DEXA) bone scan for a 65-​
year-​old patient on long-​term oral steroid (prednisolone 0 mg/​day)
is reported as T score –​2.6. The patient is on long-​term calcium D3
supplementation. What is the interpretation and correct action?
. The T score indicates normal bone density and no further therapy is necessary
A
B. The T score indicates osteopaenia, but no further bone protection therapy is necessary
C. The T score indicates osteopaenia and further protection therapy is necessary
D. The T score indicates osteoporosis and further protection therapy is necessary

58. Erythema chronicum migrans is associated with which following


organism?
A. Borrelia burgdorferi
B. Bartonella henselae
C. Treponema pallidum
D. Leptospira interrogans
Clinical Ophthalmology 2 | QUESTIONS 51

59. A 28-​year-​old male with known Budd–​Chiari syndrome and an


inflammatory arthritis presents with acute onset headaches and
horizontal diplopia. Thrombophilia screen is negative but inflammatory
markers are raised. Examination reveals bilateral VI nerve pareses and
papilloedema. What is the most likely cause for his papilloedema?
. Cerebellopontine angle tumour causing cerebrospinal fluid obstruction
A
B. CSF outflow obstruction at the level of foramen magnum
C. Dural venous sinus thrombosis
D. Pituitary apoplexy

60. A 34-​year-​old female is referred to the eye emergency department by


her optician for assessment of suspected disc swelling. The patient is
37/​40 pregnant and overweight. She reports minor visual blur but her
visual acuity is 6/​9 in both eyes. Examination reveals bilateral mild disc
swelling with sparse flame haemorrhages and cotton wool spots at
the posterior poles and peripheral retina in both eyes with macular
subretinal fluid. Her blood pressure is 53/​93. MRI of the brain is
otherwise normal. What is the most likely diagnosis?
A. Diabetic retinopathy
B. Hypertensive retinopathy
C. Posterior reversible encephalopathy syndrome
D. Purtscher-​like retinopathy

6. Which of the following about hemifacial spasm is correct?


. It is caused by anomalous vascular compression of the facial nerve
A
B. It is more common in males
C. The peak onset is usually 20–​30 years of  age
D. It has an autosomal recessive inheritance pattern

62. During a fluorescein angiography procedure, a 52-​year-​old female


becomes acutely unwell. On arrival you find her reclined, pale,
tachypnoeic, and clammy with a capillary refill time in excess of
5 seconds. An urticarial skin rash is present. Her blood pressure is
74/​35, pulse rate 20 bpm. She appears frightened and confused. The
emergency response team has been called. The acute management of
this patient should include:
. Intravenous hydrocortisone 00 mg and chlorphenamine 0 mg
A
B. Intravenous hydrocortisone 200 mg and chlorphenamine 0 mg
C. Intramuscular adrenaline (:000) 500 µg
D. 500–​000 ml bolus 0.9% saline fluid challenge
52 Clinical Ophthalmology 2 | QUESTIONS

63. Which of the following is associated with saddle nose deformity?


A. Polyarteritis  nodosa
B. Relapsing polychondritis
C. Takayasu arteritis
D. Tuberculosis

64. Which of the following combination regarding the medical condition and
deficiency of the enzyme is correct?
A. Albinism—​tyrosinase  enzyme
B. Fabry’s disease—​phytanic acid alpha-​hydrolase
C. Galactosaemia—​galactokinase  enzyme
D. Refsum disease—​hexosaminidase  A

65. Which of the following treatment options is best for a functioning


pituitary tumour producing prolactin?
. Dopamine agonist therapy
A
B. Dopamine antagonist therapy
C. Radiation therapy
D. Trans-​sphenoidal surgery

66. Which of the following statements regarding congenital Zika syndrome


(CZS) is correct?
. Around 20% of the patients will have some ocular findings
A
B. There is currently a licensed vaccine for Zika viral infection
C. Anterior segment abnormalities are most commonly reported in CKZ
D. It is transmitted to human via mosquito from the Aedes genus

67. A 30-​year-​old man presents to the accident and emergency department


with shortness of breath over the past 2 weeks. He also mentions that
he tends to bleed easily and examination shows signs suggestive of
oculocutaneous albinism. What is the most likely diagnosis?
A. Chédiak–​Higashi syndrome
B. Griscelli syndrome
C. Hermansky–​Pudlak syndrome
D. Waardenburg syndrome

68. Which of the following visual phenomenon/​syndromes describes


patients with cortical blindness who can see moving targets but not
stationary targets?
A. Anton syndrome
B. Blue field entoptic phenomenon
C. Charles Bonnet syndrome
D. Riddoch phenomenon
Clinical Ophthalmology 2 | QUESTIONS 53

69. Which of the following about migraine is true?


. Patients with migraine with aura are at increased risk of ischaemic stroke
A
B. Need to have at least three typical attacks to meet the criteria of migraine with aura
C. The headache usually precedes the visual aura
D. It is more common in men
chapter CLINICAL OPHTHALMOLOGY 2

2 ANSWERS

. Answer: B
The most likely surgical option among UK-​based surgeons is a scleral buckle and cryotherapy. Vitrectomy
with endotamponade could be performed, but due to the absence of a posterior vitreous detachment
and inferior location of the retinal break, it is less likely in the primary setting. Pneumatic retinopexy is
unlikely to be used, as the retinal break is inferiorly placed. Suprachoroidal buckling (viscoelastic injected
into the suprachoroidal space) is an option, but currently has not gained widespread popularity.
Heimann H, et al. Scleral buckling versus primary vitrectomy in rhegmatogenous retinal detachment: a
prospective randomized multicenter clinical study. Ophthalmology 2007;4:242–​54.
Shanmugam PM, et al. Novel techniques in scleral buckling. Indian J Ophthalmol 208;66:909–​5.

2. Answer: C
The term and classification of proliferative vitreoretinopathy (PVR) was first introduced in 983
by the Retina Society Terminology to describe a unique clinical entity that is characterized by
massive vitreous traction and preretinal proliferation. This was subsequently updated to its current
classification in 99 and it is divided into three categories:
Grade A—​vitreous haze, pigment clumps, and pigment clusters on inferior retina
Grade B—​wrinkling of inner retinal surface, retinal stiffness, vessel tortuosity, rolled edge of
retinal break
Grade CP—​full-​thickness retinal folds or subretinal strands posterior to equator. This can be
further divided into focal, diffuse, and subretinal subtypes.
Grade CA—​full-​thickness retinal folds or subretinal strands anterior to equator. This can be further
divided into anterior and circumferential subtypes.
Reprinted from American Journal of Ophthalmology, 2(2), Machemer, R. et al. An update classification of retinal detachment with
proliferative vitreoretinopathy. pp. 59–65. https://doi.org/0.06/S0002-9394(4)76695-4. Copyright © 99, with permission
from Elsevier Inc. All rights reserved.

For full details of the classification, please refer to the following references.
Di Lauro S, et al. Classification for proliferative vitreoretinopathy (PVR): an analysis of their use in
publication over the last 5 years. J Ophthalmol 206;206:7807596.
Machemer R, et al. An update classification of retinal detachment with proliferative vitreoretinopathy.
Am J Ophthalmol 99;2(2):59–​65.

3. Answer: B
Coagulase-​negative staphylococcus are the most commonly isolated microorganisms in bacterial
endophthalmitis post cataract surgery in Europe. The ESCRS Guidelines for Prevention and
Treatment of Endophthalmitis Following Cataract Surgery gives detailed information on the
management of endophthalmitis.
Table 2.3 offers a summary of common microorganisms in postoperative endophthalmitis.
Clinical Ophthalmology 2 | ANSWERS 55

Table 2.3  Common microorganisms in postoperative endophthalmitis


Prevalence Microorganisms
33–​77% Coagulase-​negative staphylococci (e.g. S. epidermidis)
0–​2% Staphylococcus aureus
9–​9% α-​ and β-​haemolytic streptococci, and S. pneumonia
6–​22% Gram-​negative bacteria including Pseudomonas aeruginosa
Up to 8% Fungi (Candida sp., Aspergillus sp., Fusarium sp.)

Courtesy of the European Society of Cataract and Refractive Surgeons (ESCRS). Reprinted from ESCRS
Guidelines for Prevention and Treatment of Endophthalmitis Following Cataract Surgery: Data, Dilemmas
and Conclusions 203. Peter Barry, Luis Cordoves, Susanne Gardner. https://education.escrs.org/wp-content/
uploads/208/08/ENGLISH_208_updated.pdf

Barry P, et al. ESCRS Guidelines for Prevention and Treatment of Endophthalmitis Following Cataract
Surgery: Data, Dilemmas and Conclusions, 203. Available at: https://​education.escrs.org/​wp-​content/​
uploads/​208/​08/​ENGLISH_​208_​updated.pdf

4. Answer: B
The ESCRS Guidelines for Prevention and Treatment of Endophthalmitis Following Cataract
Surgery identified a number of risk factors; see following link.
Barry P, et al. ESCRS Guidelines for Prevention and Treatment of Endophthalmitis Following Cataract
Surgery: Data, Dilemmas and Conclusions, 203. Available at: https://​education.escrs.org/​wp-​content/​
uploads/​208/​08/​ENGLISH_​208_​updated.pdf

5. Answer: A
This biomicroscopic classification is still widely used within vitreoretinal clinics. Optical coherence
tomography has also been used to classify full-​thickness macular holes and provide prognostic
indicators for the risk of non-​closure following surgery (Table 2.4).
Table 2.4  Gass classification
Grading Clinical features
Stage  A—​Foveolar detachment with a loss of foveal contour and lipofuscin-​coloured spot
B—​Foveolar detachment with lipofuscin-​coloured ring
Stage 2 Full-​thickness hole of <400 µm in diameter size + no complete PVD
Stage 3 Full-​thickness hole of >400 µm in diameter size + no complete PVD
Stage 4 Full-​thickness hole of >400 µm in diameter size + complete PVD

Adapted from American Journal of Ophthalmology, 9(6), Gass, J. Reappraisal of biomicroscopic classification of stages of
development of a macular hole. pp. 752–759. https://doi.org/0.06/S0002-9394(4)7278–3. Copyright © 995, with
permission from Elsevier Inc. All rights reserved.

Duker JS, et al. The International Vitreomacular Traction Study Group classification of vitreomacular
adhesion, traction, and macular hole. Ophthalmology 203;20:26–​9.
Gass JD. Reappraisal of biomicroscopic classification of stages of development of a macular hole. Am J
Ophthalmol 995;9:752–​9.

6. Answer: D
All of the aforementioned complications of diabetic retinopathy are indications for vitrectomy. The
Diabetic Retinopathy Vitrectomy Study (DRVS) was conducted to establish the benefit of early
56 Clinical Ophthalmology 2 | ANSWERS

vitrectomy in patients with severe vitreous haemorrhage from PDR. The main results reported are
as follows:
• Six-​hundred-​and-​sixteen eyes with recent severe diabetic vitreous haemorrhage reducing visual
acuity to 5/​200 (equivalent to 2/​60) or less for at least  month were randomly assigned to
either early vitrectomy or deferral of vitrectomy for  year.
• At 2 years’ follow-​up, vision of 0/​20 (or 6/​2) = 25% (early vitrectomy group) vs. 5%
(deferral group).
• In patients with type  diabetes, who were on the average younger and had more-​severe
proliferative retinopathy, there was a clear-​cut advantage for early vitrectomy, as reflected
in the percentage of eyes recovering visual acuity of 0/​20 or better (36% vs. 2% in the
deferral group, p = 0.000). No such advantage was found in the type 2 diabetes group (6%
in the early group vs. 8% in the deferral group), but evidence that this advantage differed by
diabetes type was of borderline significance.
[No authors]. Early vitrectomy for severe vitreous hemorrhage in diabetic retinopathy. Two-​year
results of a randomized trial. Diabetic Retinopathy Vitrectomy Study Report 2. The Diabetic
Retinopathy Vitrectomy Study Research Group. Arch Ophthalmol 985;03:644–​52.

7. Answer: D
Ocriplasmin is a small fragment of plasmin enzyme designed for enzymatic vitreolysis. The use of
ocriplasmin is supported by the evidence of two large phase 3 clinical trials:
(a) Microplasmin for Intravitreal Injection-​Traction Release without Surgical Treatment (MIVI-​
TRUST)—​6 months follow-​up
(b) Ocriplasmin for Treatment for Symptomatic Vitreomacular Adhesion Including Macular Hole
(OASIS)—​24 months follow-​up
A summary of the results can be seen in Table 2.5:

Table 2.5  Results of two large phase 3 clinical trials for use of ocriplasmin


Studies Ocriplasmin (O) Control (C) VMT release p value FTMH closure p value
MIVI-​TRUST 464 88 26.5% (O) vs. <0.00 40.6% (O) vs. <0.00
0.% (C) 0.6% (C)
OASIS 46 74 4.7% (O) vs. <0.00 30.0% (O) vs. 0.63
6.2% (C) 5.4% (C)

MIVI-​TRUST, Microplasmin for Intravitreal Injection-​Traction Release without Surgical Treatment; OASIS, Ocriplasmin for Treatment
for Symptomatic Vitreomacular Adhesion Including Macular Hole.

Ocriplasmin is recommended as an option for treating vitreomacular traction in adults, but only if:
• An epiretinal membrane is not present AND
• They have a stage II full-​thickness macular hole with a diameter of 400 micrometres or
less AND/​OR
• They have severe symptoms.
Khan MA, Haller JA. Ocriplasmin for treatment of vitreomacular traction: an update. Ophthalmol Ther
206;5:47–​59.
Clinical Ophthalmology 2 | ANSWERS 57

National Institute for Health and Care Excellence (NICE). Ocriplasmin for Treating Vitreomacular
Traction. Technology Appraisal Guidance [TA297]. Available at: https://​www.nice.org.uk/​guidance/​ta297/​
chapter/​-​Guidance

8. Answer: A
Optic disc pit maculopathy is a rare condition affecting ~50% of all people with a congenital optic
disc pit at some point in their life. The prevalence of congenital pits is thought to be approximately
:5000 and the incidence of pit maculopathy ~ in 2 million per annum in the United Kingdom.
There is no sex predilection, and the median age of presentation is ~35 years old. The commonest
fluid pattern at presentation is subretinal fluid with multilayered intraretinal fluid. Subretinal fluid
alone is rare. Congenital pits are only rarely associated with a coexisting choroidal coloboma. There
is an approximately 75% chance of anatomical success with surgery. Surgery can take a variety
of forms, mostly vitrectomy based. Resolution of sub-​and intraretinal fluid is often slow and a
prolonged follow-​up period is advised, before considering revision surgery.
Steel DHW, et al. Optic disc pit maculopathy: a two-​year nationwide prospective population-​based
study. Ophthalmology 208;pii: S06–​6420(8):30774–​7.

9. Answer: C
The development of subretinal fluid within rhegmatogenous retinal detachments follows certain
patterns. Retinal breaks can be located by looking at the distribution of the subretinal fluid and
applying certain principles. These principles or rules were first described by Harvey Lincoff in 97
and still hold true for retinal examination today (Lincoff ’s rule).
Lincoff H, Gieser R. Finding the retinal hole. Arch Ophthalmol 97;65:565–​9.

0. Answer: D
Identifying any underlying systemic or ocular causes is an important part of the investigation
and management of retinal vein occlusion. A comprehensive list of all reported associations and
management can be found within the RCOphth retinal vein occlusion guidelines. The RCOphth
recommendation on investigations for retinal vein occlusions (RVO) in the eye clinic has changed
significantly from 200 to 205. Current recommendation includes:
(a) Medical history
(b) BP measurement
(c) Serum glucose estimation
(d) FBC and ESR
Further medical tests are probably best performed by the patient’s physician. In the aforementioned
question, all the choices have been reported other than longer axial length. A number of studies
have reported on associations with central vein occlusion.
[No authors]. Risk factors for central retinal vein occlusion. The Eye Disorders Case-​Control Study
Group. Arch Ophthalmol 996;4:545–​54.
Elman MJ, et al. The risk for systemic vascular diseases and mortality in patients with central retinal
vein occlusion. Ophthalmology 990;97:543–​8.
Royal College of Ophthalmologists. Clinical Guidelines: Retinal Vein Occlusions (RVO), 205. Available
at: https://​www.rcophth.ac.uk/​wp-​content/​uploads/​205/​07/​Retinal-​Vein-​Occlusion-​RVO-​Guidelines-​
July-​205.pdf
58 Clinical Ophthalmology 2 | ANSWERS

. Answer: C
Angioid streak refers to linear, cracked-​line dehiscence of the Bruch’s membrane, with secondary
changes in the choriocapillaris and retinal pigment epithelium. It can be an ocular manifestation of
systemic disease and candidates should be familiar with the most common associated systemic
conditions. It is also a recognized cause for choroidal neovascular membrane. A useful acronym for
remembering them is ‘PAPER-​CLIP’.
P—​Pseudoxanthoma elasticum (or Gronblad–​Strandberg syndrome)
A—​Acromegaly
P—​Paget’s disease of  bone
E—​Ehler–​Danlos syndrome
R—​Red cell abnormality (e.g. sickle cell disease, haemolytic anaemia, hereditary spherocytosis)
C—​Calcification (e.g. hypercalcinosis, hyperphosphataemia)
L—​Lead poisoning
I—​Idiopathic
P—​Phakomatoses (e.g. neurofibromatosis, Sturge–​Weber syndrome, tuberous sclerosis)
Gurwood AS, Mastrangelo DL. Understanding angioid streaks. J Am Optom Assoc 997;68:309–​24.

2. Answer: B
Figure 2.2 demonstrates a choroidal neovascular membrane—​flow within a vascular network
within the ‘avascular’ zone of the retina. Optical coherence tomography angiography has become
increasingly widespread and candidates should be familiar with the basic concepts of the imaging.
Candidates should also be familiar with the NICE guidance for the management of choroidal
neovascular membranes, which is a common VIVA question. Focal argon laser would not be
appropriate in this case due to the risk of scarring and recurrence. Triamcinolone could be
considered as a treatment option; however, aflibercept and ranibizumab are licensed alternates.
Aflibercept is licensed by NICE for the treatment of myopic choroidal neovascular membranes—​a
link to the guidance is provide in the further reading.
National Institute for Health and Care Excellence (NICE). Age-​related Macular Degeneration. NICE
Guideline [NG82]. Available at: https://​www.nice.org.uk/​guidance/​NG82

3. Answer: D
Electrodiagnostic tests are an essential part of the investigation into retinal disorders and are
frequently tested within the FRCOphth Part 2 examination. An (electro) negative full field ERG
usually describes an International Society for Clinical Electrophysiology of Vision (ISCEV) standard
maximal response in which the b-​wave is smaller than a normal or minimally reduced a-​wave and
indicates dysfunction that is postphototransduction. The commonest causes are:
• X-​linked retinoschisis
• Congenital stationary night blindness (CSNB)
• Central retinal artery occlusion
• Central retinal vein occlusion (ischaemic)
• Melanoma-associated retinopathy
• Birdshot chorioretinopathy
• Batten disease
Clinical Ophthalmology 2 | ANSWERS 59

Best disease causes abnormal electro-​oculogram (EOG) result with a reduced Arden (light
peak: dark trough) of ≤ .5 (normal is ≥.8).
Robson AG, et al. Unilateral electronegative ERG of non-​vascular aetiology. Br J Ophthalmol
2005;89:620–​6.

4. Answer: D
Optic disc drusen consists of acellular intracellular and extracellular deposits that often become
calcified over time. They are typically buried early in life and generally become superficial, and
therefore visible, later in childhood, at the average age of 2 years. Most commonly they occur
in isolation, however, there are many associated ocular and systemic conditions, including retinitis
pigmentosa, pseudoxanthoma elasticum, and angioid streaks, Alagille syndrome, among others. It is
important to keep these in mind. A comprehensive overview is given in the following article:
Chang MY, Pineles SL. Optic disk drusen in children. Surv Ophthalmol 206;6:745–​58.

5. Answer: C
The most likely answer is Stickler syndrome, though all of the listed conditions can cause retinal
detachment. Stickler syndrome is a type of hereditary connective tissue disorder of fibrillar
collagen associated with retinal detachment, congenital megalophthalmos, deafness, cleft palate,
Pierre Robin sequence, joint hypermobility, and premature arthritis. It is the commonest cause of
rhegmatogenous retinal detachment in childhood. Human vitreous is primarily composed of types
2, 9, and  collagens and mutations affecting the genes encoding all these three collagens can cause
Stickler syndrome.
The risk of retinal detachment depends on the underlying genetic abnormality. The majority of
the cases seen by ophthalmologists are type  Stickler syndrome, with an inheritance pattern of
autosomal dominant. Candidates should be familiar with the most common form of inherited
vitreoretinopathy.
Table 2.6 offers a summary of the types and characteristics of Sticker syndrome.

Table 2.6  Types and characteristics of Sticker syndrome


Type Gene Features
Type  COL2A Membranous vitreous anomaly, megalophthalmos, deafness, arthropathy,
cleft palate
Type 2 COLA Beaded vitreous anomaly, megalophthalmos, deafness, arthropathy, cleft
palate
Type 3 COLA2 Normal vitreous and ocular phenotype, deafness, arthropathy, cleft palate
Type 4 COL9A, COL9A2 Recessive inheritance, deafness, myopia, vitreoretinopathy, epiphyseal
dysplasia
Ocular-​only COL2A Membranous vitreous anomaly, megalophthalmos. No systemic features
Other Unknown Hypoplastic vitreous, deafness, arthropathy, cleft palate

Reprinted by permission from Springer Nature: Nature, Eye, 25: 389–400. Stickler syndrome, ocular-only variant and a key
diagnostic role for ophthalmologists. Snead, M. et al. https://doi.org/0.038/eye.20.20. Copyright © 20, Springer Nature.

Snead MP, et al. Stickler syndrome, ocular-​only variant and a key diagnostic role for ophthalmologists.
Eye (Lond) 20;25:389–​400.
60 Clinical Ophthalmology 2 | ANSWERS

6. Answer: A
The DCCT reported the aforementioned finding. Modifiable risk factors within the management of
diabetic retinopathy include glycaemic control, blood pressure, and lipid levels.
The RCOphth guidelines (Section 6) gives an excellent summary of the findings in trials that have
looked each of the aforementioned risk factors, candidates for the FRCOphth should be similar
with these findings.
Royal College of Ophthalmologists. Diabetic Retinopathy Guidelines, December 202. Available
at: https://​www.rcophth.ac.uk/​wp-​content/​uploads/​204/​2/​203-​SCI-​30-​FINAL-​DR-​GUIDELINES-​
DEC-​202-​updated-​July-​203.pdf

7. Answer: B
All the options are criteria of CSMO, except for Option B, which is one of the criteria used by
national screening committee for referring patients with diabetic maculopathy (M grade) to
hospital eye service. M0 refers to no maculopathy.
The following excerpt is taken from the RCOphth Diabetic retinopathy guidelines and provides a
summary of the results of focal laser for CSMO. In patients with CSMO and normal visual acuity,
the ETDRS data indicated a trend towards benefit in laser treated patients, i.e. a 0% to 5%
reduction in incidence of visual loss of two lines of Snellen acuity equivalent (Level  evidence).
It is important to note that benefit in the ETDRS was taken as a delay in progression of visual
loss (i.e. that even when photocoagulation treatment was applied there was still an increasing
incidence of visual loss, albeit at a slower rate). It is also worth noting that ‘treatable lesions’ (i.e.
leaking microaneurysms or diffuse macular leakage) were identified by fluorescein angiography. In
the absence of clinically detectable retinal thickening (CSMO) fluorescein angiographic evidence
of leakage is not normally regarded as an indication for treatment in routine clinical practice. The
advent of OCT has altered the situation somewhat, in that very early intraretinal fluid that may not
be seen on fundal examination may be visualized on OCT, and the data from the ETDRS cannot
necessarily be extrapolated to that group of patients. In other words, focal laser reduces the risk of
moderate visual loss (2 Snellen-​line vision) by 50% in eyes with CSMO.
Reproduced from RCOphth Diabetic retinopathy guidelines https://www.rcophth.ac.uk/wp-content/uploads/204/2/203-SCI-
30-FINAL-DR-GUIDELINES-DEC-202-updated-July-203.pdf

Department of Health (NHS England). Public Health Functions to be Exercised by NHS England, 203.
Available at: https://​assets.publishing.service.gov.uk/​government/​uploads/​system/​uploads/​attachment_​
data/​file/​256492/​22_​nhs_​diabetic_​eye.pdf
Royal College of Ophthalmologists. Diabetic Retinopathy Guidelines, December 202. Available
at: https://​www.rcophth.ac.uk/​wp-​content/​uploads/​204/​2/​203-​SCI-​30-​FINAL-​DR-​GUIDELINES-​
DEC-​202-​updated-​July-​203.pdf

8. Answer: C
The DRS and ETDRS are landmark trails whose results were adopted worldwide. Both trials used
‘high-​risk’ examination findings to help determine which patients should undergo PRP and when.
High-​risk findings were defined as:
. NVD ≥/​3 disc area
2. Any NVD with vitreous haemorrhage
3. NVE ≥½ disc area with vitreous haemorrhage
High-​risk PDR was also defined as three or more of the following high-​risk characteristics (HRCs):
. Presence of vitreous haemorrhage or preretinal haemorrhage
Clinical Ophthalmology 2 | ANSWERS 61

2. Presence of any active neovascularization


3. Location of neovascularization on or within one-​disc diameter of the optic disc
4. NVD > /​3 disc area or NVE >½ disc area
Section 0 of the RCOphth Diabetic retinopathy guidelines provides a summary of the findings of
these trials and recommendations for management of all grades of diabetic retinopathy.
Royal College of Ophthalmologists. Diabetic Retinopathy Guidelines, December 202. Available
at: https://​www.rcophth.ac.uk/​wp-​content/​uploads/​204/​2/​203-​SCI-​30-​FINAL-​DR-​GUIDELINES-​
DEC-​202-​updated-​July-​203.pdf

9. Answer: B
Fluocinolone acetonide (FA) intravitreal implant is a NICE approved treatment for the management
of chronic diabetic macular oedema. FAME A and B were identical randomized trials that evaluate
the efficacy of fluocinolone acetonide.
The primary outcome reported in the FAME trials was the proportion of people with an
improvement of best corrected visual acuity (BCVA) by ≥5 letters at 2 years. Following are the
main results:
• Improvement of BCVA at 2 years: 28% (0.2 μg/​day) vs. 6% (sham)
• Cataract extraction: 4% vs. 5% vs. 7%
• VA at 3 years:
♦ Overall: 29% vs. 28% vs. 9%
♦ With chronic diabetic macular oedema (DMO) >3 years: 34% (FA 0.2 μg/​day) vs.
3% (sham)
♦ With chronic DMO <3 years: 22% (FA 0.2 μg/​day) vs. 28% (sham)
Interestingly the benefit was only significant in patients who have had chronic DMO for longer
than 3 years. A summary of the findings can be found in the following NICE Technology appraisal
guidance.
Protocol S reported visual outcomes in patients receiving panretinal photocoagulation versus
intravitreal ranibizumab for PDR over a 2-​year period (non-​inferiority randomized trial). The
BEVORDEX study was the first head-​to-​head randomized clinical trial of bevacizumab versus a
slow-​release intravitreal dexamethasone implant (DEX-​implant; Ozurdex; Allergan Inc., Irvine,
CA) for diabetic macular oedema. The VIVID and VISTA trials compared the efficacy and safety of
intravitreal aflibercept injection with macular laser photocoagulation for diabetic macular oedema
over 3 years.
National Institute for Health and Care Excellence (NICE). Fluocinolone Acetonide Intravitreal Implant
for Treating Chronic Diabetic Macular Oedema After an Inadequate Response to Prior Therapy. Technology
Appraisal Guidance [TA30], 203. Available at: https://​www.nice.org.uk/​guidance/​ta30/​chapter/​
3-​The-​manufacturers-​submission

20. Answer: B
Dexamethasone intravitreal implant (700 µg) is approved by NICE for the management of a centre
involving diabetic macular oedema, in pseudophakic patients with a CMT of less than 400 µm. Anti-​
VEGF therapy is also approved by NICE, however, the CMT must be greater than 400 µm. Focal
argon laser could be considered if fluorescein angiography shows areas of leakage that are affecting
the fovea. These areas may be close to the foveal avascular zone and are therefore unsuitable for
focal laser.
62 Clinical Ophthalmology 2 | ANSWERS

National Institute for Health and Care Excellence (NICE). Aflibercept for Treating Diabetic
Macular Oedema, 205. Available at: https://​www.nice.org.uk/​guidance/​ta346/​resources/​
aflibercept-​for-​treating-​diabetic-​macular-​oedema-​pdf-​8260262022
National Institute for Health and Care Excellence (NICE). Dexamethasone
Intravitreal Implant for Treating Diabetic Macular Oedema, 205.
Available at: https://​www.nice.org.uk/​guidance/​ta349/​resources/​
dexamethasone-​intravitreal-​implant-​for-​treating-​diabetic-​macular-​oedema-​pdf-​8260266240069
National Institute for Health and Care Excellence (NICE). Ranibizumab for Treating Diabetic
Macular Oedema, 203. Available at: https://​www.nice.org.uk/​guidance/​ta274/​resources/​
ranibizumab-​for-​treating-​diabetic-​macular-​oedema-​pdf-​82600624588

2. Answer: D
The most likely answer is macular telangiectasia (Mac Tel) type 2. Diabetic maculopathy and
bilateral RVO would be in the differential diagnoses; however, the OCT and confocal blue
reflectance are indicative of Mac Tel type 2. Figure 2.3A shows ‘internal limiting membrane (ILM)
drape’. This is a later imaging feature of Mac Tel type 2 and occurs secondarily to loss of the outer
nuclear layer and ellipsoid zone, which can progress into larger cysts (often called ‘cavitation’)
eventually encompassing all retinal layers. Confocal blue reflectance (Figure 2.3B)—​increased
reflectance of blue light (488 nm)—​is seen in Mac Tel type 2 and is thought to occur due to loss of
macular pigments and structural alterations.
Macular telangiectasia types:
Type —​congenital and unilateral. Possibly a variant of Coats disease. Uncommon.
Type 2—​acquired and bilateral. The most common form of the three types. Usually found in
middle-​aged or older patients.
Type 3—​poorly understood primarily occlusive phenomena which is quite rare.
American Academy of Ophthalmology. Macular Telangiectasia, 205. Available at:http://​eyewiki.aao.org/​
Macular_​telangiectasia
Charbel Issa P, et al. Macular telangiectasia type 2. Prog Retin Eye Res 203;34:49–​77.

22. Answer: D
Vasoproliferative tumours are uncommon retinal lesions that may occur in isolation (primary) or in
association with another ocular condition (secondary). They may be unilateral or bilateral and have
a predilection for the peripheral inferior temporal quadrant of the retina. Vasoproliferative tumours
can be associated with abnormalities of the macular, including epiretinal membrane formation and
cystoid macular oedema.
Rennie IG. Retinal vasoproliferative tumours. Eye (Lond) 200;24:468–​7.

23. Answer: D
A range of medical treatment has been investigated for the treatment of CSCR with no clearly
accepted preferred management. Endogenous and exogenous corticosteroids can have a role in the
pathogenesis of CSCR. Spironolactone and eplerenone are both aldosterone antagonist agents with
possible evidence of efficacy, though a recent randomized controlled trial, VICI, showed no effect
(awaiting publication). Rifampicin, but not isoniazid, is an antituberculous medication which is thought
to facilitate catabolism of endogenous steroids. It causes a proliferation of the smooth endoplasmic
reticulum and an increase in the cytochrome P- 450 content in the liver, thus affecting the metabolism
and bioavailability of endogenous corticosteroids, consequently possibly aiding in resolution of CSCR.
Clinical Ophthalmology 2 | ANSWERS 63

Nicholson B, et al. Central serous chorioretinopathy: update on pathophysiology and treatment. Surv


Ophthalmol 203;58:03–​26.
Willcox A, et al. Clinical efficacy of eplerenone versus placebo for central serous chorioretinopathy:
study protocol for the VICI randomised controlled trial. Eye (Lond) 209;33:295–303.

24. Answer: C
Vitamin B3 (or niacin/​nicotinic acid) has been associated with cystic changes at the macula.
However, there is no leakage on the fundus fluorescein angiography (FFA) despite the cystic
changes. It has been postulated that the drug causes direct toxic effect on Muller cell, resulting
in intracellular oedema. Similar clinical finding of non-​leaking macular oedema may be seen in
other conditions such as juvenile X-​linked retinoschisis, retinitis pigmentosa, and Goldmann-​Favre
syndrome (a severe form of enhanced S-​cone syndrome).
Domanico D, et al. Ocular effects of niacin: a review of the literature. Med Hypothesis Discov Inno
Ophthalmol 205;4:64–​7.

25. Answer: C
Photodynamic therapy (PDT) can be used to treat vascular abnormalities that arise within the retinal
and choroidal circulation. Photodynamic therapy with verteporfin causes release of free radicals when
the verteporfin is activated by the laser energy. The reaction that ensues between the free radicals and
blood vessel endothelial cell membranes cause locally increased histamines, thromboxane, and TNF-​α,
all immune modulation factors. The anti-​inflammatory response can lead to series of events including
vasoconstriction, thrombosis, increased vascular permeability, blood stasis, and hypoxia. Many ocular
conditions can be treated with PDT—​the following article summarizes the evidence for PDT in each
of the conditions listed earlier. Macular telangiectasia type 2 has no proven treatment unless it is
complicated by choroidal neovascularization, which can be treated by anti-​VEGF.
American Academy of Ophthalmology. Photodynamic Therapy (PDT), 207. Available at: http://​eyewiki.
aao.org/​Photodynamic_​Therapy_​(PDT)

26. Answer: A
This is a clinical vignette of myopic choroidal neovascularization (CNV). Myopic CNV is typically
seen as a small, flat, greyish membrane that may have a hyperpigmented border if chronic or
recurrent. There appears to be three main stages of myopic CNV: the first phase results in direct
damage to photoreceptors; the second phase is the regression of CNV, resulting in the formation
of a fibrous pigmented scar (also known as the Förster-​Fuchs’ spot); and the third phase is the
formation of atrophy around the regressed CNV.
Evidence has suggested that myopic choroidal neovascular membrane (CNVM) behaves differently
from age-​related CNVM whereby the myopic CNVM usually requires less intravitreal anti-​VEGF
injections. The phase 2 (REPAIR) and phase 3 (RADIANCE) trials showed that patients who
received pro re nata (PRN) regimens of ranibizumab were able to achieve significantly greater gains
in best corrected visual acuity than verteporfin photodynamic therapy (vPDT). The median of
required injections was 2–​4 injections.
Wong TY, et al. Myopic choroidal neovascularisation: current concepts and update on clinical
management. Br J Ophthalmol 205;99:289–​96.

27. Answer: B
Lampalizumab is a selective complement factor D inhibitor. Two identically designed phase 3
randomized controlled trials, Chroma and Spectri, examined its efficacy and safety for treating
64 Clinical Ophthalmology 2 | ANSWERS

geographic atrophy secondary to age-​related macular degeneration. Unfortunately, the treatment


did not reduce the enlargement of geographic atrophy compared to sham injection at 48 weeks.
The risk of endophthalmitis was similar to other intravitreal anti-​VEGF treatment studies.
Holz FG, et al. Efficacy and safety of lampalizumab for geographic atrophy due to age-​related
macular degeneration: Chroma and Spectri phase 3 randomized clinical trials. JAMA Ophthalmol
208;36:666–​77.

28. Answer: A
Tamoxifen is an antioestrogen drug frequently used for breast cancer. The incidence of tamoxifen
retinopathy is extremely low and usually presents after 3 years of treatment or after a total
cumulative dose of more than 00 g. Option B refers to the changes observed in patients who are
taking canthaxanthin, which is a naturally occurring carotenoid used for skin pigmentation in vitiligo.
Option C refers to the changes observed in patients who are taking vigabatrin, an antiepileptic
medication. Option D refers to the changes in patients taking interferon alpha, which is used in
treatment of Kaposi’s sarcoma, chronic hepatitis C, leukaemia, and lymphoma, among others.
Nencini C, et al. Retinopathy induced by drugs and herbal medicines. Eur Rev Med Pharmacol Sci
2008;2:293–​8.
Tang RJ, et al. Retinal changes associated with tamoxifen treatment for breast cancer. Eye (Lond)
997;:295–​7.

29. Answer: B
The main types of retinal vascular tumours are listed in the options. Capillary haemangioma is an
uncommon benign hamartoma of the retina, consisting of capillary-​like vessels. It is characterized
by red nodular lesions with dilatation and tortuosity of the feeding artery and draining vein.
Cavernous haemangioma is a benign hamartoma of retinal vessels, usually large-​calibre and thin-​
walled, characterized by a ‘bunch-​of-​grapes’ appearance of the lesion with blood-​filled saccules.
Racemose haemangioma is a rare retinal arteriovenous malformation, which may be associated with
Wyburn-​Mason syndrome. Vasoproliferative tumour is usually a dome-​shaped appearance retinal
lesion, commonly located at the inferior temporal peripheral retina with telangiectatic vessels on
the surface of the lesion.
Wang W, Chen L. Cavernous hemangioma of the retina: a comprehensive review of the literature
(934–​205). Retina 207;37:6–​2.

30. Answer: D
CAR is the most common intraocular paraneoplastic retinopathy. The most common primary
tumours are small-​cell lung carcinoma followed by gynaecologic and breast malignancies. The
average age of symptom onset is 65 years. The symptoms are mainly caused by the dysfunction
of rod and cone photoreceptors. Patients typically present with photosensitivity, photopsia, glare,
reduced central vision, and colour vision. CAR normally affects Cones > Rods and MAR affects
rods more than cones. The fundus usually appears normal at the initial stage but may have optic
nerve pallor, attenuated retinal arterioles, and retinal pigment epithelial thinning and mottling at
later stage, mimicking retinitis pigmentosa. Electroretinogram in CAR typically shows global retinal
dysfunction with severely reduced photopic a-​and b-​waves. OCT may show severe macular
atrophy with thinning of the outer retina (not inner retina) and loss of photoreceptors and inner
segment/​outer segment (IS/​OS) junction.
Rahimy E, Sarraf D. Paraneoplastic and non-​paraneoplastic retinopathy and optic
neuropathy: evaluation and management. Surv Ophthalmol 203;58:430–​58.
Clinical Ophthalmology 2 | ANSWERS 65

3. Answer: C
This is a clinical vignette of right Horner’s syndrome occurring after thyroidectomy due to
inadvertent damage to the oculosympathetic pathway. It is important to establish the diagnosis of
Horner’s as this enables the correct investigations to be performed so that any treatable cause is
identified. After clinical examination, confirmation is usually done using pharmacological tests. The
classic test is using cocaine 4% or 0%. This will dilate a normal pupil but not a Horner’s pupil. If
cocaine is not available, then apraclonidine 0.5% or % can be used. Cocaine blocks the reuptake
of norepinephrine at the neuromuscular junction so the normal pupil dilates as there are more
norepinephrine available; however, the pupil affected by a Horner’s has little or no norepinephrine
to be blocked so the pupil will dilate poorly.
Apraclonidine % is an alpha2-​agonist and in normal patients causes pupil constriction; however, the
drug also has weak alpha-​agonist property which promotes pupil dilatation. Therefore, in a pupil
that has been sympathetically denervated, the pupil dilator muscle develops super sensitivity to the
alpha-​agonist property of apraclonidine so the Horner’s pupil will dilate. Hydroxyamphetamine
test is useful as it will dilate a normal or preganglionic Horner’s but not a postganglionic Horner’s
pupil. In addition, adrenaline :000 and phenylephrine %, which are weak alpha-​agonists,
have also been used to localize a Horner’s pupil. They dilate a Horner’s pupil that is caused by
postganglionic lesion (due to denervation hypersensitivity) but not by central or preganglionic
lesion.
Cambron M, et al. Apraclonidine and my pupil. Clin Auton Res 20;2:347–5.
Gao Z, Crompton JL. Horner syndrome: a practical approach to investigation and management. Asia
Pac J Ophthalmol (Phila) 202;:75–​9.
Giannaccare G, et al. Horner syndrome following thyroid surgery: the clinical and pharmacological
presentations. J Ophthalmic Vis Res 206;:442–​4.

32. Answer: D
This is a clinical vignette of left Adie’s tonic pupil. Many patients with a tonic pupil present with
anisocoria. Often this has been noticed by family members and there is usually no clear history
of the time of onset. A tonic pupil is found in approximately two people per 000 in the general
population. The usual features are a dilated pupil with a sluggish response to light and poor
response to near. There are usually vermiform movements of the iris tissue. The condition is
caused by ciliary ganglion or short ciliary nerve damage. The responses in the pupil function are
related to the ciliary ganglion fibres. Most of the ciliary ganglion fibres are used for accommodation
(about 95%) and so the accommodation response of the ciliary muscles is more likely to be spared
than the iris sphincter muscles.
There is no specific cause, but a tonic pupil has been associated with herpes zoster, Lyme disease,
sarcoidosis, and inflammatory conditions such as giant cell arteritis and rheumatoid arthritis that
may cause a ciliary ganglionitis. Other local causes include orbital trauma (blunt or penetrating)
and orbital tumours. Females are affected more often that males at approximately 3:. Systemic
neurological conditions that have been associated with tonic pupils include dysautonomias such
as Shy–​Drager syndrome. It is vital to examine the eye thoroughly as paralysis of the third cranial
nerve can cause a dilated pupil, but there are usually other associated features.
Wilhelm H. Disorders of the pupil. Handb Clin Neurol 20;02:427–​66.

33. Answer: A
The vision can deteriorate over 2 weeks after the initial presentation and usually stabilizes by
2 months. The prognosis of the affected eye is usually good; about 50% will have a final vision of
66 Clinical Ophthalmology 2 | ANSWERS

6/​9 or better and 25% will have a final vision of 6/​60 or worse. Around 5–​40% eyes will recover
3 Snellen vision over time. There is a 0–​20% chance of deterioration of vision of the affected
eye at 2 years compared to the initial presentation. The risk of recurrence of the disease in the
same eye was reported at 3–​8%. The risk of fellow eye involvement is estimated at 5–​25% over
5 years. The Eyewiki website provides a very good summary on this topic.
American Academy of Ophthalmology. Non-​Arteritic Anterior Ischemic Optic Neuropathy (NAION), 209.
Available at: http://​eyewiki.aao.org/​Non-​Arteritic_​Anterior_​Ischemic_​Optic_​Neuropathy_​(NAION)

34. Answer: C
IIH is a neurological disorder characterized by increased intracranial pressure of unknown origin.
It typically affects overweight women of childbearing age. Before puberty, it affects boys and
girls equally whereas after puberty, women are nine times more likely to be affected than men.
The Friedman or modified Dandy diagnostic criteria for IIH include: () normal neurological
evaluation with exception of cranial nerve abnormalities; (2) normal neuroimaging showing normal
brain parenchyma without hydrocephalus or mass; (3) normal CSF constituents; (4) presence
of papilloedema; and (5) elevated CSF opening pressure during lumbar puncture (more than
25 cm of water in adults and more than 28 cm of water in children). The recent IIHTT showed
that acetazolamide is a well-​tolerated first-​line therapy in IIH patients with mild visual field loss
(perimetric mean deviation between –​2 dB and –​7 dB) but not in severe visual field loss. Drugs that
may be associated with intracranial pressure can be remembered as ‘STAIN-​LONG’:
S—​Steroid withdrawal (after long-​term use)
T—​Tetracycline and derivatives
A—​Vitamin A and retinoids
I—​Indomethacin
N—​Nitrofurantoin
L—​Lithium
O—​Oral contraceptive pills
N—​Nalidixic  acid
G—​Growth hormone
Madriz Peralta G, Cestari DM. An update of idiopathic intracranial hypertension. Curr Opin Ophthalmol
208;29:495–​502.

35. Answer: C
This clinical scenario is highly suggestive of giant cell arteritis (GCA), which is a systemic vasculitis
which tends to affect persons >50 years old. The incidence rate of GCA tends to increase with
increasing age. It is much more common in the Western world. GCA rarely occurs in patients aged
between 30 and 50 years but there are isolated case reports. GCA affects males and females equally
and the mean age of onset is 70 years of age. It most commonly affects cranial branches of arteries
from the arch of the aorta, and loss of vision in one or both eyes occurs in up to 50% of patients.
Classically patients present with a temporal headache and jaw claudication which can lead to
weight loss as the patient finds eating difficult. Visual symptoms may be the presenting feature of
the disease. A normal/​borderline ESR/​CRP is possible and in a patient with a high clinical suspicion
of GCA it is still prudent to treat this. The gold standard for diagnosis is still a temporal artery
biopsy. A positive temporal artery biopsy will reveal intimal thickening of the blood vessels and
there will be necrosis of parts of the arterial wall and there is formation of granulomas containing
Clinical Ophthalmology 2 | ANSWERS 67

multinucleated giant cells. Other investigations performed in some hospitals include ultrasound
scanning demonstrating a ‘halo sign’ which may be due to thickening of the artery wall.
Vodopivec I, Rizzo JF 3rd. Ophthalmic manifestations of giant cell arteritis. Rheumatology (Oxford)
208;57:ii63–​72.

36. Answer: B
The table demonstrates an ocular motility problem suggestive of right inferior rectus weakness—​
large R/​L at right gaze and down gaze. Isolated inferior rectus weakness may manifest as the
presenting sign of myasthenia gravis. This patient may have myasthenia gravis but no comment
has yet been made on the pupil responses. It is important to endure that there is no evidence of
pupil involvement as this would indicate development of a third nerve palsy, which would require
different investigations. Myasthenia gravis will not have any involvement of the pupillary muscles.
Spoor TC, Shippman S. Myasthenia gravis presenting as an isolated inferior rectus paresis.
Ophthalmology 979;86:58–​60.

37. Answer: C
The iris sphincter is highly sensitive to substances similar to acetylcholine and therefore when
a substance similar to acetylcholine is instilled then the pupil response is highly active, so the
abnormal pupil constricts even in response to weak stimulation. With time aberrant regeneration
may develop of the fibres supplying the ciliary muscle increasing nerve supply to the iris sphincter
muscles. Adie syndrome or Homes-​Adie syndrome is characterized by pupillary changes with
reduced or lost deep tendon reflexes.
Wilhelm H. Disorders of the pupil. Handb Clin Neurol 20;02:427–​66.

38. Answer: C
The ice pack test has been used now for some time and has gained more interest as it is a simple
bedside test that can be performed with little discomfort to the patient and little risk (unlike the
Tensilon test which can lead to cardiovascular collapse). The ice is applied to the dropped eye lid
for 2–​5 min and pre-​and post-​ice measurements of lid position are taken. A positive result is if the
eye lid opens by 2 mm or more. The ice causes improvement in the signs of ptosis by cooling the
skeletal muscle tissue and therefore cooling the activity of the acetyl cholinesterase enzyme. The
Tensilon test used to be readily available but it is now difficult to source edrophonium so other tests
are required. After confirming the diagnosis of myasthenia gravis (MG), chest X-​ray can be utilized
to diagnose thymoma in patients with MG but the sensitivity ranges between 45% and 80%; CT
chest is a more definite way to image thymoma. Treatment involves the use of pyridostigmine and
an immunosuppressant such as prednisolone and/​or azathioprine.
Natarajan B, et al. Accuracy of the ice test in the diagnosis of myasthenic ptosis. Neurol India
206;64:69–​72.

39. Answer: B
This clinical vignette is suggestive of LHON. It tends to affect males more than females, with
males having a higher risk of visual loss (up to 80%) than females (up to 35%). The visual loss is
painless and tends to affect both eyes eventually, often within the space of a few weeks to months.
The visual loss is often severe usually in the range of 6/​60 to hand movements with central or
centrocaecal scotomas. There is often a family history of visual loss and it is important to take a
thorough family history in any cases presenting with this type of optic neuropathy. Prognosis is poor
for most patients with LHON and the main treatment involves low vision assessments.
68 Clinical Ophthalmology 2 | ANSWERS

The differential diagnoses for bilateral sequential optic neuropathy include anterior ischaemic
optic neuropathy (which tends to affect patients >40 years old who often have associated risk
factors such as diabetes, atherosclerosis, or collagen vascular disorders such as systemic lupus
erythematosus), and demyelination, which is a possibility in this man. It is always important to
exclude neuromyelitis optica (NMO) in bilateral optic nerve disease but this man has no other
neurological symptoms. It would be important to take a blood test for Aquaporin-​4 antibody and
consider a lumbar puncture.
Kim US, et al. Leber hereditary optic neuropathy-​light at the end of the tunnel? Asia Pac J Ophthalmol
(Phila) 208;7:242–​5.

40. Answer: D
Most ischaemic third nerve palsies show signs of recovery within 3 months. If there are any signs of
aberrant regeneration in a third nerve palsy such as upper lid retraction and pupil constriction on
eye movement, then an alternative diagnosis must be sought as this is often a sign of compression
from a slow growing tumour such as meningioma in the cavernous sinus or aneurysm. Pupil-​
sparing with single muscle weakness is indicative of partial third nerve palsy, which may be a sign
of compressive lesion. Third nerve palsy will not have any diurnal variation, unlike cases of ocular
myasthenia.
Kung NH, Van Stavern GP. Isolated ocular motor nerve palsies. Semin Neurol 205;35:539–​48.

4. Answer: D
This man is describing blurring of vision in his inferior field of vision and therefore an inferior retinal
detachment would give a superior field defect. He would usually also describe floaters and flashing
lights. A macular haemorrhage would be more likely to give a central scotoma and more significant
reduction in visual acuity. The most likely diagnosis in this case is non-​arteritic anterior ischaemic
optic neuropathy (NA-​AION) and this is often associated with the ‘disc-​at-​risk’ where the cup-​to-​
disc ratio is small and the disc often appears crowded and hyperaemic. Optic nerves affected by
GCA can also be crowded, so other features need to be taken into account but a large cup-​to-​disc
ratio would be uncommon with NA-​AION.
Hayreh SS. Ischemic optic neuropathy. Prog Retin Eye Res 2009;28:34–​62.

42. Answer: A
All patients with a partial third nerve palsy (e.g. partial ptosis, preservation of some eye movements
supplied by the third nerve) that is pupil-​sparing should undergo imaging with angiography of
some type either CT angiogram or MR angiogram, particularly looking at the area of posterior
communicating artery. This is because even if the patient is high risk for an ischaemic third nerve
palsy (e.g. diabetes in this case), there is still a risk that the pupil may eventually dilate after
compression from an aneurysm.
Saito R, et al. Pupil-​sparing oculomotor nerve paresis as an early symptom of unruptured internal
carotid-​posterior communicating artery aneurysms: three case reports. Neurol Med Chir (Tokyo)
2008;48:304–​6.

43. Answer: A
This clinical scenario describes a patient who is thin with possibly malnutrition. However, there is no
option for questioning on the nutrition intake. The patient may have been suffering from a chronic
illness such as tuberculosis. Therefore, before performing any further investigations, it is important
to know if the patient had been taking any prescribed antituberculosis drugs as ethambutol can
cause optic neuropathy which can present several months after commencing the drug.
Clinical Ophthalmology 2 | ANSWERS 69

Optic neuropathy may occur in % of patients taking ethambutol at the World Health Organization
(WHO) recommended dose. Patients who develop symptoms or signs of ethambutol optic
neuropathy should be referred to the ethambutol-​prescribing physician immediately for
discontinuation or dose reduction of the drug. Several studies have examined the potential value
of optical coherence tomography in screening for ethambutol optic neuropathy (EON). There was
reduction of retinal nerve fibre layer thickness in patients with clinically significant EON; however,
its ability to detect such changes in patients taking ethambutol without visual symptoms has yet to
be confirmed.
Chamberlain PD, et al. Ethambutol optic neuropathy. Curr Opin Ophthalmol 207;28:545–​5.

44. Answer: D
This is a clinical vignette of superior oblique myokymia (SOM), which is characterized by unilateral
(often the right eye), low amplitude (less than 4°), high frequency (more than 50 Hz), torsional,
nystagmoid movement of superior oblique muscle. It normally occurs in otherwise healthy
individuals and is frequently triggered by fatigue, stress, or excessive caffeine intake. There is no
definitive treatment for SOM but various medications, including topical/​systemic β-​blockers,
carbamazepine, phenytoin, baclofen, gabapentin, memantine, botulinum toxin injections, have
been tried with success. In some cases SOM may be linked with neurovascular compression of the
trochlear nerve; therefore neuroimaging is recommended in recurrent or longstanding cases.
Zhang M, et al. Superior oblique myokymia. Surv Ophthalmol 208;63:507–​7.

45. Answer: A
Various factors have been reported in determining the risk of multiple sclerosis in patients with
optic neuritis. Factors associated with lower risk of MS include:
. Male sex
2. Optic disc swelling
3. Atypical optic neuritis features such as absent pain, vision is NPL, and severe disc swelling
with haemorrhage
Essentially, patients presenting with typical symptoms of demyelinating optic neuritis have a higher
risk of developing MS. In addition, the length of optic nerve being involved and involvement of
intracanalicular segment are poor visual prognostic factors.
Optic Neuritis Study Group. Multiple sclerosis risk after optic neuritis final optic neuritis treatment
trial follow-​up. Arch Neurol 2008;65:727–​32.

46. Answer: D
Right-​sided, non-​dominant, parietal lobe infarction may result in left-​sided visual neglect and pie-​
on-​the-​floor visual field defect (inferior quadrantanopia) as opposed to pie in the sky visual field
defect that is observed in temporal lobe infarction. Left-​sided, dominant, parietal lobe infarction
may result in Gerstmann’s syndrome, which is characterized by left-​right disorientation, finger
agnosia (inability to name or recognize fingers), dysgraphia (inability to write), and dyscalculia
(inability to calculate).
Ting DS, et al. Visual neglect following stroke: current concepts and future focus. Surv Ophthalmol
20;56:4–​34.

47. Answer: B
See-​saw nystagmus is a rare ophthalmic manifestation with less than 50 cases reported in the
literature. It is a type of disconjugate, torsional nystagmus in which one eye rises and intorts while
70 Clinical Ophthalmology 2 | ANSWERS

the other eye falls and extorts in one cycle, and reverse in the next cycle. The most common cause
is parasellar masses (including pituitary tumour, craniopharyngioma), mesodiencephalic disease,
brainstem stroke, trauma, multiple sclerosis, and congenital disease.
Drachman DA. See-​saw nystagmus. J Neurol Neurosurg Psychiat 966;29:356–​6.

48. Answer: C
Various fascicular third nerve syndromes have been described in the literature. Table 2.7 shows a
summary of the four syndromes listed in the option.

Table 2.7  Various fascicular third nerve syndromes


Syndromes Third nerve palsy + other features Lesion site
Benedikt Contralateral intention tremor Red nucleus/​paramedian midbrain
Claude Contralateral ataxia Midbrain
Nothnagel Ipsilateral ataxia Superior cerebellar peduncle
Weber Contralateral hemiparesis Cerebral peduncle

American Academy of Ophthalmology. Acquired Oculomotor Nerve Palsy, 209. Available at: http://​
eyewiki.aao.org/​Acquired_​Oculomotor_​Nerve_​Palsy

49. Answer: C
Ischaemia is the most common cause of all acquired third cranial nerve palsies (around 40%) but in
all cases of acquired third nerve palsy, an underlying aneurysm must be excluded as there is always
this risk in all cases of third nerve palsy. A pupil-​sparing third nerve palsy may well be ischaemic but
partial third nerve palsy which is pupil-​sparing may still be caused by an aneurysm. Both ischaemic
and aneurysmal third nerve palsies may be painful. Patients with a third nerve palsy should all
undergo imaging to exclude an aneurysm at the time of presentation.
Fang C, et al. Incidence and etiologies of acquired third nerve palsy using a population-​based method.
JAMA Ophthalmol 207;35:23–​8.

50. Answer: B
Parinaud dorsal midbrain syndrome is a neurological disorder caused by damage of the vertical
gaze centre located at the rostral interstitial nucleus of medial longitudinal fasciculus. This results
in a constellation of symptoms and signs, which can be remembered by the mnemonic ‘DULL-​
PC’. Saccadic movement is usually affected first followed by smooth pursuit and vestibulo-​ocular
reflex.
D—​Downward gaze in primary position (sun-​setting sign)
U—​Upward gaze palsy
L—​Light-​near dissociation
L—​Lid retraction (Collier’s sign)
P—​Papilloedema (commonly present)
C—​Convergence retraction nystagmus
It is noteworthy to mention that the doll’s head manoeuvre could elevate the eyes as this is a
supranuclear palsy. The presence of light-​near dissociation is due to the pretectal involvement
of the lesion, resulting in more damage to the afferent light pathway with relatively preserved
Clinical Ophthalmology 2 | ANSWERS 71

accommodative pathway; therefore, patients will have impaired pupillary light reflex but preserved
near response. Convergence retraction nystagmus is best elicited on upgaze saccades, which can
be brought on by asking the patient to track a downwardly rotating optokinetic drum. The co-​
contraction of all horizontal extraocular muscles results in inward pulling of the globe. As medial
rectus is a stronger muscle than lateral rectus, a convergence movement is shown. The most
common cause is dependent on the age group: pinealoma (children), multiple sclerosis (20s–​30s),
and upper brainstem stroke (elderly).
Feroze KB, Patel BC. Parinaud syndrome. In: StatPearls [Internet]. Treasure Island (FL): StatPearls
Publishing 208. Available at: https://​www.ncbi.nlm.nih.gov/​books/​NBK44892/​

5. Answer: D
This is a clinical vignette describing a patient with palinopsia, which is a visual phenomenon
characterized by persistence or recurrence of visual images after the removal of initial stimulus. It
may be caused by disease affecting the parieto-​occipital pathway or some medications, including
topiramate (an antiepileptic drug), trazadone, risperidone, mirtazapine, and acetazolamide. It
was proposed that the increased serotonergic activity secondary to 5-​HT2 receptors of these
medications predispose to the manifestation of palinopsia. Carbamazepine has been shown to
effective in treating palinopsia.
Yun SH, et al. Topiramate-​induced palinopsia: a case series and review of the literature. J
Neuroophthalmol 205;35:48–​5.

52. Answer: D
LHON is a disease caused by point mutations in the mitochondrial DNA. The most important
prognostic factor for visual recovery in patients with LHON is the gene mutation status. There are
three main types of mutation:
(a) 4484 mutation: 37—​7% chance of some degree of visual improvement
(b) 778 mutation: 4% chance of improvement
(c) 3460 mutation: similar to 778 but numbers are too small for comparison
Other positive prognostic factors include age of onset of <20 years, and especially <0 years of age.
Newman NJ. Treatment of Leber hereditary optic neuropathy. Brain 20;34:2447–​50.

53. Answer: B
Superior oblique (SO) palsy is a common cause of vertical strabismus. Bilateral SO palsy is most
commonly due to trauma. There are five features that are suggestive of bilateral SO palsy. A useful
mnemonic is ‘CRAVE’:
C—​Chin-​down head posture
R—​Reversing hyperdeviation on left gaze (R/​L) and right gaze (L/​R)
A—​Adduction failure in depression
V—​Prominent V pattern
E—​Excyclotorsion  >0°
Note that patients with bilateral SO palsy usually do not have a head tilt. Congenital SO palsy will
have a large vertical prism fusion range and high concomitance as compared to new onset acquired
lesion.
Kushner BJ. The diagnosis and treatment of bilateral masked superior oblique palsy. Am J Ophthalmol
988;05:86–​94.
72 Clinical Ophthalmology 2 | ANSWERS

54. Answer: D
At this presentation it is not possible to be certain of the definite diagnosis, so a tentative diagnosis
of unexplained visual loss is the best diagnosis. Until further examination and investigations have
been performed, you will not be able to offer a diagnosis. The patient could have any of the
suggested diagnoses, so it is best to wait until further tests are available. Often at the onset of optic
neuritis there is a more gradual visual loss rather than immediately presenting with NPL but there
are cases described with this presentation. LHON rarely presents with sudden loss of vision but
often there is not a RAPD at presentation so this diagnosis should still be considered. In addition,
there may be optic nerve head crowding/​oedema with telangiectatic vessels visible on fundus
examination. Most patients with CRAO usually have a RAPD at presentation, especially if the vision
is NPL.
Griffiths PG, Ali N. Medically unexplained visual loss in adult patients. Curr Opin Neurol 2009;22:4–​5.

55. Answer: C
CMV causes significant morbidity and mortality in immunocompromised individuals. Systemic
antiviral therapy with adjunctive intravitreal therapy is the standard approach. Intraocular treatment
alone (Option A) is only appropriate if systemic therapy is strongly contraindicated such as severe
myelosuppression or nephrotoxicity. Systemic treatment protects the other eye. Systemic antiviral
therapies for CMV are ganciclovir, valganciclovir, foscarnet, cidofovir, and leflunomide. Valganciclovir
is an oral pro-​drug and avoids intravenous administration needed for ganciclovir. Foscarnet is the
standard intravitreal medication used in the United Kingdom (ganciclovir is an alternative agent).
Intravenous acyclovir or high-​dose valaciclovir (the oral pro-​drug) are used to treat herpes simples
and varicella zoster acute retinal necrosis (ARN). In ARN there is evidence that combined systemic
and intravitreal therapy may yield greater therapeutic efficacy than systemic treatment alone. All
systemic antiviral drugs have significant toxicity profiles and require monitoring: valganciclovir is
myelotoxic and nephrotoxic and foscarnet and cidofovir are nephrotoxic.
Port AD, et al. Cytomegalovirus retinitis: a review. J Ocul Pharmacol Ther 207;33:224–​34.
Schoenberger SD, et al. Diagnosis and treatment of acute retinal necrosis: a report by the American
Academy of Ophthalmology. Ophthalmology 207;24:382–​92.

56. Answer: B
UK NICE guidelines for diabetic control are available. Candidates are not expected to recall these
guidelines in detail but should have an understanding for the FRCOphth of HbAC interpretation
to counsel patients appropriately. In type 2 diabetes, for patients on diet/​lifestyle management +/​–​
single oral drug therapy the target HbAC is <48 mol/​mol. The level here is therefore suboptimal
and diet and lifestyle advice are appropriate, and drug modification by the patient’s physician may
be necessary. In those with drug hypoglycaemia, the target is relaxed to a target level of 53 mmol/​
mol. Levels <48 mmol/​mol represent good control. In patients on a single drug with HbAC
>58 mmol/​mol intervention with diet lifestyle/​drug intervention is recommended to a target of
53 mmol/​mol.
National Institute of Health and Care Excellence (NICE). Type 2 Diabetes in Adults: NICE
Guideline [NG28], 205. Available at: https://​www.nice.org.uk/​guidance/​ng28/​chapter/​
recommendations#dietary-​advice-​2

57. Answer: D
DEXA bone T score compares the bone density against a normal, healthy young female adult. A T
score of –​2.6 SD indicates osteoporosis. Glucocorticosteroids increase the risk of fracture of the
hip and spine and loss of bone mineral density. In the United Kingdom, the femoral neck T score
Clinical Ophthalmology 2 | ANSWERS 73

is used to calculate fracture risk, using an online tool called FRAX® (available online). Important
clinical risk factors considered in the FRAX assessment are low body mass index (BMI), lack of
weight-​bearing exercise, smoking, and alcohol (>3 u/​day), secondary causes of osteoporosis,
history of fragility fractures, and family history of hip fracture and osteoporosis. In general, DEXA
bone scan is recommended for those exposed/​anticipated to continuous oral corticosteroid (any
dose) for >3 months.
The National Osteoporosis Guideline Group (NOG, March 207; see Table 2.8) recommends with
regard to glucocorticoid-​induced osteoporosis:
• Women and men age >70 years with a previous fragility fracture, or taking high doses of
glucocorticoids (>7.5 mg/​day prednisolone), should be considered for bone protective
therapy.
• In other individuals fracture probability should be estimated using FRAX with adjustment for
glucocorticoid dose.
• Bone protective treatment should be started at the onset of glucocorticoid therapy in
individuals at high risk of fracture.
• Alendronate and risedronate are first-​line treatment options.

Table 2.8  Bone densitometry scores


T score Condition
0 to –​ SD Normal
–​ SD to –​2.5 SD Osteopenia
Below –​2.5 SD Osteoporosis

NOGG 207: Clinical guideline for the prevention and treatment of osteoporosis. Available online at
https://​www.sheffield.ac.uk/​NOGG/​NOGG%20Guideline%20207.pdf
FRAX Fracture Risk Assessment Tool. Available at: https://​www.sheffield.ac.uk/​FRAX/​

58. Answer: A
Lyme borreliosis is a tick-​borne bacterial infection. B. burgdorferi is inoculated via a tick bite. After
a period of days to 3 weeks, the typical primary lesion, erythema migrans, commences. The
lesion has a target appearance with erythema and induration at the centre (the tick bite) and
at the spreading margin. The appearance is unique and can be used to diagnose Lyme disease,
supplemented by serological testing. B. henselae is responsible for cat-​scratch disease; the associated
ocular diseases are oculoglandular syndrome, keratitis, neuroretinitis, and uveitis, and a pustular
reaction may be seen at the entry site (caused by cat-​scratch). L. interrogans is associated with
a macular skin rash and T. pallidum, which causes syphilis, is associated with a painful chancre
at the primary stage and in the secondary stages, maculopapular rash (usually on the trunk),
maculopapular eruptions on the soles and palms, and condylomata.
Bush LM, Vazquez-​Pertejo MT. Tick borne illness-​Lyme disease. Dis Mon 208;64:95–​22.

59. Answer: C
The key here is avoiding confusion between the Budd–​Chiari and Arnold–​Chiari syndromes. The
latter involves anatomical anomaly at the foramen magnum predisposing to cerebellar tonsillar
descent and potential brain stem compression. Budd–​Chiari syndrome refers to thrombosis of
74 Clinical Ophthalmology 2 | ANSWERS

the hepatic portal vein. In a young male with arthritis and a major vessel thrombosis, Behçet’s
disease, which is a well-​recognized cause of dural sinus thrombosis, should be near the top of the
differential.
Yazici H, et al. Behcet syndrome: a contemporary view. Nat Rev Rheumatol 208;4:07–​9.

60. Answer: B
The concern here is pre-​eclamptic toxaemia a condition that affects 3–​5% of pregnancies and
is characterized by development of new onset hypertension in the second half of pregnancy
and proteinuria. Blood pressure tends to run lower in pregnancy with readings above 40/​90
sufficient to diagnose hypertension. Widespread endothelial dysfunction relating to arterioles
causes damage to vascular beds. Posterior reversible encephalopathy syndrome caused by pre-​
eclamptic cortical oedema (which will be highlighted on MRI brain) is a well-​recognized cause
of visual loss in this condition, however, a more marked drop in acuity would be expected in
cortical blindness. Diabetes either arising de novo in pregnancy (gestational diabetes) or gestational
acceleration of established diabetes would be more likely to produce blot haemorrhages,
microaneurysms, and exudates rather than the features described which are typical of hypertensive
retinopathy. Purtscher (traumatic cause) or Purtscher-​like (non-​traumatic cause) retinopathy is
a type of microvascular occlusive retinal disease characterized by cotton wool spots, Purtscher
flecken (pathognomonic sign), and retinal haemorrhages. The findings are usually centred around
peripapillary area.
Tadin I, et al. Hypertensive retinopathy and pre-​eclampsia. Coll Antropol 200;25Suppl:77–​8.

6. Answer: A
Hemifacial spasm (HFS) is a sporadic, rarely autosomal dominant, condition characterized
by involuntary synchronous tonic and/​or clonic contraction of facial muscles. It is caused by
dysfunctional facial nerve usually as a result of anomalous vascular compression of the facial
nerve, which can be visualized on high resolution MRI. The anterior inferior cerebellar artery and
posterior inferior cerebellar artery are the most common compressing single vessels. It has a female
predominance with a peak onset of 40–​60 years of age.
Green KE, et al. Treatment of blepharospasm/​hemifacial spasm. Curr Treat Options Neurol 207;9:4.

62. Answer: C
This patient is in anaphylactic shock. The cause here is anaphylaxis on the basis of recent exposure
to an allergen (fluorescein dye), the presence of an urticarial skin rash with acute cardiovascular
compromise, and an urticarial skin rash. The correct answer is dependent on familiarity with the
management of acute anaphylaxis. While less severe reactions (for adults) may be treated with
intravenous (IV) 200 mg hydrocortisone and intravenous 0 mg chlorphenamine, this patient
is in anaphylactic shock. The UK Resuscitation Council Guidelines emphasize the early use of
intramuscular (IM) adrenaline in anaphylaxis, usually the dose is 0.5 mg (0.5 ml of adrenaline
:000). IV adrenaline should only be used by personnel with specialist training. Once IM adrenaline
has been given, an IV fluid bolus of crystalloid fluid is recommended. Colloids should ideally be
avoided in the context of allergy as they may contain potential allergens.
UK Resuscitation Council Guidelines. Anaphylaxis, 2008. Available at: https://​www.resus.org.uk/​
anaphylaxis/​emergency-​treatment-​of-​anaphylactic-​reactions/​

63. Answer: B
Saddle nose deformity results from a depression caused by a decrease in the structural support
of the cartilaginous or bony framework deep to the nasal soft tissue envelope. It can be caused
Clinical Ophthalmology 2 | ANSWERS 75

by various conditions, including trauma, surgery, inflammatory diseases (e.g. granulomatosis with
polyangiitis, relapsing polychondritis, sarcoidosis, and Crohn’s disease), and infection (e.g. syphilis,
leprosy, and septal abscesses).
Pribitkin EA, Ezzat WH. Classification and treatment of the saddle nose deformity. Otolaryngol Clin
North Am 2009;42:437–​6.

64. Answer: A
Tyrosinase is responsible for melanin synthesis and absence or reduced tyrosinase will lead to
albinism. Fabry’s disease is caused by deficiency in alpha-​galactosidase A. Galactosaemia is caused
by deficiency in galactose-​-​phosphate uridyltransferase, whereas Refsum disease is caused by
deficiency in phytanic acid alpha-​hydrolase.
Poll-​The BT, et al. The eye in metabolic diseases: clues to diagnosis. Eur J Paediatr Neurol
20;5:97–​204.

65. Answer: A
Pituitary adenoma represents 2–​5% of symptomatic intracranial tumour. They usually occur in
adults, rarely in childhood, and are mostly isolated but 3% are associated with multiple endocrine
neoplasia type . They can be divided into macroadenoma (0 mm or more) vs. microadenoma
(<0 mm), or functioning vs. non-​functioning tumours. Functioning tumours are associated with
hormonal secretion; therefore, they are detected earlier and smaller (microadenoma). The most
common type is prolactinoma followed by growth hormone-​secreting adenoma. Prolactinoma
can result in gynaecomastia, amenorrhoea, and galactorrhoea (known as ‘Forbes-​Albright
syndrome’) and infertility in women and gynaecomastia, hypogonadism, and impotence in men.
The treatment for prolactinoma is bromocriptine—​a dopamine agonist—​which inhibits the
release of prolactin.
On the other hand, non-​functioning tumours do not produce excess hormone and may cause
hypopituitarism due to mass effect. The most common type is gonadotrophic adenoma. Most
ophthalmologic symptoms and signs are secondary to mass effects caused by macroadenoma.
Patients with pituitary adenoma need to be referred to Endocrinology immediately as many patients
develop pan-​hypopituitarism and need a full review of all the hormone levels. Many patients have a
functioning pituitary adenoma which responds to medical treatment and early intervention ensures
less comorbidity such as hypertension and cardiovascular disease. If the pituitary tumour is non-​
functioning, then the patient would be referred for surgery.
Oki Y. Medical management of functioning pituitary adenoma: an update. Neurol Med Chir (Tokyo)
204;54:958–​65.

66. Answer: D
Zika virus is an arbovirus primarily transmitted to humans via the Aedes genus mosquito. It can lead
to Guillain–​Barré syndrome and congenital birth abnormalities, including neurologic, ophthalmic,
audiologic, and skeletal abnormalities (now called CZS). Around 70% of the patients will some form
of ocular abnormalities. Posterior segment abnormalities such as loss of retinal pigment epithelium,
perivascular choroidal inflammatory infiltrates, chorioretinal atrophy, and retinal haemorrhages,
are the most common ophthalmic findings in CZS. Other ophthalmic abnormalities include iris
coloboma, glaucoma, cataract, and lens subluxation findings. Currently there is no licensed vaccine
or treatment for Zika viral infection.
de Oliveira Dias JR, et al. Zika and the eye: pieces of a puzzle. Prog Retin Eye Res 208;66:85–​06.
76 Clinical Ophthalmology 2 | ANSWERS

67. Answer: C
All the options listed are syndromic forms of albinism. Chédiak–​Higashi syndrome (CHS) is a rare
autosomal recessive disorder characterized by recurrent severe pyogenic infections, progressive
neurologic abnormality, and mucosal disease. The many ‘i’s in the name of the syndrome can
help candidates to link the disease to Infection. Griscelli syndrome is characterized by neurologic
deficit, with or without immunologic impairment. It is distinguished from CHS by the lack of giant
intracellular granules seen in CHS. Hermansky–​Pudlak syndrome is platelet deficiency (resulting in
bleeding), pulmonary disease, and granulomatous colitis. Waardenburg syndrome is characterized
by iris heterochromia, broad nasal root, and white forelock.
American Academy of Ophthalmology. Albinism, 204. Available at: http://​eyewiki.aao.org/​Albinism

68. Answer: D
These are several interesting visual phenomenon/​syndromes that the candidates should be familiar
with. Anton syndrome refers to patients with cortical blindness who deny their blindness. This is
due to bilateral retrochiasmal visual pathway diseases such as bilateral occipital lobe infarction. Blue
field entoptic phenomenon describes the phenomenon of multiple tiny dots moving rapidly along
wavy lines in the visual field, which is caused by the moving white blood cells within the superficial
retinal capillaries. Charles Bonnet syndrome refers to formed and unformed visual hallucination
in patients with bilateral poor vision. They usually have good insights when they are experiencing
hallucination. Pulfrich phenomenon refers to patients seeing targets moving towards them when the
targets are moving perpendicular to the line of sight (i.e. laterally). It usually occurs in optic nerve
diseases. Riddoch phenomenon refers to the preserved awareness of moving but not stationary
stimuli.
Arcaro MJ, et al. Psychophysical and neuroimaging responses to moving stimuli in a patient
with the Riddoch phenomenon due to bilateral visual cortex lesions. Neuropsychologia 208;
pii: S0028–​3932(8):30204–​5.

69. Answer: A
Migraine is a debilitating chronic neurologic disorder with a global prevalence of 2%. It is more
common in female than male, with an estimated ratio of 3:. Migraine is typically unilateral,
pulsating, moderate-​severe pain, lasting 4–​72 hours, and associated with nausea and/​or vomiting or
photophobia/​phonophobia. The criteria for migraine with aura are defined by at least two typical
attacks of migraine with some form of aura symptoms (e.g. visual, sensory, motor, etc.). A recent
meta-​analysis found an increased risk of ischaemic stroke and myocardial infarction in patients with
migraine with aura.
Meir RW, Dhadwal S. Primary headaches. Dent Clin North Am 208;62:6–​28.
Vgontzas A, Burch R. Episodic migraine with and without aura: key differences and implications for
pathophysiology, management and assessing risks. Curr Pain Headache Rep 208;22:78.
chapter CLINICAL OPHTHALMOLOGY 3

3 QUESTIONS

. Which of the following statements about childhood sight impairment in


the United Kingdom is true?
. The prevalence of childhood amblyopia with an acuity worse than LogMAR 0.3 is 5%
A
B. The mortality rate among children in the year following diagnosis of severe visual
impairment is %
C. Annual age group specific incidence childhood visual impairment was reported to be
highest in the first year of  life
D. Congenital cataract remains an important cause of severe sight impairment

2. Which of the following statement regarding Sturge–​Weber syndrome


(SWS) is correct?
A. Ocular changes are seen in over 75% of patients on the ipsilateral side to the port
wine stain
B. The incidence of SWS without port wine stain is 0%
C. 90% of the glaucoma cases seen in SWS patients are of congenital or early onset
D. Choroidal haemangioma is seen in up to 25%

3. Which of the following eyes with retinopathy of prematurity can be


followed conservatively?
. Zone , Stage 3 with Plus disease
A
B. Zone , Stage 3 without Plus disease
C. Zone 2, Stage 3 without Plus disease
D. Zone 2, Stage 2, with Plus disease

4. The minimum required diagnostic criteria to diagnose congenital


(infantile) esotropia is:
. The range of the angle of strabismus is 40–​50 prism dioptres
A
B. Some children neurological status is abnormal
C. The refractive error correction eliminates strabismus in some children
D. Asymmetric optokinetic nystagmus characterized by robust temporal to nasal response
and erratic nasal to temporal response
78 Clinical Ophthalmology 3 | QUESTIONS

5. These are the features of Sticklers syndrome, EXCEPT:


A. Stickler syndrome is usually autosomal dominant and caused by type 2 collagen
abnormality
B. Stickler syndrome type III is commonly associated with ocular problems
C. Up to 0% may develop glaucoma
D. Stickler syndrome is the leading cause of retinal detachment in children, and retinal tears or
detachments occur in up to 50% of sufferers

6. Congenital cataract features include:


A. Persistent fetal vasculature can be associated with a posterior plaque outside or involving
the posterior capsule of a clear lens
B. Posterior polar cataract is genetically determined with autosomal recessive the common
mode of inheritance
C. Posterior subcapsular opacities involve the cortex and posterior capsule
D. Spontaneous rupture of the posterior lenticonus is common and results in total hydrated
cataract

7. Which of the following type of strabismus is usually associated with


stable angle of deviation?
. Convergence excess esotropia
A
B. Microtropia
C. Nystagmus blockage syndrome
D. Uncorrected anisometropia

8. The standard management option for type II Duane syndrome includes:


. Surgery of the extraocular muscle can eliminate innervation abnormality in some cases
A
B. Abnormal head posture can be corrected by surgery in over 80% of the cases
C. Surgery is commonly performed to improve fusion
D. Prisms commonly used to treat anomalous head posture

9. The characteristics features of juvenile X-​linked retinoschisis include:


. DNA sequencing detects RS mutation in 50% of  cases
A
B. Electronegative electroretinogram (ERG) is seen in both focal and macular ERG and
full-​field  ERG
C. In X-​linked juvenile retinoschisis, optical coherence tomography (OCT) reveals cystic
spaces primarily in the outer nuclear and inner plexiform layers of the retina
D. Fluorescein angiography may reveal leakage at the macula cystic area
Clinical Ophthalmology 3 | QUESTIONS 79

0. Which of the following statements concerning primary congenital


glaucoma is true?
. Primary congenital glaucoma occurs sporadically and is familial in less than 5% of cases
A
B. Epiphora is present on first presentation in under 5% of children with primary congenital
glaucoma
C. Neuroretinal rim thinning preferentially occurs superiorly and inferiorly resulting in vertical
cupping of the disc
D. Ketamine has little effect on the intraocular pressure measurement during examination
under anaesthesia

. A seven-​year girl presents to the eye emergency department with


sudden onset of convergent squint. Her management options include:
. Base-​in prisms on the contralateral side
A
B. Bimedial recession as an urgent procedure
C. Botox on the ipsilateral lateral rectus muscle
D. Cycloplegic refraction and prescribe glasses

2. Which of the following features of congenital monocular elevation


deficiency (MED) is correct?
. The affected eye is commonly the fixating eye
A
B. Hypotropia of the affected eye on the affected side present as orthophoria in primary gaze
C. It is usually due to inferior oblique weakness
D. Chin-​down as compensatory head position

3. Which of the following features of strabismus is usually observed in


thyroid eye disease?
A. Inferior rectus is the most commonly affected extraocular muscle with hypertropia as the
presenting feature
B. Pain with eye movement is seen in at least 30% of affected patient
C. During superior rectus recession, connection points between the superior rectus and
upper eyelid elevators improve upper eyelid retraction
D. The order of surgery is orbital decompression followed by corrective eyelid procedures
then strabismus surgery

4. A nine-​year-​old child presents with intermittent right divergent squint


and she measures 45 prism dioptres base-​in for distance and 20 dioptres
base-​in for near. Her ocular movements are full. Factors taken into
consideration during management plan include:
A. Tenacious proximal fusion in addition to AC/​A ratio play important roles in distance/​near
discrepancy
B. Both distance and near stereopsis is normal during the manifest phase
C. Magnitude of deviation is the single factor that gives information about the ‘quality’ of
control
D. Evidence of fusion in the presence of a manifest deviation is indicative of normal
correspondence
80 Clinical Ophthalmology 3 | QUESTIONS

5. An 8-​month-​old child has been referred with sudden onset of


strabismus and abnormal red reflex. On examination a solid unilateral
retinal lesion is identified and a provisional diagnosis of retinoblastoma
is made. Which of the following statements regarding management of
retinoblastoma is correct?
. The primary objective in the management is to preserve vision
A
B. Unilateral tumour stage III, IV, and V are generally managed with enucleation
C. Bilateral tumours are treated with chemoreduction for at least one of their two
involved eyes
D. Plaque radiotherapy is generally reserved for small tumours less than 3 mm in size

6. Which of the following statements about rhabdomyosarcoma is correct?


. The most common extracranial solid childhood tumour
A
B. Embryonal and alveolar subtypes have distinct genetic alterations that may play in the
pathogenesis of the tumours
C. Orbital tumours are more likely to have alveolar histologic subtype
D. Orbital tumours commonly present with ophthalmoplegia

7. Kearns–​Sayre syndrome features include:


. Onset before 8 years of  age
A
B. Cerebellar syndrome
C. Cerebrospinal fluid protein level greater than 0 mg/​dL
D. Cardiomyopathy

8. A 26-​year-​old female presents to an eye department with vertical


diplopia and defective elevation of the eye. The feature that is
suggestive of acquired Brown syndrome include:
. Diplopia when the patient looks up and to the ipsilateral side
A
B. Lack of significant hypotropia observed in the primary position
C. Compensatory head posture with chin down and ipsilateral face turn
D. Audible or palpable click on ocular rotations up and contralateral side

9. Which of the following statements about neurofibromatosis type  is


correct?
. More than 50% of patients with NF have learning difficulties
A
B. Lab tests are useful in the diagnosis of  NF
C. Lisch nodules is the most characteristic feature in children over six years of age
D. Choroidal hamartomas are well-​defined, elevated lesions found in the midperiphery of
the retina
Clinical Ophthalmology 3 | QUESTIONS 81

20. Which of the following features describes the acute phase in Leber’s
hereditary optic neuropathy?
. Bilateral simultaneous acute loss of  vision
A
B. Visual loss is severely reduced to 6/​60 or less in two weeks
C. Central scotoma is the characteristic field defect
D. Pupillary reflexes are preserved

2. Which of the following statements regarding the management of


congenital fibrosis of extraocular muscles (CFEOM) is true?
. Successful surgery at a young age may avoid the loss of vision in one or both eyes
A
B. The order of management is to correct the lid position followed by strabismus surgery
C. One of the goals of the surgery is to eliminate abnormal eye movements
D. Forced duction test has very limited value in the management of CFEOM

22. A two-​and-​a-​half-​year-​old child presents to the eye department with


a squint. On examination her near angle of deviation was 45 prism
dioptres (PD) and her distant angle of deviation was 30 PD. Her
refractive error was +2.25 D in each eye after cycloplegic refraction. Her
ocular movements were full. What is the most likely working diagnosis?
. Early onset esotropia
A
B. Non-​refractive accommodative esotropia
C. Partial accommodative esotropia
D. Refractive accommodative esotropia

23. Which of the following features is most commonly seen in child with
X-​linked ocular albinism?
. Compound myopic astigmatism
A
B. Positive-​angle  Kappa
C. Normal binocular function
D. Progressive reduction in visual acuity

24. A newborn is referred with tense, non-​pulsatile swelling located below


the medial canthus. The immediate management include:
A. Liaise with paediatrician and commence systemic antibiotics as prophylaxis as the risk of
secondary infection is high
B. Observe for 2 weeks, during which time most spontaneously improve
C. Organize an urgent MRI scan to make an accurate diagnosis
D. Surgical treatment is necessary to remove the swelling in most cases
82 Clinical Ophthalmology 3 | QUESTIONS

25. In a 6-​year-​old child with an orbital floor fracture due to blunt trauma of
the inferior orbital rim you would expect:
. Red inflamed eye
A
B. Restricted  upgaze
C. No vomiting or fainting
D. No long-​term sequelae with conservative management

26. In acute severe traumatic optic neuropathy (TON) with associated with
major head trauma, which of the following statements is most likely to
be true?
. Computed tomography (CT) of the optic canal is commonly abnormal
A
B. Fundoscopy is commonly abnormal
C. High-​dose steroids are indicated
D. Intervention is unlikely to result in improvement

27. Which of the following signs would you expect to see in a patient
presenting with a suspected direct carotico-​cavernous fistula (CCF) after
a deceleration injury but not in a spontaneous indirect CCF?
. Acute painful proptosis
A
B. Cranial  bruit
C. Dilated episcleral vessels
D. Raised intraocular pressure (IOP)

28. Which individual variable at presentation is the strongest predictor of


visual outcome using the ocular trauma score (OTS)?
A. Endophthalmitis
B. Globe rupture
C. Presence of relative afferent pupillary defect
D. Visual  acuity

29. Which of the following features is pathognomonic for abusive head


trauma (or shaken baby syndrome)?
A. Haemorrhagic retinoschisis
B. Trilaminar retinal haemorrhages
C. Perimacular  folds
D. No pathognomonic sign has been identified
Clinical Ophthalmology 3 | QUESTIONS 83

30. A patient is shot in the face with a shot gun. The eye is hypotonous
with a total hyphaema so there is no fundal view. On CT scan the globe
appears disrupted with metallic foreign bodies in the retro-​orbital space
but no intraocular foreign body. Which of the following best classifies
her ocular injury according to the Birmingham Eye Trauma Terminology
system (BETT)?
A. Lamellar laceration
B. Penetrating laceration
C. Perforating laceration
D. Ruptured  globe

3. A 4-​year-​old child presents as an emergency with a 2-​day history of


unilateral periocular swelling, redness, and proptosis. Which of the
following is NOT an essential emergency investigation?
. Full blood count
A
B. Temperature
C. Plain film X-​ray face
D. Weight

32. Which of the following is most commonly found in ptosis secondary to


aponeurotic dehiscence?
. Low skin crease
A
B. Normal levator function
C. Reduced Bell’s phenomenon
D. Reduced marginal reflex distance (MRD)-​2

33. Which of the following is the commonest cause of trichiasis worldwide?


A. Entropion
B. Ocular cicatricial pemphigoid (OCP)
C. Steven–​Johnson syndrome  (SJS)
D. Trachoma
84 Clinical Ophthalmology 3 | QUESTIONS

34. A 53-​year-​old lady, with a background of asthma, presented with a 5-​


day history of periorbital inflammation, initially thought to be cellulitis
but non-​responsive to antibiotics. She rapidly developed proptosis
and restricted eye movements. CT orbits showed inflammatory
stranding throughout periorbital and intraconal fat, and oedema of the
extraocular muscles. Orbital biopsy of the rectus muscles and orbital
fat showed eosinophilic rich infiltration extending through the walls of
medium-​sized blood vessels with extravascular granulomas. Which of
the following is the most likely diagnosis?
A. Churg–​Strauss syndrome
B. Granulomatosis with polyangiitis (GPA)
C. Idiopathic orbital inflammatory disease
D. Sarcoidosis

35. Which of the following is associated with type 2 blepharophimosis


syndrome?
A. Epicanthus tarsalis
B. Gene mutation in FOXL2
C. Primary ovarian failure
D. Increased interpupillary distance

36. Which of the following is the recommended management of dysthyroid


optic neuropathy (DON)?
A. Intravenous steroid
B. Oral steroid
C. Radiotherapy
D. Selenium

37. Which of the following investigations must be performed prior to


surgery for upper lid ptosis in an 8-​year old child with neurofibromatosis
type  (NF)?
A. Clotting  assay
B. CT  orbits
C. MRI  orbits
D. Visual field test (VFT)

38. In a patient treated for myasthenia with long-​term oral steroids, which
of the following would be the best regime for steroid cover for a patient
undergoing ptosis correction?
. 00 mg intramuscular hydrocortisone just before the local anaesthetic
A
B. 00 mg intravenous hydrocortisone just before the local anaesthetic
C. Double oral dose for 24 hours postoperative
D. 00 mg intramuscular hydrocortisone just before the local anaesthetic and double oral
dose for 24 hours postoperative
Clinical Ophthalmology 3 | QUESTIONS 85

39. A 48-​year-​old male presents with a 6-​day history of unilateral 2 mm


proptosis and painful eye movements with mild upper lid swelling and
ptosis. For which of the following radiological appearances would it be
appropriate to undertake a therapeutic trial of steroids without orbital
biopsy?
. Single muscle enlargement on MRI
A
B. Adjacent sinus mucosal thickening
C. Diffuse contrast enhanced orbital signal on MRI with extension though the superior orbital
fissure
D. Enlarged lacrimal gland on CT

40. During a surgical decompression for acute compressive optic


neuropathy which of the paranasal sinuses will NOT be entered?
A. Ethmoid
B. Frontal
C. Maxillary
D. Sphenoid

4. A 52-​year-​old Caucasian male patient, with a background of treated


colonic cancer, is referred to the oculoplastic clinic for a growing eyelid
lesion. He has a strong family history for colonic cancer. Ophthalmic
examination reveals lid changes suggestive of sebaceous cell carcinoma.
What is the most likely diagnosis?
A. Bazex syndrome
B. Gardner syndrome
C. Gorlin–​Golz syndrome
D. Muir–​Torre syndrome

42. Which of the following is the best indicator of activity in thyroid eye
disease?
A. Diplopia
B. Upper lid oedema
C. Pain
D. Reduced colour vision

43. A 40-​year-​old female presents with a 3-​month history of painful swelling


in the superotemporal quadrant of the left orbit. She undergoes an
incision biopsy which demonstrates glandular tubules with lumina,
excess basement membrane and mucin, and islands of anaplastic cells
without squamous differentiation. Which of the following is the most
likely diagnosis?
. Adenoid cystic carcinoma
A
B. Dacryoadenitis
C. Pleomorphic adenoma
D. Sarcoidosis
86 Clinical Ophthalmology 3 | QUESTIONS

44. A 62-​year-​old patient is listed for post-​Mohs reconstruction of the


lower lid following Mohs excision of a basal cell carcinoma (BCC). The
original tumour size measured 8 mm diameter but was clinically well
circumscribed. Which best describes the likely choice of reconstruction
for an anticipated full thickness lid defect?
A. Direct closure
B. Full thickness skin graft
C. Hughes tarsoconjunctival flap
D. Mustarde cheek rotation flap

45. What is the most common cause of unilateral axial proptosis in a


45-​year-​old  man?
A. Cavernous haemangioma
B. Lymphoproliferative disease
C. Orbital inflammatory disease
D. Thyroid eye disease

46. An MRI is done for investigation of headaches in an otherwise fit and


healthy 52-​year-​old female. A well circumscribed intraconal 20 mm
round lesion is noted in the orbits. It is homogenous, high signal on T2.
Which of the following is the most likely diagnosis?
A. Cavernous haemangioma
B. Metastasis from a breast primary
C. Pleomorphic adenoma
D. Solitary fibrous tumour

47. Which of the following is NOT currently recommended for 7 mm


superficial BCC located in the upper lid?
. Mohs micrographical surgical excision
A
B. Radiotherapy
C. Surgical excision with 3–​4 mm margin
D. Topical imiquimod

48. In a patient with an isolated lacrimal gland mass, what test would be
the most sensitive to differentiate between lymphoma and orbital
inflammatory disease (OID)?
. Contrast enhanced MRI
A
B. Orbital  biopsy
C. Serum  LDH
D. Steroid response
Clinical Ophthalmology 3 | QUESTIONS 87

49. Which of the following statements about orbital rhabdomyosarcoma is


correct?
. Most commonly occurs between first and second decade of  life
A
B. Most commonly located in the superotemporal quadrant
C. Most commonly of embryonal cell type
D. Most commonly arises from extraocular muscles

50. Which of the following is recommended for 6-​monthly follow-​up of


patients who have had enucleation for ocular melanoma?
A. CT abdomen
B. PET  CT
C. Serological liver function tests
D. Ultrasound of the liver

5. A 30-​year-​old male patient has a history of treated unilateral


retinoblastoma. What is the chance of his children developing
retinoblastoma?
A. %
B. 7%
C. 5%
D. 40%

52. Which of the following features of choroidal naevus is associated with


increased risk of transformation to choroidal melanoma?
. Absence of ultrasonographic hollowness
A
B. Absence of  halo
C. Tumour thickness of   mm
D. Presence of  drusen

53. A 26-​year-​old female presents with an acute unilateral visual loss. The
visual acuity in the affected eye is 6/​30 with a relative afferent pupillary
defect (RAPD), numerous grey/​white retinal lesions at the posterior
pole (<00 μm in size), and optic disc swelling. There are no anterior
chamber or vitreous cells. The fellow eye is normal. The patient is
systemically well but describes a flu-​type illness prior to the onset of
visual symptoms. What is the most likely diagnosis?
. Acute posterior multifocal placoid pigment epitheliopathy (APMPPE)
A
B. Birdshot chorioretinopathy
C. Multiple evanescent white dot syndrome (MEWDS)
D. Punctate inner choroidopathy (PIC)
88 Clinical Ophthalmology 3 | QUESTIONS

54. A 28-​year old male presents with a history of recurrent unilateral


anterior uveitis. He has evidence of posterior synechiae and an irregular,
large pupil. Iris retroillumination demonstrates marked sectoral iris
atrophy. The vitreous is clear. The other eye is normal. What is the
most likely diagnosis?
. Fuchs’ heterochromic uveitis
A
B. Herpetic uveitis
C. Human leukocyte antigen-​B27 (HLA-​B27)-​related uveitis
D. Ocular sarcoidosis

55. The Standardization of Uveitis Nomenclature (SUN) grading system


includes which of the following criteria?
. Granulomatous keratitic precipitates
A
B. Macular  oedema
C. Vitreous  cells
D. Vitreous  haze

56. A 28-​year-​old Caucasian male presents with his first episode of uveitis.
He is found to have unilateral panuveitis with a mobile hypopyon and
a focus of retinitis associated with vasculitis. His past medical history
includes deep vein thrombosis and pustular skin lesions. What is the
most likely diagnosis?
A. Behçet’s disease
B. Herpetic uveitis
C. HLA-​B27 related uveitis
D. Syphilis

57. Which of the following HLA types is associated with birdshot


chorioretinopathy?
A. HLA-​A29
B. HLA-​B27
C. HLA-​B5
D. HLA-​DR
Clinical Ophthalmology 3 | QUESTIONS 89

58. A 68-​year-​old female patient presents with severe bilateral necrotizing


anterior scleritis and has a history of chronic sinusitis. There is no
evidence of intraocular inflammation. She describes recent onset weight
loss, epistaxis, arthralgia, haemoptysis, and chronic cough. A CT chest
shows evidence of a cavitating pulmonary lesion. Routine inflammatory
markers are elevated and renal impairment is detected. Quantiferon
test is negative. What is the most likely diagnosis?
. Granulomatosis with polyangiitis
A
B. Lupus-​associated scleritis
C. Rheumatoid arthritis-​associated scleritis
D. Tuberculosis-​associated scleritis

59. The following are appropriate treatment regimes in toxoplasma uveitis,


EXCEPT:
. Observation alone for a peripheral focus of toxoplasma retinitis
A
B. Oral clindamycin plus oral corticosteroid
C. Oral spiramycin during pregnancy
D. Oral steroid alone in a patient with allergy to co-​trimoxazole

60. A 36-​year-​old male presents with a 5-​day history of ocular pain, redness,
and photophobia affecting his left eye. He reports generalized malaise
of several months’ duration, abdominal discomfort, oral and perianal
ulceration, and has had to give up running as this causes pain under the
ball of his right foot. Examination reveals an acute left anterior uveitis
complicated by hypopyon and posterior synechiae. His erythrocyte
sedimentation rate (ESR) and C-​reactive protein (CRP) are both
elevated. Which of the following is the most likely diagnosis?
A. Behçet’s disease
B. Crohn’s disease
C. Infectious endogenous endophthalmitis
D. Ulcerative colitis

6. Which of the following features is characteristic of Fuchs’ heterochromic


cyclitis?
. Absence of vitritis
A
B. Inferiorly located stellate keratic precipitates
C. Presence of posterior synechiae
D. Usually asymptomatic
90 Clinical Ophthalmology 3 | QUESTIONS

62. A 58-​year-​old gentleman underwent retinal detachment repair to the


right eye 4 months previously. Unfortunately, this was complicated by
severe postoperative inflammation, requiring medication with high-​
dose systemic corticosteroids. Following the tapering of the steroids, he
developed sympathetic ophthalmia of the left eye. He is now established
on immunosuppression and attends for regular follow-​up visit. He has
developed gastrointestinal (GI) upset on mycophenolate mofetil and
feels nauseous. The left anterior segment is quiet and there is no vitritis.
The OCT of his left eye is shown in Figure 3.A.
. He will need to increase his oral immunosuppressive medication
A
B. He will need to be treated with pulsed intravenous methylprednisolone or equivalent
C. His systemic symptoms will subside as he perseveres with his medication; his ocular
inflammation is well controlled
D. The inflammation affecting the left eye will settle once the right eye is enucleated

63. A 65-​year-​old type  diabetic female inpatient is referred to the eye


clinic for 2 days’ history of right eye pain with worsening blurred vision.
She is currently being treated for a liver abscess. She denied past ocular
history of trauma or surgery. Examination confirms vision 6/​60 in the
right eye with hypopyon anterior uveitis. There are no granulomatous
features. Mydriatic examination reveals intense vitritis through which
fundus haemorrhages in the macular area are just visible in the macular
area. The asymptomatic left eye is normal. Which of the following
statement is correct about this condition?
A. Streptococcus pneumonia is likely to be the main culprit
B. Bilateral involvement is rare
C. The mortality can be as high as 50% in the presence of  sepsis
D. Approximately 20% cases end up with vision of no light perception (NPL) or evisceration
Clinical Ophthalmology 3 | QUESTIONS 91

(a)

(b)

Figure 3.  
chapter CLINICAL OPHTHALMOLOGY 3

3 ANSWERS

. Answer: C
The prevalence of childhood amblyopia with acuity worse than LogMAR 0.3 is %. The
mortality rate among children in the year following diagnosis of severe visual impairment is
0%, when compared with the total child population. Annual age group specific incidence
of childhood visual impairment was reported to be highest in the first year of life at 4.0 per
0 000. Congenital cataract remains an important cause of severe visual impairment in the
developing world.
According to the most recent UK study (2007–​200), cerebral visual impairment (2–​3%)
and optic nerve disorders (6%) have emerged as the most common causes for severe sight
impairment, whereas congenital globe anomalies (8%) and retinal dystrophy (7%) are the two
most common causes for sight impairment.
Mitry D, et al. Causes of certifications for severe sight impairment (blind) and sight impairment (partial
sight) in children in England and Wales. Br J Ophthalmol 203;97:43–​6.
Rahi JS, Cable N, British Childhood Visual Impairment Study Group. Severe visual impairment and
blindness in children in the UK. Lancet 2003;362:359–​65.

2. Answer: B
Sturge–​Weber syndrome (SWS) is a type of phakomatoses that is characterized by hamartomas
involving the brain, eye, and skin. It affects approximately :50 000 babies, with no gender
predilection. It is related to sporadic mutation of the GNAQ gene, which leads to stimulation of
cell proliferation and inhibition of apoptosis.
Pathological ocular changes are seen in up to 50% of patients ipsilateral to the Prader–​Willi
syndrome (PWS) in SWS. The risk of glaucoma in SWS ranged between 30% and 70% and is
related to anterior chamber malformation, high episcleral venous pressure, and changes in ocular
haemodynamics. Over 75% have bilateral PWS, however only upper eyelid PWS is seen in 20%.
Of those with glaucoma, 60% cases are of early onset and 40% manifest glaucoma in later life.
Choroidal haemangiomas may be present up to 70% of patients with SWS and typically exist
in diffuse form, which gives rise to a bright red or red-​orange colour appearance of the fundus
(‘tomato ketchup appearance’). The incidence of SWS and central nervous system manifestations
without a PWS or ocular abnormality is between 5% and 5%. This group is referred as type III or
encephalofacial angiomatosis
Mantelli F, et al. Ocular manifestations of Sturge–​Weber syndrome: pathogenesis, diagnosis, and
management. Clin Ophthalmol 206;0:87–​8.
Sullivan TJ, et al. The ocular manifestations of Sturge–​Weber syndrome. J Pediatr Ophthalmol
Strabismus 992;29:349–​56.
Clinical Ophthalmology 3 | ANSWERS 93

3. Answer: C
The current recommended treatment technique by RCOphth is transpupillary diode laser to
give near-​confluent laser burn to the entire avascular retina when prethreshold criteria is met.
A summary of treatment criteria is given in Table 3..
Table 3.  Treatment criteria for retinopathy of prematurity
Zone Plus disease Stage Treatment
I + Any stage Treat
I − ≥3 Treat
II + ≥3 Treat
II + 2 Consider

Data from Cryotherapy for Retinopathy of Prematurity Cooperative Group. Multicenter Trial of Cryotherapy for Retinopathy
of Prematurity: ophthalmological outcomes at 0 years. Arch Ophthalmol 200;9:0–8; and The Early Treatment for
Retinopathy of Prematurity Cooperative Group Revised indications for treatment of retinopathy of prematurity: results of
the early treatment for retinopathy of prematurity randomized trial. Arch Ophthalmol 2003;2:684–6.

To answer this question, the candidates should be familiar with three main studies concerning the
treatment of retinopathy of prematurity, namely CRYO-​ROP, ETROP, and BEAT-​ROP.
Cryotherapy for Retinopathy of Prematurity (CRYO-​ROP) study
• A randomized controlled trial evaluating the efficacy and safety of cryotherapy versus
observation for threshold ROP
• Defines threshold ROP (all criteria required):
♦ Zone  I/​II
♦ Plus disease
♦ Stage 3
♦ 5 continuous clock hour or 8 non-​continuous, cumulative clock hours
Conclusion: Cryotherapy in ‘threshold ROP’ reduces the risk of adverse outcome (defined

as retinal detachment, macular fold, or retrolental mass) by 50% compared to no treatment
Early Treatment of Retinopathy of Prematurity (ETROP) study
• A randomized controlled trial evaluating the efficacy and safety of early laser therapy versus
observation for prethreshold ROP
• Defines prethreshold ROP into type  and type 2 (not important)
• Type  prethreshold ROP is defined as:
♦ Zone , any stage of ROP with Plus
♦ Zone , Stage 3 without Plus
♦ Zone 2, Stage 2, or Stage 3 with plus disease
• Results supports the use of early laser for type  ROP and ‘watch and wait’ strategy for type
2 ROP, which is similar to the management of mild ROP.
• Unfavourable visual outcome at 9 months: 4% vs. 20% (reduced by 30%)
• Unfavourable structural outcome at 9 months: 9% vs. 6% (reduced by 40%)
Conclusion: Early laser therapy in high-​risk prethreshold ROP reduced unfavourable visual

and structural outcomes.
Cryotherapy for Retinopathy of Prematurity Cooperative Group. Multicenter Trial of Cryotherapy
for Retinopathy of Prematurity: ophthalmological outcomes at 0 years. Arch Ophthalmol
200;9:0–​8.
94 Clinical Ophthalmology 3 | ANSWERS

The Early Treatment for Retinopathy of Prematurity Cooperative Group Revised indications for
treatment of retinopathy of prematurity: results of the early treatment for retinopathy of prematurity
randomized trial. Arch Ophthalmol 2003;2:684–​6.

4. Answer: D
The minimum required findings for diagnosis of congenital (infantile) esotropia according to the
Pediatric Eye Disease Investigator Group, are:
. Esotropia—​usually 40–​50 dioptres, but with a range of 0–​90 PD
2. Normal neurologic status (except for strabismus)
3. Refractive error expected for age (usually low to moderate hyperopia), correction of which
does not eliminate esotropia
4. Asymmetric optokinetic nystagmus characterized by robust temporal to nasal response and
erratic nasal to temporal response
Other common clinical findings often present but not essential to the diagnosis are:
• Manifest nystagmus
• Oblique muscle dysfunction
• Dissociated strabismus either vertical deviation or a horizontal deviation primarily an
exodeviation of one eye (DHD)
• Variable angle
• Latent nystagmus/​manifest latent nystagmus
• Torticollis
American Academy of Ophthalmology. Strabismus: Infantile Esotropia. Available at: https://​www.aao.org/​
disease-​review/​strabismus-​infantile-​esotropia

5. Answer: B
Stickler syndrome type I (STL) is responsible for approximately 70% of reported cases and
presents with a wide variety of symptoms affecting the eye, ear, facial appearance, palate, and
musculoskeletal system and occurs due to mutations over the entire COL2A gene on chromosome
2q3..
Type I has the highest risk of retinal detachment. Type II also includes eye abnormalities, but type
III does not (and is often called non-​ocular Stickler syndrome). Types IV, V, and VI are very rare and
have each been diagnosed in only a few individuals.
Stickler syndrome type III (STL3) has been described as the non-​ocular form of Stickler syndrome,
affecting the joints and hearing without involving the eyes. Stickler syndrome type III is caused by
mutations of the COLA2 gene on chromosome 6p2.3.
Some patients will exhibit congenital abnormalities of the anterior chamber drainage angle,
which may predispose them to a higher risk of glaucoma. Experience suggests, however, that
this is a relatively uncommon finding and that in most patients with glaucoma, this is a chronically
progressive angle closure phenomenon resulting from retinal detachment and proliferative
vitreoretinopathy causing secondary angle closure.
Alshahrani ST, et al. Rhegmatogenous retinal detachments associated to Stickler syndrome in a tertiary
eye care center in Saudi Arabia. Clin Ophthalmol 206;0:–​6.
Snead MP, et al. Stickler syndrome, ocular-​only variants and a key diagnostic role for the
ophthalmologist. Eye (Lond) 20;25:389–​400.
Clinical Ophthalmology 3 | ANSWERS 95

6. Answer: A
The opacity of posterior polar cataract is in the capsule itself. Posterior polar cataracts are
genetically determined with autosomal dominant inheritance and some have been associated with
mutations in PITX3. The lens opacities in patients with persistent fetal vasculature are generally
capsular and can be associated with shrinkage, thickening, and vascularization of the capsule. There
may be a posterior plaque outside or involving the lens capsule with a clear lens that nonetheless
must be treated as a cataract. Posterior subcapsular cataract can be congenital but are more
commonly acquired as a result of injury or steroid use. The opacities are cortical and do not involve
the capsule proper.
Posterior lenticonus, the central and sometimes paracentral posterior capsule, is thin and bulges
posteriorly. This usually occurs at the location where the hyaloid system attaches to the eye. The
distortion can cause a localized area of extreme myopic refraction. There may or may not be
subcapsular cortical opacification. Interference with vision can be the result of optical distortion or
of capsular opacification. Most cases are unilateral, although bilateral and familial cases have been
reported. Surgery is associated with good visual outcomes in most cases. Spontaneous rupture of
the lens can rarely occur, leading to abrupt progression to total cataract.
Gillespie RL, et al. Personalized diagnosis and management of congenital cataract by next-​generation
sequencing. Ophthalmology 204;2:224–​37.
Xu LT, Traboulsi EI. Genetics of congenital cataracts. In: Wilson ME, Trivedi RH (eds). Pediatric
Cataract Surgery: Techniques, Complications and Management. Philadelphia, PA: Lippincott Williams &
Wilkins, 204.

7. Answer: B
Variable angle of strabismus include:
• Uncorrected refractive error
• Anisometropia
• Nystagmus compensation (blockage) syndrome
• Convergence excess esotropia
Microtropia is associated with Inconspicuous shift or no shift on cover test, stable alignment, the
fixation is central or parafoveal in one eye. Mild amblyopia is frequent with reduced or absent
stereopsis. Microtropia may exist in two forms, with and without identity. Studies have shown that
microtropia with identity is a reliable indicator of presence of amblyopia and possible need for
occlusion therapy.
Lysons D, Tapley J. Is microtropia a reliable indicator of the presence of amblyopia in anisometropic
patients? Strabismus 208;26:8–​2.

8. Answer: B
Standard management of Duane syndrome (DS) may involve surgery. The indications for surgery
include:
• Elimination or improvement of an unacceptable head turn
• Elimination or reduction of significant misalignment of the eyes
• Reduction of severe retraction
• Improvement of upshoots and downshoots
Surgery does not eliminate the fundamental abnormality of innervation and no surgical technique
has been completely successful in eliminating the abnormal eye movements. Simple horizontal
96 Clinical Ophthalmology 3 | ANSWERS

muscle recession procedures, vertical transposition of the rectus muscle, or combinations of the
two may be successful in improving or eliminating head turns and misalignment of the eyes. The
choice of procedure must be individualized. A prism can be placed on the patient’s glasses to
correct for the face turn (though this is not commonly used). The success rate in eliminating an
abnormal head position is 79–​00%. Surgery does not improve motility or stereopsis/​fusion. A risk
of diplopia may be present with or without surgery.
Merino P, et al. Horizontal rectus surgery in Duane syndrome. Eur J Ophthalmol 202;22:25–​30.
Pressman SH, Scott WE. Surgical treatment of Duane’s syndrome. Ophthalmology 986;93:29–​38.

9. Answer: B
In X-​linked recessive juvenile retinoschisis (XLRS), foveal changes are seen in all cases and
peripheral retinoschisis in one-​half of cases. Maculopathy is characterized by stellate spoke-​like
appearance with microcysts. Vitreous veils are a common feature of X-​linked juvenile retinoschisis.
The mutation in RS can be detected in 90–​95% of patients who have a clinical diagnosis. It not only
helps confirm the diagnosis but also provides useful genetic information of the patient and offspring.
In juvenile X-​linked retinoschisis (XLR), ERG findings show electronegative ERG responses (i.e.
normal a-​wave, reduced b-​wave). ERG dysfunction is found throughout the retina and is not limited
to schitic areas. Therefore, both focal and macular ERG and full-​field ERG yield similar results.
OCT reveals cystic spaces primarily in the inner nuclear and outer plexiform layers of the retina.
Although there are cystic changes at the macula, there is no sign of leakage at the cystic areas on
fundus fluorescein angiography, which is in contrast to the usual cystoid macular oedema.
Indocyanine green angiography (ICGA) performed on patients with XLRS shows a distinct
hyperfluorescence in the macular region that is associated with radial lines of hypofluorescence
centred on the foveola in the early phase. This feature disappears in the late phase of the ICGA.
Sikkink SK, et al. X-​linked retinoschisis: an update. J Med Genet 2007;44:225–​32.
Souied EH, et al. Indocyanine green angiography of juvenile X-​linked retinoschisis. Am J Ophthalmol
2005;40:558–​6.

0. Answer: B
Most primary congenital glaucoma (PCG) cases occur sporadically. They are familial in 0–​40%
of cases, usually with autosomal recessive inheritance and variable penetrance. The classic triad
of symptoms in PCG is epiphora, photophobia, and blepharospasm, but could be absent in rare
occasions. In fact, photophobia and blepharospasm are found in only 7.5% of patients at first
presentation, while epiphora is only present in 3.3%. Cloudy cornea and buphthalmos account for
the most common presenting sign, found in over 40% of patients.
In PCG, the glaucomatous cup enlarges circumferentially, as the scleral canal is uniformly stretched
in all directions. In adult onset glaucoma, rim thinning preferentially occurs at the inferior and
superior rim due to the abundance of nerve fibre layer in that area. Most general anaesthetics and
central nervous system depressants decrease the intraocular pressure (IOP), except for ketamine
which increases the muscle tone of the extraocular muscles, paradoxically increasing the IOP.
Chloral hydrate has the least effect on IOP, followed by ketamine.
Tamcelik N, et al. Demographic features of subjects with congenital glaucoma. Indian J Ophthalmol
204;62:565–​9.
Wadia S, et al. Ketamine and intraocular pressure in children. Ann Emerg Med 204;64:385–​8.
Yu Chan JY, et al. Review on the management of primary congenital glaucoma. J Curr Glaucoma Pract
205;9:92–​9.
Clinical Ophthalmology 3 | ANSWERS 97

. Answer: D
Acute onset of concomitant esotropia is an uncommon form of strabismus. In the clear majority of
cases it will have no obvious underlying neurological cause.
• Check for history of previous strabismus, occlusion therapy or monocular visual loss, or
myopia cause little worry
• No apparent cause for the acute concomitant esotropia—​the possibility of an underlying
neurological disease should at least be considered
• Presence of nystagmus or the inability to restore binocularity in any of these patients should
be considered sufficiently abnormal and warrants neurological investigation
Indications for and specific types of treatment need to be individualized for each patient.
• Optical correction: cycloplegic refraction and prescription of glasses
• Prisms to eliminate diplopia and to re-​establish binocular vision: base out prisms in the
ipsilateral eye
• Chemodenervation: botox of the ipsilateral medial rectus muscle to reduce medial rectus
contracture
• Extraocular muscle surgery in stable deviations that are too large to allow spontaneous
binocular fusion
Chen J, et al. Acute acquired concomitant esotropia: clinical features, classification, and etiology.
Medicine (Baltimore) 205;94:e2273.

2. Answer: B
Congenital MED or known as double elevator palsy can be caused by:
• Paralysis of the superior rectus (SR) muscle without any involvement of the inferior oblique
muscle.
• Primary inferior rectus (IR) restriction or secondary restriction due to long-​standing SR palsy
• Lesion in the supranuclear pathway of upgaze located in the pretectum
MED features include:
• Inability to elevate the eye above midline in abduction, adduction, or from primary position
of  gaze
• Hypotropia of the affected eye on the affected side present as orthophoria in primary gaze.
Rarely the affected eye may fixate
• Ptosis of the affected eye or sometimes pseudoptosis may be seen
• Chin-​up as compensatory head position
Indications for surgery are:
• Vertical deviation in primary gaze
• Deviation-​induced amblyopia
• Diplopia in primary gaze
• Restricted binocular fields
The goal of surgery is to:
• Improve the position of the affected eye in primary gaze
• Increase the field of binocular vision
98 Clinical Ophthalmology 3 | ANSWERS

If restriction to upgaze is demonstrated on the forced duction test (FDT), IR restriction is


present. An IR recession with conjunctival recession should be done in such patients. If the FDT
is non-​restrictive, the affected patient has either SR paresis or supranuclear MED, and the Knapp
procedure should be performed.
Bagheri A, et al. Double elevator palsy, subtypes and outcomes of surgery. J Ophthal Vis Res
2008;3:08–​3.
Kim JH, Hwang JM. Congenital monocular elevation deficiency. Ophthalmology 2009; 6:580–​4.

3. Answer: B
Thyroid eye disease (TED) affects extraocular muscles in a predictable manner:
• The IR and medial rectus are most commonly involved
• This presents as hypotropia and/​or esotropia
• Pain with eye movement, characterized as dull, deep orbital pain affects 30% of patients
• 40% of patients are affected by restrictive extraocular myopathy
Most TED patients with diplopia due to strabismus will not require surgical intervention, as most
can be effectively managed with prism spectacles. Indications for strabismus surgery include:
• Intractable diplopia in primary gaze or with reading
• Abnormal head posture
• Cosmetically unacceptable globe position
Following surgical approach is recommended in TED patients affected by proptosis, strabismus,
and lid retraction: orbital decompression l strabismus surgery l lid surgery. This is because
orbital decompression can alter/​cause strabismus. Sometimes extraocular muscle recession may
worsen proptosis. In addition, strabismus surgery can affect eyelid position; therefore, it should be
undertaken prior to any corrective eyelid procedures:
• Large IR muscle recession can result in lower eyelid retraction, which is largely due to
adherence between the IR muscle and the capsulopalpebral fascia of the lower eyelid.
• With SR recession, connection points between the SR and upper eyelid elevators may worsen
upper eyelid retraction.
Bartley GB, et al. Clinical features of Graves’ ophthalmopathy in an incidence cohort. Am J Ophthalmol
996;2:284–​90.

4. Answer: A
Intermittent exotropia (IXT) is classified into three types: basic exotropia (BE), distance exotropia
(DE), and convergence insufficiency (CI). In BE, the distance deviation is within 0 PD of the near
deviation. In DE, the distance deviation is greater than nearby 0 PD. DE can be further classified
intro true vs. simulated divergence excess. In true divergence excess, the near deviation remains
less than the distance deviation after a brief period of occlusion. In simulated divergence excess,
however, the near deviation approaches distance deviation after occlusion. In CI type IXT, the near
deviation is greater than distance by 0 PD.
Exam findings in intermittent exotropia typically reveal normal visual acuities at distance and
near, good stereopsis at near (during phoric phase), no diplopia, suppression, or anomalous
correspondence, or a combination of latter two. Proportion of time the deviation is manifest is as
important as the magnitude of deviation as it gives information about the ‘quality’ of control. True
fusion experienced during the phoric phase or anomalous fusion during the tropic phase. Evidence
of fusion in the presence of a manifest deviation is indicative of anomalous correspondence.
Clinical Ophthalmology 3 | ANSWERS 99

Clarke MP. Intermittent exotropia. J Pediatr Ophthalmol Strabismus 2007;44:53–​7.


Kushner BJ, Morton GV. Distance/​near differences in intermittent exotropia. Arch Ophthalmol
998;6:478–​86.

5. Answer: C
The most important objective in the management of a child with retinoblastoma is survival of
the patient, and the second most important goal is preservation of the globe. The focus on visual
acuity comes later, after safety of the patient and globe is established. Therapy is tailored to each
individual case and based on the overall situation, including threat of metastatic disease, risks for
second cancers, systemic status, laterality of the disease, size and location of the tumour, and
estimated visual prognosis.
In recent years, eyes with unilateral retinoblastoma are generally managed with enucleation if the
eye is classified as Reese–​Ellsworth group V; for those eyes in groups I to IV, chemoreduction
or focal measures are used. For bilateral retinoblastoma, chemoreduction is utilized in most
cases unless there is an extreme asymmetric involvement, with one eye having advanced disease
necessitating enucleation while the other eye has minimal disease, treatable with focal methods.
Most children with bilateral retinoblastoma are treated with chemoreduction for at least one of
their two involved eyes. Selective ophthalmic arterial injection therapy has been shown to be a
promising treatment for intraocular retinoblastoma, with eye preservation rate ranging from 00%
(group A) to 30% in group E according to International Classification of Intraocular Retinoblastoma.
Focal therapies include laser photocoagulation, thermotherapy, cryotherapy, and plaque
radiotherapy. Most of these therapies are employed for small tumours, especially those that have
been reduced by chemoreduction. Commonly, focal therapies are applied to an eye while the child
is receiving chemoreduction, and they are repeated to each tumour at each chemotherapy session.
Plaque radiotherapy is generally reserved for tumours that fail other focal therapies, even those
that reach a moderate size, up to 8 or 0 mm in thickness. The remainder of the focal therapies are
reserved for small tumours, generally those under 3 mm in greatest dimension.
Shields CL, Shields JA. Recent developments in the management of retinoblastoma. J Pediatr
Ophthalmol Strabismus 999;36:8–​8.
Shields CL, et al. Chemoreduction plus focal therapy for retinoblastoma: factors predictive of need for
treatment with external beam radiotherapy or enucleation. Am J Ophthalmol 2002;33:657–​64.
Suzuki S, et al. Selective ophthalmic arterial injection therapy for intraocular retinoblastoma: the long-​
term prognosis. Ophthalmology 20;8:208–​7.

6. Answer: B
Rhabdomyosarcoma (RMS) is the most common soft-​tissue sarcoma of childhood. After
neuroblastoma and Wilms’ tumour, it is the third most common extracranial childhood solid
tumour. Orbital tumours being characterized by embryonal histology in most cases. On the other
hand, extremity tumours are more commonly found in adolescents and are more likely to have an
alveolar histologic subtype.
It is the most common cause of primary malignant orbital tumour in childhood (around 4%). It
usually begins around 8 years of age with slight male predilection. It can be extraconal (35%),
intraconal (5%), or both (50%). It preferentially affects the superior nasal quadrant and the
main signs include non-​axial unilateral exophthalmos, often with inflammatory character, rapid
progressive ptosis, and may mimic orbital cellulitis.
The two histologic subtypes of RMS, embryonal and alveolar, have been found to have distinct
genetic alterations that may play a role in the pathogenesis of these tumours. Alveolar RMS has
100 Clinical Ophthalmology 3 | ANSWERS

been demonstrated to have a characteristic translocation between the long arm of chromosome
2 and the long arm of chromosome 3, referred to as t(2;3)(q35;q4). Embryonal RMS is known
to have loss of heterozygosity at the p5 locus with loss of maternal genetic information and
duplication of paternal genetic information. Orbital tumours produce proptosis, and, occasionally,
ophthalmoplegia. Embryonal RMS is the most common subtype (50%; can be remembered as
Everyone has it), whereas alveolar RMS has a worse outcome (which can be remembered as awful
outcome).
Malempati S, Hawkins DS. Rhabdomyosarcoma: review of the Children’s Oncology Group (COG)
Soft-​Tissue Sarcoma Committee experience and rationale for current COG studies. Pediatr Blood
Cancer 202;59:5–​0.
Pater LE, et al. Rhabdomyosarcoma Review 207. Children’s Oncology Group. Available at: https://​www.
qarc.org/​COG/​Rhabdomyosarcoma_​.pdf

7. Answer: B
Kearns-​Sayers syndrome (KSS) is a type of mitochondrial myopathy that demonstrates the
following: (a) chronic progressive external ophthalmoplegia, onset before age 20 years, and
(b) pigmentary retinopathy. KSS also has at least one of the following: (a) cardiac conduction
defects; (b) cerebrospinal fluid (CSF); (c) protein level greater than 00 mg/​dL; and (d) cerebellar
syndrome. Other abnormalities in KSS can include mental retardation, Babinski sign, hearing loss,
seizures, short stature, delayed puberty, and various endocrine disorders, such as diabetes mellitus,
hypoparathyroidism, and hearing loss.
Chinnery PF. Mitochondrial Disorders Overview. GeneReviews® [Internet] 204. Available at: http://​
www.ncbi.nlm.nih.gov/​books/​NBK224/​

8. Answer: D
Patients with acquired Brown syndrome in late childhood or adulthood experience diplopia when
tropic. Diplopia may occur when the patient looks up and to the contralateral side of the affected
eye. Patients with congenital Brown syndrome rarely complain of diplopia, because most patients
have developed suppression. Limited elevation in adduction, an invariable sign, is the hallmark of
Brown syndrome. Even in severe cases of congenital Brown syndrome, there is minimal hypotropia
in primary position and no hypotropia in downgaze. In contrast, much larger hypotropias have
been observed in cases of Brown syndrome associated with trauma or periorbital surgery. Patients
often present with compensatory head-​posturing, their chin up, and a contralateral face turn to
avoid the hypotropia that increases in upgaze and gaze to the contralateral side of the affected eye.
A feature that often is associated with acquired Brown syndrome is an audible or palpable superior
nasal click on ocular rotations up and nasal ward; sometimes, the pain is associated with this ocular
movement.
Parks MM, Brown M. Superior oblique tendon sheath syndrome of Brown. Am J Ophthalmol
975;79:82–​6.
Suh SY, et al. Size of the oblique extraocular muscles and superior oblique muscle contractility in
Brown syndrome. Invest Ophthalmol Vis Sci 205;56:64–​20.

9. Answer: A
Neurofibromatosis type  (NF), also known as von Recklinghausen NF or peripheral NF, is
characterized by multiple café au lait spots (patches of tan or light brown skin) and neurofibromas
(soft, fleshy growths) on or under the skin. Enlargement and deformation of bones and curvature
of the spine (scoliosis) may also occur. Occasionally, tumours may develop in the brain, on cranial
nerves, or on the spinal cord. About 50–​75% of people with NF also have learning disabilities.
Clinical Ophthalmology 3 | ANSWERS 101

Lisch nodules are the most common ocular clinical finding in adults older than 20 years with NF-​.
Hamartomas of the choroid are usually in the posterior pole and are flat, ill-​defined lesions. No
laboratory tests are pertinent in evaluating ophthalmologic manifestations of NF-​. However, tissue
biopsy of skin lesions is occasionally necessary to confirm the diagnosis of NF-​.
Savar A, Cestari DM. Neurofibromatosis type I: genetics and clinical manifestations. Semin Ophthalmol
2008;23:45–​5.

20. Answer: D
Leber’s hereditary optic neuropathy (LHON) carriers remain asymptomatic until they experience
blurring or clouding of vision in one eye. In the vast majority of cases, visual dysfunction is bilateral, the
fellow eye becoming affected either simultaneously (25%) or sequentially (75%), with a median intereye
delay of 8 weeks. Visual acuity usually reaches low in 4–​6 weeks after the first start of symptoms and is
severely reduced to 6/​60 or less. The characteristic field defect in LHON is a centrocaecal scotoma.
Other clinical features include the early impairment of colour perception but, more importantly,
pupillary reflexes are preserved and patients usually report no pain on eye movement.
Fundoscopy provides other diagnostic clues and in classical cases the following abnormalities can
be observed: vascular tortuosity of the central retinal vessels, a circumpapillary telangiectatic
microangiopathy, and swelling of the retinal nerve fibre layer. However, it must be stressed that in
∼20% of LHON cases, the optic disc looks entirely normal in the acute phase.
Yu-​Wai-​Man P, et al. Leber hereditary optic neuropathy. J Med Genet 2002;39:62–​9.

2. Answer: A
The standard management of CFEOM may involve surgery. The goal of surgery is the elimination
or improvement of an unacceptable head position, the reduction of ptosis, and the elimination or
reduction of significant misalignment of the eyes. Successful surgery at a young age may avoid loss
of vision in one or both eyes. However, surgery does not eliminate the fundamental abnormality,
and no surgical technique has been completely successful in eliminating the abnormal eye
movements. These patients require a stepwise surgical approach to correct strabismus and eyelid
position. The vertical and horizontal misalignments are addressed first followed by the ptosis repair,
as extraocular muscle surgery can alter eyelid position. FDT is useful in differentiating other causes
of non-​restrictive misalignment of the eyes.
Whitman M, et al. Congenital Fibrosis of the Extraocular Muscles. In: GeneReveiws at GeneTests: Medical
Genetics Information Resource; Seattle, WA: University of Washington, 993–​208. Available
at: https://​www.ncbi.nlm.nih.gov/​books/​NBK348/​

22. Answer: B
Accommodative esotropia is defined as a convergent deviation of the eyes associated with
activation of the accommodation reflex. It comprises more than 50% of all childhood esotropias
and can be classified into three forms: () refractive; (2) non-​refractive; and (3) partially
accommodative or decompensated. All three forms possess the following characteristics:
• Onset usually between 6 months and 7 years of age, averaging 2.5 years
• Intermittent at onset, then becoming constant over time
• Often initiated by trauma or illness
• Frequently associated with amblyopia
• May be associated with diplopia in older children, but later disappears as a suppression
scotoma develops
• Often has a hereditary basis
102 Clinical Ophthalmology 3 | ANSWERS

Early onset esotropia normally presents before the age of 6 months, with a constant, large angle
of strabismus (>30 PD), no or mild amblyopia, small to moderate hyperopia, latent nystagmus,
dissociated vertical deviation, limitation of abduction (although the patients in fact have normal
abduction, they appear to have limitation due to cross fixation), and absent or reduced binocular
vision, in the absence of nervous system disorders. Refractive accommodative esotropia usually
occurs in a child between 2 and 3 years of age. The average cycloplegic refractive error in refractive
accommodative esotropia is +4.75 D. The angle of deviation is typically the same for distance and
near, averaging between 20 and 40 PD. Despite full spectacle correction, if the distance esotropia is
still noted to be high, the patient has a partially accommodative esotropia. A subgroup of patients
with accommodative esotropia have significantly larger esotropia at near, that is, non-​refractive or
high accommodative convergence (AC:A) ratio accommodative esotropia. They usually present
between 2 and 3 years of age. The refractive error in this condition may be hyperopic, emmetropic,
or myopic. The average refractive error is +2.25 D.
Rutstein RP. Update on accommodative esotropia. Optometry 2008;79:422–​3.

23. Answer: B
In most individuals with X-​linked ocular albinism (XLOA), the best corrected visual acuity is
between 20/​40 (6/​2) and 20/​200 (6/​60). XLOA is a non-​progressive disorder and the visual
acuity typically slowly improves until mid-​to-​late teens and then remains stable throughout life.
Hypersensitivity to light, often called ‘photophobia’, is present in most affected individuals but
varies in intensity and significance from one individual to another. Substantial refractive errors
are common, most often as hypermetropia with oblique astigmatism. High myopia or compound
myopic astigmatism may occur in some affected individuals. Most affected individuals have reduced
or absent binocular functions as a consequence of misrouted optic pathway projections, and ocular
misalignment (strabismus). A positive angle Kappa is often found in individuals with albinism.
Brodsky MC, Fray KJ. Positive angle Kappa: a sign of albinism in patients with congenital nystagmus. Am
J Ophthalmol 2004;37:625–​9.
Lewis RA. Ocular albinism, X-​linked. In: Adam MP, et al. (eds). GeneReviews. Seattle, WA: University of
Washington, Seattle, 993–​208.

24. Answer: B
A dacryocystocele usually presents with tense, blue, non-​pulsatile swelling BELOW the medial
canthus that is evident at or shortly after birth. The clinical appearance is classic, but if in doubt
organize MRI scan to exclude meningocele and meningomyelocele, in which the lesions typically
present ABOVE the medial canthus. Treatment is observation for 2 weeks, during which most
spontaneously get better and surgery is only indicated if it does not settle in 2 weeks, acute
dacryocystitis sets in, or respiratory difficulties develop.
Sullivan TJ, et al. Management of congenital dacryocystocoele. Aust N Z J Ophthalmol 992;20:05–​8.
Wong RK, VanderVeen DK. Presentation and management of congenital dacryocystocele. Pediatrics
2008;22:e08–​2.

25. Answer: B
A ‘green-​stick’ floor fracture is also known as a ‘white eye blow-​out fracture’ where the force of
the injury has been transmitted via the bony orbital floor, causing the bone to fracture then snap
back into place often causing entrapment of the IR or associated soft tissues. This can result in a
restrictive vertical gaze palsy and secondary oculo-​cardiac reflex when attempting upgaze from the
entrapped muscle, resulting in vomiting and vasovagal syncope. Severe fibrosis and restricted eye
movement can result due to muscle ischaemia if not surgically released.
Clinical Ophthalmology 3 | ANSWERS 103

Lane K, et al. Evaluation and management of pediatric orbital fractures in a primary care setting. Orbit
2007;26:83–​9.

26. Answer: D
Most commonly TON is due to direct or vascular injury to the optic nerve within the optic canal.
Fracture through the optic canal is less common, and the fundus examination is normal acutely
unless there is an optic nerve avulsion or a very anterior nerve injury. High-​dose steroids have been
shown to worsen prognosis in severe head injury.
Timlin H, et al. RCOphth FOCUS article: Traumatic Orbital Emergencies. Available at: https://​www.
rcophth.ac.uk/​wp-​content/​uploads/​205/​02/​Focus-​Autumn-​205.pdf

27. Answer: B
The clinical findings of the indirect fistulas are almost always less dramatic than those of a direct
carotid cavernous fistula, although over time the low-​flow state of an indirect dural sinus fistula may
become a greater flow as new arterial connections develop. Other than by the tell-​tale symptom of
a cranial bruit, differentiating high flow from low flow is best determined by angiographic studies.
Chaudhry IA, et al. Carotid cavernous fistula: ophthalmological implications. Middle East Afr J
Ophthalmol 2009;6:57–​63.

28. Answer: D
The higher the raw score sum, the better probability of achieving higher visual acuity at 6 months.
Visual acuity at presentation represents the biggest single variable in calculating the OTS score (see
Table 3.2).
Computational method for deriving the OTS score:
Raw score sum = sum of raw points

Table 3.2  Calculation of the OTS score


Initial visual factor Raw points

A. Initial raw score (based on initial VA) NPL 60


PL or HM 70
/​200 to 9/​200 80
20/​200 to 20/​50 90
≥ 20/​40 00
B. Globe rupture −23
C. Endophthalmitis −7
D. Perforating injury −4
E. Retinal detachment −
F. Relative afferent pupillary defect −0

Reproduced from Scott, R. (206). The ocular trauma score. Community Eye Health, 28(9):44-5. CC BY-NC 4.0.
104 Clinical Ophthalmology 3 | ANSWERS

29. Answer: D
There are a number of characteristic ocular fundus findings reported in abusive head trauma
or shaken baby syndrome; however, none of them are pathognomonic. Characteristic features
include severe retinal haemorrhages (preretinal, intraretinal, and subretinal), perimacular folds, and
haemorrhagic retinal cysts and retinoschisis. Optic nerve sheath haemorrhages are more common
in abuse than in other conditions in autopsy studies.
The Royal College of Paediatrics and Child Health and The Royal College of Ophthalmologists. Abusive
Head Trauma and the Eye in Infancy, 203. Available at: https://​www.rcophth.ac.uk/​wp-​content/​uploads/​
204/​2/​203-​SCI-​292-​ABUSIVE-​HEAD-​TRAUMA-​AND-​THE-​EYE-​FINAL-​at-​June-​203.pdf

30. Answer: C
A lamellar laceration is a closed globe injury. A ruptured globe is caused by a blunt force where the
eye will rupture at the weakest point, often an old surgical wound site or just behind the muscle
insertions. A penetrating injury requires an entrance wound and a perforating injury requires both
entrance and exit wound. See Figure 3.2.

Injury

Closed globe Open globe

Contusion Lamellar laceration Laceration Rupture

Penetrating IOFB Perforating

Figure 3.2  

This figure was published in Ophthalmology Clinics of North America, 5, 2, Kuhn F, et al., Birmingham Eye Trauma Terminology (BETT):
Terminology and classification of mechanical eye injuries, pp. 39–43, Copyright Elsevier 2002.

3. Answer: C
Both full blood count and temperature are essential to establish a likelihood of infective orbital
cellulitis, the commonest cause of acute paediatric unilateral proptosis. A weight is required to
calculate the dosage of both antibiotics and analgesia which are required for emergency treatment.
Neoplastic causes such as orbital rhabdomyosarcoma should be considered in the differential,
but CT or MRI imaging would be required; plain film X-​ray is non-​diagnostic and unnecessary
exposure to radiation. The other important differential diagnosis to be considered in this situation
is orbital neuroblastoma, which usually occurs in patients less than 2 years old. It typically presents
Clinical Ophthalmology 3 | ANSWERS 105

as unilateral or bilateral periorbital ecchymosis (or known as ‘raccoon eyes’). Other ophthalmic
features include proptosis, periorbital swelling, Horner’s syndrome, opsoclonus, optic neuropathy,
and strabismus.
Sindhu K, et al. Aetiology of childhood proptosis. J Paediatr Child Health 998;34:374–​6.

32. Answer: B
Features of aponeurotic dehiscence include high skin crease, normal levator function, reduced
MRD, unaffected Bell’s phenomenon, and normal orbicularis function.
Low skin crease is found in congenital ptosis. Reduced Bell’s phenomenon and limited upgaze are
associated with chronic progressive external ophthalmoplegia. Reduced MRD2 means the lower lid
is higher than normal, which is not related to the upper lid. Important features of various types of
ptosis are summarized in Table 3.3, next. It is noteworthy to mention that all the elements in the
first column should be examined for during the clinical examination of ptosis.

Table 3.3  Important features of various types of ptosis


Aponeurotic Myogenic Myasthenic Congenital

MRD Low Low Low Low


Levator function Normal Reduced Variable Reduced
Skin crease High/​lost Normal Normal Low/​absent
Orbicularis function Normal Weak Weak Normal
Bell’s phenomenon Normal Poor Normal unless Normal unless
significant upgaze associated elevator
restriction palsy

Other features Deep superior May have significant Improvement in Lid lag in down gaze
sulcus ocular motility MRD after rest/​ice
restriction

Leatherbarrow B. Oculoplastic Surgery, 2nd edition. Chapter 7: Blepharoptosis. London, UK: Informa


Healthcare, 20.

33. Answer: D
Trichiasis is defined as misdirected lashes arising from a lid margin of normal position. Entropion is
pseudotrichiasis. OCP and SJS are rare. Trachoma is the seventh most prevalent cause of blindness
worldwide according to the World Health Organization Global data on visual impairment in the
year 2002.
Resnikoff S, et al. Global Data on Visual Empairment in the Year 2020. Available at: http://​www.who.int/​
bulletin/​volumes/​82/​/​en/​844.pdf?ua=

34. Answer: A
Churg–​Strauss syndrome (CSS), or currently known as eosinophilic granulomatosis with polyangiitis
(EGPA), is a small-​and medium-​vessel vasculitis characterized by asthma, hypereosinophilia, and
multisystem vasculitis. There are usually distinct clinical phases of CSS. The disease begins with
asthma and atopic allergies that may begin in childhood before progression to the eosinophilic
phase, which is characterized by eosinophilic infiltration with granulomatous inflammation and can
106 Clinical Ophthalmology 3 | ANSWERS

have an array of clinical presentations including orbital inflammation. A vasculitic phase usually
follows this. Presentation of sarcoid and GPA are usually much less acute; sarcoid usually affecting
the lacrimal gland and GPA almost invariably involves sinus or respiratory tract symptoms.
Akella SS, et al. Ophthalmic eosinophilic granulomatosis with polyangiitis (Churg-​Strauss syndrome): a
systematic review of the literature. Ophthalmic Plast Reconstr Surg 209;35():7–​6.

35. Answer: B
Blepharophimosis, ptosis, and epicanthus inversus syndrome (BPES) is a rare autosomal dominant
disease that mainly affects the development of the eyelids. People with this condition have
Blepharophimosis (narrowing of the eye opening), Ptosis, and Epicanthus inversus (an upward
fold of the skin of the lower eyelid near the inner canthus) but not epicanthus tarsalis (fold more
prominent in upper eyelid). In addition, there is an increased distance between the inner corners
of the eyes (telecanthus) but not the interpupillary distance. Type  BPES is also associated with
primary ovarian failure which can lead to subfertility and type 2 BPES has no systemic association.
FOXL2 gene mutation is implicated in both type  and 2 BPES.
Yang L, et al. Identification of a novel FOXL2 mutation in a single family with both types of
blepharophimosis—​ptosis-​epicanthus inversus syndrome. Mol Med Rep 207;6:5529–​32.

36. Answer: A
DON can be treated by systemic glucocorticoids, surgery, or both. Orbital radiotherapy is not
recommended in the case of DON unless as an adjunct to proven therapies. High-​dose intravenous
steroids administered in pulses are more efficacious and associated with fewer adverse effects than
oral or retrobulbar steroids. Selenium has only been shown to be beneficial in mild TED.
Bartalena L, et al. Consensus statement of the European Group on Graves’ orbitopathy (EUGOGO)
on management of GO. Eur J Endocrinol 2008;58:273–​85.
Marcocci C, et al. Selenium and the course of mild Graves’ orbitopathy. N Engl J Med
20;364:920–​3.

37. Answer: C
NF is an autosomal dominant condition that can cause a range of ophthalmic manifestations.
Plexiform neurofibromas may involve the eyelid, orbit/​periorbital, and facial structures (termed
OPPN) and could cause visual loss in children. Most OPPN track along the distribution of trigeminal
nerve and can be categorized into three types:
a. Isolated upper eyelid (~33%)—​usually assumes a characteristic ‘S’-​shaped deformity, ptosis
is usually mild, and progression into orbit is unlikely. Rarely a ptosis may be secondary to an
encephalocoele due to an absent sphenoid wing.
b. Eyelid and periorbital region—​affecting V and V2 distribution of trigeminal nerve. Ptosis
may be severe, which can cause amblyopia, and progression to orbit is possible.
c. Orbit with/​without lid involvement—​invades lateral orbit and potentially cavernous sinus.
There are no clotting abnormalities associated with NF, although surgery of plexiform
neurofibroma can be complicated by difficulty controlling haemostasis due to the vascularity of the
lesion; however, there is no role for routine clotting testing. CT can be used to assess the bony
orbits to look for absent sphenoid wing, but radiation should be avoided in children and the bony
deformities can be detected by MRI. MRI is useful for delineating soft-​tissue abnormalities, the
extent of plexiform neurofibroma (including orbital involvement), and bony deformities (figures are
available in the reference). VFT can be used to examine any visual field obstruction and is helpful in
supporting the need for ptosis surgery but is not essential.
Clinical Ophthalmology 3 | ANSWERS 107

Avery RA, et al. Orbital/​periorbital plexiform neurofibromas in children with neurofibromatosis type
: multidisciplinary recommendations for care. Ophthalmology 207;24:23–​32.

38. Answer: D
It is essential to provide extra glucocorticoid cover for patients on long-​term steroid treatment
because of their acquired adrenal insufficiency. Ptosis correction would be classified as ‘minor’
surgery. Intramuscular hydrocortisone is preferred to intravenous injection for its more sustained
duration.
Addison’s Disease Self-​Help Group. ADSHG Surgical Guidelines, 207. Available at: https://​www.
addisons.org.uk/​surgery

39. Answer: A
A patient with OID should only have a therapeutic trial of steroids if they have specific features
consistent with myositis. All other presentations of inflammatory orbital disease should be
investigated further to exclude underlying causes before they can be labelled as idiopathic and
treated accordingly.
Mombaerts I, et al. Consensus on diagnostic criteria of idiopathic orbital inflammation using a modified
Delphi approach. JAMA Ophthalmol 207;35:769–​76.

40. Answer: B
Surgical decompression for optic neuropathy would involve removal of the medial orbital wall,
predominantly towards the apex in order to expand the space around the optic nerve. This
necessitates access into the ethmoid and sphenoid sinuses. Maxillary sinus is commonly opened
inferoposteriorly for drainage and to aid decompression. The frontal sinus being superiorly aids
neither.
Rootman DB. Orbital decompression of thyroid eye disease. Surv Ophthalmol 208;63:86–​04.

4. Answer: D
Muir–​Torre syndrome is an uncommon autosomal dominant condition characterized by sebaceous
carcinomas of the skin and visceral malignancies (most commonly colorectal carcinoma). It is
a phenotypic variant of hereditary non-​polyposis colorectal cancer. Some autosomal recessive
cases have been documented. The median age of onset is 53 years of age and most cases were
reported in Caucasian patients. Bazex syndrome is characterized by skin disorders with underlying
malignancies, most commonly squamous cell carcinoma of the head and neck. Gardner syndrome
is an autosomal dominant disorder characterized by familial adenomatous polyposis and it is
associated with congenital hypertrophy of retinal pigment epithelium. Gorlin–​Golz syndrome, also
known as naevoid BCC syndrome, is a rare autosomal dominant disease characterized by multiple
BCC, hypertelorism, strabismus, myelinated nerve fibres, retinal abnormalities, and other systemic
features such as palmar and plantar pits and ectopic calcification of the brain.
Chen JJ, et al. Review of ocular manifestations of nevoid basal cell carcinoma syndrome: what an
ophthalmologist needs to know. Middle East Afr J Ophthalmol 205;22:42–​7.
Gay JT, Gross GP. Muir-​Torre Syndrome. StatPearls [Internet]. Treasure Iland, FL: StatPearls Publishing,
208. Available at: https://​www.ncbi.nlm.nih.gov/​books/​NBK5327/​

42. Answer: B
Activity is characterized by the inflammatory phase of the disease. It can be graded by clinical
assessment using tools such as the clinical activity severity score; with a score >3 suggesting activity.
108 Clinical Ophthalmology 3 | ANSWERS

Some features of activity can persist when the acute inflammation has settled, such as pain due to
congestion or diplopia due to muscle fibrosis. Reduced colour vision is a sign of optic neuropathy,
which is a sign of severity, not activity. Upper lid oedema is secondary to inflammation which
resolves in the inactive phase.
Mourits MP, et al. Clinical activity score as a guide in the management of patients with Graves’
ophthalmopathy. Clin Endocrinol (Oxf ) 997;47:9–​4.

43. Answer: A
Painful mass in the region of the lacrimal gland suggests either inflammation or neoplasm with
perineural invasion. The histological features given are those of a biopsy from glandular tissue,
but a benign lesion would not contain anaplastic cells. A pleomorphic adenoma would usually
present with a longer duration of painless growth and there would be squamous metaplasia on
the biopsy.
Gunduz AK, et al. Overview of benign and malignant lacrimal gland tumors. Curr Opin Ophthalmol
208;29:458–​68.

44. Answer: C
A full thickness lid defect needs to have posterior lamella reconstruction with tissue to replace
the rigidity of the tarsus lined with a mucosal surface. Ideally this is done with like-​for-​like
donor tarsal plate, either from the ipsilateral upper lid with a Hughes flap or a free tarsal graft.
It would not be possible to close a 0–​2 mm full-​thickness defect in the lower lid by direct
closure. Both a full thickness skin graft and a Mustarde flap would only reconstruct the anterior
lamella. For an elderly patient, the size of skin defect can be classified into small (30%),
moderate (30–​60%), and large (>60%). The following table (Table 3.4) provides a very good
summary on the methods of reconstruction of upper and lower eyelid defects based on the
size of the defect.
Subramanian N. Reconstructions of eyelid defect. Indian J Plast Surg 20;44:5–​3.

Table 3.4  Reconstruction methods for upper and lower eyelid defects, based on the size
of the defect
Size of defect Upper eyelid defect Lower eyelid defect
Small -​ DC with lateral cantholysis or -​ DC with or without lateral cantholysis or
Tenzel’s semicircular flap Tenzel’s semicircular flap
Moderate -​ Mustarde’s lid switch flap -​ AL: Advancement of cheek skin, full
-​ Cutler–​Beard method thickness skin graft, Tripier flap unipedicle
-​ PL: Hughes’ tarsoconjunctival flap
Large -​ Mustarde’s lid switch flap -​ AL: Mustarde’s cheek rotation flap,
-​ Cutler–​Beard method nasolabial flap, median forehead flap, lateral
temporal flap
-​ PL: Chondromucous graft nasal septum
-​ AL + PL: Full thickness skin graft or Tripier
bipedicle flap with Hughes’ flap
Other -​ AL: Fricke’s flap, lateral temporal flap,
midline forehead flap
-​ PL: Free mucous membrane graft,
tarsoconjunctival flap

Data from Subramanian N. Reconstructions of eyelid defect. Indian J Plast Surg 20;44:5–3.
Clinical Ophthalmology 3 | ANSWERS 109

45. Answer: D
TED or Grave’s disease is the most common cause for unilateral or bilateral proptosis (whether
or not axial or non-​axial) among adults. The differential diagnosis of unilateral proptosis can be
remembered as ‘VEIN’:
Vascular—​CCF, arteriovenous malformation, lymphangiectasia
Endocrine—​TED
Infective—​orbital cellulitis
Inflammatory—​OID, myositis, sarcoidosis, granulomatosis with polyangiitis, etc.
Neoplasms—​cavernous haemangioma, lymphoma, etc.
Kamminga N, et al. Unilateral proptosis: the role of medical history. Br J Ophthalmol 2003;87:370–​.

46. Answer: A
Cavernous haemangioma is the most common primary benign orbital tumours in adults, commonly
affecting women at fourth and fifth decades of life. It is a slow growing and often asymptomatic
lesion; therefore, it is commonly an incidental finding. The most common sign is axial proptosis due
to intraconal location.
Orbital metastases usually present late in multisystem disease but may manifest as the first sign
in 5% cases. The lesions are usually located at superior lateral extraconal quadrant. Common
primary sites include breast, skin (melanoma), and prostate. Radiographic features are variable,
ranging from well-​defined round lesions to infiltrating lesions. Pain and diplopia are common
symptoms.
Pleomorphic adenomas arise from the lacrimal gland which is an extraconal structure. Solitary
fibrous tumour is heterogeneous in signal with low intensity areas which correspond to dense
acellular collagen, and is relatively less common.
Calandriello L, et al. Cavernous venous malformation (cavernous hemangioma) of the orbit: Current
concepts and a review of the literature. Surv Ophthalmol 207;62:393–​403.
Valenzuela AA, et al. Orbital metastasis: clinical features, management and outcome. Orbit
2009;28:53–​9.

47. Answer: B
Radiotherapy is not advisable in the upper lid due to collateral damage to the conjunctiva, which can
result in keratinization. All the other treatments have been recommended for superficial BCC.
The Royal College of Ophthalmologists. Focus Article: Periocular Basal Cell Carcinoma. Winter 20.
Available at: https://​www.rcophth.ac.uk/​wp-​content/​uploads/​204/​08/​Focus-​Winter-​20.pdf

48. Answer: B
Lymphoma and OID are indistinguishable on clinical and radiological examination, and both improve
with steroid treatment. Orbital lymphoma is not reliably associated with serological markers, unlike
more disseminated lymphoma which can be associated with a rise in serum lactate dehydrogenase.
Mombaerts I, et al. Consensus on diagnostic criteria of idiopathic orbital inflammation using a modified
Delphi approach. JAMA Ophthalmol 207;35:769–​76.

49. Answer: C
Rhabdomyosarcoma (RMS) is a highly malignant tumour and is one of the few life-​threatening diseases
that present first to the ophthalmologist. It is the most common soft-​tissue sarcoma of the head and
110 Clinical Ophthalmology 3 | ANSWERS

neck in childhood and comprises 4% of all paediatric malignancies, with 0% of all cases occurring in
the orbit. Most of these tumours occur in the first decade of life; however, RMS has been reported
from birth to the eighth decade. Patients with orbital RMS usually present with proptosis developing
rapidly over weeks (80–​00%), or globe displacement (80%) which is usually downward and outward
because two-​thirds of these tumours are superonasal. The mass is usually close to extraocular muscles,
but there is no enlargement of the muscle belly. Previously, RMS was believed to arise from extraocular
muscles, but now it is thought that it originates from pluripotent mesenchymal cells that have the ability
to differentiate into skeletal muscle. The embryonal type comprises 50–​70% of orbital RMS.
Jurdy L, et al. Orbital rhabdomyosarcomas: a review. Saudi J Ophthalmol 203;27:67–​75.

50. Answer: D
Patients judged at high risk of developing metastases should have 6-​monthly lifelong surveillance
incorporating a clinical review, nurse specialist support, and liver-​specific imaging by a non-​ionizing
modality such as ultrasound. Liver function tests alone are an inadequate tool for surveillance.
Nathan P, et al. Uveal Melanoma National Guidelines—​Melanoma Focus, 205. Available at: http://​
melanomafocus.com/​wp-​content/​uploads/​205/​0/​Uveal-​Melanoma-​National-​Guidelines-​Full-​v5.3.pdf

5. Answer: B
The genetic mechanism of retinoblastoma is governed by the Knudson’s two-​hit hypothesis
whereby both Rb tumour suppressor genes need to be mutated for the development of
retinoblastoma. The pattern of inheritance is autosomal dominant, but phenotypically it is recessive
at cellular level. It can exist in either hereditary/​germline or somatic/​sporadic form (whereby
there is only unilateral involvement). Hereditary form usually results in bilateral and multifocal
retinoblastoma and is associated with an increased risk of secondary tumours, notably pinealoma
(or known as trilateral retinoblastoma). It has a 90% penetrance, which means the offspring of
the affected patients will have a 45% chance of developing retinoblastoma (50% inheriting the
gene × 90% penetrance). It is also important to remember that around 0–​5% of the unilateral
cases have hereditary form of mutation; therefore, in a parent who has unilateral retinoblastoma,
there is up to 7% chance of his/​her children developing retinoblastoma [5% (hereditary) × 50%
(dominant inheritance) × 90% (incomplete penetrance) = around 7%]. If the parent has a unilateral
disease and the first child develops retinoblastoma (indicative of germ line mutation), the second
child has a 45% risk of developing retinoblastoma. If there is no family history and a child develops
retinoblastoma, the risk of sibling developing retinoblastoma is 2% (if the first child has bilateral
disease) or % (if the first child has unilateral disease).
Draper GJ, et al. Patterns of risk of hereditary retinoblastoma and applications to genetic counselling.
Br J Cancer 992;66:2–​9.

52. Answer: B
Choroidal naevus is the most common benign intraocular tumour, occurring in about  in 0
people. In view of its high prevalence, it is important to familiarize with the risk factors for
transformation into melanoma from clinicians’ and patients’ perspective. Shields et al. have
conducted an analysis on 254 consecutive cases of choroidal naevi and have found the following
risk factors based on multivariate analysis. This can be memorized by a mnemonic: ‘To Find Small
Ocular Melanoma Using Helpful Hints Daily’. Patients with three or more of these factors are
likely to develop melanoma.
T—​Thickness > 2 mm
F—​Fluid (subretinal)
S—​Symptoms
Clinical Ophthalmology 3 | ANSWERS 111

O—​Orange pigment
M—​Margin of tumour within 3 mm of the optic disc
UH—​Ultrasonographic hollowness
H—​Halo  absent
D—​Drusen  absent
Shields CL, et al. Melanoma of the eye: revealing hidden secrets, one at a time. Clin Dermatol
205;33:83–​96.

53. Answer: C
MEWDS is usually a self-​limiting condition, typically affecting young females (F:M = 4:) and often
preceded by a viral-​type illness. It is almost always unilateral with no or very little vitritis, small
subtle white-​grey lesions (00–​200 μm) at the posterior pole to midperiphery and a granular fovea
appearance. A RAPD and swollen optic disc may be observed. The other diseases have different
features and lesion morphology to that described. In APMPPE, larger white-​creamy placoid retinal
pigment epithelium lesions are seen acutely followed by chorioretinal scarring. Birdshot is a chronic
disease with multiple creamy chorioretinal lesions (often oval), vitritis, and retinal vasculitis, and
depigmentation. In birdshot, anterior uveitis is mild or absent, but disc swelling can be observed.
PIC usually affects young myopic patients, typically female, with no-​mild vitritis and multiple white
chorioretinal lesions (usually 50–​00 μm) which become atrophic +/​–​pigmented in time. PIC may
present with secondary choroidal neovascularization (CNV) as a complication. RAPD and granular
macular appearance would not be observed in typical PIC and are key features of MEWDS.
Salvatore S, et al. Multimodal imaging in acute posterior multifocal placoid pigment epitheliopathy
demonstrating obstruction of the choriocapillaris. Ophthalmic Surg Lasers Imaging Retina
206;47:677–​8.
Tavallali A, Yannuzzi LA. MEWDS, common cold of the retina. J Ophthalmic Vis Res 207;2:32–​4.

54. Answer: B
Understanding the features of anterior uveitis syndromes is important for answering such questions,
with need to focus on the disease course, iris involvement, type of keratitic precipitates, and
posterior synechiae. The description of sectoral iris atrophy, large irregular pupil, and posterior
synechiae are typical descriptors of herpetic uveitis, alongside raised pressure and/​or corneal
disease. Fuchs’ uveitis typically causes heterochromia, diffuse rather than sectoral iris atrophy,
vitreous opacity is common, and posterior synechiae are absent. HLA-​B27 uveitis is typically
recurrent non-​granulomatous fibrinous uveitis with risk of posterior synechiae. Similarly, ocular
sarcoidosis can present with unilateral or bilateral anterior uveitis (granulomatous or non-​
granulomatous) and synechiae.
Tay-​Kearney ML, et al. Clinical features and associated systemic diseases of HLA-​B27 uveitis. Am J
Ophthalmol 996;2:47–​56.
Van der Lelij A, et al. Anterior uveitis with sectoral iris atrophy in the absence of keratitis: a distinct
clinical entity among herpetic eye diseases. Ophthalmology 2000;07:64–​70.

55. Answer: D
Systems for classification of uveitis have been assessed in FRCOphth examinations. The SUN
grading system is an internationally accepted system for standardization of reporting clinical data
in uveitis. This includes anatomical classification (anterior, intermediate, posterior, or panuveitis)
based on the site(s) of uveitis and not on the presence of complications (such as cystoid macular
112 Clinical Ophthalmology 3 | ANSWERS

oedema (CMO)). Criteria for the onset, duration, and course of the uveitis were also established.
Standardized grading schemes for grading anterior chamber cells, anterior chamber flare, and
vitreous haze were developed. Although presence of vitreous cells is an important feature, the
grading system does not include vitreous cells. Macular oedema can be reported as present or
absent but is not part of the grading criteria. Similarly the type of keratitic precipitates is not
graded.
Jabs DA, et al. Standardization of Uveitis Nomenclature (SUN) Working Group. Standardization of
uveitis nomenclature for reporting clinical data. Results of the First International Workshop. Am J
Ophthalmol 2005;40:509–​6.

56. Answer: A
Both the ocular features and systemic manifestations described are typical of Behçet’s disease.
Behçet’s disease is a chronic, multisystem inflammatory disease characterized by orogenital
ulceration. The ocular disease can involve all segments of the eye with non-​g ranulomatous
anterior uveitis with a mobile hypopyon and retinitis and occlusive vasculitis (arterial or venous)
described. The systemic disease spectrum can be diverse including neurological (headache,
venous sinus thrombosis, cranial nerve palsies, vasculitis), intestinal, cutaneous (pustular folliculitis,
erythema nodosum), vascular (venous thrombosis), and urogenital (epididymitis, orchitis)
manifestations.
Sakane T, et al. Behçet’s disease. N Engl J Med 999;34:284–​9.

57. Answer: A
HLA disease associations are an important list to compile for the exam, with particular focus on
diseases with ophthalmic involvement. Birdshot chorioretinopathy is strongly associated with
HLA-​A29. HLA-​A29 is found in up to 7% of the Caucasian population and >95% of patients
with birdshot chorioretinopathy. HLA-​B5 is associated with Behçet’s disease but is not part of
the diagnostic criteria. HLA-​B27 is associated with multiple inflammatory conditions including
spondyloarthropathy, inflammatory bowel disease, psoriatic arthropathy, and non-​granulomatous
anterior uveitis. HLA-​DR is associated with rheumatoid arthritis.
Suarez-​Almazor ME, et al. HLA-​DR, DR4, and DRB disease-​related subtypes in rheumatoid
arthritis. Association with susceptibility but not severity in a city wide community based study. J
Rheumatol 995;22:2027–​33.
Wee R, Papaliodis G. Genetics of birdshot chorioretinopathy. Semin Ophthalmol 2008;23:53–​7.

58. Answer: A
The aetiology of scleritis includes inflammatory, infective (e.g. syphilis, tuberculosis, and herpetic)
and masquerade conditions (lymphoma and myeloma). Important inflammatory systemic
associations are rheumatoid arthritis, granulomatosis with polyangiitis (GPA), and less commonly
sarcoidosis. GPA (formerly known as Wegener’s granulomatosis) is characterized by presence
of antineutrophil cytoplasmic antibodies (ANCA). Around 80–​95% of cases are associated with
cytoplasmic-​ANCA (c-​ANCA) directed against proteinase 3 antibodies (PR3) the remainder are
perinuclear antibodies (pANCA) against myeloperoxidase (MPO).
The features described are typical features of GPA. GPA has a spectrum of manifestations
especially ear, nose, and throat (ENT) and respiratory plus renal, rheumatological, cutaneous,
and neurological disease. Peak incidence is 64–​75 years old. Typical upper and lower respiratory
complications are epistaxis, nasal congestion, chronic sinusitis, haemoptysis, and cough.
Clinical Ophthalmology 3 | ANSWERS 113

Constitutional symptoms include weight loss, fatigue, and night sweats. The age of onset of joint
problems in this patient is very atypical for rheumatoid arthritis. Gamma-​interferon assay testing
(Quantiferon®) is used for tuberculosis testing and a negative result indicates no previous exposure
to Mycobacterium Tuberculosis.
Kubaisi B, et al. Granulomatosis with polyangiitis (Wegener’s disease): An updated review of ocular
disease manifestations. Intractable Rare Dis Res 206;5:6–​9.
Schonermarck U, et al. Prevalence and spectrum of rheumatic diseases associated with proteinase 3-​
antineutrophil cytoplasmic antibodies (ANCA) and myeloperoxidase-​ANCA. Rheumatology (Oxford)
200;40:78–​84.

59. Answer: D
The indications for treatment of toxoplasma retinochoroiditis are: lesions involving/​threatening a
major vessel, the disc, or macula; significant vitritis and immunocompromised status. Observation
may be appropriate for non-​sight threatening peripheral lesions in immunocompetent patients.
Oral spiramycin is used for treatment of toxoplasmosis pregnancy (on a named patient basis). Oral
steroids must NOT be used without antitoxoplasmosis therapy and can cause significant worsening
of inflammation. Treatment regimes include co-​trimoxazole, clindamycin, azithromycin, atovaquone,
and sulfadiazine/​pyrimethamine/​folinic acid (not folate) with oral corticosteroid therapy. There is
no definite consensus on the best treatment regimen.
Lima GS, et al. Current therapy of acquired ocular toxoplasmosis: a review. J Ocul Pharmacol Ther
205;3:5–​7.
Ozgonul C, Besirli CG. Recent developments in the diagnosis and treatment of ocular toxoplasmosis.
Ophthalmic Res 207;57:–​2.

60. Answer: B
The differential of acute anterior uveitis associated with oral and anal ulceration is between
Behçet’s disease and Crohn’s disease. No mention is made of genital ulceration or other systemic
features of Behçet’s disease. Ulceration in ulcerative colitis (UC) is mostly restricted to the large
bowel. However, UC is associated with HLA-​B27 which is frequently associated with enthesitis
such as plantar fasciitis or Achilles tendonitis. Crohn’s disease is associated with both HLA-​B27 and
oral and perianal ulceration unlike infectious endogenous endophthalmitis, which should always be
in the differential of hypopyon but is not easily linked to either of the other features. All of these
conditions would be expected to elevate ESR and CRP.
Chang JH, et al. Acute anterior uveitis and HLA-​B27. Surv Ophthalmol 2005;50:364–​88.

6. Answer: D
Fuchs’ heterochromic cyclitis (FHC) is a chronic, low-​grade anterior segment uveitis that accounts
for around 3% of all uveitis cases. Affected patients are usually asymptomatic and may sometimes
complain of mild blurry vision or floaters. It is commonly unilateral (90%) and typical ocular findings
include widespread stellate keratic precipitates (cf. inferiorly located keratic precipitates in HLA-​
related anterior uveitis), mild anterior chamber reaction with mild but persistent cells and flare,
absence of posterior synechiae, iris nodules (around 20%), presence of abnormal vessels in iris and
trabecular meshwork which may lead to spontaneous or surgical-​induced haemorrhage (Amsler’s
sign), and presence of vitritis. The cause of FHC is unknown but rubella virus has been implicated in
the pathogenesis.
Bonfioli AA, et al. Fuchs’ heterochromic cyclitis. Semin Ophthalmol 2005;20:43–​6.
114 Clinical Ophthalmology 3 | ANSWERS

62. Answer: B
The notable feature shown in Figure 3.A is the marked thickening of the choroid, in the range
of 500 microns, suggestive of active choroiditis. The absence of obvious anterior chamber or
vitreous inflammation or subretinal fluid should not lull the clinician into a false sense of security.
This is an only eye situation that requires urgent rescue with high-​dose systemic corticosteroids.
Further options may then be considered. These include introduction of a calcineurin inhibitor
or an antitumour necrosis factor (TNF) agent. He will also likely need to reduce the dose
of mycophenolate mofetil in the face of GI upset. Following the high-​dose intravenous
methylprednisolone, the choroidal thickening significantly improved (shown in Figure 3.B).
Arevalo JF, et al. Update on sympathetic ophthalmia. Middle East Afr J Ophthalmol 202;9:3–​2.

63. Answer: C
This is a clinical vignette suggestive of endogenous endophthalmitis and the most likely culprit in this
case is Klebsiella pneumonia in view of the underlying liver abscess and diabetes. Around 95–​00%
patients with Klebsiella endophthalmitis are associated with underlying pyogenic liver abscess and
50–​70% of them also have diabetes. The mortality is around 0–​20% but can be as high as 50% in
patients with septic shock. Around 67–​80% of the affected patients end up with vision of NPL
or evisceration. Poor visual prognostic factors include diabetes, presence of hypopyon, diffuse
posterior involvement, and delayed treatment for more than 24 hours from the onset.
Sheu SJ, et al. Risk factors for endogenous endophthalmitis secondary to Klebsiella pneumonia liver
abscess: 20-​year experience in Southern Taiwan. Retina 20;3:2026–​3.
Tan YM, et al. Ocular manifestations and complications of pyogenic liver abscess. World J Surg
2004;28:38–​42.
chapter PHARMACOLOGY, THERAPEUTICS,
AND INVESTIGATIONS
4 QUESTIONS

. A 34-​year-​old female with a history of severe active bilateral panuveitis is


being counselled on systemic immunosuppression therapies. She is keen
to start a family and has expressed that she wishes to become pregnant.
Which of the following drugs is absolutely contraindicated in pregnancy?
A. Azathioprine
B. Ciclosporin
C. Methotrexate
D. Prednisolone

2. A -​year-​old boy with congenital glaucoma requires treatment for raised


intraocular pressure (IOP). Which of the following topical treatments is
contraindicated?
A. Brimonidine
B. Brinzolamide
C. Latanoprost
D. Timolol

3. In a patient with a history of allergy to sulphonamide antibiotic therapy


there is a possibility of cross-​reactivity to which of the following drugs?
A. Azathioprine
B. Clarithromycin
C. Co-​trimoxazole
D. Trimethoprim

4. An assay for thiopurine methyltransferase (TPMT) is recommended


before starting which immunosuppressive agent?
A. Azathioprine
B. Mycophenolate mofetil
C. Mycophenolate  sodium
D. Tacrolimus
116 Pharmacology, Therapeutics, and Investigations | QUESTIONS

5. In the management of acute anterior uveitis associated with HLA B27,


subconjunctival ‘mydricaine’ is often used to break posterior synechiae.
Which of the following statement about mydricaine preparation is true?
. It contains phenylephrine, procaine, and atropine
A
B. It contains adrenaline, procaine, and atropine
C. It should not be used in the second trimester of pregnancy
D. It is licensed for use in the treatment of uveitis in the United Kingdom

6. What is the mechanism of action of vigabatrin (Sabril)?


. Selective irreversible inhibitor of gamma-​aminobutyric acid (GABA)-​transaminase
A
B. Non-​selective irreversible inhibitor of GABA-​transaminase
C. Selective reversible inhibitor of GABA-​transaminase
D. Non-selective reversible inhibitor of GABA-transaminase

7. Tocilizumab, an anti-​interleukin 6 receptor antibody, is approved by the


National Institute for Health and Care Excellence (NICE) for treatment
of which of the following conditions?
. Giant cell arteritis
A
B. Granulomatosis with polyangiitis
C. Multiple sclerosis
D. Thyroid eye disease

8. Which of the following mechanisms of action best describes aflibercept?


. Inhibits vascular endothelial growth factor (VEGF)-​A
A
B. Inhibits  VEGF-​B
C. Inhibits VEGF-​A  and  -​B
D. Inhibits VEGF-​A and -​B and placental growth factor

9. A 27-​year-​old male presented to casualty with a one-​week history of


bilateral reduced vision. An optical coherence tomography (OCT)
performed in casualty revealed bilateral retinal striae. Which of the
following is a possibility?
A. He had seen his neurologist 0 days earlier who had increased his topiramate from 75 mg
to 00 mg
B. He had seen his neurologist 0 days earlier who had increased his sodium valproate from
500 mg OD to BD
C. He had seen his neurologist 0 days earlier who had stopped his phenytoin
D. He had seen his neurologist 0 days earlier who had stopped his vigabatrin
Pharmacology, Therapeutics, and Investigations | QUESTIONS 117

0. What is the mechanism of action of the commonly used fusidic acid (or
fucithalmic acid)?
. Inhibits cell wall synthesis
A
B. Inhibits DNA replication
C. Inhibits folic acid metabolism
D. Inhibits protein synthesis

. Tacrolimus and cyclosporine have similar mechanisms of action.


However, which of the following is correct?
. Tacrolimus is ten times more potent than cyclosporine
A
B. Tacrolimus is 00 times more potent than cyclosporine
C. Cyclosporine is ten times more potent than tacrolimus
D. Cyclosporine is 00 times more potent than tacrolimus

2. A 45-​year-​old male presented to the corneal clinic with severe keratitis,
which had not improved despite treatment with tear substitutes.
He was prescribed topical cyclosporine ( mg/​ml or 0.%). He was
particularly concerned about the main adverse events. Which is the
commonest investigator-​related adverse event?
A. Blurred  vision
B. Erythema of the eyelid
C. Instillation site pain
D. Ocular hyperaemia

3. What is the mechanism of action of vismodegib—​a drug that is used in


basal cell carcinoma (BCC)?
. Causes alkylation of  DNA
A
B. Inhibits Hedgehog pathway
C. Inhibits interleukin 2
D. Inhibits tumour necrosis factor (TNF)-​alpha

4. A 79-​year-​old keen gardener with age-​related macular degeneration


attends the clinic after purchasing a box of MacuShield, which contains
the ingredient of meso-​zeaxanthin. He asks which plant is used:
A. Allium tricoccum
B. Allium tuberosum
C. Daffodil
D. Marigold
118 Pharmacology, Therapeutics, and Investigations | QUESTIONS

5. A 45-​year-​old Caucasian patient attends eye casualty. He reports


distorted vision. On examination he has evidence of  mm punched out
yellow spots, a macular choroidal neovascular membrane (CNVM) seen
on optical coherence tomography angiography with scarring adjacent
to the optic disc. He reports a recent sabbatical to the Ohio-​Mississippi
River Valley. Which of the following is the correct treatment?
. Oral terbinafine 250 mg daily for six weeks
A
B. Oral terbinafine 500 mg daily for four weeks
C. Oral tetracycline 500 mg daily for two days
D. Focal  laser

6. Which of the following is not a known side effect of gefitinib—​an


epidermal growth factor receptor (EGFR) inhibitor?
A. Blepharoconjunctivitis
B. Corneal erosion
C. Metamorphopsia
D. Trichomegaly

7. A 63-​year-​old female patient presents to the eye clinic with chronic
conjunctivitis. Examination reveals cicatricial conjunctival conjunctivitis
with symblepharon and subtarsal conjunctival scarring. She also
discloses a history of glucose-​6-​phosphate dehydrogenase (G6PD)
deficiency. Which of the following medications is contraindicated?
A. Dapsone
B. Mycophenolate mofetil
C. Rituximab
D. Tacrolimus

8. What is the mechanism of action of imiquimod, a drug that is approved


for treating superficial BCC?
A. Anti-​interleukin  (IL)-​2
B. Anti-​IL-​2
C. Interferon β-​a antagonist
D. Toll-​like receptor 7 agonist

9. Which of the following conditions is NOT associated with the following
OCT image in an adult patient (shown in Figure 4.)?
. Central serous chorioretinopathy
A
B. Harada’s disease
C. Macular telangiectasia
D. Pre-​eclampsia
Pharmacology, Therapeutics, and Investigations | QUESTIONS 119

Figure 4.  

20. A patient presents with blurred vision with evidence of uveitis and
multifocal serous retinal detachment. The patient has headache,
tinnitus, and neck stiffness. Lumbar puncture shows evidence of
pleocytosis. A wide-​field fundus fluorescein angiography (FFA) is
performed and shows multiple foci of pinpoint hyperfluorescence
followed by pooling of fluorescein within the subretinal space. What is
the most likely diagnosis?
A. Posterior scleritis
B. Primary central nervous system (CNS) lymphoma
C. Sympathetic ophthalmia
D. Vogt–​Koyanagi–​Harada (VKH) disease

2. If FFA of CNVM shows early lacy hyperfluorescence and late leakage,
which of the following is correct?
. The lesion is a classic lesion
A
B. The lesion is a predominantly classic lesion
C. The lesion is a minimally classic lesion
D. The lesion is an occult lesion with no classic features

22. In testing for Mycobacterium tuberculosis which statement is most true?


. A positive gamma-​interferon test result indicates active Mycobacterium tuberculosis infection
A
B. Gamma-​interferon testing is not influenced by prior Bacillus Calmette–​Guérin (BCG)
vaccination
C. Gamma-​interferon testing is negative in patients who have received a full course of
antituberculous therapy
D. Gamma-​interferon testing in interpreted by quantification of degree of skin induration at
the site of subcutaneous injection of plasma protein derivative (PPD).
120 Pharmacology, Therapeutics, and Investigations | QUESTIONS

23. In FFA, what is the normal arm-​eye time?


A. 2–​3 seconds
B. 0–​2 seconds
C. 20 seconds
D. 60 seconds

24. Which of the following medication is associated with the OCT


appearance shown in Figure 4.2?
A. Hydroxychloroquine
B. Nicotinic  acid
C. Rifabutin
D. Tamoxifen

Figure 4.2  

25. Which of the following statement regarding culture media for corneal
scraping is true?
. Cooked meat broth specifically grows aerobic bacteria
A
B. Blood agar grows most bacteria and fungi
C. Chocolate agar is particularly good for meningococci and streptococci
D. Brain heart infusion grows anaerobic bacteria

26. Which of following about ophthalmic ultrasonography is true?


A. The use of lower frequency transducer allows generation of images with much higher
resolution
B. The use of lower frequency transducer allows greater depth penetration
C. Orbital ultrasound uses a high frequency ultrasonic transducer
D. B-​scan can be used to measure axial length
Pharmacology, Therapeutics, and Investigations | QUESTIONS 121

27. What is the following imaging technique shown in Figure 4.3 and what is
the most likely diagnosis?
. Fluorescein angiography; wet macular degeneration
A
B. Indocyanine green angiography; wet macular degeneration
C. Autofluorescence; adult vitelliform disease
D. Red free imaging, adult vitelliform disease

Figure 4.3  

28. Which of the following statements regarding indocyanine green


angiography (ICGA) is true?
. Is safer in pregnancy than fluorescein angiography
A
B. Is contraindicated in patients with iodine allergy
C. Has an excitation peak at 490 nm and emission of  530 nm
D. Does not stain the stool

29. Which of the following options regarding anterior segment optical


coherence tomography (AS-​OCT) is true?
. Uses a shorter wavelength light source compared to posterior OCT
A
B. Is inferior to ultrasound biomicroscopy (UBM) in visualization of the lens
C. Does not easily allow assessment of the drainage angle
D. Is inferior to UBM in the assessment of anterior stromal scars
122 Pharmacology, Therapeutics, and Investigations | QUESTIONS

30. Which of the following statements about A-​scan is true?


. The axial length of the globe may be artefactually longer in the presence of silicone oil
A
B. The axial length of the globe may be artefactually longer in eyes with asteroid hyalosis
C. Can be used to locate intraocular foreign body
D. It obtains the axial length measurement from the retinal pigment epithelium

3. Visual evoked potential (VEP):


A. Is a reliable test of the optic nerve function in a patient with advanced macular
degeneration
B. Cannot detect subclinical optic nerve demyelination
C. Shows a decreased amplitude and increased peak-​time of P00 in optic neuritis
D. Flash VEP provides more information than pattern reversal VEP

32. What is the principle of imaging utilized by Pentacam machine?


A. Placido  disc
B. Placido disc and scanning slit technique
C. Purkinje  images
D. Scheimpflug imaging

33. The requirements to complete a Lees screen test to measure ocular


deviation is:
. Normal vision in at least one eye
A
B. Normal colour vision in both eyes
C. Normal retinal correspondence
D. Normal visual field

34. Which of the following statements about prism adaptation test (PAT) is
correct?
A. It is effective in predicting postoperative sensory and motor fusion especially in patients
with congenital esotropia
B. The angle increase after PAT is generally smaller than diagnostic occlusion test
C. Preoperative prism correction is not useful in the determination of the amount of surgery
D. The increase in the squint angle after PAT is caused by an anomalous sensorial relationship
between the two eyes

35. Which of the following orthoptic tests CANNOT measure torsion?


A. Hess  chart
B. Maddox  rod
C. Maddox  wing
D. Synoptophore
Pharmacology, Therapeutics, and Investigations | QUESTIONS 123

36. Which of the following about accommodative convergence/​


accommodation (AC/​A) ratio is correct?
. Gradient AC/​A method requires measurement of interpupillary distance (IPD)
A
B. Heterophoria AC/​A requires a plus lens for the test
C. The normal AC/​A ratio is 3–​5 degree/​dioptre
D. AC/​A ratio is usually low in simulated intermittent distance exotropia (IDEX)

37. Which of the following neuroimaging tests is shown to be best in


imaging intracranial aneurysm?
. Computed tomography (CT)
A
B. CT angiography
C. Magnetic resonance imaging (MRI) with no contrast
D. MR angiography

38. A 62-​year-​old man presented to clinic describing reduction in vision. He


had thought it had been coming on over 3 months. He had no headaches
and his recent refraction was unchanged from previous attendance at
optometry. On examination his visual acuity was 6/​2 in both eyes and
his colour vision was reduced to 3/​4 in both eyes. There was bilateral
temporal pallor of both optic nerves but otherwise no relative afferent
pupillary defect. OCT of each optic nerve showed loss of temporal nerve
fibres bilaterally. Visual fields to confrontation showed a bitemporal
hemianopia. What radiological investigations would you perform next?
. Computed tomography angiogram (CTA) of circle of  Willis
A
B. CT  head
C. MRI of  orbit
D. MRI brain with gadolinium

39. Which of the following features is most likely found in the condition
shown in Figure 4.4?
. Anterior segment dysgenesis
A
B. Cataract
C. Optic nerve hypoplasia
D. Retinal dystrophy

40. What is the choice of neuroimaging in patients with suspected optic


neuritis?
. CT head with contrast
A
B. CTA of circle of  Willis
C. MRI using fluid attenuated inversion recovery (FLAIR) sequence
D. MR head and orbit with gadolinium contrast
124 Pharmacology, Therapeutics, and Investigations | QUESTIONS

Figure 4.4 
Reprinted by permission from Springer Nature: Nature, European Journal of Human Genetics, 8(4):393–397, Septooptic dysplasia.
Webb, E., and Dattani, M. https://doi.org/0.038/ejhg.2009.25. Copyright © 2009, Springer Nature.

4. A 53-​year-​old man, a heavy smoker, presents to the eye emergency


clinic with 3 days’ history of significant right-​sided ear pain, mild ptosis,
and anisocoria, suggestive of possible Horner’s syndrome. What is the
most appropriate radiological investigation for this patient?
. CT head and CTA head and neck
A
B. MRI head and base of skull with gadolinium
C. MRI head and magnetic resonance angiography (MRA) Circle of Willis
D. MRI internal auditory meatus

42. Which of the following about dacryoscintigram (DSG) is correct?


. It requires injection of a radio-​opaque contrast medium into the lacrimal drainage system
A
B. Patient is instructed to lie flat during the procedure
C. It is not useful for proximal obstruction
D. It is a useful investigation for functional epiphora
chapter PHARMACOLOGY, THERAPEUTICS,
AND INVESTIGATIONS
4 ANSWERS

. Answer: C
Methotrexate at any dose is contraindicated in pregnancy due to teratogenic effects and must be
stopped at least 3 months before conception. Mycophenolate mofetil is also not safe for use in
pregnancy with risk of teratogenicity and spontaneous abortion. Azathioprine, ciclosporin, and
tacrolimus can be safely used in pregnancy if prescribed within recommended doses. Prednisolone
is compatible with all stages of pregnancy.
Flint J, et al. BSR and BHPR guideline on prescribing drugs in pregnancy and breastfeeding—​part
I: standard and biologic disease modifying anti-​rheumatic drugs and corticosteroids. Rheumatology
(Oxford) 206;55:693–​7.

2. Answer: A
This is an important safety topic in pharmacology. Brimonidine can cause Central nervous system
(CNS) depression in children including drowsiness or lethargy. The drug is contraindicated in
neonates and infants (<2 years old). Alternative IOP lowering therapy should be considered
in children under 2 years and used with caution and close monitoring. There are no apparent
contraindications to β-​blocker therapy in this patient (namely asthma, chronic obstructive
pulmonary disease (COPD), bradycardia, and heart block) and topical brinzolamide is effective
and safe to use in children. Prostaglandin therapy is also safe in children.
Al-​Shahwan S, et al. Side-​effect profile of brimonidine tartrate in children. Ophthalmology
2005;2:243.
Coppens G, et al. The safety and efficacy of glaucoma medication in the pediatric population. J Pediatr
Ophthalmol Strabismus 2009;46:2–​8.

3. Answer: C
Co-​trimoxazole is a combined antibiotic of trimethoprim and sulfamethoxazole, which contains
the sulfonamide chemical group. It is usually used for the treatment of toxoplasma uveitis. In the
context of ophthalmology, it is noteworthy to mention about acetazolamide, which is a sulfonamide
derivative. The British National Formulary (BNF) highlights that acetazolamide is contraindicated if
there is confirmed sulfonamide hypersensitivity.
Mayo Clinic. Sulfa allergy: Which medications should I avoid? Available at: https://​www.mayoclinic.org/​
diseases-​conditions/​drug-​allergy/​expert-​answers/​sulfa-​allergy/​faq-​20057970

4. Answer: A
Azathioprine (AZA) is metabolized to its active metabolite by a series of enzyme steps, including
TPMT. TPMT enzyme activity is highly variable: 90% of individuals have high/​normal activity, 0%
have intermediate activity, and 0.3% low/​absent activity. The British Society of Rheumatology
disease-​modifying antirheumatic drug (DMARD) guidelines recommend performing TPMT assay
126 Pharmacology, Therapeutics, and Investigations | ANSWERS

before starting AZA. The assay provides additional risk of toxicity but does not replace routine
monitoring. Patients with low enzyme activity are at potential risk of profound neutropenia (which
may be delayed by several months). Reduced dosing or an alternative drug should be considered.
Baseline laboratory organ function (FBC, U&E, LFTs) are mandatory before starting all the
medications but no specific enzyme assay is required for the other medications.
Ledingham J, et al. BSR and BHPR guideline for the prescription and monitoring of non-​biologic
disease-​modifying anti-​rheumatic drugs. Rheumatology (Oxford) 207;56:865–​68.
McLeod HL, et al. Analysis of thiopurine methyltransferase variant alleles in childhood acute
lymphoblastic leukaemia. Br J Haematol 999;05:696–​700.

5. Answer: B
Mydricaine for subconjunctival injection is marketed by Moorfields Pharmaceuticals in 0.3 ml
vials, indicated for use in acute anterior uveitis to achieve maximal mydriasis. Caution is advised in
pregnancy but use of mydricaine is not contraindicated. The product is not licensed for use in the
United Kingdom but is routinely used on an off-​license basis in the absence of a licensed alternative.
It comes in two preparations; mydricaine No. is used in paediatric or elderly patients and it
contains procaine 3 mg, atropine 0.5 mg, and adrenaline 08 µg, whereas mydricaine No. 2 is used
in adult patients and it contains procaine 6 mg, atropine  mg, and adrenaline 26 µg (effectively
double the dose of the No.  preparation).
Steel DH, Thorn J. The incidence of systemic side-​effects following subconjunctival mydricaine no. 
injection. Eye (Lond) 999;3:720–​2.

6. Answer: A
Vigabatrin acts as a selective irreversible inhibitor of GABA-​transaminase. The drug is water
soluble and rapidly absorbed by the gastrointestinal tract. Maximal efficacy is usually seen in the
2–​3 g/​day range (children 50–​00 mg/​kg/​day). It is predominantly used in partial epilepsy, with or
without secondary generalization. It is the one of the main treatments for West syndrome (infantile
spasms). Dr West first described the condition in his 4-​month-​old son in 84.
The Royal College of Ophthalmologists. The Ocular Side-​Effects of Vigabatrin (Sabril) Information and
Guidance for Screening. 2008. Available at: https://​www.rcophth.ac.uk/​wp-​content/​uploads/​205/​0/​
2008-​SCI-​020-​The-​Ocular-​Side-​Effects-​of-​Vigabatrin-​Sabril.pdf

7. Answer: A
Tocilizumab (RoActemra) is an immunosuppressive drug. It is a humanized monoclonal antibody
that targets against the interleukin-​6 receptor (IL-​6R). Tocilizumab is approved by NICE for the
treatment of rheumatoid arthritis and giant cell arteritis. It can be used in combination with
a tapering course of glucocorticoid or alone following discontinuation of glucocorticoid. It is
given every week via subcutaneous injection (62 mg). The main clinical evidence for the use of
tocilizumab came from GiACTA, a multicentre, double-​blind, randomized controlled trial.
National Institute for Health and Care Excellence (NICE). Tocilizumab for Treating Giant Cell Arteritis
[TA58]. Available at: https://​www.nice.org.uk/​guidance/​ta58

8. Answer: D
Aflibercept, also known as Eylea or VEGF Trap, is a novel soluble decoy receptor which utilizes
the fusion of components from multiple endogenous receptors. It inhibits VEGF-​A, VEGF-​B, and
placental growth factor (PlGF). It has a much higher affinity to VEGF-​A than ranibizumab and
bevacizumab. More importantly it is the only anti-​VEGF agent that targets VEGF-​B and PlGF.
Pharmacology, Therapeutics, and Investigations | ANSWERS 127

Papadopoulos N, et al. Binding and neutralization of vascular endothelial growth factor (VEGF) and
related ligands by VEGF Trap, ranibizumab and bevacizumab. Angiogenesis 202;5:7–​85.

9. Answer: A
The use of topiramate is becoming increasingly popular for the management of epilepsy, migraine,
trigeminal neuralgia, and depression. The anterior segment ocular side effects have been extensively
reported but the documentation and mechanism of a pure topiramate maculopathy is less well
understood. This is highlighted by the omission of any reference of a pure maculopathy in the
RCOphth guidelines. A thorough drug history including any recent change in dosage when faced
with a similar clinical scenario is required. It is imperative the underlying diagnosis behind the
use of topiramate is established and changes in dosage or discontinuation must be carried out
in consultation with the patient’s GP and/​or neurologist. Topiramate maculopathy is not a life-​
threatening condition, whereas status epilepticus is.
Severn P, et al. Topiramate maculopathy secondary to dose titration: first reported case. Eye (Lond)
205;29:982–​4.

0. Answer: D
Fusidic acid is a bacteriostatic antibiotic that inhibits protein synthesis by preventing the turnover
of elongation factor G (EF-​G) from the ribosome. It primarily works on Gram-​positive bacteria.
Following is a summary table of the mechanisms of action (MOA) of commonly used antibiotics
(Table 4.).

Table 4.  Mechanisms of action for commonly used antibiotics


MOA Antibiotics
Inhibit cell wall synthesis (therefore effective Penicillin, cephalosporin, vancomycin
against Gram-​positive bacteria)
Inhibit protein synthesis: Aminoglycoside, tetracycline
-​ Inhibit 30 S subunits of ribosome Chloramphenicol, macrolide
-​ Inhibit 50 S subunits of ribosome Fusidic acid
-​ Inhibits EF-​G of ribosome
Inhibit DNA synthesis Fluoroquinolone
Inhibit folic acid metabolism Sulfonamide, trimethoprim

Kapoor S, et al. Action and resistance mechanisms of antibiotics: a guide for clinicians. J Anaesthesiol
Clin Pharmacol 207;33:300–​5.

. Answer: B
Tacrolimus and cyclosporine are both immunosuppressive agents that inhibit calcineurin, which then
subsequently inactivates nuclear factor of activated T cells (NFAT) and inhibits IL-​2. Tacrolimus, or
also known as fujimycin or FK506, is used mainly after allogenic transplants to decrease the risk of
organ rejection. It was first described in 987 from the fermentation broth of a Japanese soil that
contained the bacterium Streptomyces tsukubaensis. The main enzyme responsible for its metabolism
is CYP3A5. Tacrolimus is 50–​00 times more potent than cyclosporine and has been shown to
be effective in the treatment of immune-​mediated diseases such as corneal graft rejection, ocular
inflammation, ocular pemphigoid, and uveitis.
PHARMGKB. Tacrolimus/​Cyclosporine Pathway, Pharmacodynamics. Available at: https://​www.pharmgkb.
org/​pathway/​PA65985892
128 Pharmacology, Therapeutics, and Investigations | ANSWERS

2. Answer: C
Topical ciclosprorin drops (Ikervis) are indicated for severe keratitis in adult patients with dry
eye disease that has not improved despite treatment with tear substitutes. The main adverse
events with ciclosporin eye drops appear to be related to ocular discomfort when administering
the medicine. Pooled data from SANSIKA, SICCANOVE, and two phase II studies indicate that
the most common ocular adverse events considered by the investigator as possibly related to
ciclosporin were: instillation site pain (6%), instillation site irritation (9%), eye irritation (8.8%),
eye pain (3.5%), instillation site lacrimation (2.9%), lacrimation increased (2.%), instillation site
erythema (.9%), ocular hyperaemia (.9%), conjunctival hyperaemia (.7%), erythema of eyelid
(.7%), eyelid oedema (.3%), and blurred vision (.2%).
Scottish Medicines Consortium. Cyclosporin (Ikervis). SMC No. (089/​5). Available at: https://​www.
scottishmedicines.org.uk/​medicines-​advice/​ciclosporin-​ikervis-​fullsubmission-​0895/​

3. Answer: B
Vismodegib is an approved medication for locally advanced or metastatic BCC. It works
by inhibiting the Hedgehog pathway, which is important for cell growth and differentiation
during embryogenesis. A dysregulated Hedgehog signalling pathway has been implicated in the
pathogenesis of BCC. Mitomycin-​C is an immunosuppressant commonly used in glaucoma surgery
and its primary mechanism of action is alkylation of DNA. Tacrolimus and cyclosporine inhibit
interleukin-​2, whereas infliximab and adalimumab are the common anti-​TNF-​alpha treatment.
Aditya S, Rattan A. Vismodegib: a smoothened inhibitor for the treatment of advanced basal cell
carcinoma. Indian Dermatol Online J 203;4:365–​68.

4. Answer: D
Macushield is a food supplement containing the antioxidant carotenoids, zeaxanthin, and meso-​
zeaxanthin. Meso-​zeaxanthin is extracted from marigolds. It is a xanthophyll carotenoid and
contains three stereoisomers of zeaxanthin. In 203, the Age-​Related Eye Disease Study 2
(AREDS2) reported a reduced risk of visual loss and a reduced risk of disease progression in
patients with non-​advanced age-​related macular degeneration. However, the AREDS2 preparation
did not contain meso-​zeaxanthin, which is the dominant carotenoid at the very centre of the
macula, and the presence of which is essential for maximum collective antioxidant effect.
Nolan JM, et al. What is meso-​zeaxanthin, and where does it come from? Eye (Lond)
203;27:899–​905.

5. Answer: D
The patient is suffering from presumed ocular histoplasmosis syndrome (POHS) and has the
classical triad of yellow spots, a macular CNVM, and atrophy/​scarring adjacent to the optic disc.
In addition, linear rows of histo-​spots might be visible in the peripheral fundus. POHS is a type of
multifocal chorioretinitis caused by Histoplasma capsulatum and is endemic in Ohio and Mississippi
river valleys. It is likened to HLA DRw2 and B7. There is a common misconception that the
condition is treated with oral antifungal treatment, which is not the case. For a well-​defined lesion,
the patient can be offered focal or photodynamic therapy laser. Surgical removal of the lesion could
be considered if the lesion remains too central. In addition, antivascular endothelial growth factor
(VEGF) treatment and intravitreal triamcinolone (4 mg) steroids have also been tried with varying
degrees of success.
Iyengar SS, Dyer DS. Diagnosing and treating histoplasmosis. 209. Available at: https://​www.aao.org/​
eyenet/​article/​diagnosing-​treating-​histoplasmosis
Pharmacology, Therapeutics, and Investigations | ANSWERS 129

6. Answer: C
EGFR inhibitors are used for the treatment of many solid tumours, including non-​small cell bronchial
carcinoma, pancreatic carcinoma, colorectal carcinoma, and BCC. EGFR is one of the key receptors
in wound healing of the cornea. Epidermal growth factor (EGF) stimulates proliferation of the
epithelial cells of the meibomian glands in the eyelids. If the effect of EGF is inhibited, corneal
wound healing is delayed and meibomian glands become inflamed.
Hager T, Seitz B. Ocular side effects of biological agents in oncology: what should the clinician be
aware of? Onco Targets Ther 204;7:69–​77.

7. Answer: A
Dapsone is an antibiotic which also possess anti-​inflammatory and immunosuppressive properties.
It has been shown to be effective in treating mild ocular cicatricial pemphigoid. However, it should
not be used in patients with G6PD deficiency as it increases the risk of haemolysis. In addition,
systemic treatment needs to be escalated to either azathioprine, methotrexate, mycophenolate, or
cyclophosphamide if the condition is not adequately controlled with dapsone within 3 months.
Neff AG, et al. Treatment strategies in mucous membrane pemphigoid. Ther Clin Risk Manag
2008;4:67–​26.

8. Answer: D
Imiquimod is an approved topical medication for treating superficial BCC. It acts as a toll-​like
receptor 7 agonist and it modifies the immune response through upregulation of the cytokines.
While surgery is the gold standard for treating BCC, imiquimod has been shown to be a relatively
effective, less invasive, and cheaper treatment option for superficial BCC.
Kamath P, et al. A review on imiquimod therapy and discussion on optimal management of basal cell
carcinomas. Clin Drug Investig 208;38:883–​9.

9. Answer: C
All these conditions are associated with serous retinal detachments, except macular telangiectasia.
The image shown in Figure 4. is a case of central serous chorioretinopathy (CSCR). Candidates for
the Fellowship of the Royal College of Ophthalmologists (FRCOphth) Part 2 exam should be aware
of the differential diagnosis for serous retinal detachments at the macula. These include:
• Vascular: Age-​related macular degeneration (including idiopathic polypoidal choroidal
vasculopathy), retinal macroaneurysm
• Idiopathic—​CSCR
• Ischaemia (choroidal hypoperfusion): Pre-​eclampsia, malignant hypertension
• Inflammation: Harada’s disease
• Neoplasia: Choroidal tumours
Hikichi T, et al. Causes of macular serous retinal detachment in Japanese patients 40 years and older.
Retina 2009;29:395–​404.

20. Answer: D
The diseases listed are all differential diagnoses for serous retinal detachment. VKH is a bilateral
granulomatous uveitis disorder associated with serous detachments, vitritis, and disc oedema.
VKH presents in four different phases: prodrome, acute, convalescent, and recurrent. Associated
systemic manifestations include headache, meningism, tinnitus, poliosis, and vitiligo. The FFA findings
described are highly-​characteristic of this condition. Ultrasound B-​scan is useful for differentiating
130 Pharmacology, Therapeutics, and Investigations | ANSWERS

from posterior scleritis as there will be absence of T-​sign, which is due to sub-​Tenon’s fluid.
Lymphoma is a masquerade disease but the cerebrospinal fluid (CSF) findings and systemic features
exclude this answer. Sympathetic ophthalmia is also a bilateral granulomatous disorder but is not
associated with systemic disease and a history of trauma or intraocular surgery/​procedure is
necessary for diagnosis.
O’Keefe GA, Rao NA. Vogt-​Koyanagi-​Harada disease. Surv Ophthalmol 207;62:–​25.

2. Answer: A
The FFA features of CNVM are important, including features differentiating between different
subtypes. Classic CNVMs are described as early bright or lacy hyperfluorescence followed by late
leakage. Predominantly classic lesions are where classic choroidal neovascularization (CNV) forms
at least 50% of the lesion and minimally classic where this is less than 50%.
Occult membranes are classified as fibrovascular pigment epithelial detachments (PED) or late
leakage of undetermined origin. The former shows stippled or irregular hyperfluorescence followed
by leakage in the later stages of the angiogram. Classic membranes are located in the subretinal
space and are type 2 membranes, whereas occult membranes are subretinal pigment epithelium and
termed type  membranes.
Arias L, Mones J. Fluorescein angiography. Last revision October 20. Available at: http://​www.
amdbook.org/​content/​fluorescein-​angiography-​0

22. Answer: B
NICE published UK guidelines for the diagnosis and management of active and latent tuberculosis
in 206. Testing for tuberculosis involves Mantoux skin testing (if available) and gamma-​interferon
assay ((G-​IFN) Quantiferon®) blood test. G-​IFN is a quantitative analysis of G-​IFN release by T
cells on exposure to M. tuberculosis antigen. Mantoux test is a subcutaneous injection of PPD of
M. tuberculosis and the degree of reaction is quantified in mm of reaction at 48–​72 hours. G-​IFN
is not influenced by prior BCG, whereas Mantoux testing is and therefore must be interpreted
according to history of previous BCG. G-​IFN is NOT useful for distinguishing between active and
latent disease; it indicates prior exposure and not activity. G-​IFN therefore remains positive after a
full course of antituberculous therapy.
National Institute for Health and Care Excellence (NICE). Tuberculosis. NICE Guideline [NG33]. January
206. Available at: https://​www.nice.org.uk/​guidance/​ng33

23. Answer: B
FFA progresses through stages: (a) choroidal flush; (b) arterial; (c) arteriovenous; (c) venous; and
(d) late re-​circulation. The time from injection in the arm to appearance in the central retinal artery
(arm–​eye time) is 0–​2 seconds, –​2 seconds after the initial choroidal flush. A delay in the arm–​
eye time can reflect cardiovascular disease, such as carotid disease and is seen in ocular ischaemic
syndrome. The venous stage is maximal at 30 seconds.
American Academy of Ophthalmology. Fluorescein angiography. 209. Available at: https://​eyewiki.aao.
org/​Fluorescein_​Angiography

24. Answer: A
The OCT appearance in Figure 4.2 is consistent with hydroxychloroquine maculopathy. It demonstrates
the ‘flying saucer’ sign. An ovoid appearance of the central fovea created by preservation of central
foveal outer retinal structures surrounded by perifoveal loss of the photoreceptor inner segment/​outer
segment (IS/​OS) junction, and perifoveal outer retinal thinning. The RCOphth had recently released
Pharmacology, Therapeutics, and Investigations | ANSWERS 131

the screening guidelines for hydroxychloroquine retinopathy. It is also important that candidates for the
FRCOphth Part 2 examination are similar with causes of drug-​induced maculopathy.
American Academy of Ophthalmology. Talk: drug-​induced maculopathy. 208. Available at: http://​
eyewiki.aao.org/​Talk%3ADrug_​induced_​maculopathy
Royal College of Ophthalmologists. Clinical Guidelines: Hydroxychloroquine and Chloroquine
Retinopathy: Recommendations. February 208. Available at: https://​www.rcophth.ac.uk/​wp-​content/​
uploads/​208/​03/​Hydroxychloroquine-​and-​Chloroquine-​Retinopathy-​Screening-​Guideline-​and-​
Recommendations.pdf

25. Answer: B
Following is a summary table of culture media for various microorganisms (Table 4.2).
Microbeonline. Bacterial Culture Media: classification, types and uses. 200. Available at: https://​
microbeonline.com/​types-​of-​bacteriological-​culture-​medium/​

Table 4.2  Culture media and their corresponding microorganisms


Culture media Intended microorganisms
Blood agar Most bacteria and fungi
Brain heart infusion Streptococci and meningococci
Chocolate agar Haemophilus influenza, Neisseria spp., and Moraxella spp.
Cooked meat broth Anaerobic and fastidious bacteria
Non-​nutrient agar with E. coli overlay Acanthamoeba spp.
Sabouraud dextrose agar Fungi

26. Answer: B
Higher frequency and shorter wavelength are usually associated with higher resolution of images
but poorer penetration. For example, ultrasound biomicroscopy (UBM) for anterior segment
imaging uses ultrasound frequencies in the 50–​00 MHz range, whereas the commonly used
diagnostic B-​scan uses frequencies of 8–​0 MHz and orbital ultrasound uses 4–​5 MHz. A-​scan can
be used to measure the axial length.
Kendall CJ, et al. Diagnostic ophthalmic ultrasound for radiologists. Neuroimaging Clin N Am
205;25:327–​65.

27. Answer: C
Fundus autofluorescence (FAF) is a novel imaging technique to examine the distribution of lipofuscin
deposition in the retina. Hyper-​FAF indicates dysfunctional retinal pigment epithelium. In Figure 4.3
the optic disc shows hypo-​FAF and the deposit in adult vitelliform, which is usually rich in lipofuscin,
hyperfluoresces on atrial fibrillation (AF). In red free imaging, the disc appears white and not dark.
Schmitz-​Valckenberg S, et al. Fundus autofluorescence imaging: review and perspectives. Retina
2008;28:385–​409.

28. Answer: B
Safety data on the use of indocyanine green (ICG) in pregnancy is scarce and generally considered a
relative contraindication. ICG contains 5% iodine hence it is contraindicated in patients with iodine
allergy. It has an excitation peak at 80 nm and emission of 830 nm. The longer wavelength light
132 Pharmacology, Therapeutics, and Investigations | ANSWERS

enhances depth penetration, especially in cases of retinal haemorrhage. Side effects of ICG include
nausea/​vomiting, sneezing, pruritus, staining of stool, backache, syncope, and anaphylaxis.
Lim JI. Recent developments in indocyanine green angiography. Curr Opin Ophthalmol 996;7:46–​50.

29. Answer: B
Anterior segment OCT uses a longer wavelength light compared to posterior segment OCT
(30 nm vs. 800 nm). UBM allows better visualization of the lens due to greater depth
penetration. It is superior in assessment of the cornea, including post-​graft position, post-​LASIK
graft thickness, and thickness of anterior stromal scars.
Konstantopoulos A, et al. Recent advances in ophthalmic anterior segment imaging: a new era for
ophthalmic diagnosis? Br J Ophthalmol 2007;9:55–​7.

30. Answer: A
Ultrasound travels slower in silicone oil compared to vitreous, hence the axial length can be
artefactually longer if the correct setting is not used. In contrast, ultrasound travels faster with
asteroid hyalosis and the axial length can be measured shorter than the true axial length. B-​scan
rather than A-​scan is used to locate intraocular foreign body and detect calcification on optic disc
drusen. It is also important to remember that, unlike optical biometry (e.g. IOLMaster) which uses
partial coherence intereferometry, A-​scan or acoustic biometry measure the axial length from inner
limiting membrane and not retinal pigment epithelium.
Silverman RH. Focused ultrasound in ophthalmology. Clin Ophthalmol 206;0:865–​75.

3. Answer: C
VEP records gross electrical response from the visual cortex in response to a changing visual
stimulus such as multiple flash or changing pattern stimuli. It requires relatively normal retinal/​
macular function to be a reliable test. It is often used to detect subclinical optic nerve demyelination,
chiasmal and retrochiasmal dysfunction, and non-​organic visual loss. Pattern reversal VEP provides
more information than flash VEP but flash VEP is useful in poorly cooperative patients.
Creel D. Visually evoked potentials (202). In: Kolb H, et al. (eds). Webvision: The Organization of the
Retina and Visual System [Internet]. Salt Lake City, UT: University of Utah Health Sciences Center, 995.
Available at: https://​www.ncbi.nlm.nih.gov/​books/​NBK0728/​

32. Answer: D
Various anterior segment imaging systems are currently used in clinical practice. Orbscan utilizes
the principle of slit-​scanning and Placido disc technology, whereas Pentacam uses a single rotating
Scheimpflug camera and a static camera in combination with a monochromatic slit-​light source
around the optical axis to obtain multiple slit images.
Oliveira CM, et al. Corneal imaging with slit-​scanning and Scheimpflug imaging techniques. Clin Exp
Optom 20;94:33–​42.

33. Answer: C
In Hess tests, both eyes are dissociated using lenses of different colours. In the Lees screen test,
the eyes are dissociated using two opalescent glass screens at right angles to each other, bisected
by a two-​sided plane mirror. A Hess screen test requires a decent visual acuity and normal colour
vision of each eye to be useful. It also requires a normal retinal correspondence since the results
will be inaccurate if the patient cannot superimpose two macular images. The fovea of each eye
Pharmacology, Therapeutics, and Investigations | ANSWERS 133

should have a common visual direction or else the deviation that shall be mapped out will not be
the right one.
The success of the Lees screen test 50 years ago is due to the fact that this method did not need
the complementary red-​green colours—​hence normal colour vision is not necessary in this test.
The Lees screen test, such as the red-​green screen tests, requires a decent visual acuity, normal
retinal correspondence, and adequate dimming of the room. There should not be a large visual field
defect.
Timms C. Lees screen test. Am Orthopt J 2006;56:80–​3.

34. Answer: D
PAT is effective in predicting postoperative sensory and motor fusion in patients with acquired
esotropia. During PAT, the patients are given prisms of adequate power to re-​align the visual axis.
This sometimes stimulates the restoration of some form of binocular vision. Other patients may
also benefit from prism adaptation. The angle change is generally smaller after diagnostic occlusion
of one eye than after prism adaptation. Preoperative prism correction allows a more accurate
determination of the amount of surgery, prevents the risk of undue overcorrection, and promotes
the development of binocularity. The increase in the squint angle after prism adaptation is caused by
an anomalous sensorial relationship between the two eyes. Surgery tailored to the squint angle after
prism adaptation seems advisable in patients with normosensoric esotropia.
Kiyak Yilmaz A, et al. The impact of prism adaptation test on surgical outcomes in patients with
primary exotropia. Clin Exp Optom 205;98:224–​7.

35. Answer: B
Torsion can be measured by various orthoptic tests, including Hess chart, double Maddox rod,
Maddox wing, Bagolini glasses, and synaptophore. Maddox rod only measures latent/​manifest
horizontal and vertical deviation. Maddox wing measures heterophoria at near fixation and allows
the measurement of horizontal, vertical, and cyclo-​deviation.
Guyton DL. Clinical assessment of ocular torsion. Am Orthop J 983;33:7–​5.

36. Answer: C
AC/​A ratio is the measurement of the convergence induced by accommodation for every dioptre
of accommodation. The normal AC/​A ratio is around 3–​5 degree/​dioptre. It is important in the
diagnosis and/​or treatment of some types of strabismus, including IDEX. In simulated IDEX, the
AC/​A ratio is usually high, masking the near exotropia. This can be unmasked by a patch test or
a +3.0 D lens. It can be measured either gradient or heterophoria method. Gradient method is
measured using a minus or a plus lens and the change in phoria with the additional lens compared
to without the lens yields the AC/​A ratio. The heterophoria method is calculated by the following
formula:
AC/A=IPD+near fixation distance in metres × (near phoria–distance phoria)
Wybar K. Relevance of the AC/​A ratio. Br J Ophthalmol 974;58:248–​54.

37. Answer: B
Both CT and MR angiogram have been shown to be good in detecting intracranial aneurysms.
However, CTA is more sensitive than MRA in detecting small unruptured aneurysms.
Numminen J, et al. Detection of unruptured cerebral artery aneurysms by MRA at 3.0
tesla: comparison with multislice helical computed tomographic angiography. Acta Radiol
20;52:670–​4.
134 Pharmacology, Therapeutics, and Investigations | ANSWERS

38. Answer: D
This is a clinical scenario suggestive of pituitary adenoma. Lesions of the chiasm are usually slow
growing and therefore cause chronic progressive visual loss. Many patents do not describe definite
visual field loss but a gradual reduction in bilateral visual function. Bitemporal hemianopia is most
commonly caused by a pituitary adenoma but there are many causes of this visual field defect
including craniopharyngioma, meningiomas, and a suprasellar aneurysm of the carotid artery.
Patients who are found to have this field defect need imaging sequences, most commonly MRI
with contrast, as this will show a hypodense region in T imaging and they are solid tumours with
homogenous enhancement with contrast on T2. There is occasionally a cystic component. They
most commonly compress the inferior chiasm, so the superior visual field on the temporal side will
be affected first before progressing to bitemporal hemianopia.
Lucas JW, Zada G. Imaging of the pituitary and parasellar region. Semin Neurol 202;32:320–​3.

39. Answer: C
The coronal MRI scan shown in Figure 4.4 demonstrates the absence of septum pellucidum, which
is a diagnostic feature of septo-​optic dysplasia (SOD), or also known as De Morsier’s syndrome.
SOD is a neurological disorder characterized by a classic triad of optic nerve hypoplasia, pituitary
hormone abnormalities, and midline brain defect, including agenesis of septum pellucidum and/​or
corpus callosum. The diagnosis is made when there is present of two features. Around 60% of the
patients have hypopituitarism and absent septum pellucidum. Significant visual impairment is found
in around 20–​30% of the patients. The affected child may present with strabismus, nystagmus, or
other visual abnormalities. Multidisciplinary teamwork, involving ophthalmologist, endocrinologist,
paediatricians, and neurodevelopmental team, is essential in the management of these patients.
Webb EA, Dattani MT. Septo-​optic dysplasia. Eur J Hum Genet 200;8:393–​7.

40. Answer: C
MRI FLAIR sequence is similar to a T2-​weighted image but the former has the ability to suppress
signals from cerebrospinal fluid (CSF), highlighting periventricular hyperintense lesions such as
multiple sclerosis-​associated plaques. The following references provide good neuroimaging pictures
demonstrating the distinguishing features of MRI T, T2, and FLAIR sequences.
Rovira A, et al. Recommendations for using and interpreting magnetic resonance imaging in multiple
sclerosis. Neurologia 200;25:248–​65.
Trip SA, Miller DH. Imaging in multiple sclerosis. J Neurol Neurosurg Psychiatry 2005;76:iii–​8.

4. Answer: A
Most patients that have developed a new onset Horner’s, in particular a painful Horner’s, must
have imaging performed. The course of the sympathetic fibres supplying the iris dilator muscle
is very long and therefore any interruption of the sympathetic pathway could cause a Horner’s
lesion. Therefore, the imaging protocol must reflect the course of the pathway. In this case the
likely diagnosis is a carotid artery dissection and the imaging of choice would be a study to evaluate
the entire sympathetic pathway of head, neck, and upper chest to a level of T2 with contrast.
This is followed by an MRA of the head and neck. This extensive imaging is time-​consuming in the
emergency setting and in this patient CT and CTA is the best option.
Gao Z, Crompton JL. Horner syndrome: a practical approach to investigation and management. Asia
Pac J Ophthalmol (Phila) 202;:75–​9.
Pharmacology, Therapeutics, and Investigations | ANSWERS 135

42. Answer: D
DSG is a useful test for diagnosing functional epiphora. The patient is instructed to sit upright during
the procedure while a drop of radioactive tracer isotope (usually technetium-​99m) is administered
to the inferior fornix of both eyes. The patient then blinks normally and images are taken at regular
intervals. It is useful for proximal obstruction and distal obstruction of nasolacrimal drainage system.
In contrast dacryocystogram requires the injection of a radio-​opaque contrast via the puncta, which
may mask the proximal obstruction. The patient is instructed to lie supine during the procedure.
Peter NM, Pearson AR. Comparison of dacryocystography and lacrimal scintigraphy in the
investigation of epiphora in patients with patent but nonfunctioning lacrimal systems. Ophthalmic Plast
Reconstr Surg 2009;25:20–​5.
chapter BASIC SCIENCE AND MISCELLANEOUS

5 QUESTIONS

. Bruch’s membrane is a:
A. Bilaminar structure
B. Trilaminar structure
C. Pentalaminar structure
D. Hexalaminar structure

2. Which of the following statements about the superior orbital fissure


(SOF) is true?
. It communicates with the cavernous sinus
A
B. It transmits the ophthalmic artery
C. The oculomotor nerve travels via the SOF outside the tendinous ring
D. The trochlear nerve travels via the SOF within the tendinous ring

3. Which of the following muscles is supplied by the contralateral


oculomotor nucleus?
A. Inferior  rectus
B. Inferior oblique
C. Medial  rectus
D. Superior  rectus

4. Which of the following statement regarding the lens is true?


. Contains an anterior Y-​shaped suture and a posterior inverted Y-​shaped suture
A
B. Has a thinner anterior lens capsule than the posterior capsule
C. The anterior-​posterior diameter of lens does not change with age
D. Contains lens epithelium at the posterior but not the anterior lens capsule

5. All of the following are pathologic characteristics of retinoblastoma,


EXCEPT:
A. Calcification
B. Fleurettes
C. Flexner–​Wintersteiner rosettes
D. Psammoma  bodies
138 Basic Science and Miscellaneous | QUESTIONS

  6. Which glucose receptor is found in adipose tissues and striated muscle


(skeletal muscle and cardiac muscle)?
A. Glut  
B. Glut  2
C. Glut  3
D. Glut  4

  7. Which of the following structures derives from neural crest cells?


A. Conjunctival epithelium
B. Lacrimal  glands
C. Retinal pigment epithelium
D. Trabecular meshwork

  8. Which of the following phakomatoses conditions has a different


inheritance mode among the others?
A. Neurofibromatosis-​  (NF-​)
B. Sturge–​Weber syndrome
C. Tuberous sclerosis
D. Von Hippel–​Lindau (VHL) syndrome

  9. The retinoblastoma protein (protein name abbreviated pRb; gene name


abbreviated RB or RB) is a tumour suppressor protein. It is located on:
A. Chromosome  3
B. Chromosome  4
C. Chromosome  5
D. Chromosome  6

0. The following genes have been associated with primary open angle
glaucoma, EXCEPT:
A. LAMA
B. MYOC
C. OPTN
D. WDR36

. Following is a family tree (Figure 5.) with a Disease A running in the
family. What is the most likely inheritance pattern of Disease A?
A. Autosomal dominant
B. Autosomal recessive
C. Mitochondrial
D. X-​linked recessive
Basic Science and Miscellaneous | QUESTIONS 139

Patient 1

Figure 5.  

2. Which of the following has a similar spherical equivalent as –​2.00/​


+3.50 × 90?
A.  –​.00/​+4.50  ×  90
B.  –​3.00/​+2.50  ×  80
C. +.00/​–​2.50  ×  90
D. +2.00/​–​3.50  ×  80

3. A glaucoma consultant asks you to analyse his data. The excel
spreadsheet contains 20 observations of intraocular pressure (IOP); 60
of which were made after using drug A and 60 of which were made after
using drug B. This is an example of:
. Unit of analysis issue
A
B. Unilateral analysis bias
C. Bilateral analysis
D. Bilateral  bias

4. Ten trainees are operating in ten different theatres during the week.
The number of surgeries performed by each trainee is different.
Following is the number of cases performed by each trainee:

, 2, 3, 3, 3, 4, 4, 7, 8, 0

Based on the information given, what is the mean, median, mode, and
skew in terms of the number of cases being performed by the trainees?
. 4.5, 3, 3.5, positive skew
A
B. 4.5, 3, 4, negative skew
C. 4.5, 3.5, 3, positive skew
D. 4.5, 4, 3, negative skew
140 Basic Science and Miscellaneous | QUESTIONS

5. Incorrect rejection of the null hypothesis. This is an example of:


. Type  error
A
B. Type 2 error
C. Type 3 error
D. Type 4 error

6. Which of the following is true about the incidence and progression
of age-​related macular degeneration (AMD) in the Blue Mountains
Eye Study?
. Male sex was independently associated with early AMD incidence
A
B. The presence of complement factor D was independently associated with early AMD
incidence
C. Current smoking was associated with early AMD incidence
D. Fish consumption was inversely associated with late AMD incidence

7. When there is no information about a study population, which of the


following statistical tests can be employed to test differences between
two groups?
A. Wilcoxon signed-​rank  test
B. Paired sample  T-​test
C. One-​way analysis of variance (ANOVA)
D. Independent sample  T-​test

8. A 50-​year-​old type 2 diabetic attends the eye clinic and asks about
cholesterol management and eye disease. Which combination is
evidence based?
. Simvastatin 40 mg + atorvastatin 80 mg
A
B. Simvastatin 40 mg + ezetimibe 0 mg
C. Simvastatin 40 mg + fenofibrate 60 mg
D. Simvastatin 40 mg + pravastatin 40 mg

9. Which of the following is correct about the Collaborative Ocular


Melanoma Study (COMS)?
A. 0% of the small sized choroidal melanoma demonstrated evidence of tumour growth at
5 years
B. Brachytherapy was as effective as enucleation for medium sized choroidal melanoma in
terms of tumour-​related and all-​cause mortality at 5 years
C. Addition of preoperative external beam radiotherapy to enucleation significantly reduced
the tumour-​related mortality at 5 years compared to enucleation alone
D. Younger age was shown to be a poor prognostic factors for mortality
Basic Science and Miscellaneous | QUESTIONS 141

20. Which of the following about Herpetic Eye Disease Study (HEDS) is
correct?
. Steroid treatment led to worse outcomes in patients with stromal keratitis
A
B. There was no apparent benefit in the addition of oral aciclovir to the treatment regime
of topical corticosteroids and topical antiviral treatment in patients with acute stromal
keratitis
C. There was no benefit in adding oral aciclovir to the treatment of herpes simplex virus
(HSV) iridocyclitis in patients receiving topical corticosteroids and trifluridine prophylaxis
D. Psychological stress appears to be a trigger of recurrences of ocular HSV disease

2. Based on the findings of Optic Neuritis Treatment Trial (ONTT), which
is the best treatment option for acute optic neuritis?
. 3 days of IV methylprednisolone 250 mg QID alone
A
B. 3 days of IV methylprednisolone 250 mg QID then  days of oral prednisolone  mg/​kg
C. 7 days of oral methylprednisolone 500 mg
D. 4 days of oral prednisolone  mg/​kg

22. According to the North America Symptomatic Carotid Endarterectomy


(NASCET) study, which of the following regarding carotid
endarterectomy (CEA) is correct?
A. CEA has been shown to significantly reduce the risk of stroke in symptomatic patients with
mild (<50%) to severe (70–​99%) carotid stenosis
B. CEA is recommended in patients with asymptomatic severe carotid stenosis
C. In patients with severe carotid stenosis, the risk of ipsilateral stroke at 2 years after
combined CEA and aspirin is 0%
D. There is a twofold increase risk of perioperative severe stroke or death in patients who
have CEA compared to medical treatment only

23. Which of following nutritional supplements was NOT found to be


beneficial in reducing the risk of developing advanced AMD?
A. Copper
B. Lutein
C. Omega-​3
D. Vitamin  C

24. A 5-​year-​old boy has asked for surgery to correct a cosmetic squint. His
parents refuse to give their consent. He is deemed Gillick competent.
Can he have the surgery?
. Gillick competence requires one parent to consent if both alive
A
B. Gillick competence requires the child to be older than 6 years
C. The Fraser clause overrules consent in cosmetic procedures
D. He can have the surgery
142 Basic Science and Miscellaneous | QUESTIONS

25. A Lithuanian taxi driver who speaks no English attends the eye clinic
with cataract. He does not understand any English and his friend
translates in broken English. He wants to have the cataract surgery
but is anxious about procedure. Which consent form would be most
appropriate for cataract surgery with sedation?
A. Consent  
B. Consent  2
C. Consent  3
D. Consent  4

26. The four principles used as a common framework for the analysis of
medical ethics are:
. Autonomy, beneficence, justice, and non-​maleficence
A
B. Autonomy, beneficence, capacity, and non-​maleficence
C. Autonomy, beneficence, justice, and maleficence
D. Autonomy, beneficence, capacity, and maleficence

27. A female patient comes to your clinic wearing revealing clothes. She
then comes up very close to you and starts asking personal questions in
a seductive tone. What would be the appropriate response?
. Continue to examine her with the door open
A
B. Refer her to another doctor
C. Call in a nurse as a chaperon
D. Refuse to examine her

28. A 40-​year-​old patient attends the glaucoma clinic for the first time.
Which of the following is most likely to be TRUE regarding the
implications of visual field loss on his current licence?
A. If he holds a group  licence, he needs to inform the Driver and Vehicle Licensing Agency
(DVLA) if his horizontal field does not extend to 60°, with extension of at least 70° to left
and right
B. If he holds a group  licence, he needs to inform the DVLA if he has blepharospasm
C. If he holds a group 2 licence, he needs to inform the DVLA only when there is a defect
within 20° of fixation above or below the horizontal meridian
D. If he holds a group 2 licence he needs to inform the DVLA if he has any defect in the
central 30° in either eye
Basic Science and Miscellaneous | QUESTIONS 143

29. A 70-​year-​old patient presents to the eye casualty with a transient


episode history of decreased vision, described as a curtain coming
down. The symptom has fully resolved by the time he arrives at the
eye casualty. His past medical history includes hypertension and type 2
diabetes. The presenting vision is 6/​6 in both eyes and confrontational
visual field is full in both eyes. Slit-​lamp examination shows mild
non-​proliferative diabetic retinopathy and silver wiring of the retinal
arterioles. What advice should be given in terms of driving?
A. The patient can continue to drive but stop if notices another similar episode of visual
disturbance
B. The patient should stop driving for at least  week
C. The patient should stop driving for at least 2 weeks
D. The patient should stop driving for at least  month

30. Which of the following analytic methods refers to analysis of all the costs
and consequences of an intervention in monetary term?
A. Cost-​benefit analysis
B. Cost-​effectiveness analysis
C. Cost-​minimization analysis
D. Cost-​utility analysis

3. There have been a number of cases of bacterial endophthalmitis in your


unit over the past month. Which of the following statements should
raise particular concern?
. An incidence of  .0%
A
B. Each case has arisen from a different theatre list
C. Microbiological analysis indicating a different organism in each case
D. The finding that each case has been operated on by a different member of staff

32. The majority of vigabatrin related defects:


. Extend to within 5◦ of fixation
A
B. Extend to within 0◦ of fixation
C. Extend to within 20◦ of fixation
D. Extend to within 30◦ of fixation

33. Which of the following is true about the NHS Diabetic Eye Screening
Programme (NDESP)?
A. All patients with treated proliferative diabetic retinopathy will need to be reviewed at least
once a year in the hospital eye service
B. 0% of the patients with negative primary grading results need to undergo internal quality
assurance process via secondary grading
C. The screening service may screen diabetic patients up to every 6-​monthly
D. Digital surveillance could only provide annual screening
144 Basic Science and Miscellaneous | QUESTIONS

34. A 65-​year old type 2 insulin-​dependent diabetic of 0 years was found


to have reduced vision by his optician. On examination, his visual acuity
was 6/​5. There were multiple areas of microaneurysm on the macula
and an area of hard exudates 500 microns from the fovea. Optical
coherence tomography showed a central retinal thickness (CRT) of
402 mm. He has no other past ocular history otherwise. He runs his
own company and tells you that he is a very busy man. What is the most
appropriate management plan?
. Offer the patient focal laser to the area of hard exudates
A
B. Offer the patient intravitreal aflibercept every month for 3 months, then 2 monthly
C. Offer the patient intravitreal aflibercept every month for 5 months, then 2 monthly
D. Offer the patient illuvien injection

35. Based on the Royal College of Ophthalmologists’ (RCOphth) guideline


on instrument decontamination, which of the following is true?
. 70% alcohol wipes have been shown to be sufficient to disinfect diagnostic contact lenses
A
B. Diagnostic contact lenses should be soaked in % sodium hypochlorite solution for 0
minutes between patients
C. Tonometer prisms should be soaked in % sodium hypochlorite solution for 5 minutes
between patients
D. Single-​use instruments have bene shown to be cost-​effective in reducing the possible
transmission risk of Creutzfeldt–​Jakob disease (CJD)

36. Which of the following options regarding the UK eye retrieval and eye
bank services is correct?
. The death-​to-​retrieval time should be 8 hours or less
A
B. The use of demented patients’ ocular tissues for transplantation is considered a
contraindication in most cases
C. Hypothermic storage is the common method used in UK eye bank due to the advantage of
longer storage time
D. The endothelial cell count needs to be more than 2500 cells/​mm2 to be considered
suitable for penetrating/​endothelial keratoplasty
chapter BASIC SCIENCE AND MISCELLANEOUS

5 ANSWERS

. Answer: C
Bruch’s membrane (BM) is a pentalaminar structure, which is located between the retinal
pigment epithelium (RPE) and the fenestrated choroidal capillaries. From inside to outside, it is
composed of basement membrane of RPE, inner collagenous zone, central band of elastic fibres,
outer collagenous zone, and basement membrane of choriocapillaris. BM is a dynamic structure
involved actively or passively in diseases such as the onset and progression of diseases like
retinitis pigmentosa, AMD, pseudoxanthoma elasticum, Sorsby’s fundus dystrophy, and Malattia
Leventinese.
Booij JC, et al. The dynamic nature of Bruch’s membrane. Prog Retin Eye Res 200;29:–​8.

2. Answer: A
The SOF is bordered between greater and lesser wings of the sphenoid bone and it connects
to cavernous sinus. The mnemonic ‘SINA-​LFTs’ can be used to remember the structures that
transmit through SOF within or outside the tendinous ring. The optic canal transmits ophthalmic
artery and optic nerve.
Structures that travel within the tendinous ring include:
Superior and Inferior divisions of III nerve
Nasociliary nerve of  V
Abducens nerve
Structures that travel outside tendinous ring include:
Lacrimal nerve of  V
Frontal nerve of  V
Trochlear nerve
Superior ophthalmic vein.
Shumway CL, Wade M. Anatomy, Head and Neck, Orbit Bones. StatPearls [Internet]. Treasure Island,
FL: StatPearls Publishing, 208. Available at: https://​www.ncbi.nlm.nih.gov/​books/​NBK53490/​

3. Answer: D
Fibres to the inferior oblique, inferior rectus, and medial rectus muscles supply the ipsilateral eye
whereas fibres to the superior rectus muscle decussate and supply the contralateral eye. The
decussating fibres pass through the opposite superior rectus nucleus; thus, damage to the right
oculomotor nucleus might have bilateral superior rectus muscle involvement.
Brazis PW. Localization of lesions of the oculomotor nerve: recent concepts. Mayo Clin Proc
99;66:029–​35.
146 Basic Science and Miscellaneous | ANSWERS

4. Answer: A
As lens fibres grow anteriorly and posteriorly, the ends of the fibres meet and interdigitate with the
ends of fibres arising on the opposite side of the lens, forming a pattern of cell association known
as sutures. At 8 weeks’ gestation, an erect Y-​suture appears anteriorly and an inverted Y-​suture
appears posteriorly. The anterior lens capsule is thicker than the posterior lens capsule. The lens
epithelium is beneath the anterior capsule but not the posterior capsule.
Bassnett S, Sikic H. The lens growth process. Prog Retin Eye Res 207;60:8–​200.

5. Answer: D
There are various pathologic characteristics of retinoblastoma, depending on the extent of retinal
differentiation. These included Homer–​Wright pseudo-​rosettes (neuroblastic differentiation),
Flexner–​Wintersteiner rosettes (early retinal differentiation), and fleurettes (photoreceptor
differentiation). Calcification is also a common feature of retinoblastoma. Psammoma bodies are
associated with meningioma and other tumours, but not retinoblastoma.
Eagle RC Jr. The pathology of ocular cancer. Eye (Lond) 203;27:28–​36.

6. Answer: D
GLUTs are integral membrane proteins that contain 2 membrane-​spanning helices with both the
amino and carboxyl termini exposed on the cytoplasmic side of the plasma membrane. 4 GLUTS
are encoded by the human genome. Glut 4 is the principle glucose transporter involved in regulating
whole body glucose homeostasis.
Stöckli J, et al. GLUT4 exocytosis. J Cell Sci 20;24:447–​59.

7. Answer: D
Following is the summary of the embryologic origin of various ocular structures:
Surface ectoderm: epidermis of the eyelids, conjunctiva, corneal epithelium, lens, and
lacrimal glands
Neuroectoderm: epithelial lining of iris and ciliary body, RPE, and optic nerve
Neural crest cells: corneal stroma, Descemet membrane and endothelium, sclera, iris stroma, ciliary
body stroma and muscles, and trabecular meshwork
Mesoderm: vitreous, choroid, sclera, endothelial lining of ocular blood vessels and extraocular
muscles
Graw J. Eye development. Curr Top Dev Biol 200;90:343–​86.
Williams AL, Bohnsack BL. Neural crest derivatives in ocular development: discerning the eye of the
storm. Birth Defects Res C Embryo Today 205;05:87–​95.

8. Answer: B
Phakomatoses is a group of conditions that affects brain, eye, and skin. These include NF-​, NF-​
2, tuberous sclerosis, and VHL. They all have autosomal dominant inheritance. Sturge–​Weber
syndrome and Wyburn–​Mason syndrome are different from ‘true phakomatoses’ where they
occurred sporadically.
Rosser T. Neurocutaneous disorders. Continuum (Minneap Minn) 208;24:96–​29.

9. Answer: A
The protein is encoded by the RB gene located on chromosome 3 (3q4.-​q4.2). If both
alleles of this gene are mutated early in life, the protein is inactivated and results in development of
Basic Science and Miscellaneous | ANSWERS 147

retinoblastoma cancer, hence the name Rb. Two forms of retinoblastoma were noticed: a bilateral,
familial form, and a unilateral, sporadic form. Sufferers of the former were six times more likely
to develop other types of cancer later in life. This highlights the fact that mutated Rb could be
inherited and supported the two-​hit hypothesis.
Kleinerman RA, et al. Risk of new cancers after radiotherapy in long-​term survivors of
retinoblastoma: an extended follow-​up. J Clin Oncol 205;23: 2272–​9.

0. Answer: A
All the genes have been implicated in primary open angle glaucoma except LAMA, which encodes
one of the alpha  subunits of laminin. It is associated with Poretti-​Boltshauser syndrome, which is
characterized by cerebellar dysplasia, cerebellar cyst, high myopia, variable retinal dystrophy, and
eye movement abnormalities.
Sakurada Y, Mabuchi F. Advances in glaucoma genetics. Prog Brain Res 205;220:07–​26.

. Answer: D
This is a common genetic question presented in a family tree fashion, which should be familiarized
by all candidates. Generally speaking, square refers to male and circle refers to female; fully shaded
shape refers to having the disease, non-​shaded shape refers to no disease and half shaded refers
to carrier of the disease. In this question, there are three generations involved. The grandfather
of patient  has Disease A and only his female children is affected. This disease is not possible to
have an autosomal recessive inheritance pattern because all the children of the grandfather should
be a carrier instead of being fully affected. Similarly autosomal dominant inheritance pattern is
unlikely because the male and female children of the grandfather should have 50% chance of being
affected, though it is still possible for both male children to be unaffected. Mitochondrial inheritance
is not possible in this case because mitochondria is only inheritable exclusively from the mother.
Therefore, none of the children or grandchildren should be affected. Patient  is a carrier and her
mother is a carrier and her brother is affected. Therefore, the condition is an X-​linked recessive
disease because the affected females are all carriers and the males are fully affected.
NHS. Genetic and Genomic Testing. Available at: https://​www.nhs.uk/​conditions/​genetics/​inheritance/​

2. Answer: C
This is a simple question examining the candidates’ knowledge on calculation of the mean refractive
spherical equivalent (MRSE). Following is the equation: MRSE (dioptre) = sphere + cylinder/​2
(regardless of the axis)
The given refraction has a MRSE of –​0.25, which is the same as Option C.
Wilkinson ME. Introduction to Optics and Refractive Errors of the Eye, 205. Available at: https://​webeye.
ophth.uiowa.edu/​eyeforum/​video/​Refraction/​Intro-​Optics-​Refract-​Errors/​index.htm

3. Answer: A
This scenario illustrates what is known as the ‘unit of analysis’ issue. The patient is the sampling unit
and should be the unit of analysis. Multiple observations may be made on patients, but the statistical
analysis must not ignore the fact that these observations are made on individuals. Failure to do so
violates the assumption made by the majority of statistical tests that each data value is independent.
Multiple observations from the same patient falsely inflate your sample size, sometimes dramatically
so, leading to spurious statistical significance.
Altman DG, Bland JM. Statistics notes. Units of analysis. BMJ 997;34:874.
Bunce C, et al. Ophthalmic statistics note : unit of analysis. Br J Ophthalmol 204;98:408–​2.
148 Basic Science and Miscellaneous | ANSWERS

4. Answer: C
The total amount of cases performed is 45 cases. ‘Mean’ refers to the average of the total amount;
in this case is 45 cases/​0 trainees = 4.5. ‘Median’ refers to the middle value of a set of ordered
data; in this case there are ten numbers, so the value will be the average between fifth and sixth,
which is (3 + 4)/​2 = 3.5. Mode refers to the number that occurs most often, so in this case is
3. Positive skew has a longer right tail whereas negative skew has a longer left tail; in this case
there are more lower numbers (compared to the mean value of 4.5) in this 0-​data set, so it has
a positive skew. The other way to interpret is that if the mean value is larger (i.e. on the right side
of the horizontal axis) than the median value, it is positively skewed and vice versa. Refer to the
following useful website for graphical explanation.
[No authors]. A Look at Skewed Distributions. Available at: http://​www.cvgs.k2.va.us/​digstats/​main/​
descriptv/​d_​skewd.html

5. Answer: A
Type I error:
• Incorrect rejection of the true null hypothesis
• Maximum probability is set in advance as alpha
• Is not affected by sample size as it is set in advance
• Increases with the number of tests or end points
Type II error:
• Failure to reject false null hypothesis
• Probability is beta
• Beta depends upon sample size and alpha
• Cannot be estimated except as a function of the true population effect
• Beta gets smaller as the sample size gets larger, the number of tests or end points increases
Banerjee A, et al. Hypothesis testing, type I and type II errors. Ind Psychiatry J 2009;8:27–​3.

6. Answer: D
The Blue Mountains Study is a large population-​based cohort study to assess the 5-​year incidence
and progression of AMD in an older Australian population. The following are the main results
of this study: ‘Age was strongly associated with early and late AMD incidence. Female sex and
the presence of both risk alleles of CFH-​rs0670 or ARMS2-​rs0490924 were independently
associated with early AMD incidence whereas current smoking and presence of ≥ risk allele
of CFH-​rs0670 or ARMS2-​rs0490924 were associated with late AMD incidence. Fish
consumption was inversely associated with late but not early AMD incidence. Severity of early
AMD lesion characteristics was a strong predictor of progression to late AMD.’
Joachim N, et al. The incidence and progression of age-​related macular degeneration over
5 years: the Blue Mountains Eye Study. Ophthalmology 205;22:2482–​9.

7. Answer: A
A statistical test is a formal technique used to reach conclusion concerning the reasonableness
of the hypothesis by relying on the probability distribution. This hypothetical testing related to
differences can be classified into parametric and non-​parametric tests. Parametric tests (sample
T-​tests, one-​way analysis of variance, etc.) are used when there is complete information about
the population whereas non-​parametric tests (Wilcoxon signed-​rank tests, Chi-​square test,
Basic Science and Miscellaneous | ANSWERS 149

Mann–​Whitney, Kruskal–​Wallis test, etc.) are used when information about the population is
unavailable.
Ali Z, Bhaskar SB. Basic statistical tools in research and data analysis. Indian J Anaesth 206;60:662–​9.

8. Answer: C
In the ACCORD Eye study, fenofibrate (60 mg daily) with simvastatin resulted in a 40% reduction
in the odds of retinopathy progressing over 4 years, compared with simvastatin alone. In the
FIELD (Fenofibrate Intervention and Event Lowering in Diabetes study) fenofibrate reduced the
requirements for laser therapy in patients with pre-​existing retinopathy (numbers need to treat
(NNT) to avoid first laser = 7) and prevented disease progression (NNT = 9) in patients with
pre-​existing retinopathy.
ACCORD Study Group; ACCORD Eye Study Group, Chew EY, et al. Effects of medical therapies on
retinopathy progression in type 2 diabetes. N Engl J Med 200;363:233–​44.
Keech AC, et al. Effect of fenofibrate on the need for laser treatment for diabetic retinopathy (FIELD
study): a randomised controlled trial. Lancet 2007;370:687–​97.

9. Answer: B
The Collaborative Ocular Melanoma Study is a three-​arm study, including an observational
study of small choroidal melanomas, a randomized controlled trial comparing the effectiveness
of brachytherapy to enucleation for medium sized choroidal melanomas, and a randomized
controlled trial to examine the additional benefit of preoperative external beam radiotherapy for
large choroidal melanomas. The size of choroidal melanoma was defined as small (.5–​2.4 mm
height and 5–​6 mm diameter), medium (2.5–​0 mm apical height and ≤6 mm diameter)
and large (>0 mm apical height and >6 mm diameter). The primary outcomes measures
were tumour-​related and all-​cause mortality at 5 years. In conclusion brachytherapy serves as
an effective alternative treatment to enucleation for medium sized choroidal melanomas and
preoperative radiotherapy does not improve the survival. Older age, instead of younger age, at
presentation is a poor prognostic factor for mortality. Following is a summary table of the main
findings (Table 5.).

Table 5.  The main findings of the Collaborative Ocular Melanoma Study


Small* Medium size Large size
5-​year tumour-​related mortality % 9% (B) vs. % (E) 28% (E only) vs.
26% (E + B)
5-​year all-​cause mortality 6% 8% (B) vs. 9% (E) 43% (E only) vs.
38% (E + B)
*3% of the small sized choroidal melanomas demonstrated evidence of tumour growth at 5 years.
B, brachytherapy; E, enucleation.
Adapted with permission from Margo, C. E. (2004) ‘The Collaborative Ocular Melanoma Study: An Overview’, Cancer Control, pp.
304–309. doi: 0.77/073274804000504. Copyright © 2004, © SAGE Publications.

20. Answer: B
There are two parts to the HEDS:
Conclusions from HEDS  (consisting of three randomized controlled trials (RCTs)):
. Patients with acute stromal herpes simplex keratitis (HSK) who received prednisolone
phosphate drops had faster resolution and fewer treatment failures (reduced by 68%)
150 Basic Science and Miscellaneous | ANSWERS

2. There was also no apparent benefit in the addition of oral acyclovir of therapeutic dose
(400 mg five times a day) to the treatment regimen of a topical corticosteroid and a topical
antiviral for acute stromal keratitis
3. For patients with HSV iridocyclitis, adding oral acyclovir of therapeutic dose in addition to
topical corticosteroids and trifluridine prophylaxis might be beneficial, thought the sample
size was too small to achieve statistical significance
Conclusions from HEDS 2 (consisting of two RCTs and one observational study):
. There was no benefit from the addition of oral acyclovir to treatment with topical Trifluridine
in preventing the development of stromal keratitis or iritis in patients with epithelial HSK
2. It was found that oral acyclovir (400 mg BD), reduced the probability that any form of
herpes of the eye would return in patients who had the infection in the previous year by
4%, particularly stromal keratitis
3. Psychological stress does not appear to be a trigger of recurrences of ocular HSV disease
Kalezic T, et al. Herpetic eye disease study: lessons learned. Curr Opin Ophthalmol 203;29:340–​6.

2. Answer: B
The ONTT enrolled 457 patients between 988 and 99 in order to study the use of
corticosteroids to treat acute optic neuritis as there was no consensus on the best treatment
regime. As part of the study’s further analysis, the long-​term risk of developing MS was also
analysed, and the patients were followed up for 5 years post recruitment. All patients had an
unenhanced MRI at recruitment and yearly follow-​up scans until 997, then at 0 years and then
5 years follow-​up. The patients were randomized into three groups:
. IVMP 250 mg QID 3/​7 followed by OP  mg/​kg for  days (group )
2. OP  mg/​kg for 4 days (group 2)
3. Placebo (group 3)
Following are the main results of ONTT, which can be memorized as V-​RRR (see also Table 5.2).
(a) Visual outcome: Neither treatment regime influenced the visual outcome at any time point
up to 5 years 94% have VA ≥6/​2 at 5 years; 3% have ≤6/​60 at 5 years
(b) Recovery of vision: Group  has more rapid improvement in symptoms vs. other 2 groups
but no effect on final visual outcome at 6 months or 5 years
(c) Recurrence of optic neuritis: Group 2 has twofold increased risk of recurrence (28%) vs.
other two groups
(d) Risk of MS: IVMP reduced the rate of clinically definite multiple sclerosis during the first
2 years, but similar to other two groups by the third year

Table 5.2  The main findings of the Optic Neuritis Treatment Trial (ONTT)


Risk of MS 5-​year 0-​year 5-​year
Overall 30% 38% 50%
No lesion on MRI 6% 22% 25%
 or more lesions on MRI 33%* 56% 72%
* 5% if three or more lesions on MRI. MS, multiple sclerosis.

Optic Neuritis Study Group. Multiple sclerosis risk after optic neuritis final Optic Neuritis Treatment
Trial follow-​up. Arch Neurol 2008;65:727–​32.
Basic Science and Miscellaneous | ANSWERS 151

22. Answer: D
NASCET study is a landmark randomized controlled trial comparing the benefit and safety of CEA
and aspirin (Group ) to aspirin alone (Group 2) in symptomatic and asymptomatic patients with
mild (<50%), moderate (50–​69%) and severe (70–​99%) carotid stenosis. Following are the main
results:
• Mild stenosis: No significant difference of treatment failure rate between surgical (5%) and
medical treatment (9%) at 5 years
• Moderate stenosis: 5-​year rate of ipsilateral stroke = 6% vs. 22% (NNT was 5). Marginal
beneficial effect in reducing risk of stroke at 2 years
• Severe stenosis:
◆ Absolute risk reduction of ipsilateral stroke was 7% (NNT was 6)
◆ Risk of perioperative (within 30 days) severe complication (severe stroke/​death): 2.%
vs. 0.9%
◆ Risk of ipsilateral stroke at 2 years: .6% vs. 2.2%
In summary, CEA is recommended for:
• Symptomatic stenosis of 50–​99% if perioperative risk of stroke or death is <6%
• Asymptomatic stenosis of 60–​99% if perioperative risk of stroke or death is <3%
Barnett HJ, et al. Benefit of carotid endarterectomy in patients with symptomatic moderate or severe
stenosis. North American Symptomatic Carotid Endarterectomy Trial Collaborators. N Engl J Med
998;339:45–​25.
Cina CS, et al. Carotid endarterectomy for symptomatic carotid stenosis. Cochrane Database Syst Rev
2000;2:CD0008.

23. Answer: C
The original Age-​related Eye Disease Study (AREDS)  formulation contained vitamin C, vitamin E,
zinc, copper, and b-​carotene. AREDS 2 looked at improving the original formulation. Omega-​3 fatty
acids as well as the antioxidants, lutein, and zeaxanthin, which are in the same family of nutrients
as β-​carotene, were tried as β-​carotene was found to be associated with an increased risk of lung
cancer in smokers in previous study. AREDS 2 found that while omega-​3 fatty acids had no effect
on the formulation, lutein and zeaxanthin together appeared to be a safe and effective alternative to
β-​carotene.
Age-​related Eye Disease Study 2 Research Group. Lutein + zeaxanthin and omega-​3 fatty acids for
age-​related macular degeneration: the Age-​related Eye Disease Study 2 (AREDS2) randomized clinical
trial. JAMA 203;309:2005–​5.

24. Answer: D
Children under 6 years of age can consent to medical treatment if they understand what is being
proposed. It is up to the doctor to decide whether the child has the maturity and intelligence to
fully understand the nature of the treatment, the options, the risks involved, and the benefits.
A child who has such understanding is considered Gillick competent (or Fraser competent). The
parents cannot overrule the child’s consent when the child is judged to be Gillick competent.
Children under 6 who are not Gillick competent and very young children cannot either give or
withhold consent. Those with parental responsibility need to make the decision on their behalf. The
‘Fraser guideline’ specifically relates to contraception and sexual health.
Gillick v West Norfolk and Wisbech AHA, 985. Available at: http://​www.bailii.org/​uk/​cases/​UKHL/​
985/​7.html
152 Basic Science and Miscellaneous | ANSWERS

Lennings NJ. Forward, Gillick: Are competent children autonomous medical decision makers? New
developments in Australia. J Law Biosci 205;2:459–​68.

25. Answer: A
The patient has full capacity. He simply does not understand English. The patient should ideally
be rebooked with a medical translator and a Lithuanian consent form, which can be downloaded
from the internet. Consent : Adult with capacity; Consent 2: Parental agreement for a child
or young person; Consent 3: Procedure where consciousness is not impaired (therefore this
cannot be used in this case as sedation is required); and Consent 4: Adults who are unable to
consent.
Good Practice in Consent Implementation Guide: Consent to examination or treatment. Available at: http://​
www.wales.nhs.uk/​publications/​impguide-​e.pdf (see p. 4)

26. Answer: A
A common framework used in the analysis of medical ethics are the ‘four principles’ approach
postulated by Tom Beauchamp and James Childress in their textbook Principles of Biomedical Ethics.
It recognizes four basic moral principles. Ideally, for a medical practice to be considered ‘ethical’, it
must respect all four of these principles: autonomy, justice, beneficence, and non-​maleficence.
Beauchamp J. Principles of Biomedical Ethics. Oxford, UK: Oxford University Press, 203.
Pollard BJ. Autonomy and paternalism in medicine. Med J Aust 993;59:797–​802.

27. Answer: C
The first thing one should do in this case is to call in a nurse as a chaperon. If patient continues to
behave inappropriately she can then be warned, and the doctor has the right to refuse examination.
Davies M. Crossing boundaries: dealing with amorous advances by doctors and patients. BMJ
205;35:h5368.

28. Answer: D
Visual standards for driving is an extremely common exam topic. You are required to recall all of
the specific details. Be aware not only of the differences between group  and 2 drivers, but also
implications of acute monocularity, diplopia, blepharospasm, and also grandfather rights. Group 
includes car and motorcycles and Group 2 includes large lorries and buses. Option A is incorrect as
this is a Group 2 parameter. Option B is incorrect as the guidance states that this must be ‘severe’
blepharospasm. There are no criteria stipulated for severity, but it is noted if the condition is mild
or treated, they may drive. Option C is incorrect as this is a Group  parameter.
Driver and Vehicle Licensing Agency (DVLA). Assessing Fitness to Drive: A Guide for
Medical Professionals, September 209. Available at: https://​www.gov.uk/​guidance/​
assessing-​fitness-​to-​drive-​a-​guide-​for-​medical-​professionals

29. Answer: D
This is a common clinical scenario suggestive of amaurosis fugax. These patients should be treated
as ocular transient ischaemic attack (TIA) and be referred to the TIA team for further assessment
and management. Affected patients should stop driving for at least  month.
Driver and Vehicle Licensing Agency (DVLA). Assessing Fitness to Drive: A Guide for
Medical Professionals, September 209. Available at: https://​www.gov.uk/​guidance/​
assessing-​fitness-​to-​drive-​a-​guide-​for-​medical-​professionals
Basic Science and Miscellaneous | ANSWERS 153

30. Answer: A
There are several types of economic evaluation in healthcare. Following is the summary of the
commonly used analytic methods (Table 5.3).

Table 5.3  Commonly used methods of economic evaluation in healthcare


Analysis Description
Cost-​benefit Analysis of all the costs and consequence of an intervention in monetary terms
Cost-​effectiveness Comparison of different drugs or programmes which have a common health
outcome. The results are usually presented as costs per life year gained
Cost-​minimization Analysis that focuses on the cost alone and the cheapest option is chosen, provided
the consequences of two or more interventions being compared are similar
Cost-​utility Analysis of costs and benefits intervention and the results are usually presented as
cost per quality adjusted life year (QALY)

Data from Kernick DP. Introduction of health economics for the medical practitioner. Postgrad Med J 2003;79:47–50.

3. Answer: A
The correct management of an outbreak of endophthalmitis within a unit is key to ensuring patient
safety, both at a local and national level. The Royal College of Ophthalmologists has a document
that covers this subject and gives key factors that should be taken into consideration. An incidence
of >0.8% should be taken extremely seriously.
Royal College of Ophthalmologists. Ophthalmic Services Guidance: Managing an Outbreak of
Postoperative Endophthalmitis, July 206. Available at: https://​www.rcophth.ac.uk/​wp-​content/​uploads/​
206/​07/​Managing-​an-​outbreak-​of-​postoperative-​endophthalmitis.pdf

32. Answer: D
Vigabatrin is associated with bilateral, concentric, predominantly nasal constriction of the visual
field. The majority of defects extend to within 30◦ of fixation, defects outside that eccentricity,
and therefore not detected by standard 30◦ threshold tests have been reported. This finding is in
accord with a study that found that peripheral rod-​derived dark-​adapted visual fields were also
constricted in patients having visual field constriction attributable to vigabatrin (VAVFC) on light-​
adapted fields. Risk factors for VAVFC include male gender, treatment dose and duration, and
increasing age.
The Royal College of Ophthalmologists. The Ocular Side-​Effects of Vigabatrin (Sabril) Information and
Guidance for Screening, 2008. Available at: https://​www.rcophth.ac.uk/​wp-​content/​uploads/​205/​0/​
2008-​SCI-​020-​The-​Ocular-​Side-​Effects-​of-​Vigabatrin-​Sabril.pdf

33. Answer: B
The NDESP provides specific guidance on the diabetic screening and monitoring in the United
Kingdom. All patients with diabetes will be invited to the routine digital screening test where
digital photograph is obtained and primary grading is performed. Patients who have primary
grading result of R, R2, M, or non-​DR lesions will have a secondary grading and further referral
outcome grading. Patients who meet the referral criteria will be suspended from the routine
digital screening and sent to either of the three services: () slit-​lamp biomicroscopy surveillance;
154 Basic Science and Miscellaneous | ANSWERS

(2) digital surveillance (where it requires monitoring more frequently than annually); and (3) referral
to hospital eye services for further assessment and management. For patients who have negative
primary grading result (R0M0), 0% of the cases will undergo internal quality assurance via
secondary grading and the subsequent steps are similar as above. The screening service only
recalls patients for annual screening and not more frequently. Digital surveillance can provide more
frequent monitoring such as every 3 or 6 monthly. Stable treated proliferative diabetic retinopathy
can be reviewed in digital surveillance or annual screening service. Following link provides a good
summary of the flow of the NDESP service.
Gov.UK. NHS Diabetic Eye Screening (DES) Programme. Available at: https://​www.gov.uk/​health-​and-​
social-​care/​population-​screening-​programmes-​diabetic-​eye

34. Answer: C
The current options for the management of diabetic macular oedema where CRT are more than
400 mm are aflibercept, ranibizumab, Ozurdex, and illuvien. Focal laser will not help in the case of
diffuse macular oedema.
NICE guideline recommends aflibercept to be given as a single 2 mg intravitreal injection every
month for 5 consecutive months, followed by one injection every 2 months with no requirement
for monitoring between visits. After the first 2 months, the treatment interval may be extended
based on visual and anatomic outcomes. Answer B is incorrect as the regime mentioned is not the
recommended regime.
Ranibizumab should be given monthly and continued until maximum visual acuity is reached
(until visual acuity has been stable for 3 consecutive months). Thereafter, visual acuity should be
monitored monthly. Treatment is resumed if monitoring indicates a loss of visual acuity caused by
diabetic macular oedema, and continued until visual acuity has remained stable for 3 consecutive
months. Ozurdex and illuvien are recommended for use in pseudophakic cases which have failed to
respond to non-​corticosteroid treatment, or such treatment is unsuitable.
National Institute for Health and Care Excellence (NICE). Aflibercept for Treating Diabetic Macular
Oedema, 205. Available at: https://​www.nice.org.uk/​guidance/​ta346
National Institute for Health and Care Excellence (NICE). Dexamethasone Intravitreal Implant for
Treating Diabetic Macular Oedema, 205. Available at: https://​www.nice.org.uk/​guidance/​ta349
National Institute for Health and Care Excellence (NICE). Fluocinolone Acetonide Intravitreal Implant for
Treating Chronic Diabetic Macular Oedema After Inadequate Response to Prior Therapy, 203. Available
at: https://​www.nice.org.uk/​guidance/​ta30

35. Answer: B
All candidates should be familiar with the RCOphth guideline on ophthalmic instrument
decontamination for multiple-​choice question and VIVA purposes. The guideline advises that all
non-​surgical reusable instruments (such as tonometer prisms, diagnostic contact lenses, etc.)
should be decontaminated with % sodium hypochlorite (Milton) solution for 0 minutes between
patients. Approx. 70% alcohol wipes have been shown to be insufficient to inactivate adenovirus
and other viruses. Single-​use instruments have not been shown to be cost-​effective in reducing the
possible transmission of  CJD.
Royal College of Ophthalmologists. Ophthalmic Services Guidance Ophthalmic Instrument
Decontamination, 206. Available at: https://​www.rcophth.ac.uk/​wp-​content/​uploads/​204/​2/​
Ophthalmic-​Instrument-​Decontamination.pdf
Basic Science and Miscellaneous | ANSWERS 155

36. Answer: B
All trainees should be familiar with the UK eye retrieval and eye bank services. The death-​to-​
retrieval time should be 24 hours or less. Once the eyes are retrieved, they are stored in the eye
bank using the organ culture method at 34°C, which has the advantage of longer storage time
(30 days) compared to hypothermic storage method at 4°C (7–​0 days), which is the method
used in the United States. The minimum cut-​off limit of endothelial cell density for penetrating/​
endothelial keratoplasty is 2200 cells/​mm2 in the United Kingdom. A list of contraindications
for the use of ocular tissues for corneal transplantation can be found on the Royal College
of Ophthalmologist guideline. The contraindications include blood-​borne viral infections,
haematological malignancies, previous ocular inflammation or corneal surgeries, and central nervous
system diseases such as dementia (most cases), multiple sclerosis, and Parkinson disease, among
many others.
Royal College of Ophthalmologists. RCOphth Clinical Guidelines on Standards for the Retrieval of
Human Ocular Tissue Used in Transplantation, Research and Training. Available at: https://​www.
rcophth.ac.uk/​publications/​current-​clinical-​guidelines/​
chapter MOCK EXAM

6 QUESTIONS

. A 23-​year-​old patient presents to the eye emergency department after


a splash of bleach in the eye. His presenting visual acuity is 6/​36 and
examination showed a central corneal defect of 80% with mild corneal
oedema, 20% conjunctival staining,  clock-​hour limbal ischaemia,
and raised intraocular pressure of 24 mmHg. The cornea is otherwise
clear. What is the likely clinical outcome of the patient if managed
appropriately?
. The patient is likely to recover without any long-​term ocular sequelae
A
B. The patient is likely to develop persistent epithelial defect that requires long-​term topical
treatment
C. The patient is likely to require surgical interventions such as amniotic membrane transplant
to recover good vision
D. The patient is likely to have a poor visual outcome

2. Which of the following statements regarding corneal collagen cross-​


linking is correct?
. It has been shown to be an effective adjunct treatment for acanthamoeba keratitis
A
B. It is safe to be used in patients with corneal thickness of 350 microns
C. The treatment regime utilizes ultraviolet-​A and vitamin B2 drops
D. Accelerated treatment protocols have been shown to be more effective than the
conventional Dresden protocol (3 mW/​min for 30 min)

3. A 42-​year-​old man has been referred to the eye clinic with dry eyes and
photophobia associated with some form of corneal stromal changes.
Systemic examination reveals bilateral facial nerve paresis and skin
laxity. Which of the following statements best describes the condition of
this patient?
A. The ocular examination is likely to reveal multiple grey reticular opacities at the
subepithelial layer
B. This patient is at increased risk of developing cardiac and renal failures
C. This condition is associated with mutation in transforming growth factor β-​induced
(TGFBI) gene
D. Recurrent corneal erosion is rare in this condition
158 Mock Exam | QUESTIONS

4. Which of the following options regarding the complication of herpes


zoster ophthalmicus (HZO) is correct?
. Post-​herpetic neuralgia is more common in younger patients with HZO
A
B. Male patients are more likely to develop post-​herpetic neuralgia than the female patients
C. Abducens nerve palsy is the most common ocular motility problem reported in patients
with HZO
D. HZO increases the risk of stroke and cardiac event within a year of the diagnosis of HZO

5. Which of the following diseases has the highest risk of recurrence in


corneal graft?
A. Granular dystrophy
B. Lattice dystrophy
C. Macular dystrophy
D. Reis-​Buckler dystrophy

6. A 35-​year-​old patient presented to the eye casualty with a -​week


history of right eye pain. She was a soft contact lens wearer and had
recently recovered from an episode of cold sores at the mouth corner.
Slit-​lamp examination showed an area of dendritic-​like changes on the
paracentral cornea. The patient was started on topical aciclovir and
was reviewed in a week’s time. One week later, there was worsening of
the dendritic-​like changes on the corneal epithelium with new onset of
stromal oedema, keratic precipitates, and anterior chamber activity.
Otherwise there were no stromal or perineural infiltrates. What is the
most appropriate next step in this case?
. Stop topical aciclovir, perform corneal scrape, and start the patient on topical chlorhexidine
A
B. Change topical aciclovir to topical ganciclovir due to lack of treatment efficacy
C. Continue with topical aciclovir and add topical prednisolone 0.5% 4×/​day
D. Change topical aciclovir to topical ganciclovir and add topical prednisolone 0.5% 4×/​day
and oral aciclovir 400 mg 5×/​day

7. Which of the following statements concerning primary Sjogren’s


syndrome (PSS) is correct?
. Male is more commonly affected than female
A
B. Negative anti-​SSA (or anti-​Ro) and anti-​SSB (or anti-​La) antibodies preclude the
diagnosis of  PSS
C. There is an increased risk of T-​cell lymphoma in patients with PSS
D. The classic triad of symptoms consist of dryness of the eyes and mouth, fatigue, and pain

8. Which of the following conditions/​drugs is NOT associated with vortex


keratopathy?
A. Atovaquone
B. Fabry’s disease
C. Lowe syndrome
D. Tamoxifen
Mock Exam | QUESTIONS 159

9. A 65-​year-​old gentleman presents to the eye casualty with a 5-​day


history of right eye pain. He has recently investigated for recurrent
episodes of nosebleed. Slit-​lamp examination revealed a very injected
right eye with engorged conjunctival and deep episcleral vessels
associated with an area of peripheral corneal thinning spanning 3 to 6
o’clock with no infiltrate. Serologic testing revealed markedly elevated
antiproteinase 3 antibody level. The patient was started on high-​dose
systemic prednisolone but was not able to fully control the ocular
disease. What is the best next step-​up treatment to further control this
condition?
A. Azathioprine
B. Methotrexate
C. Mycophenolate mofetil
D. Rituximab

0. Which of the following statements concerning pterygium is true?


A. The presence of Hudson–​Stahli line at the head of pterygium is an indicative sign of
chronicity of pterygium
B. Pterygium grows across the limbus and damages the Bowman’s layer
C. Ultraviolet (UV) radiation plays an important role in the cases of recurrent pterygium
D. Evidence suggests that sutures may result in lower risk of recurrence than fibrin glue for
fixing the conjunctival graft during pterygium surgery

. Which of the following anterior segment dysgenesis should always have
chromosomal analysis to exclude a WT deletion, which is associated
with an important systemic pathology?
A. Aniridia
B. Axenfeld-​Rieger syndrome
C. Peter’s anomaly
D. Trabeculodysgenesis

2. A 55-​year-​old patient is referred to the glaucoma clinic. During the


assessment you find an intraocular pressure of 28 mmHg in the right eye
(OD) and 3 mmHg in the left eye (OS), and open iridocorneal angles.
There is evidence of neuroretinal rim thinning and arcuate defects
on perimetric testing. What treatment would be most appropriate in
this case?
A. Latanoprost
B. Timolol
C. Tiopex
D. Xalatan
160 Mock Exam | QUESTIONS

3. A 32-​year-​old female patient with glaucoma presents to your clinic


with a rise in intraocular pressure and suspected further damage to the
retinal nerve fibre layer/​neuroretinal rim as identified on perimetry and
imaging. Her current medication is topical latanoprost once at night.
She informs you that she is pregnant. Your management plan is most
likely to include the following:
. Change latanoprost to latanoprost/​timolol combination
A
B. Continue with latanoprost and start brimonidine
C. Stop latanoprost and start brimonidine
D. Stop latanoprost and observe

4. In any patient with posterior segment ischaemia, which of the following
best describes the mechanism of disruption of aqueous circulation in
those who develop neovascular glaucoma?
A. Neovascular membranes grow across the angle, causing fibrosis, contraction, and
synechiael closure of the iridocorneal angle
B. Hormones released as a consequence of ischaemia causes a secondary open angle
glaucoma through an unknown mechanism
C. Mechanical blockage of the trabecular meshwork with inflammatory particles released
from friable new vessels
D. New vessels in the anterior segment cause an increase in blood supply to the ciliary body
and increased aqueous production

5. A 43-​year-​old male with elevated intraocular pressure (IOP) at the


optometrist is referred to your clinic. On examination you can see
pigment on the corneal endothelium, midperipheral transillumination
defects within the iris, and heavily pigmented trabecular meshwork.
Which of the following is MOST accurate with regard to this
gentleman’s condition?
. Patients tend to have mild to moderate hypermetropia
A
B. Trabeculectomy for these patients has a similar success rate as for primary open angle
glaucoma
C. Laser trabeculoplasty tends not to be effective in this condition
D. Laser iridotomy may help deepen the iridocorneal angle and reduce the risk of angle
closure

6. Please select the MOST ACCURATE statement from the following
options with regard to the properties of intraocular lenses used during
routine cataract surgery.
A. Hydrophobic acrylic intraocular lens (IOLs) can be folded easily, have low rates of
posterior capsule opacification (PCO), and a high refractive index
B. Round edged IOLs are associated with lower rates of  PCO
C. Silicone IOLs can be folded easily and have a high refractive index
D. Rigid polymethylmethacrylate (PMMA) lenses have a much lower rate of PCO but require
a larger incision.
Mock Exam | QUESTIONS 161

7. Based on the current guidelines, the following findings would suggest
repeating biometry:
. Soft contact lens wearer who has not worn contact lenses for more than  week
A
B. Mean corneal power difference greater than 0.7 D between the two eyes
C. Axial length greater than 0.5 mm difference between the two eyes
D. Patient has had biometry performed after dilation and tonometry

8. All of the following conditions are linked with microspherophakia,


EXCEPT:
A. Ehlers–​Danlos syndrome
B. Marfan’s syndrome
C. Peters anomaly
D. Weill–​Marchesani syndrome

9. For the following clinical scenario, please select the MOST appropriate
option with regard to the formation of lens opacity. A 42-​year-​old
woman taking long-​term treatment with amiodarone to control her
arrhythmia has bilateral asymptomatic lens opacities.
. Anterior subcapsular deposits
A
B. Posterior subcapsular opacities
C. Sunflower cataract
D. Vossius  ring

20. With regard to congenital cataract, which of the following statements is


correct?
. Sealing the wound with a suture leads to more astigmatism
A
B. Bilateral cataract is more amblyogenic than unilateral cataract, and therefore should be
operated on sooner
C. Galactosaemia can result in an oil drop cataract and renal failure
D. All of the above

2. A 53-​year-​old female presents with a recurrent retinal detachment


secondary to inferior proliferative vitreoretinopathy (Grade C). She
undergoes further vitrectomy surgery to reattach her retina. Which is
the most likely choice of endotamponade?
A. Hexafluoroethane  (C2F6)
B. Perfluorocarbon liquid (PFCL)
C. Silicone  oil
D. Sulphur hexafluoride (SF6) gas
162 Mock Exam | QUESTIONS

22. A 52-​year-​old male presents with a retinoschisis-​related retinal


detachment (RSRD). Which of the following statements is correct?
. RSRD retinal detachments occur predominantly in a temporal location
A
B. RSRD can only occur if inner and outer leaf breaks are present
C. Surgical outcomes are equivalent to those of conventional rhegmatogenous retinal
detachments
D. At presentation RSRD are rarely associated with proliferative vitreoretinopathy

23. A patient presents 4 days after uncomplicated cataract surgery.


Vision in the operated eye is perception of light. There is a 4 mm
hypopyon present, no fundal view. A diagnosis of presumed bacterial
endophthalmitis is made in the eye emergency department. Which of
the following management options is most appropriate?
. Vitreous biopsy and intravitreal antibiotics
A
B. Hourly topical steroid, with a review in 6 hours to see if there is any improvement
C. Three port pars plana vitrectomy and intravitreal antibiotics
D. Anterior chamber washout and topical antibiotics

24. Which of the following is the commonest cause of sympathetic


ophthalmia within the United Kingdom?
A. Brachytherapy
B. Cataract surgery
C. Ocular  trauma
D. Vitreoretinal surgery

25. Which of the following statement regarding familial exudative


vitreoretinopathy (FEVR) is correct?
. It is most commonly inherited in an autosomal recessive fashion
A
B. Wide-​field fluorescein angiography is the gold standard in assessing FEVR
C. A negative family history is useful in excluding the diagnosis of FEVR
D. Early laser photocoagulation does not reduce the risk of disease progression

26. Which of the following options about retinal arterial macroaneurysm


(RAM) is correct?
. RAM may result in trilaminar retinal haemorrhages
A
B. Exudative RAM is usually associated with a better visual prognosis than haemorrhagic RAM
C. It is most commonly associated with hyperlipidaemia
D. Haemorrhagic RAM usually requires treatment compared to exudative RAM

27. Which of the following is true with regard to the natural history of
retinal vein occlusions?
A. In central retinal vein occlusions 30% of eyes with a visual acuity of worse than 6/​60 go on
to develop rubeosis
B. 50–​60% of untreated branch retinal vein occlusion cases retain a visual acuity of better
than 6/​2 at  year
Mock Exam | QUESTIONS 163

. 20% of central retinal vein occlusions present with bilateral involvement


C
D. 20% of branch retinal vein occlusions will have fellow eye involvement over time

28. Which of the following steroid preparations has National Institute for
Health and Care Excellence (NICE) approval for the management of
macular oedema secondary to central retinal vein occlusion?
. Dexamethasone intravitreal implant 0.7 mg
A
B. Fluocinolone acetonide intravitreal implant 0.9 mg
C. Methylprednisolone acetate
D. Triamcinolone acetonide

29. A patient presents with painless loss of vision in the left eye and the
following retinal appearance (Figure 6.) following uneventful cataract
surgery. The intracameral use of which medication has been associated
with this condition?
A. Amphotericin
B. Bevacizumab
C. Cefuroxime
D. Vancomycin

Figure 6. 
Reprinted from Ophthalmology: Journal of the American Academy of Ophthalmology, 24, 5, Witkin, A. et al., Vancomycin-Associated
Hemorrhagic Occlusive Retinal Vasculitis: Clinical Characteristics of 36 Eyes, pp. 583–595, Copyright 206, with permission from
Elsevier. https://doi.org/0.06/j.ophtha.206..042. Published by Elsevier on behalf of the American Academy of Ophthalmology.
164 Mock Exam | QUESTIONS

30. Which of the following conditions is associated with retinitis pigmentosa


and steatorrhoea?
A. Bardet–​Biedl syndrome
B. Bassen–​Kornzweig syndrome
C. Refsum syndrome
D. Waardenburg syndrome

3. Based on the ocular motility problem illustrated in Figure 6.2, where is
the most likely location of the lesion?
. Left medial longitudinal fasciculus
A
B. Left sixth nerve nucleus
C. Right paramedian pontine reticular formation
D. Right parietal lobe
Right Eye Left Eye

–3 –3

Figure 6.2  

32. A 58-​year-​old man presented with a 4-​day history of discomfort around


his right eye and new onset diplopia which he noticed when watching
TV. The diplopia was worse on right gaze and on examination he was
found to have an esotropia in the primary position of 0 dioptres and of
30 dioptres on right gaze. He had no other significant medical history
and was on no medication. What is the most likely diagnosis?
. Idiopathic intracranial hypertension
A
B. Microvascular sixth nerve palsy
C. Thyroid eye disease
D. Traumatic injury to the eye

33. Which of the following statements regarding Marcus-​Gunn jaw-​winking


syndrome (MGJWS) is correct?
. It represents 30% cases of congenital ptosis
A
B. Amblyopia may develop in patients with MGJWS and is usually caused by the ptotic eyelid
C. It is caused by synkinetic movement of internal pterygoid muscle and levator palpebrae
superioris muscle
D. It may present bilaterally
Mock Exam | QUESTIONS 165

34. A 32-​year-​old man presented with a week’s history of right ptosis and
ear pain. Examination revealed a 2 mm ptosis of the right upper lid and
he continued to describe significant ear pain. He had already received a
-​week course of oral antibiotics. In the light, his right pupil measured
2 mm and his left measured 3 mm, and in a dimly lit room his right pupil
measured 2.5 mm and the left measured 4 mm. What is the most likely
cause of the anisocoria?
. Argyll Robertson pupil
A
B. Damage to oculosympathetic system
C. Longstanding Adie pupil
D. Physiological anisocoria

35. A 29-​year-​old female presented to her GP with pain around her left eye
for 3 days. Her GP found her vision to be slightly reduced in the left eye
so suggested she was seen in an urgent clinic in the eye department. On
examination she had visual acuity of 6/​6 OD and 6/​2 OS. Colour vision
revealed an Ishihara plate of 5/​5 OD and 7/​5 OS. When she was seen
in the eye department she also mentioned that colours seemed washed
out in her left eye compared to her right. She had a left relative afferent
pupillary defect (RAPD) and on examination of her fundus, there was
no sign of any abnormality. What is the most likely finding on visual field
testing of this patient?
. Altitudinal field defect
A
B. Bitemporal hemianopia
C. Central scotoma
D. Junctional scotoma

36. Which of the following syndromes best describes ipsilateral sixth and
seventh nerve palsies with contralateral hemiparesis?
A. Eight-​and-​a-​half syndrome
B. Foville syndrome
C. Millard–​Gubler syndrome
D. Raymond-​Céstan syndrome

37. Which of the following feature is most suggestive of non-​arteritic


anterior ischaemic optic neuropathy (NA-​AION)?
. Altitudinal field defect
A
B. Family history of visual loss
C. Jaw claudication
D. Recent foreign travel
166 Mock Exam | QUESTIONS

38. Which of the following statements about myotonic dystrophy (MD) is


correct?
A. It is an autosomal recessive disease due to cytosine-​thymine-​guanine (CTG)
trinucleotide repeat
B. Respiratory failure is the leading cause of death in patients with MD
C. The most common ocular motility problem is deficit in elevation of the eyes
D. It is most commonly found in Eastern European and Asian populations

39. A patient with a history of lupus presents with a new onset posterior
uveitis and undergoes baseline investigation including syphilis testing.
The following result is obtained: syphilis immunoglobulin G (IgG) with
a treponemal specific antibody test is positive and rapid plasma regain
result is highly positive What does this test result mean?
. The patient has untreated syphilis and requires treatment
A
B. The result is a false positive and no further treatment is necessary
C. The patient appears to have previously undergone treatment for syphilis
D. The patient has no evidence of active syphilis and no treatment is necessary

40. A 30-​year-​old female is brought to the accident and emergency


(A&E) department with hearing loss and altered mental state. She
also mentions that her vision is blurred in both eyes. Ophthalmic
examination identifies bilateral multiple branch retinal artery
occlusions. Magnetic resonance imaging (MRI) of the brain
demonstrates multiple lesions within the corpus callosum. What is the
most likely diagnosis?
. Cerebral venous thrombosis
A
B. Cogan syndrome
C. Multiple sclerosis
D. Susac’s syndrome

4. A 44-​year-​old female presents with new onset polyarthralgia, including


her hands, elbows, ankles, and knees. She is also noted to have a
painful and tender rash affecting the shins of both legs. Inflammatory
markers are high and autoantibodies were negative. Serum angiotensin-​
converting enzyme (ACE) was normal. A chest X-​ray showed bilateral
hilar lymphadenopathy. What is the most likely diagnosis?
A. Heerfordt’s syndrome
B. Löfgren’s syndrome
C. Rheumatoid arthritis
D. Reactive arthritis
Mock Exam | QUESTIONS 167

42. The following are features of Alport syndrome, EXCEPT:


A. Anterior lenticonus
B. Glomerulonephritis
C. Posterior subcapsular cataract
D. Sensorineural deafness

43. A 52-​year-​old lady is referred to the outpatient clinic to exclude an


ocular cause for her intractable right-​sided ocular pain. The pain has
been ongoing for 9 months with no pain-​free intervals. There is no
exacerbating or relieving factor. She describes a persistent aching and
occasionally stabbing behind the eye which can be associated with lid
drooping and sometimes the eye can look pink. There is no movement
exacerbation and the pain can make her restless. Examination of both
eyes is normal as is the MRI of the brain requested from primary care.
Which of the treatment is most likely to alleviate her symptoms?
. High-​flow oxygen via venture mask with rebreathable bag
A
B. Oral aciclovir
C. Oral indomethacin
D. Subcutaneous sumatriptan

44. A 3-​month child is referred because the parents are worried about its
vision. On complete eye exam no apparent cause can be found. What do
you do next?
. Referral to a paediatrician or a paediatric neurologist and review the child at aged 6 months
A
B. Referral for visual electro physiology and review the child at age 6 months
C. Referral for a magnetic resonance image of the head and review the child with result
D. Referral for visual electrophysiology and paediatric workup and review the child with
the result

45. Which of the following is a characteristic feature of euryblepharon?


. Asymmetric enlargement of the horizontal palpebral fissure
A
B. Elongated lid margins
C. Upward and lateral displacement of the lateral canthus
D. Vertical lengthening of the eyelid skin

46. A 3-​month-​old child is referred with a segmental capillary haemangioma


of the upper eyelid more than 5 cm in size. The first line of
management includes:
. Intralesional steroid injection as there is a risk of amblyopia
A
B. MRI/​MRA scan of the head and neck
C. Pulsed-​dye laser treatment to regress the lesion
D. Surgical excision of the haemangioma
168 Mock Exam | QUESTIONS

47. Which of the following statements regarding orbital cellulitis and


subperiosteal abscess is correct?
A. Orbital cellulitis occurs as a secondary extension of acute or chronic bacterial sinusitis,
especially the frontal sinusitis
B. Surgical intervention is more likely if the child is under 9 years of age as it is commonly
polymicrobial
C. Clinical improvement correlates well with repeat computed tomography (CT) scan analysis
D. Blindness occurs in up to % of  cases

48. A 4-​month-​old child was referred with features of midfacial hypoplasia,


shallow orbit, proptosis, corneal exposure, finger and toe abnormality,
partial syndactyly, and normal intelligence. These distinctive features
are suggestive of:
A. Apert syndrome
B. Crouzon syndrome
C. Muenke syndrome
D. Pfeiffer syndrome

49. A 2-​day-​old unwell child was seen with bilateral red eye, non-​purulent
discharge, and hazy cornea. The management includes:
A. Topical aciclovir ointment five times daily and intravenous acyclovir 60 mg/​kg/​day in three
divided doses
B. Topical cefuroxime and gentamicin drops four times daily and broad-​spectrum antibiotics
C. Topical erythromycin 2-​hourly and intravenous benzylpenicillin (30 mg/​kg/​day) in three
divided doses or cefotaxime 00 mg/​kg as a single dose
D. Topical tetracycline ointment four times a daily and 4-​week course of oral erythromycin
(50 mg/​kg/​day) in four divided doses

50. Which of the following statements concerning congenital megalocornea


is correct?
. It is commonly unilateral and thought to be due to defective growth of the optic cup
A
B. The model of inheritance is an X-​linked recessive trait in more than 90%
C. It is a slowly progressive condition with a corneal diameter more than 5 mm
D. Hypermetropia is the most common refractive error associated with this condition, often
accompanied with by-​the-​rule astigmatism

5. A 63-​year gentle man presented with vertical diplopia after an injury.
On examination he had a right hypertropia increasing on right gaze and
left head tilt. The most likely extraocular muscle that is paretic is:
. Right inferior rectus
A
B. Right superior oblique
C. Left inferior oblique
D. Left superior rectus
Mock Exam | QUESTIONS 169

52. A 5-​year-​old patient presents to the eye emergency department


complaining of double vision, with a history of blunt trauma to his right
cheekbone. Which of the following signs would require urgent surgical
intervention?
. Bradycardia in upgaze
A
B. Maxillary sinus opacification
C. 2 mm enophthalmos
D.  –​2 restriction in elevation

53. All of the following facial injection sites are associated with high risk for
inadvertent ophthalmic artery intra-​arterial injection embolus causing
central retinal artery occlusion, EXCEPT:
A. Glabellar
B. Nasal  dorsum
C. Nasojugal
D. Temporalis

54. Which of the following signs is the best indicator of severe thyroid eye
disease?
A. RAPD
B. Chemosis
C. 40^ esotropia
D. Lateral flare of the upper lid

55. A 4-​year-​old boy is reviewed in clinic for a congenital left upper


eyelid ptosis. His unaided visual acuity is recorded as right: 0.25 and
left: 0.300 LogMAR equivalent on Kay Pictures. He has normal right
eye examination and a 3 mm ptosis on the left upper eyelid with
levator function of 9 mm. Which of the following would be the best
management option for him?
A. Fasanella  Servat
B. Frontalis  sling
C. Levator aponeurosis advancement
D. Patching of the right eye

56. Which of the following could cause a cicatricial ectropion?


A. Dacryoadenitis
B. Horizontal lid laxity
C. Lower lid basal cell carcinoma (BCC)
D. Ocular cicatricial pemphigoid (OCP)
170 Mock Exam | QUESTIONS

57. A 45-​year-​old female is referred to clinic with concern regarding


facial asymmetry; she thinks one eye looks sunken compared to the
other. She has no significant past medical history and no history of
trauma. Visual acuity and eye movements are normal, there is 3 mm
of enophthalmos on the left with a deep superior sulcus but no lid
malposition. Which of the following is the most likely diagnosis?
. Previous blow out fracture
A
B. Sclerosing metastasis
C. Scleroderma
D. Silent sinus syndrome

58. Which of the following features is more common in squamous cell


carcinoma (SCC) than BCC?
A. Bleeding
B. Pain
C. Telangiectasia
D. Ulceration

59. Which of the following is associated with an increase of choroidal


melanoma metastasis?
A. Monosomy  3
B. Younger age at presentation
C. Female patient
D. Loss of chromosome 8q

60. Which of the following is the most common histopathological diagnosis


for orbital malignancies in an adult?
. Adenoid cystic carcinoma
A
B. Lymphoma
C. Metastasis
D. Rhabdomyosarcoma

6. Retinal astrocytoma may be associated with the following neoplasms,


EXCEPT:
A. Cardiac rhabdomyoma
B. Neurofibroma
C. Pheochromocytoma
D. Pulmonary lymphangioleiomyoma
Mock Exam | QUESTIONS 171

62. A 48-​year-​old male presents with a 3-​month history of reduced vision in


one eye. His vision is count fingers with a total retinal detachment with
a large retinal tear superiorly at the ora serrata. He has evidence of +
anterior chamber cells and the IOP is 34 in the affected eye. The other
eye is normal. He reports blunt ocular trauma prior to the onset of
symptoms. What is the most likely diagnosis?
. Chronic uveitis complicated by exudative retinal detachment
A
B. Posner–​Schlossman syndrome
C. Schwartz syndrome
D. Sympathetic ophthalmia

63. In patients with juvenile idiopathic arthritis, which of the following has
the highest risk of developing chronic anterior uveitis?
. ANA +ve, oligoarthritis, younger age at diagnosis
A
B. ANA +ve, polyarthritis, older age at diagnosis
C. ANA –​ve, oligoarthritis, younger age at diagnosis
D. ANA –​ve, polyarthritis, older age at diagnosis

64. An 82-​year-​old retired engineer presents with right ocular redness and
visual loss due to blurry vision and floaters over 3 weeks. His medical
background is notable only for previously treated malaria. Examination
reveals a right panuveitis. Through the moderately hazy vitreous a large
pale patch is observed involving the macula of right fundus. There are
no fundus haemorrhages and the left eye is normal. Optical coherence
tomography (OCT) reveals derangement of the architecture of the
outer retina. Which of the following is the most likely diagnosis?
A. Herpes  zoster
B. Small cell carcinoma of the lung
C. Syphilis
D. Tuberculosis

65. Which of the following describes the method of action of adalimumab


therapy in the treatment of uveitis?
A. Anti-​CD20
B. Antitumour necrosis factor alpha
C. Anti-​vascular endothelial growth factor (VEGF)
D. Interferon–​βa
172 Mock Exam | QUESTIONS

66. What is the recommended use of corticosteroid therapy in the


United Kingdom for prevention of Jarisch–​Herxheimer reaction in the
treatment of neurological or ophthalmic syphilis?
. A 3-​day course of
A oral corticosteroid starting 24 hours before antibiotic therapy
B. A 5-​day course of oral corticosteroid starting concurrently with antibiotic therapy
C. A 3-​day course of oral corticosteroid starting 48 hours after initiation of antibiotic therapy
D. A 5-​day course of oral corticosteroid starting 48 hours after initiation of antibiotic therapy

67. Which of the following best describes the mechanism of latanoprost in


reducing intraocular pressure?
. Increase uveoscleral outflow
A
B. Reduce aqueous production
C. Increase uveoscleral outflow and reduce aqueous production
D. Increase conventional/​trabecular outflow and reduce aqueous production

68. Which of the following medications is associated with increased risk of


tendon rupture?
A. Cefuroxime
B. Ciclosporin
C. Ciprofloxacin
D. Cyclophosphamide

69. Quetiapine can cause:


A. Corneal verticillata
B. Dopamine maculopathy
C. Intraoperative floppy iris syndrome (IFIS)
D. Open angle glaucoma

70. What is the mechanism of action of botulinum toxin?


. Inhibits the presynaptic release of acetylcholine (ACh)
A
B. Inhibits the postsynaptic ACh receptor
C. Inhibits the reuptake of acetylcholinesterase
D. Inhibits the voltage-​gated calcium channels at the neuromuscular junction

7. Which of the following statements regarding the microbiological


staining technique is true?
. Bacteria that allow crystal violet dye to wash off are Gram positive
A
B. Giemsa stain is for bacteria, fungi, and acanthamoeba
C. Gomori methenamine silver stains for acanthamoeba
D. Periodic acid Schiff stain is useful for bacteria
Mock Exam | QUESTIONS 173

72. Which of the followings regarding fundus fluorescein angiography (FFA)


is correct?
. It incorporates a yellow green excitation filter and a blue barrier filter
A
B. Sodium fluorescein is 40–​50% bound to plasma albumin
C. Sodium fluorescein is metabolized and excreted by the kidney in 24 hours
D. Pregnancy is not an absolute contraindication

73. In acute posterior multifocal pigment placoid epitheliopathy (APMPPE),


what is the typical lesion appearance on FFA?
. Persistent hypofluorescence throughout the angiogram
A
B. Early hypofluorescence and late staining
C. Early hyperfluorescence and retinal vascular leakage
D. Early hyperfluorescence and evidence of retinal vascular occlusion

74. Which of the following is correct about pattern electroretinogram


(pERG)?
. Assesses the peripheral retina
A
B. Uses a reversing checkerboard to evoke small potentials that arise from the inner retina
C. N95 is photoreceptor-​driven and is key to assessing macular cone function
D. P50 originates from the macular ganglion cells

75. What is the clinical interpretation of the Worth four-​dot (W4D) test
result in the following scenario? A patient wears a green lens in front of
the right eye and a red lens in front of the left eye. She sees three green
lights on the W4D test.
. The left eye is suppressed
A
B. The right eye is suppressed
C. The patient is likely to have diplopia
D. The patient is likely to have binocular single vision (BSV)

76. Which of the following orthoptic tests can be carried out without
glasses?
A. Frisby
B. Hess  chart
C. Titmus
D. TNO

77. Which of the following describes the neuroimaging shown in Figure 6.3?


A. MRI T-​weighted
B. MRI T-​weighted with gadolinium enhancement
C. MRI T2-​weighted
D. MRI T2-​weighted with fluid attenuated inversion recovery (FLAIR) sequence
174 Mock Exam | QUESTIONS

Figure 6.3 
Reproduced with permission from Yau, G. et al. (204) Neuromyelitis Optica Spectrum Disorder in a Chinese Woman with
Ocular Myasthenia Gravis: First Reported Case in the Chinese Population. Neuro-Ophthalmology, 38(3): 40–44. https://doi.
org/0.309/065807.203.879903. Rights managed by Taylor & Francis.

78. Which of the following autoantibodies is NOT implicated in myasthenia


gravis?
. Antiacetylcholine receptors (AChR) antibody
A
B. Anticyclic citrullinated peptide (CCP)
C. Anti-​low-​density lipoprotein receptor-​related protein (LRP4) antibody
D. Antityrosine kinase muscle-​specific kinase (MuSK) antibody

79. The optic vesicle is formed by day:


. 2 of
A gestation
B. 25 of gestation
C. 32 of gestation
D. 42 of gestation

80. Which of the following cell types of choroidal melanoma is associated


with the worst prognosis?
A. Epithelioid  cell
B. Spindle  A
C. Spindle  B
D. Mixed  type
Mock Exam | QUESTIONS 175

8. Patients with gyrate atrophy result from a deficiency of:


. Ornithine aminotransferase (OAT), located on chromosome 0
A
B. Ornithine aminotransferase (OAT), located on chromosome 2
C. Ornithine hexaminase (Ohex), located on chromosome 4
D. Ornithine hexaminase (Ohex), located on chromosome 6

82. What kind of image does slit-​lamp fundus lenses (e.g. +90 D lens)
produce?
. Real, mirrored, and magnified
A
B. Real, inverted, and magnified
C. Virtual, mirrored, and magnified
D. Virtual, inverted, and magnified

83. Which of the following tests is used to compare the mean of three or
more groups?
. Analysis of variance (ANOVA)
A
B. Contingency  table
C. Mann–​Whitney  U  test
D. T-​tests

84. Which of the following statements regarding the Tube Versus


Trabeculectomy (TVT) study is correct?
A. The rate of reoperation was significantly lower in the tube group versus the
trabeculectomy group
B. The number of glaucoma medications in the tube group was significantly lower than the
trabeculectomy group at 5 years
C. The cumulative probability of failure during 5 years of follow-​up was the same in both tube
and trabeculectomy groups
D. The rate of early postoperative complications was significantly higher in the tube group

85. A 76-​year-​old patient attends the age-​related macular degeneration


clinic keen to supplement his diet in zinc. Which food has the highest
content of zinc?
. One cup of sliced apricots
A
B. One cup of cubed avocado
C. Three  dates
D. One of sliced kiwi
176 Mock Exam | QUESTIONS

86. A 68-​year-​old patient was recently diagnosed with right eye primary
open angle glaucoma and left ocular hypertension. His best-​corrected
visual acuity is 6/​6 in either eye. He is currently receiving treatment in
both eyes. He normally drives a four-​seater car. According to the Driver
and Vehicle Licensing Agency (DVLA) driving standard, which of the
following advices should be given to this patient in this circumstance?
. Patient can continue to drive and does not need to inform DVLA
A
B. Patient can continue to drive but needs to inform DVLA
C. Patient should stop driving and needs to inform DVLA
D. If patient continues to drive, you may have to breach the patient’s confidentiality and
inform DVLA

87. Capacity should be assessed in a semi-​structured direct interview with


the patient. Capacity evaluation is assessed in a:
A. One-​step process
B. Two-​step process
C. Three-​step process
D. Four-​step process

88. The quality-​adjusted life year or quality-​adjusted-​life-​year (QALY)


equates to:
.  year of
A life ×  year of ill health
B.  year of ill health ×  utility value
C.  year of life ×  utility value
D.  year of ill health ×  year of  life

89. A patient attends the outpatient department with a putative diagnosis


of Creutzfeldt–​Jakob disease (CJD). Which procedure carries the
highest risk of transmission?
A. Brow  lift
B. Dacryocystorhinostomy
C. Lateral rectus recession
D. Scleral buckling

90. Patients receiving long-​term hydroxychloroquine should receive the


following after 5 years of therapy:
A. 0–​2 Humphrey visual field, spectral domain optical coherence tomography (SD-​OCT),
and fundus autofluorescence (FAF)
B. 0–​2 Humphrey visual field, SD-​OCT, and multifocal electroretinography
C. 30–​2 Humphrey visual field and SD-​OCT
D. 30–​2 Humphrey visual field and FAF
chapter MOCK EXAM

6 ANSWERS

. Answer: A
Chemical eye injury is a common ophthalmic emergency that requires immediate medical attention.
The question is testing the candidates’ knowledge on the assessment and prognostic factors of
chemical eye injury, which can be based on Roper-​Hall or modified Hughes classification. In this
classification, limbal ischaemia—​a proxy for limbal stem cell damage—​and corneal haze are the two
main prognostic factors for vision (Table 6.).

Table 6.  Limbal ischaemia and corneal haze are the two main prognostic factors for vision

Grading Cornea Limbal ischaemia Prognosis


 Clear Nil Good
2 Hazy; iris details visible </​3 Good
3 Opaque; iris details obscured /​3–​/​2 Guarded
4 Opaque; iris details obscured >/​2 Poor

Reproduced with permission from Roper-Hall, M. (965) Thermal and chemical burns. Transactions of the Ophthalmological Societies
of the United Kingdom, 85:63-53. Courtesy of The Royal College of Ophthalmologists.

2. Answer: C
Corneal collagen cross-​linking (CXL) was first introduced in 2003 by Wollensak et al. to treat
progressive keratitis. It is a minimally, invasive procedure that combines the use of ultraviolet-​A light
of 365–​370 nm and riboflavin/​vitamin B2 drops. The NICE guideline has recommended the use
of CXL in patients with corneal ectasia with a corneal thickness of 400 micros or more to avoid
corneal endothelial damage by UV irradiation. The original Dresden protocol requires a 30-​minute
instillation of topical riboflavin solution followed by UVA irradiation at 3 mW/​cm2 for 30 minutes
(fluence of 5.4 J/​cm2). Over the recent years, various accelerated protocols, using higher UVA
irradiation with shorter time span, have been proposed and examined. So far the evidence showed
similar efficacy between accelerated and conventional protocols.
Photoactivated chromophore for keratitis (PACK)-​CXL has been shown to be a potentially
effective adjuvant treatment to topical antibiotic treatment for bacterial keratitis, but the effect on
acanthamoeba keratitis has not been proven.
Berra M, et al. Treatment of Acanthamoeba keratitis by corneal cross-​linking. Cornea 203;32:74–​8.
National Institute for Health and Care Excellence (NICE). Photochemical Corneal Collagen Cross‑Linkage
Using Riboflavin and Ultraviolet A for Keratoconus and Keratectasia. Interventional Procedures Guidance
[IPG466], 203. Available at: https://​www.nice.org.uk/​guidance/​ipg466
178 Mock Exam | ANSWERS

Shajari M, et al. Comparison of standard and accelerated corneal cross-​linking for the treatment of
keratoconus: a meta-​analysis. Acta Ophthalmol 209;97():e22–​35.

3. Answer: B
This clinical scenario describes a patient suffering from a rare type of familial systemic amyloidosis
called Meretoja’s syndrome (or type 2 lattice corneal dystrophy). It was first described in 969 by
Dr Meretoja, a Finnish ophthalmologist. It is an autosomal dominant disease caused by mutation at
the gelsolin gene at chromosome 9q, whereas type  lattice dystrophy is caused by gene mutation
in the BIGH3 or TGFBI. The symptoms/​signs of Meretoja’s syndrome usually starts from the third
decade of life and the first sign of disease is usually corneal lattice dystrophy. The typical diagnostic
triad includes corneal lattice dystrophy, progressive bilateral facial nerve palsy, and skin laxity. The
affected patients are at increased risk of developing renal failure, cardiac failure, and conduction
abnormalities. The amyloid stains with Congo red and demonstrates apple green birefringence and
dichroism at polarizing microscopy.
Carrwik C, Stenevi U. Lattice corneal dystrophy, gelsolin type (Meretoja’s syndrome). Acta Ophthalmol
2009;87:83–​9.
Meretoja syndrome—​recent articles. Available at: https://​www.ncbi.nlm.nih.gov/​medgen/​30243

4. Answer: D
HZO is associated with a wide spectrum of ocular and non-​ocular complications. Post-​herpetic
neuralgia (PHN) is one of the most common and debilitating complication following HZO. The
risk of PHN rises significantly with age, from 4% in patients younger than 50 years old to 30–​40%
in patients over 80 years old. Female gender has been shown as a risk factor for PHN but the
evidence is not conclusive. Third nerve palsy is the most common ocular motility disorder reported
in patients with HZO but other types of ocular motility problem such as fourth, sixth, and multiple
nerve palsies have been reported. HZO significantly increases the risk of stroke and cardiac events
within a year of diagnosis of HZO. This is likely due to the migration of virus from neurons to the
cerebral and coronary vasculatures, causing local inflammatory responses, vascular occlusion, and
ischaemia.
Erskine N, et al. A systematic review and meta-​analysis on herpes zoster and the risk of cardiac and
cerebrovascular events. PloS One 207;2:e08565.
Marsh RJ, et al. External ocular motor palsies in ophthalmic zoster: a review. Br J Ophthalmol
977;6:677–​82.
Schutzer-​Weissmann J, Farquhar-​Smith P. Post-​herpetic neuralgia—​a review of current management
and future directions. Expert Opin Pharmacother 207;8:739–​50.

5. Answer: D
All the conditions mentioned in the answer list can be associated with recurrence of the disease in
the corneal graft. The risk of recurrence can be memorized by the mnemonic of ‘Rude Little Green
Man’ in descending order of frequency.
R—​Reis-​Buckler dystrophy (most common)
L—​Lattice dystrophy
G—​Granular dystrophy
M—​Macular dystrophy (least common)
Marcon AS, et al. Recurrence of corneal stromal dystrophies after penetrating keratoplasty. Cornea
2003;22:9–​2.
Mock Exam | ANSWERS 179

6. Answer: A
This is a clinical vignette of acanthamoeba keratitis (AK) masquerading as herpes simplex keratitis
(HSK), which is a commonly encountered scenario in clinical practice. Epithelial HSK usually
responds well and quickly to topical antiviral treatment. Therefore, one should always have a
low threshold of suspecting AK in contact lens wearers who present with non-​specific epithelial
changes or ‘dendritic-​like’ changes, especially when the condition has not improved on topical
antiviral treatment. It is also noteworthy to mention that sometimes AK can coexist with HSK,
therefore a positive herpes simplex virus (HSV) swab result does not exclude the diagnosis of AK.
Approximately 85–​90% cases of AK are related to contact lens wear. It normally progresses from
epithelial to stromal disease and patients usually have disproportionate pain to clinical signs, but
absence of pain does not preclude the diagnosis.
Dart JK, et al. Acanthamoeba keratitis: diagnosis and treatment update 2009. Am J Ophthalmol
2009;48:487–​99.

7. Answer: D
PSS is a common systemic autoimmune disease that may occur in isolation or with associated
organ-​specific autoimmune diseases such as thyroiditis or primary biliary cirrhosis. Secondary SS is
referred to when the disease occurs in association with another systemic autoimmune disease such
as rheumatoid arthritis, systemic lupus erythematosus, scleroderma, or dermatomyositis. PSS has a
female-​to-​male predominance of 9: with a peak incidence at around 50 years old. The classic triad
of symptoms include dryness of the mouth and eyes, fatigue, and pain. PSS increases the risk of
B-​cell lymphoma (not T-​cell lymphoma) by 5–​20 times compared to the general population. The
207 American College of Rheumatology, European League against Rheumatism (ACR-​EULAR)
classification specifies the diagnostic criteria for PSS, which include focus score of ≥ on minor labial
salivary gland biopsy, presence of anti-​SSA (or anti-​Ro) antibodies, SICCA ocular staining score of
≥5, Schirmer’s test of ≤5 mm per 5 min, and unstimulated whole salivary flow of ≤0. ml per min.
Absence of anti-​SSA or anti-​Ro does not exclude the diagnosis of PSS but will require the presence
of focal lymphocytic sialadenitis on minor labial salivary gland biopsy to confirm the diagnosis.
Mariette X, Criswell LA. Primary Sjogren’s syndrome. N Engl J Med 208;378:93–​9.

8. Answer: C
Vortex keratopathy, or also known as cornea verticillata, is a type of deposition keratopathy
characterized by whorl-​like changes at the corneal subepithelial layer. It may be associated with
a wide range of drugs and conditions. Many of these drugs have cationic amphiphilic structures
that allow them to cross the cell membranes, leading to intracellular phospholipid accumulation.
Vortex keratopathy usually has no effect on the vision and is not an indication for discontinuing
the treatment. Fabry’s disease and Lowe syndrome are both metabolic disorders that can cause
congenital cataract. However, Lowe syndrome does not result in vortex keratopathy.
The following mnemonic (CAT-​FANGS) can be used to help memorize the list of drugs/​condition
associated with vortex keratopathy:
C—​Chloroquine (common)
C—​Chlorpromazine (common)
A—​Amiodarone (common)
T—​Tamoxifen (common)
T—​Tilorone
F—​Fabry’s disease
180 Mock Exam | ANSWERS

A—​Atovaquone
N—​NSAIDs
G—​Gold
G—​Gentamicin
S—​Suramin
Raizman MB, et al. Drug-​induced corneal epithelial changes. Surv Ophthalmol 207;62:286–​30.

9. Answer: D
This is a clinical scenario of peripheral ulcerative keratitis (PUK) with scleritis in a patient with
undiagnosed granulomatosis with polyangiitis (GPA), formerly known as Wegener’s granulomatosis.
GPA is a type of systemic vasculitides that affects small-​and medium-​sized vessels, with primary
involvement of upper respiratory tract and kidney. Patients may suffer from recurrent nose
bleeds, saddle-​nose deformity, pulmonary haemorrhage, scleritis, PUK, glomerulonephritis, and
arthritis. Antineutrophilic cytoplasmic antibodies (ANCA) are autoantibodies produced by a
person’s immune system that attack proteins within the person’s neutrophil. Proteinase 3 (PR3)
and myeloperoxidase (MPO) are the two most common subsets of ANCA. 85% of the samples
with cytoplasmic (cANCA) will have anti-​PR3 antibodies (which is highly associated with GPA) and
90% of the perinuclear (pANCA) will have anti-​MPO antibodies (which is highly associated with
microscopic polyangiitis). In uncontrolled GPA-​related PUK, cyclophosphamide or rituximab has
been shown to best control disease and induce remission.
Ebrahimiadib N, et al. Successful treatment strategies in granulomatosis with polyangiitis-​associated
peripheral ulcerative keratitis. Cornea 206;35:459–​65.

0. Answer: B
Pterygium is a common degenerative conjunctival fibrovascular lesion extending from the
conjunctiva to the cornea. It is characterized by elastotic degeneration of the substantia propria.
UV light plays an important role in primary pterygium but not recurrent pterygium, which is more
related to surgical trauma. Presence of Stocker’s iron line at the head of pterygium is an indicative
sign of chronicity of pterygium. The Hudson–​Stahli line is an innocuous iron line commonly found at
the inferior /​3 of the cornea in older people. Evidence suggests that fibrin glue may result in lower
risk of recurrence and lesser operating time than sutures for fixing the conjunctival graft during
pterygium surgery.
American Academy of Ophthalmology. Pterygium, 205. Available at: http://​eyewiki.aao.org/​Pterygium
Romano V, et al. Fibrin glue versus sutures for conjunctival autografting in primary pterygium surgery.
Cochrane Database Syst Rev 206;2:CD0308.

. Answer: A
The WT tumour suppressor gene lies next to the PAX6 gene on p3, so can also be affected in
sporadic aniridics. This is associated with Wilms tumour (nephroblastoma). The most commonly
affected gene involved in trabeculodysgenesis, or primary congenital glaucoma is CYPB (Chr2p)
and also MYOC (Chrq). Peters anomaly is usually sporadic, with multiple genes isolated as the
cause including PAX6, PIT2X, FOXC, CYPB, and MYOC. Questions in the exam therefore are
unlikely to focus on specific genes for Peters anomaly.
Gould DB, John SW. Anterior segment dysgenesis and the developmental glaucoma are complex
traits. Hum Mol Genet 2002;:85–​93.
Mock Exam | ANSWERS 181

2. Answer: A
This answer reflects the new NICE guidelines which suggest: ‘Offer a generic prostaglandin
analogue to people with suspected chronic open angle glaucoma and intraocular pressure of
24 mmHg or more, in line with the recommendations on treatment for people with ocular
hypertension.’ This is different to the previous iteration of the guidelines, which focussed on
different primary medications based on corneal thicknesses and age.
Reproduced from National Institute for Health and Care Excellence. Glaucoma: Diagnosis and Management (NG8) November
207. Available on: www.nice.org.uk/guidance/ng8

3. Answer: C
This is a complex scenario and is not without some controversies, but raises the importance of
understanding implications of medicines on the pregnant patient and the developing fetus.
The US Food and Drug Administration (FDA) has classified glaucoma medications as follows:
• Class B (medications have varying and /​or contradictory human and animal
data.)—​Brimonidine  only
• The rest of the glaucoma medications are classified as class C (side effects in animal
models, or where inadequate animal and human studies are available)—​β-​
blockers, carbonic anhydrase inhibitors (topical or systemic), and prostaglandin analogues.
This classification is based on the little existing evidence. There is some varied clinical practice, but
for the purposes of the exam, brimonidine is the safest medication during pregnancy. Importantly
post-​partum this should be stopped if the intention is to breastfeed, as it passed to the milk and can
cause respiratory depression in the newborn. Prostaglandin is probably the least safe, as it is similar
to the hormones that stimulate uterine contractions and has been shown to induce miscarriage
in animals. Option D is not an option as the vignette describes glaucoma progression. It should
be noted though that during pregnancy in those with ocular hypertension/​primary open angle
glaucoma the IOP tends to decrease, so if there was no progression monitoring could be an option.
Razeghinejad MR, et al. Pregnancy and glaucoma. Surv Ophthalmol 20;56:324–​5.
Salim S. Glaucoma in pregnancy. Curr Opin Ophthalmol 204;25:93–​7.

4. Answer: A
This is the main cause though the new vessels themselves cause some degree of mechanical
blockage of the trabecular meshwork. Option B is incorrect—​although VEGF and pigment
epithelium derived factor (PEDF) may independently affect aqueous drainage, the extent of
their effect is currently not known and is not supposed to contribute the elevation of intraocular
pressure significantly. Option C is incorrect as there are no specific inflammatory particles that
are produced; however, there is often some level of anterior segment inflammation. Option D
is incorrect. There is no known glaucoma that occurs from overproduction of aqueous humour,
although certain medications can increase the production to varying degrees.
McLaren JW, et al. Effect of ibopamine on aqueous humor production in normotensive humans. Invest
Ophthalmol Vis Sci 2003;44:4853–​8.
Rodrigues GB, et al. Neovascular glaucoma: a review. Int J Retina Vitreous 206;2:26.

5. Answer: B
The vignette describes pigment dispersion syndrome (PDS) with elevated intraocular pressure.
Trabeculectomy success rates are similar to that of primary open angle glaucoma, but with
reported increased rates of hypotony maculopathy.
182 Mock Exam | ANSWERS

Glaucoma may develop in 33–​50% of patients with PDS. It can be clinically similar to
pseudoexfoliation (PXF), although in PXF the iris transillumination defects tend to be closer to
the pupillary margin, and the pigmentation of the angle is more sporadic and heterogeneous.
There may also be evidence of pseudoexfoliative materials in the anterior segment. Patients
with PDS tend to be myopes, with some evidence suggesting the degree of myopia in those
who develop glaucoma is higher than in those who do not. Laser trabeculoplasty tends to be
effective; however, a greater incidence in post-​laser IOP spikes has been reported. Peripheral
iridotomy has been purported as a potential treatment for PDS to equalize the pressure
between anterior and posterior chambers and reduce the posterior bowing of the iris but
not to deepen the anterior chamber angle. However, a recent Cochrane collaboration review
found insufficient evidence of high quality on the effectiveness of peripheral iridotomy for
pigmentary glaucoma or PDS. The vignette describes an open angle, and PDS is an open
angle glaucoma. If you see a patient with occludable or narrow but visibly pigmented angles,
the diagnosis is likely to be different (e.g. primary angle closure or primary angle closure
glaucoma).
Michelessi M, Lindsley K. Peripheral iridotomy for pigmentary glaucoma. Cochrane Database Syst Rev
206;2:CD005655.
Niyadurupola N, Broadway DC. Pigment dispersion syndrome and pigmentary glaucoma—​a major
review. Clin Exp Ophthalmol 2008;36:868–​82.

6. Answer: A
Square-​edged IOLs are associated with lower rates of PCO. Silicone IOLs have a lower refractive
index, but otherwise accurate. PMMA lens have a higher rate of PCO but otherwise accurate.
Denniston AKO, Murray PI. Oxford Handbook of Ophthalmology, 3rd edition. Oxford, UK: Oxford
University Press, 204; pp. 324–​8.

7. Answer: D
Biometry readings are considered acceptable if a soft contact lens (CL) wearer has not worn the
CLs for  week or more. For rigid gas permeable contact lenses, the requirement is 4 weeks.
The other findings suggesting repeat biometry are:
(a) Axial length >0.7 mm difference between the two eyes
(b) Mean corneal power difference greater than 0.9 D between the two eyes
(c) Axial length <2.2 mm or >26.6 mm in either eye
(d) Mean corneal power <4 D or >47 D in either eye
(e) Delta K (corneal astigmatism) >2.5 D in either eye
(f ) If the patient has biometry performed after dilation or tonometry
Knox-​Cartwright N, et al. The Cataract National Dataset electronic multicentre audit of 55,567
operations: when should optical biometric measurements be rechecked? Eye (Lond) 200;24:894–​900.
Royal College of Ophthalmologists. Cataract Surgery Guidelines, September 200;
p. 45. Available at: https://​www.rcophth.ac.uk/​wp-​content/​uploads/​204/​2/​
200-​SCI-​069-​Cataract-​Surgery-​Guidelines-​200-​SEPTEMBER-​200-​.pdf

8. Answer: A
Microspherophakia is the clinical term for a small, spherical lens but no criteria are used to
determine specific size or curvature. It can be inherited as an isolated abnormality (autosomal
dominant familial microspherophakia), associated with conditions Weill–​Marchesani (AR), Marfan’s
Mock Exam | ANSWERS 183

syndrome (AD), homocystinuria (AR), Peters anomaly, hyperlysinaemia (AR), Alport syndrome
(X-​linked dominant), and congenital rubella. It is associated with an increased risk of acute or
chronic angle closure glaucoma. Patients who develop this may benefit from lens extraction.
A clinical triad of acute angle closure, shallow anterior chamber, and myopia is highly suggestive of
microspherophakia.
American Academy of Ophthalmology. Microspherophakia, 209. Available at: http://​eyewiki.aao.org/​
Microspherophakia

9. Answer: A
Gold and chlorpromazine are also known to cause inconsequential lens opacities. Medication
induced posterior subcapsular lenticular opacities are commonly seen in patients with long-​term
systemic or topical steroid use. Sunflower cataract is typically caused by Wilson’s disease, a disorder
of copper metabolism that can lead to abnormal deposits of copper in the lens and peripheral
cornea (Kayser–​Fleischer ring), as well as the liver, brain, and other parts of the body. Vossius ring
is an anterior lens opacity, caused by compression of the iris onto the anterior lens surface due to
blunt trauma.
Ikäheimo K, et al. Visual functions and adverse ocular effects in patients with amiodarone medication.
Acta Ophthalmol Scand 2002;80:59–​63.

20. Answer: C
Unilateral cataract causes higher risk of amblyopia than bilateral cataract. As a general guide
unilateral cataract should be operated on prior to 6 weeks to avoid amblyopia, and bilateral
cataract within 0 weeks. A very simple aide memoir is ‘Single eye Six weeks, Two eyes Ten
weeks’. As the structures of the eye are much more pliable in younger patients, leaving the corneal
wound unsutured will result in higher amount of astigmatism.
Lim ME, et al. Update on congenital cataract surgery management. Curr Opin Ophthalmol
207;28:87–​92.

2. Answer: C
Proliferative vitreoretinopathy remains the leading cause for retinal re-​detachment and can result in
poor visual outcomes. The aim of revision surgery is to relieve the underlying retinal traction (either
by peeling the membrane off or by using a relieving retinectomy). Silicone oil endotamponade is
the most likely choice in a patient with Grade C proliferative vitreoretinopathy. Perfluoropropane
(C3F8) is shown to be similarly effective as silicone oil in terms of achieving at least 5/​200 vision
and macular attachment at a minimum of one year. Perfluorocarbon can be used as a short-​
acting endotamponade (usually in the management of giant retinal tears with associated retinal
detachment). Shorter-​acting gas tamponade is unlikely to be used in this setting due to the higher
risk of failure.
Schwartz SG, et al. Tamponade in surgery for retinal detachment associated with proliferative
vitreoretinopathy. Cochrane Database Syst Rev 204;(2):CD00626.

22. Answer: A
RSRD is a relatively rare form of retinal detachment (RD), accounting for –​2% of all
rhegmatogenous RD. They mostly occur in a temporal location and surgical outcomes are inferior
when compared with conventional rhegmatogenous RD. They can occur when an outer leaf break
is present only (fluid from the schitic cavity can move into the subretinal space). The associated
proliferative vitreoretinopathy (PVR) rate at presentation is reported as 2% (Grade B and C).
184 Mock Exam | ANSWERS

Jeourdi AM, et al. Management of degenerative retinoschisis-​associated retinal detachment.


Ophthalmology Retina 207;:266–​7.
Xue K, et al. Incidence, mechanism and outcomes of schisis retinal detachments revealed through a
prospective population-​based study. Br J Ophthalmol 207;0:022–​6.

23. Answer: C
Postoperative bacterial endophthalmitis should be considered as an ophthalmic emergency and
prompt treatment is required. The Endophthalmitis Vitrectomy Study (EVS) found that patients
presenting with vision of perception of light only, had a significantly better visual outcome when
randomized to the pars plana vitrectomy group. The European Society of Cataract & Refractive
Surgeons (ESCRS) Guidelines for Prevention and Treatment of Endophthalmitis Following Cataract
Surgery gives recommendations on the time scale for the management of this condition.
Endophthalmitis Vitrectomy Study Group. Results of the Endophthalmitis Vitrectomy Study.
A randomized trial of immediate vitrectomy and of intravenous antibiotics for the treatment of
postoperative bacterial endophthalmitis. Arch Ophthalmol 995;3:479–​96.

24. Answer: D
Sympathetic ophthalmia (SO) is an uncommon bilateral granulomatous panuveitis. Ocular surgery,
predominantly retinal surgery, is the commonest cause within the United Kingdom. The incidence
has been reported as 0.03/​00 000. Prompt diagnosis and treatment can lead to good visual
outcomes. The role of enucleation in the management of SO is still debated.
Kilmartin DJ, et al. Prospective surveillance of sympathetic ophthalmia in the UK and Republic of
Ireland. Br J Ophthalmol 2000;84:259–​63.

25. Answer: B
FEVR is a heritable vitreoretinopathy characterized by anomalous retinal vascular development.
The hallmark of the disease is the avascularity of the peripheral retina, which can lead to exudation,
haemorrhage, neovascularization, and RD. The most common inheritance pattern is autosomal
dominant, followed by autosomal recessive and X-​linked recessive. Wide-​field fluorescein
angiography is the gold standard in diagnosing and monitoring patients with FEVR because more
than half of asymptomatic family members of patients with FEVR can have subclinical findings that
are not visible on slit-​lamp examination. This is also the reason why a negative family history is not
useful in excluding the diagnosis of FEVR. Early laser photocoagulation of the avascular peripheral
retina has been shown to reduce the risk of disease progression.
Tauqeer Z, Yonekawa Y. Familial exudative vitreoretinopathy: pathophysiology, diagnosis, and
management. Asia Pac J Ophthalmol (Phila) 208;7:76–​82.

26. Answer: A
RAM is an acquired saccular or fusiform dilatation of the large arterioles of the retina, usually within
the first three orders of bifurcation. It is most commonly observed in elderly women and is associated
with systemic vascular conditions such as hypertension. RAM can cause visual loss via exudation
(exudative RAM) or preretinal, intraretinal, subretinal or trilaminar haemorrhages (haemorrhagic
RAM). It is widely recognized that haemorrhagic RAM usually does not require treatment as it has
a tendency to thrombose. Haemorrhagic RAM usually has a better visual prognosis than exudative
RAM, especially when macular oedema and exudates persist for several months in exudative type.
The most common treatment for RAM is laser photocoagulation either directly to or around the
aneurysm to prevent bleeding or to exudative lesion to reduce macular oedema.
Mock Exam | ANSWERS 185

Pitkänen L, et al. Retinal arterial macroaneurysms. Acta Ophthalmol 204;92:0–​4.

27. Answer: B
It is recommended that all candidates sitting the Fellowship of the Royal College of
Ophthalmologists (FRCOphth) exam are similar with this guideline. The document contains a
section on both the epidemiology and natural history of retinal vein occlusions. The Royal College
of Ophthalmologists (RCOphth) published guidelines on the retinal vein occlusion in July 205; see
following link.
Royal College of Ophthalmologists. Clinical Guidelines: Retinal Vein Occlusion, 205. Available at: https://​
www.rcophth.ac.uk/​wp-​content/​uploads/​205/​07/​Retinal-​Vein-​Occlusion-​RVO-​Guidelines-​July-​205.pdf

28. Answer: A
Dexamethasone intravitreal implant has NICE approval for the management of macular oedema
secondary to both branch and central retinal vein occlusions. Fluocinolone acetonide is licensed by
NICE for the management of chronic diabetic macular oedema but not retinal vein occlusion.
Haller JA, et al. Dexamethasone intravitreal implant in patients with macular edema related to branch
or central retinal vein occlusion twelve-​month study results. Ophthalmology 20;8:2453–​60.
National Institute for Health and Care Excellence (NICE). Dexamethasone Intravitreal Implant for
the Treatment of Macular Oedema Secondary to Retinal Vein Occlusion. Technology Appraisal Guidance
[TA229], 20. Available at: https://​www.nice.org.uk/​guidance/​ta229
National Institute for Health and Care Excellence (NICE). Fluocinolone Acetonide Intravitreal Implant
for Treating Chronic Diabetic Macular Oedema After an Inadequate Response to Prior Therapy. Technology
Appraisal Guidance [TA30], 203. Available at: https://​www.nice.org.uk/​guidance/​ta30

29. Answer: D
Figure 6. shows a fundal appearance consistent with haemorrhagic occlusive retinal vasculitis
(HORV). This is a rare, but devastating, complication of cataract surgery in which intracameral
vancomycin has been used.
The American Society of Cataract and Refractive Surgery (ASCRS) and the American Society of
Retina Specialists (ASRS) formed a joint task force to define the clinical characteristics of HORV and
to study its prevalence, cause, treatment, and outcomes.
Witkin AJ, et al. Vancomycin-​associated hemorrhagic occlusive retinal vasculitis: clinical characteristics
of 36 eyes. Ophthalmology 207;24:583–​95.

30. Answer: B
This is a favourite topic in FRCOphth Part 2 exam. The most common general health problems
associated with retinitis pigmentosa (RP) (so-​called systemic) are obesity and hearing loss. An
awareness of systemic associations is important as treatment can be initiated to prevent further
progression, e.g. a high-​calorie diet devoid of foods rich in phytanic acid (such as butter and animal
fat) combined with plasmapheresis in Refsum syndrome. Common systemic syndromes and their
distinguishing features are listed next:
A. Syndromes with RP and hearing loss
i. Usher syndrome
ii. Refsum syndrome: ataxia, ichthyosis, anosmia
iii. Alport syndrome: posterior polymorphous corneal dystrophy (PPCD), anterior
lenticonus, fleck retinopathy, renal failure
186 Mock Exam | ANSWERS

iv. Waardenburg syndrome: heterochromia, dystopia canthorum, white forelock


B. Syndromes with RP without hearing loss
i. Bassen–​Kornzweig syndrome (abetalipoproteinaemia): steatorrhoea, fat-​soluble vitamin
deficiency and ataxia, progressive external ophthalmoplegia
ii. Bardet–​Biedl syndrome: truncal obesity, polydactyly, short stature, and renal failure
iii. Lawrence–​Moon syndrome: same as Bardet–​Biedl but without obesity and polydactyly
iv. Mucopolysaccharidoses: all types except 6 (Maroteaux–​Lamy syndrome) and 7 (Sly)
v. Neuronal ceroid lipofuscinosis: dementia, seizures, and pigment retinopathy
(a) Infantile: Jansky–​Bielschowsky
(b) Juvenile: Vogt–​Spielmeyer–​Batten disease
(c) Adult: Kufs syndrome
vi. Kearn–​Sayre syndrome: chronic progressive external ophthalmoplegia (CPEO), ataxia,
heart block
National Organization for Rare Disorders (NORD). Retinitis Pigmentosa. Available at: https://​
rarediseases.org/​rare-​diseases/​retinitis-​pigmentosa/​

3. Answer: B
The illustration describes a case of left horizontal gaze palsy, which is most likely caused by a left
sixth nerve nucleus lesion. Understanding of the anatomy and physiology of the supranuclear
pathway is essential in reaching the accurate diagnosis of this type of ocular motility problem.
Here follows a summary of the important structures and functions of the supranuclear pathway
(Table 6.2).

Table 6.2  Important structures and functions of the supranuclear pathway


Structures Functions Examples
6th nerve Supplies ipsilateral LR and contralateral MR Left 6th nucleus damage l
nucleus Left horizontal gaze palsy
PPRF Horizontal gaze centre, supplies 6th nerve Left PPRF damage l
nucleus Left horizontal gaze palsy
POT Smooth pursuit centre, supplies ipsilateral Left POT/​parietal damage l
6th nerve nucleus Left smooth pursuit deficit
MLF Connects ipsilateral MR and contralateral Left MLF dysfunction l Left INO l Left
6th nerve nucleus adduction deficit and right abduction
nystagmus
MR, medial rectus; LR, lateral rectus; PPRF, paramedian pontine reticular formation; POT, parietal-​occipital-​temporal; MLF, medial
longitudinal fasciculus; INO, internuclear ophthalmoplegia.

Denniston AKO, Murry PI. Oxford Handbook of Ophthalmology, 3rd edition. Chapter 6: Neuro-​
ophthalmology, pp. 696–​7. Oxford, UK: Oxford University Press, 204.

32. Answer: B
In most adults over the age of 50 years the most likely cause of a sixth nerve palsy is a
microvascular event. Many patients have associated risk factors such as diabetes and cardiovascular
risk factors; however, this man had no obvious risks and there is a group of patients who have an
isolated sixth nerve palsy where no cause is found. It is vital, however, to rule out any significant
Mock Exam | ANSWERS 187

pathology that may cause a sixth nerve palsy, especially when there is no spontaneous recover of
nerve palsy after 3 months from the onset of symptoms.
Kung NH, Van Stavern GP. Isolated ocular motor nerve palsies. Semin Neurol 205;35:539–​48.

33. Answer: D
MGJWS ptosis is a type of congenital ptosis caused by synkinetic movement of the external
pterygoid (not internal pterygoid) muscle and the levator palpebrae superioris muscle. It represents
2–​3% cases of congenital ptosis. It usually presents unilaterally, but may rarely present bilaterally.
Amblyopia is usually caused by strabismus and anisometropia instead of ptosis.
Pearce FC, et al. Marcus-​Gunn jaw-​winking syndrome: a comprehensive review and report of four
novel cases. Ophthalmic Plast Reconstr Surg 207;33:325–​8.

34. Answer: B
This patient has features of a Horner’s syndrome with a unilateral ptosis and anisocoria. However,
some of the changes in this case are subtle and need further investigations, and this is frequently the
case in Horner’s syndrome. The patient has already been treated with antibiotics for a supposed
ear infection, which may be misleading but often a carotid artery dissection can present as ear or
neck pain. Often there is a history of trauma, but dissection can be spontaneous and can occur
even after minor trauma. Argyll Robertson and longstanding Adie’s syndromes will cause miotic
pupils but should not cause ptosis.
Patel RR, et al. Cervical carotid artery dissection: current review of diagnosis and treatment. Cardiol
Rev 202;20:45–​52.

35. Answer: C
This is a clinical vignette suggestive of optic neuritis. The visual acuity is usually reduced in one
eye and may worsen over a 2-​week period then starts to improve after a further 4 weeks. There
is often pain around or behind the eye and is often worse on eye movement. The patents usually
have reduced vision in the affected eye ranging from 6/​6 to no perception of light with reduced or
absent colour vision. There is usually a RAPD and fundal examination may reveal a slightly swollen
or normal optic nerve (>60% cases).
Patients with optic neuritis can develop many types of visual field defects including an altitudinal
field defect or an arcuate field defect but the most common field defect is a central scotoma.
A junctional scotoma would imply involvement of the optic nerve and chiasm, so would be very
rare in optic neuritis. Optic neuritis means inflammation of the optic nerve, it does not imply any
diagnosis.
Toosy AT, et al. Optic neuritis. Lancet Neurol 204;3:83–​99.

36. Answer: C
Various nuclear and fascicular sixth nerve syndromes have been described in the literature. The
following is a summary of the syndromes (Table 6.3).
Silverman IE, et al. The crossed paralyses. The original brain-​stem syndromes of Millard–​Gubler,
Foville, Weber, and Raymond–​Cestan. Arch Neurol 995;52:635–​8.

37. Answer: A
NA-​AION usually presents as unilateral visual disturbance and can present with a varying degree
of visual loss. In addition to optic nerve head swelling, there is often an altitudinal visual field
defect with crowded disc. A large cup:disc ratio in the fellow eye should raise the suspicion
188 Mock Exam | ANSWERS

Table 6.3  Syndromes associated with nerve palsies and contralateral hemiparesis


Syndromes Clinical features Lesion site
Eight-​and-​a-​half Ipsilateral gaze palsy, ipsilateral INO, and 6th and 7th nerve nuclei, PPRF
ipsilateral 7th nerve palsy + MLF
Foville Ipsilateral gaze palsy, ipsilateral 5th and 7th nerve Dorsolateral pons
palsies, and contralateral hemianaesthesia
Millard–​Gubler Ipsilateral 6th and 7th nerve palsies and Ventral pons
contralateral hemiparesis
Raymond–​Cestan Ipsilateral 6th nerve palsy and contralateral Ventral pons
hemiparesis

INO, internuclear ophthalmoplegia; PPRF, paramedian pontine reticular formation; MLF, medial longitudinal fasciculus.

of other causes. Many patents have a history of vascular disorders such as hypertension,
hypercholesterolaemia, or diabetes, but some patients have no obvious risk factors at all. The use
of steroid treatment in NA-​AION at the early stages to prevent further visual loss remains a highly
debatable and controversial topic.
Kerr NM, et al. Non-​arteritic anterior ischaemic optic neuropathy: a review and update. J Clin Neurosci
2009;6:994–​000.

38. Answer: B
MD is an uncommon autosomal dominant disease caused by expanded CTG trinucleotide repeat,
which can lead to earlier and more severe disease in successive generation. It is characterized
by inability to relax the muscle (myotonia) and muscle wasting (dystrophy). It is most commonly
reported in French Canadians. Respiratory failure is the most common cause (30%) of death in MD,
followed by cardiac conduction abnormality (20%). The most common feature of ophthalmoplegia
is the deficit in adduction, resulting in pseudointernuclear ophthalmoplegia. It is also associated
with other ocular features such as Christmas tree cataract, miosis, low intraocular pressure (due to
ciliary body detachment), and retinal degeneration.
American Academy of Ophthalmology. Ocular Manifestations of Myotonic Dystrophy. Available at: http://​
eyewiki.aao.org/​Ocular_​Manifestations_​of_​Myotonic_​Dystrophy

39. Answer: A
Syphilis testing includes treponemal testing and non-​treponemal testing (RPR and VDRL).
Treponemal tests remain positive after acquisition of syphilis, whether treated or not. Non-​
treponemal tests are quantified and can distinguish between active disease (ratio >:6) and
inactive, treated disease. Quantitative VDRL/​RPR helps stage the infection and can indicate the
need for treatment in some cases, where the patients have been previously treated and may have
been re-​infected. The presence of positive treponemal testing alongside high RPR indicates active
infection and need for systemic therapy. False positive results may occur with any of the serological
tests and causes include lupus, RA, and Lyme disease; the titre is usually low in false positive results
in comparison to most true positives (Table 6.4).
Mock Exam | ANSWERS 189

Table 6.4  Syphilis testing


Treponemal test
Positive Negative
Positive Active, untreated syphilis False positive
Negative Previously treated syphilis or inactive untreated syphilis True negative
(confirm with history)
False positive (confirm with second treponemal test)

Kingston M, et al. UK national guidelines on the management of syphilis 205. Int J STD AIDS
206;27:42–​46.

40. Answer: D
Syndromes with ophthalmic manifestations are popular topics. Susac’s syndrome is a rare condition
characterized by the clinical triad of encephalopathy, sensorineural hearing loss, and branch retinal
artery occlusion(s) (BRAO). On FFA sectoral vessel hyperfluorescence is seen. MRI shows a distinctive
pattern of corpus callosum lesions that differ from those seen in demyelination. Cogan syndrome is
characterized by vertigo, hearing loss, and interstitial keratitis; BRAOs are not a feature. In demyelinating
disease, intraocular involvement can include intermediate uveitis and vessel sheathing, due to perivascular
leucocytes, but not BRAOs. Venous sinus thrombosis can cause headache, cranial nerve palsies,
papilloedema, and neurological disturbance but is not characterized by the development of BRAO.
Susac JO, et al. MRI findings in Susac’s syndrome. Neurology 2003;6:783–​7.

4. Answer: B
Löfgren’s syndrome is an acute form of sarcoidosis characterized by polyarthritis or polyarthralgia,
bilateral hilar lymphadenopathy, and erythema nodosum. Variant forms are sometimes observed.
Serum ACE levels are elevated in approximately 60% of patients with sarcoidosis and can therefore
be within normal limits. Rheumatoid arthritis is an important differential but negative autoantibodies
and the presence of bilateral hilar lymphadenopathy (BHL) support the diagnosis of Löfgren’s
syndrome. Heerfordt’s syndrome is a different manifestation of sarcoidosis characterized by facial
nerve palsy, fever, anterior uveitis, and parotid enlargement.
Iannuzzi MC, et al. Sarcoidosis. N Engl J Med 2007;357:253–​65.

42. Answer: C
Alport syndrome is a disorder of basement membrane type IV collagen and features ocular, renal,
and cochlear involvement manifesting as sensorineural deafness, progressive renal failure, and
characteristic ocular signs. Renal disease progresses from microscopic haematuria to progressive
proteinuria and insufficiency. Ocular examination can be very useful for the diagnosis of the
condition, particularly the lens, for evidence of anterior/​posterior lenticonus or cataract (anterior
polar or cortical). Fleck retinal changes are also observed and sometimes posterior polymorphous
dystrophy. A description of anterior lenticonus, fleck retina, and early onset renal failure is strongly
suggestive of this condition. Posterior subcapsular cataract is not a feature of Alport syndrome and
is observed in uveitis, corticosteroid, and radiation exposure, MD, and as age-​related lens change.
Colville DJ, Savige J. Alport syndrome. A review of the ocular manifestations. Ophthalmic Genet
997;8:6–​73.
Savige J, et al. Ocular features in Alport syndrome: pathogenesis and clinical significance. Clin J Am Soc
Nephrol 205;0:703–​9.
190 Mock Exam | ANSWERS

43. Answer: C
Headache and eye pain are a common presentation and cause diagnostic confusion for neurologists
and ophthalmologists alike. The key to this question is knowledge of primary and secondary
headache syndromes and being able to distinguish them from secondary headaches. High-​flow
oxygen can be effective for cluster headache (CH) and is used as a therapeutic trial to aid diagnosis.
However, CH is defined as attacks of excruciating pain lasting 30–​80 minutes with absence of pain
between attacks. Hemicranial continua is a much rarer primary headache disorder defined as strictly
unilateral, relentless headache or facial pain. The response to indomethacin is swift and exquisite.
Sumatriptan is the treatment of choice for abortion of migraine attacks but the clinical features are
more consistent with hemicranias than migraine. Herpes zoster can convincingly mimic primary
headaches, but usually either abates or declares itself the typical skin lesions or at least one of its
ocular manifestations.
Prakash S, Adroja B. Hemicrania continua. Ann Indian Acad Neurol 208;2:S23–​30.

44. Answer: D
The diagnosis of delayed visual maturation is really done retrospectively. It can be isolated in a child
with no other anomalies, or associated with mental retardation and/​or seizure. It can also include
children with a primary visual abnormality and a superimposed visual maturation delay. This is an
area where the ophthalmologist and the paediatrician should work together well. The other causes
of normal eye exam and subnormal vision are cortical visual impairment in whom the visual evoked
potential (VEP) will be abnormal but not in all children. There might be neurodevelopmental
abnormalities.
In the early stages of inherited retinal dystrophies, the eye exam can be normal, however it is
normally associated with high myopic or hypermetropic refractive error, positive family history, and
abnormal ERG.
Pehere N, et al. Cerebral visual impairment in children: causes and associated ophthalmological
problems. Indian J Ophthalmol 208;66:82–​5.
Weiss AH, et al. The infant who is visually unresponsive on a cortical basis. Ophthalmology
200;08:2076–​87.

45. Answer: B
Diagnosis of euryblepharon is based on several clinical features:
• Bilateral symmetrical enlargement of the horizontal palpebral apertures
• Elongated lid margins. The horizontal palpebral fissure length is increased to approximately
35 mm from the average length of 28–​30 mm
• Vertical shortening of eyelid skin
• Downward and lateral displacement of outer canthi
• Other features include lateral ectropion, reduced blink rate, lagophthalmos, with exposure
keratopathy
McCord CD Jr, et al. Congenital euryblepharon. Ann Ophthalmol 979;:27–​24.

46. Answer: B
The ophthalmologist should be concerned for PHACES syndrome when a haemangioma is
segmental and over 5 cm. The acronym PHACE stands for:
P—​Posterior fossa brain malformations
Mock Exam | ANSWERS 191

H—​Haemangiomas
A—​Arterial lesions (blood vessel abnormalities in the head or neck)
C—​Cardiac abnormalities (aortic coarctation)
E—​Eye abnormality
An echocardiogram to exclude cardiac anomalies such as coarctation of aorta, other aortic arch
anomalies, and other vascular anomalies is necessary. An MRI/​MRA scan of the head and neck is
mandatory to exclude Dandy–​Walker malformation and other posterior fossa malformations.
Haggstrom AN, et al. Risk for PHACE syndrome in infants with large facial hemangiomas. Paediatrics
200;26:48–​26.
Metry DW, et al. The many faces of PHACE syndrome. J Pediatr 200;39:7–​23.

47. Answer: D
Orbital cellulitis occurs as a secondary extension of acute or chronic bacterial sinusitis, especially
the ethmoid sinusitis. Intravenous antibiotic treatment is the choice for children of <9 years of age
with orbital subperiosteal abscess as it is commonly caused by single microbial organisms, whereas
surgical intervention for orbital subperiosteal abscess is more likely to be required if the children are
over 9 years of age. In addition, the latter group is more commonly associated with polymicrobial
and anaerobic infection. Clinical improvement does not correlate well with repeat CT scan analysis
as it can take up to 72 hours to see radiological improvement. Blindness occurs in up to %
of cases.
Liao JC, Harris GJ. Subperiosteal abscess of the orbit: evolving pathogens and the therapeutic protocol.
Ophthalmology 205;22:639–​47.
Nageswaran S, et al. Orbital cellulitis in children. Pediatr Infect Dis J 2006;25:695–​9.

48. Answer: D
All four conditions are a form of craniosynostosis, which refers to partial or complete premature
fusion of cranial sutures. Ocular hypertelorism, proptosis, beaking of the nose, and midfacial
hypoplasia are common facial features of the craniosynostosis. These four conditions can be
remembered with the mnemonic ‘CAMP’.
(a) Crouzon syndrome is associated with normal intelligence and normal hands and feet. The
severity of facial deformity is milder than of  Apert.
(b) Apert syndrome in addition has severe symmetrical syndactly of fingers and toes. Learning
disability requiring special education is also a common accompaniment.
(c) Muenke syndrome features include unilateral or bilateral coronal synostosis, proptosis,
downward slanting palpebral fissure, hearing loss, developmental delay, and specific bone
anomalies of the hand and feet.
(d) Pfeiffer syndrome type  has the classic phenotype of brachycephaly, midface hypoplasia,
broad, radially deviated thumbs, and/​or big toes along with normal intelligence.
Kutkowska-​Kaźmierczak A, et al. Craniosynostosis as a clinical and diagnostic problem: molecular
pathology and genetic counseling. J Appl Genet 208;59:33–​47.

49. Answer: A
The features are suggestive of viral conjunctivitis. The diagnosis and management of ophthalmia
neonatorum are based on the time of onset and characteristics of conjunctivitis. Some commonly
192 Mock Exam | ANSWERS

used treatment regimes are given next, but always refer to the local microbiology/​infectious disease
guideline or discuss with the respective team (Table 6.5).

Table 6.5  Commonly used treatment regimes for viral conjunctivitis


Types of conjunctivitis Onset Features Management
Chemical (% silver nitrate) At birth Watery % silver nitrate solution
is no longer in common
use
Viral –​4 days Non-​purulent discharge, hazy corneaSee Option A
due to dendritic ulcer/​stromal
keratitis
Gonococcal <5 days Severe purulent discharge See Option C
Bacterial, non-​gonococcal 4–​5 days Purulent discharge See Option B
Chlamydia 5–​4 days Mucopurulent discharge See Option D

Data from Allen UD, Robinson JL. Prevention and management of neonatal herpes simplex virus infections. Paediatr Child Health
204;9:20-6; and Ophthalmia neonatorum. BCSC series: Pediatric Ophthalmology and Strabismus. American Academy of
Ophthalmology 200.

50. Answer: B
Congenital megalocornea is a non-​progressive, enlarged cornea with a horizontal diameter of
more than 3 mm in the absence of congenital glaucoma. Myopia is the most common associated
refractive error. It is rare and usually bilateral, inherited as a X-​linked trait in most instances and
90% are males. The condition maps to Xq2–​22. Autosomal dominant inheritance has also been
reported. Management consists of careful observation for complications such as cataract formation,
dislocated lens, and glaucoma.
Meire FM, et al. X-​linked megalocornea. Ocular findings and linkage analysis. Ophthalmic Paediatr Genet
99;2:53–​7.

5. Answer: A
The Parks–​Bielschowsky or Park’s three-​step test is used to isolate single paretic muscle in acquired
vertical diplopia. The test works by observing the vertical deviation in primary gaze, left and right gaze,
and right and left head tilt. Where the hyperdeviations are of the greatest magnitude indicate where
the paretic muscle should be working maximally. The following is a diagram (Figure 6.4) for reference.
Step : If the right eye is the hyperdeviated eye, we know that the muscle that is failing is either
responsible for pulling the right eye down or for pulling the left eye up. Our suspects in the right
eye are the right superior oblique (RSO) or right inferior rectus (RIR). In the left eye, it is the left
superior rectus (LSR) or LIO.
Step 2: When we look at the right gaze, the muscles most responsible for the eyes’ vertical
position are the right superior and inferior rectus muscles and the left superior and inferior oblique
muscles.
Step 3: When the head is tilted to the left, the right eye needs to turn outwards (excyclotorsion)
and the left eye needs to turn inwards (incyclotorsion). The muscles responsible for these
movements are the RIO, RIR, and the LSO, LSR.
Mock Exam | ANSWERS 193

Right eye Left eye


SR IO IO SR

LR MR MR LR

IR SO SO IR

Figure 6.4  

In this case by process of elimination the affected muscle is the RIR.


American Academy of Ophthalmology. Three Step Test for Cyclovertical Muscle Palsy, 209. Available
at: http://​eyewiki.aao.org/​Three_​Step_​Test_​for_​Cyclovertical_​Muscle_​Palsy

52. Answer: A
Greenstick orbital floor fractures occur in paediatric population where the fracture fragment can
act as a ‘trap door’ entrapping the interior rectus muscle. This can elicit the oculocardiac reflex
producing bradycardia, nausea, syncope, and rarely bradycardia on attempted eye movement
against the entrapped muscle. It therefore requires urgent surgical release. All the other criteria are
traditional indicators for surgical correction of an orbital fracture but not on an urgent basis.
Boyette JR, et al. Management of orbital fractures: challenges and solutions. Clin Ophthalmology
205;9:227–​37.

53. Answer: D
Because of the extensive anastomoses of facial vessels, it is possible to inject intra-​arterial embolus
of filler inadvertently in almost any region of the face. Highest risk areas are injections around the
glabellar, nasal dorsum, nasojugal, and nasolabial folds.
Humzah MD, et al. The treatment of hyaluronic acid aesthetic interventional induced visual loss
(AIIVL): a consensus on practical guidance. J Cosmet Dermatol 209; 8():7–​6.

54. Answer: A
Severe thyroid eye disease is characterized by signs of sight-​threatening pathology. The most
common of these is dysthyroid optic neuropathy due to compression or stretch resulting in a
RAPD, unless equally bilateral. Other presentations of severe disease are corneal exposure, sight-​
threatening raised intraocular pressure, and globe subluxation.
Chemosis, esotropia, and lid malposition can occur in moderate TED.
Perros P, et al. Thyroid eye disease. BMJ 2009;338:b560.

55. Answer: C
Fasanella Servat is a destructive and outdated operation that has a high rate of postoperative
complications, so its use is in decline. Frontalis sling is required when the levator function is poor,
usually below 4 mm. Levator advancement can be achieved via an anterior or posterior approach,
194 Mock Exam | ANSWERS

and can be adjusted according to the degree of ptosis and levator function. Conservative amblyopia
management will not have long-​term success when the visual axis is occluded.
Finsterer J. Ptosis: causes, presentation and management. Aesthetic Plast Surg 2003;27:93–​204.

56. Answer: C
Cicatricial ectropion is the turning out of the lid due to shortage within the anterior lamella, usually
contracture of the skin. This can be caused by trauma, burns, scarring from infection (e.g. HZO),
skin tumours (e.g. BCC), and medications. Lid laxity can be a risk factor for developing a non-​
cicatricial entropion or ectropion. OCP can cause a cicatricial entropion due to contracture of the
conjunctiva turning the lid in. Dacryoadenitis can cause a mechanical ptosis.
Bedran EG, et al. Ectropion. Semin Ophthalmol 200;25:59–​65.

57. Answer: D
The causes of enophthalmos can be broadly divided into three categories: () structural alternation
in bony orbit; (2) orbit fat atrophy; and (3) retraction. Small eye may cause apparent enophthalmos.
All the options listed in this question are causes of enophthalmos, but silent sinus syndrome (SSS)
is the most common where there is no history of trauma. As the name suggested, SSS usually
presents in a silent/​asymptomatic manner. Although it is supposedly linked with a history of
sinusitis, there is often very little history. Sclerosing metastases usually arise from breast primary
and, like scleroderma, it can cause muscle fibrosis and orbital fat atrophy, resulting in enophthalmos.
Athanasiov PA, et al. Non-​traumatic enophthalmos: a review. Acta Ophthalmol 2008;86:356–​64.

58. Answer: B
All except pain can be features of both SCC and BCC. SCC is an aggressive tumour, has a
propensity to spread via perineural invasion so can cause pain, dysaesthesia, or anaesthesia. The
overall rate of regional lymph node metastases ranges from 0% to 25%.
Thosani MK, et al. Periocular squamous cell carcinoma. Dermatol Surg 2008;34:585–​99.

59. Answer: A
Various clinical, histopathologic, cytogenetic features and gene expressions have been implicated in
the prognosis of uveal melanoma, in terms of risk of metastasis and mortality. These include older
age at presentation, male gender, larger tumour basal diameter and thickness, ciliary body location,
epithelioid cell type, high mitotic activity, monosomy 3, loss of chromosomes p, 6q, or 8p, and gain
of 8q. The following review paper provides a very good summary on this subject.
Kaliki S, et al. Uveal melanoma: Estimating prognosis. Indian J Ophthalmol 205;63:93–​02.

60. Answer: B
Non-​Hodgkin’s lymphomas constitute one-​half of the malignancies arising in the orbit and the
ocular adnexae. Mucosa-​associated lymphoid tissue (MALT)-​type lymphoma is the most common
histological category in this anatomic region. The following is a useful summary table of the most
common intraocular and orbital tumours in children and adults (Table 6.6):
Castillo BV Jr, Kaufman L. Pediatric tumors of the eye and orbit. Pediatr Clin North Am 2003;50:49–​72.
Margo CE, Mulla ZD. Malignant tumors of the orbit. Analysis of the Florida cancer registry.
Ophthalmology 998;9:835–​46.
Ting DS, et al. A 0-​year review of orbital biopsy: the Newcastle Eye Centre Study. Eye (Lond)
205;29:62–​6.
Mock Exam | ANSWERS 195

Table 6.6  The most common intraocular and orbital tumours in children and adults
Children Adult
Benign Intraocular Very rare Choroidal naevus
Orbital Dermoid/​epidermoid cyst; capillary Cavernous haemangioma
haemangioma (2nd)
Malignant Intraocular Retinoblastoma Primary: Choroidal melanoma
Secondary: Choroidal metastases
Orbital Primary: Rhabdomyosarcoma Lymphoproliferative tumour
Secondary: Neuroblastoma

6. Answer: C
This is a question examining the candidate’s knowledge on the systemic association of retinal
astrocytoma, which is a rare benign glioma that typically presents in childhood and adolescence. It
is related to tuberous sclerosis and neurofibromatosis. Tuberous sclerosis is associated with cardiac
rhabdomyoma, pulmonary lymphangioleiomyoma, renal angiomyolipoma, subependymal giant
cell astrocytoma, and adenoma sebaceum. Pheochromocytoma is linked to Von Hippel–​Lindau
syndrome.
O’Shea WF, Powers JE. Solitary retinal astrocytoma. J Am Optom Assoc 99;62:59–​24.
Rowley SA, O’Callaghan FJ, Osborne JP. Ophthalmic manifestations of tuberous sclerosis: a population-​
based study. Br J Ophthalmol 200;85:420–​3.

62. Answer: C
Schwartz–​Matsuo syndrome is elevated IOP and mild anterior uveitis (AU) associated with
rhegmatogenous RD. The RD is typically shallow, with peripheral retinal breaks particularly retinal
dialysis. The only evidence of uveitis is anterior chamber (AC) cells; posterior synechiae, flare,
keratotic precipitates or posterior uveitis indicate an inflammatory condition. The IOP is typically
high with fluctuations. AC photoreceptor outer segments and aqueous outflow obstruction is a
proposed mechanism for raised IOP. Exudative RD can complicate uveitis involving the posterior
segment—​there is only AU here so Option A is incorrect. The presence of a tear indicates this is a
rhegmatogenous rather than an exudative detachment. Posner–​Schlossman syndrome is a form of
recurrent unilateral AU, with very few cells, fine KPs, and very high IOP but is not associated with
RD. Ocular trauma is a risk factor for sympathetic ophthalmia—​this is excluded because the fellow
eye is normal.
Matsuo T. Photoreceptor outer segments in aqueous humor: key to understanding a new syndrome.
Surv Ophthalmol 994;39:2–​33.
Mitry D, et al. Photoreceptor outer segment glaucoma in rhegmatogenous retinal detachment. Arch
Ophthalmol 2009;27:053–​4.

63. Answer: A
Juvenile idiopathic arthritis (JIA) is the commonest rheumatic disease in children and JIA-​related
uveitis is the most common extra-​articular manifestation. The risk factors for chronic AU in JIA
patients include presence of antinuclear antibody (ANA), young age at diagnosis, female gender,
and oligoarticular disease (involvement of up to four joints). In contrast, male gender, HLA-​B27
positivity, and enthesitis-​related arthritis are risk factors for acute AU.
196 Mock Exam | ANSWERS

Sen ES, Ramanan AV. Juvenile idiopathic arthritis-​associated uveitis. Best Pract Res Clin Rheumatol
207;3:57–​34.

64. Answer: C
Syphilis can reactivate in the eye many years after initial primary infection. Manifestations of ocular
syphilis are many but the pale macular lesion with disruption at the level of the photoreceptors
is typical of posterior placoid syphilitic uveitis. A metastatic deposit would cause a raised, fundus
lesion, its colour dependent on composition and the state of the retinal pigment epithelium.
Cancer-​associated retinopathy may cause outer retinal damage but there will not be any macular
lesion. Tuberculosis can manifest in a variety of ways but would not be expected to cause the
described OCT changes. The absence of haemorrhage and inner retinal disturbance goes against
zoster-​associated acute retinal necrosis.
Wells J, et al. Ocular syphilis: the re-​establishment of an old disease. Eye (Lond) 208;32:99–​03.

65. Answer: B
Adalimumab (Humira®) is a biologic agent. It is humanized antitumour necrosis factor alpha (TNF-​
α) monoclonal antibody. TNF-​alpha is an important target in non-​infectious uveitis and infliximab
is another anti-​TNF agent. Pivotal trials for the efficacy of adalimumab in the treatment of uveitis
are the SYCAMORE and VISUAL I and II studies. Rituximab is a different biologic agent and is
monoclonal antibody against CD20, a B-​cell marker, used for treatment of systemic vasculitides.
Anti-​VEGF therapy is used for treatment of AMD, RVO, and diabetic maculopathy. Interferon
therapy is indicated for the treatment of Behçet’s disease, including uveitis.
Ramanan AV, et al. Adalimumab plus methotrexate for uveitis in juvenile idiopathic arthritis. N Engl J
Med 207;376:637–​46.

66. Answer: A
The Jarisch–​Herxheimer reaction is an acute febrile illness with headache, myalgia, chills, and rigours
which resolves within 24 hours. This is common in early syphilis and is usually not clinically significant
unless there is neurological or ophthalmic involvement. The UK syphilis guidelines recommend
steroid therapy when treating neurological (includes ophthalmic) syphilis with prednisolone 40–​
60 mg for 3 days starting 24 hours before antitreponemal antibiotics. A longer course
of oral steroid may be necessary for severe ophthalmic involvement but it is important that the
timing of initiation is carefully coordinated with antibiotic therapy because steroid exposure without
appropriate antitreponemal treatment can cause worsening of syphilitic disease.
Kingston M, et al. UK national guidelines on the management of syphilis 205. Int J STD AIDS
206;27:42–​46.

67. Answer: A
Following is the summary of the pharmacology of topical intraocular pressure-​lowering medication
(Table 6.7).
Mock Exam | ANSWERS 197

Table 6.7  Pharmacology of topical intraocular pressure-​lowering medications


Medications Mechanisms Examples
Prostaglandin analogue i uveoscleral outflow Latanoprost, bimatoprost
β-​blocker* d aqueous production Betaxolol, timolol, carteolol
Carbonic anhydrase inhibitor d aqueous production Brinzolamide, dorzolamide
Alpha-​2 agonist i uveoscleral outflow + Apraclonidine, brimonidine
d aqueous production
Cholinergic i trabecular outflow Pilocarpine

*Betaxolol is a β-​ agonist (cardioselective), which has less pulmonary side effects. Carteolol is a non-​selective β-​agonist but has
intrinsic sympathomimetic activity, which increases the systolic blood pressure, heart rate, and cardiac contractility.
Data from Tataru CP, Purcarea VL. Antiglaucoma pharmacotherapy. J Med Life 202;5:247–​5.

68. Answer: C
Fluoroquinolone (FQ) is an antibiotic that inhibits bacterial DNA synthesis via interaction with
DNA gyrase and topoisomerase. It has been reported to induce tendinitis and tendon rupture.
Ciprofloxacin was found to be the causal agent in 90% of the cases. Risk factors for FQ-​induced
tendon rupture include age >60 years, steroid use, renal impairment, diabetes mellitus, and a
history of musculoskeletal (MSK) disorders. The tendinopathy may occur between a few hours
and few months after the start of FQ treatment, with a median onset of 6 days, and the effect
is dose dependent. The pathophysiology remains elusive; however, various concepts have been
introduced. These include FQ-​related direct cytotoxic effect on enzymes found in MSK tissues,
the chelating properties against several metal ions, which can cause direct toxicity to type 
collagen synthesis and promote collagen degradation. Ciclosporin can cause a range of side effects,
including hypertension, hyperglycaemia, hyperlipidaemia, gingival hyperplasia, and hirsutism among
others. Cyclophosphamide can cause myelosuppression, cardiac, pulmonary, and liver toxicity,
haemorrhagic cystitis, and neoplasms.
Kim GK. The risk of fluoroquinolone-​induced tendinopathy and tendon rupture: What does the
clinician need to know? J Clin Aesthet Dermatol 200;3:49–​54.

69. Answer: C
IFIS was first described by Chang and Campbell in 2005. The current use of tamsulosin, alfuzosin,
terazosin, benzodiazepines, quetiapine, and finasteride are all associated with IFIS. The duration
of α-​blocker use was not found to be associated with IFIS. The recent review also highlighted the
elevated risk of IFIS in patients taking rivastigmine with a short axial length. Quetiapine may also
increase the risk of acute primary angle closure due to its anticholinergic effect.
Chatziralli IP, et al. Risk factors for intraoperative floppy iris syndrome: a prospective study. Eye (Lond)
206;30:039–​44.

70. Answer: A
Botulinum toxin is a neurotoxin that is derived from Clostridium botulinum, a Gram-​positive
anaerobic bacterium. It primarily acts by binding to the presynaptic high-​affinity recognition
sites on the cholinergic nerve terminals and thereby decreasing the presynaptic release of ACh.
Myasthenia gravis is caused by autoantibodies blocking the postsynaptic ACh receptors whereas
Lambert–​Eaton myasthenic syndrome is caused by autoantibodies blocking the presynaptic
voltage-​gated calcium channels at the neuromuscular junction, reducing the release of presynaptic
198 Mock Exam | ANSWERS

ACh. Organophosphate inhibits acetylcholinesterase, leading to accumulation of ACh at the


neuromuscular junction.
Barnes M. Botulinum toxin—​mechanisms of action and clinical use in spasticity. J Rehabil Med 2003;(4
Suppl):56–​9.

7. Answer: B
Bacteria that take up crystal violet dye are Gram positive and those that allow crystal violet dye to
wash off are Gram negative. The following are the common staining techniques used for corneal
samples (Table 6.8).

Table 6.8  Common staining techniques for corneal samples


Staining techniques Intended microorganisms
Gram Bacteria, fungi, and acanthamoeba
Giemsa Bacteria, fungi, and acanthamoeba
Gomori methenamine silver (GMS) Fungi
Calcofluor white Fungi and acanthamoeba
Periodic acid Schiff Fungi and acanthamoeba
Ziehl–​Neelsen Fungi, acanthamoeba, mycobacteria, and nocardia

Denniston AKO, Murray PI. Oxford Handbook of Ophthalmology, 3rd edition. Chapter 7: Cornea; p. 223.
Oxford, UK: Oxford University Press, 204.
Sharma S. Diagnosis of infectious diseases of the eye. Eye (Lond) 202;26:77–​84.

72. Answer: D
FFA incorporates a blue excitation filter and a yellow green barrier filter. The sodium fluorescein
used is 70–​80% bound to plasma albumin. It is metabolized by the liver and excreted by the kidney
in 24 hours. FFA is not found to cause a high rate of birth anomalies or complications during
pregnancy; however, it is generally avoided unless its use is deemed absolute necessary. With the
current advancing techniques in retinal imaging such as OCT and OCT angiography, the use of FFA
in pregnancy can usually be avoided.
Halperin LS, et al. Safety of fluorescein angiography during pregnancy. Am J Ophthalmol
990;09:563–​6.

73. Answer: B
On fluorescein angiography, APMPPE has a characteristic appearance with lesion hypofluorescence
in the early stages, due to blockage, followed by lesion staining in the late stages of the angiogram.
On indocyanine green angiography (ICGA) the lesions remain hypocyanescent throughout the
angiogram and lesions may be more numerous than detected clinically. APMPPE is typically a
bilateral disease, preceded by a prodromal illness, and has no sex predilection (Table 6.9).
Salvatore S, et al. Multimodal imaging in acute posterior multifocal placoid pigment epitheliopathy
demonstrating obstruction of the choriocapillaris. Ophthalmic Surg Lasers Imaging Retina
206;47:677–​8.
Mock Exam | ANSWERS 199

Table 6.9  Appearance and presentation characteristics of APMPPE


History Clinical FAF FFA ICG
MEWDS Young Unilateral; Multiple hyper-​AF Early Hypo-​F lesions
females; flu No-​little vitritis; grey/​ spots hyperfluorescence
type illness white dots-​spots 00–​ (hyper-​F) spots;
200 µm lesions, macular Late hyper-​F
to mid periphery optic Wreath like
disc swelling; foveal macular change
granularity
APMPPE Young Bilateral white-​yellow Hypo-​AF lesions Early: hypo-​F Hypo-​F
patients; flu placoid lesions (can be lesions lesions.
type illness large) posterior pole to Late: hypo-​F lesions More
mid periphery. Vitritis numerous than
may be present seen on FFA
PIC Young No-​mild vitritis. Hypo-​AF lesions, Hyper-​F lesions Hypo-​F
myopic; Multiple white lesions edge hyper-​FAF if Window defects More
typically (50–​200 µm), become active. Edge fades as if scars numerous than
female. atrophic scars +/​–​ becomes inactive. seen on FFA
May present pigmentation in time. Atrophic lesions
with CNV Peripapillary atrophy. hypo-​FAF
Linear lesions in
periphery. CNV as
complication
MFC Young Multiple lesions, can Hypo-​AF lesions Hyper-​F lesions Hypo-​F
patients be large 00–​300 µm corresponding
or bigger in posterior to chorioretinal
pole and periphery. atrophy. Hyper-​FAF
Vitritis present. CNV is can be seen within
complication active lesions

CNV, choroidal neovascularization.

74. Answer: B
pERG assesses macular function. P50 assesses macular cone function and N95 assesses macular
ganglion cells function. Amplitudes, peak times, and N95/​P50 ratio (typically >.) are key
components in interpreting the pattern electroretinogram (PERG).
Holder GE. Pattern electroretinography (PERG) and an integrated approach to visual pathway
diagnosis. Prog Retin Eye Res 200;20:53–​6.

75. Answer: A
W4D test is a useful orthoptic test that can be used with both distance and near fixation to
differentiate between suppression, abnormal retinal correspondence, and BSV. However, it can only
be interpreted if a manifest squint is present or absent. The patient is asked to wear a green lens
in front of the right eye to filter out all colours except green, and a red lens in front of the left eye
to filter out all colours except red. He or she will be asked to view a box with four lights: one red
(top), two green (middle) and one white (bottom).
200 Mock Exam | ANSWERS

Following is the interpretation of the results:


(A) All four lights = presence of BSV or harmonious abnormal retinal correspondence (ARC) (if
manifest deviation is present)
(B) Three green lights = suppression of left eye (because the right eye sees the green light only)
(C) Two red lights = suppression of right eye
(D) Three green and two red lights = diplopia
Kanski JJ, Bowling B. Clinical Ophthalmology: A Systematic Approach, 7th edition.
Chapter 8: Strabismus: Clinical evaluation; pp. 750–​4. Edinburgh, UK/​New York, NY: Elsevier/​
Saunders, 20.

76. Answer: A
The binocular status such as stereopsis can be measured with several types of orthoptic tests,
including Frisby, Lang, Titmus, and TNO. Frisby and Lang tests do not require any glasses whereas
Titmus and TNO require polaroid glasses and red-​green glasses, respectively. It is also important
to remember that some of these tests such as Frisby, Lang, and Titmus tests may give monocular
clues. Hess chart also requires red-​green glasses for dissociating the eyes for measurement of
ocular deviation.
Lee J, McIntyre A. Clinical tests for binocular vision. Eye (Lond) 996;0:282–​5.

77. Answer: D
The image shown in Figure 6.3 is an MRI T2-​weighted scan with FLAIR sequence. On the first
glance, the candidate might confuse this with a T-​weighted scan as the vitreous and cerebrospinal
fluid (CSF) appear dark. However, in the T2-​weighted scan, with or without FLAIR sequence, the
grey matter should appear in lighter grey (or relatively hyperintense) as compared to the white
matter, which should appear in darker grey (or relatively hypointense). This is the opposite for T-​
weighted scan. MRI FLAIR sequence is similar to a T2-​weighted image but the former has the ability
to suppress signals from cerebrospinal fluid (CSF).
Simha A, et al. Magnetic resonance imaging for the ophthalmologist: a primer. Indian J Ophthalmol
202;60:30–​0.
Yau GS, et al. Neuromyelitis optica spectrum disorder in a Chinese woman with ocular myasthenia
gravis: first reported case in the Chinese population. Neuroophthalmology 204;38:40–​4.

78. Answer: B
Serological measurements of antiacetylcholine receptor (AChR) antibody are highly sensitive for
generalized myasthenia gravis (MG; around 80%) but lower in ocular myasthenia gravis (around
45%). In 20% of patients with MG that have negative anti-​AChR antibody, 30–​40% were found to
have antityrosine kinase MuSK antibody. These patients may have symptoms very similar to AChR-​
positive MG, but some patients may have a bulbar form with few ocular symptoms. Anti-​low-​
density lipoprotein receptor-​related protein (LRP4) antibody has recently been found in patients
with double antibodies (AChR and MuSK) negative MG. Anticyclic citrullinated peptide (CCP) is
useful in diagnosing rheumatoid arthritis, with high specificity, present early in the disease process,
and has the ability to identify severe cases.
Niewold TB, et al. Anti-​CCP antibody testing as a diagnostic and prognostic tool in rheumatoid
arthritis. QJM 2007;00:93–​20.
Rivner MH, et al. Muscle-​specific tyrosine kinase and myasthenia gravis owing to other antibodies.
Neurol Clin 208;36:293–​30.
Mock Exam | ANSWERS 201

79. Answer: B
About day 22, two small grooves develop either side of the developing forebrain in the neural folds.
These are called the optic sulci. As the neural tube closed, the grooves become out pockets known
as optic vesicles. They are formed from the diencephalon at approximately day 25 of gestation.
The following is the timeline of ocular embryology (in gestation):
• 23 days: optic pits appearance
• 25 days: invagination of optic vesicle
• 28 days: induction of  lens
• 33 days: closure of embryonic fissure (starts between optic nerve and iris, then progresses
anteriorly and posteriorly)
Barishak YR. Embryology of the eye and its adnexae. Dev Ophthalmol 992;24:–​42.

80. Answer: A
There are typically four cell types in choroidal melanoma, namely type A, type B, epithelioid cells,
and a mixture of all three cell types. It has been shown that epithelioid cell type is associated with
the worst prognosis compared to other cell types.
Sedoon JM, et al. Death from uveal melanoma. Number of epithelioid cells and inverse SD of
nucleolar area as prognostic factors. Arch Ophthalmol 987;05;80–​6.

8. Answer: A
OAT deficiency is also known as gyrate atrophy (GA) of the choroid and retina. This is an
autosomal recessive dystrophy caused by mutations in the gene for ornithine aminotransferase
(OAT), located on chromosome 0. This presents with progressive chorioretinal degeneration,
myopia, night blindness, and eventually complete blindness in the fourth or fifth decade. OAT
normally maintains a stable ornithine level in mitochondria. In OAT deficiency, hyperornithinaemia
inhibits arginine:glycine amidinotransferase (AGAT), causing creatine deficiency. Dietary restriction
of arginine has been used to treat some GA patients, the diet is very difficult to maintain and must
be monitored by paediatricians with experience in metabolic disease. Vitamin B6 (or pyridoxine)
treatment lowers the plasma ornithine levels in a small percentage of GA patients.
Kaiser-​Kupfer MI, et al. Gyrate atrophy of the choroid and retina: long-​term reduction of ornithine
slows retinal degeneration. Arch Ophthalmol 99;09:539–​48.

82. Answer: B
Slit-​lamp fundus lenses and indirect ophthalmoscopy with condensing lenses produce inverted,
magnified, and real images.
American Academy of Ophthalmology. BCSC Series, 20–​202: Chapter 8: Telescopes and optical
instruments, pp. 243–​82. San Francisco, CA: American Academy of Ophthalmology, 202.

83. Answer: A
ANOVA test is used to compare the mean of three or more groups. Mann–​Whitney U test (or
Wilcoxon rank-​sum test) is a non-​parametric test for examining the mean between two groups
and T-​test is the parametric test for examining the mean between two groups. Contingency
tables, including chi-​squared test and Fisher’s exact test, is used for examining categorical variables
between two or more groups. The following reference provides a good summary table of the
commonly used statistical tests.
Nayak BK, Hazra A. How to choose the right statistical test? Indian J Ophthalmol 20;59:85–​6.
202 Mock Exam | ANSWERS

84. Answer: A
The original TVT study looked at the 5-​year outcome of Tube Versus Trabeculectomy surgery in
patients with uncontrolled glaucoma who had previous ocular surgery. The main findings were:
• Tube shunt surgery had a higher success rate compared to trabeculectomy with mitomycin C
(MMC) during 5 years of follow-​up
• Both procedures were associated with similar IOP reduction and use of medical therapy at
5 years
• Additional glaucoma surgery was needed more frequently after trabeculectomy with MMC
than tube shunt placement
Following the TVT study, the PTVT (Primary Tube Versus Trabeculectomy) study was published
looking at patients with uncontrolled glaucoma with no previous ocular surgery. The study found
that patients in the trabeculectomy group had a higher success rate.
Gedde SJ, et al. Treatment outcomes in the Tube Versus Trabeculectomy (TVT) study after five years
of follow-​up. Am J Ophthalmol 202;53:789–​803.

85. Answer: B
One cup of fresh sliced apricots has 0.33 mg of zinc, one cup of cubed avocado has 0.96 mg of
zinc, three dates contain 0.32 mg of zinc, and one cup of sliced kiwi has 0.25 mg of zinc. The
original Age-​Related Eye Disease Study (AREDS) formulation contained 80 mg as zinc oxide and
AREDS2 contained 25 mg.
National Eye Institute (NIH). Available at: https://​nei.nih.gov/​faqs/​
macular-​degeneration-​areds-​and-​areds2

86. Answer: A
The DVLA driving standards is a popular question in both Part 2 written and oral exams.
Candidates are advised to familiarize with these standards. For a class  driver (car and motorcycle),
patient does not have to inform DVLA when only one eye is affected, unless vision of the affected
eye is non-​perceptive to light (NPL). In addition, patient will need to inform DVLA if the eye with
ocular hypertension (OHT) progresses to glaucoma.
Driver and Vehicle Licensing Agency (DVLA). Assessing Fitness to Drive: A Guide for Medical
Professionals, September 209. Available at: https://​www.gov.uk/​government/​uploads/​system/​uploads/​
attachment_​data/​file/​596959/​assessing-​fitness-​to-​drive-​a-​guide-​for-​medical-​professionals.pdf

87. Answer: B
Capacity evaluation is a two-​step process. Does the patient have mild cognitive impairment or
dementia (mini-​mental state examination and test letter and word fluency)? Regarding the decision,
can the patient understand, appreciate, give reasons, and communicate their decision? The patient
should have adequate and relevant information about the issue under discussion (disease, treatment
options, and so on). The clinician should use open-​ended questions. Capacity can be rated as
adequate, inadequate, and marginal.
Hegde S, Ellajosyula R. Capacity issues and decision-​making in dementia. Ann Indian Acad Neurol
206;9:S34–​9.
Wong JG, et al. Capacity to make health care decisions: Its importance in clinical practice. Psychol Med
999;29:437–​46.
Mock Exam | ANSWERS 203

88. Answer: C
The QALY is a measure of the value of health outcomes which makes the assumption that health is
a function of life and quality of life. A perfect health year is calculated as following:  year of life × 
utility value. The value is normally expressed 0–​ (0 is dead and perfect health ). For example, bed
ridden has a utility value of 0.5. Therefore,  × 0.5 = 0.5 QALY. NICE have used QALYs to evaluate
health since 203.
Weinstein MC, et al. QALYs: the basics. Value Health 2009;2:S5–​9.

89. Answer: D
Scleral buckling has the highest risk of transmission due to the likelihood of drainage of subretinal
fluid. CJD is a fatal human form of transmissible spongiform encephalopathy associated with an
accumulation of prion proteins in brain tissue. In humans, the disease can be familial (autosomal
dominant) or sporadic (usually in older patients). Variant CJD (vCJD), thought to be related to
ingestion of contaminated beef, usually occurs in younger patients. All are rare. Because the disease
can be very slowly progressive and can have a prolonged preclinical phase, there is a concern about
potential transmission in patients without any symptoms or signs of the disease. Posterior segment
and some orbital procedures are considered high risk; anterior segment operations are considered
low risk.
Gov.UK. Minimise Transmission Risk of CJD and vCJD in Healthcare Settings: Managing CJD/​vCJD Risk
in Ophthalmology. Annex L, 20. Available at: https://​assets.publishing.service.gov.uk/​government/​
uploads/​system/​uploads/​attachment_​data/​file/​209770/​Annex_​L_​-​_​Managing_​CJD_​vCJD_​risk_​in_​
ophthalmology.pdf

90. Answer: A
Patients planning to take hydroxychloroquine long term (i.e. over 5 years) should have a baseline
examination in a hospital eye department ideally within 6 months, but definitely within 2 months,
of starting therapy with a colour retinal photograph and SD-​OCT scans of the macula. Patients
should be referred for annual screening after 5 years of therapy and be reviewed annually thereafter
while on therapy. At each screening visit, patients should undergo 0–​2 Humphrey visual field
testing, followed by pupillary dilation and imaging with both SD-​OCT and wide-​field fundus
autofluorescence imaging (FAF). If wide-​field FAF is not available, FAF can be acquired in several
photographic fields to encompass the macula and extramacular areas. Patients with abnormalities
on wide-​field FAF with normal 0–​2 visual field test results should undergo 30–​2 visual field
testing on another date. Patients with persistent and significant visual field defects consistent with
hydroxychloroquine retinopathy, but without evidence of structural defects on SD-​OCT or FAF
may be considered for multifocal electroretinography. Screening before 5 years should only be
considered if additional factors exist (high-​dose prescribed, concomitant tamoxifen therapy, or renal
insufficiency). A useful aide memoir for guidelines is 5 × 5 rule (ideally keep dosage <5 mg/​kg/​day
and screen after 5 years of drug use).
The Royal College of Ophthalmologists. Hydroxychloroquine and Chloroquine
Retinopathy: Recommendations on Screening, 208. Available at: https://​www.rcophth.ac.uk/​wp-​
content/​uploads/​208/​07/​Hydroxychloroquine-​and-​Chloroquine-​Retinopathy-​Screening-​Guideline-​
Recommendations.pdf
INDEX

Notes
Tables and figures are indicated by t or f following the page number.
Page numbers in q refer to Question and a refer to Answer
vs. indicates a comparison or differential diagnosis

A alpha-​galactosidase deficiency, Fabry’s disease  75a


abetalipoproteinaemia (Bassen–​Kornzweig syndrome)  186a Alport syndrome  167q, 183a, 185a, 189a
abusive head trauma (shaken baby syndrome)  82q, 104a anterior lenticonus  33a
acanthamoeba keratitis (AK)  158q, 179a alveolar rhabdomyosarcoma  99–​100a
accommodative convergence/​accommodation (AC/​A) amaurosis fugax  143q, 152a
ratio 123q, 133a amblyopia, unilateral cataracts  183a
accommodative esotropia  81q, 101–​102a AMD (age-​related macular degeneration)  117q, 128a,
ACCORD Eye study  149a 175q, 202a
ACE (angiotensin-​converting enzyme)  166q American Society of Cataract and Refractive Surgery
acetazolamide 9q, 26a (ASCRS) 185a
acquired Brown syndrome  80q, 100a American Society of Retina Specialists (ASRS)  185a
acquired third nerve palsies  48q, 70t amniotic membrane  6q, 23a
acute anterior uveitis  116q, 126a ANA (antinuclear antibody)  195a
acute compressive optic neuropathy  85q, 107a analytic methods  143q, 153a
acute iatrogenic secondary angle glaucoma  10q, 28a anaphylactic shock, fluorescein angiography  51q, 74a
acute inferior macula, retinal detachment  36q, 54a ANCA (antineutrophil cytoplasmic antibodies)  112a, 159q
acute posterior multifocal pigment placoid epitheliopathy aneurysm, intracranial  123q, 133a
(APMPPE) 111a, 173q, 198a, 199t angiography
acute postoperative endophthalmitis  11q, 30a fluorescein  see fluorescein angiography
acute retinal necrosis (ARN)  72a indocyanine green see indocyanine green
acyclovir 150a angiography (ICGA)
adalimumab (Humira®) 171q, 196a angioid streaks  37q, 58a
Adie’s syndrome  67a, 187a angiotensin-​converting enzyme (ACE)  166q
Adie’s tonic pupil  44q, 65a aniridia 7q, 25a
adipose tissue, glucose receptor  138q, 146a anisocoria 165q, 187a
Advanced Glaucoma Intervention Study (AGIS)  8q, 25a Horner’s syndrome  187a
aflibercept (Eylea/​VEGF Trap)  58a, 116q, 126–​127a ANOVA test  201a
diabetic macular oedema  154a anterior ischaemic optic neuropathy, bilateral sequential optic
AGAT (arginine:glycine amidinotransferase)  201a neuropathy vs. 68a
Age-​Related Eye Disease Study (AREDS)  151a, 202a anterior lenticonus  33a
Age-​Related Eye Disease Study  2 (AREDS2) 128a anterior segment dysgenesis (Axenfeld–​Rieger syndrome)  7q,
age-​related macular degeneration (AMD)  117q, 128a, 140q, 21a, 25a
148a, 175q, 202a chromosomal analysis  159q, 180a
nutritional supplements  141q, 151a anterior segment optical coherence tomography
AGIS (Advanced Glaucoma Intervention Study)  8q, 25a (AS-​OCT)  121q, 132a
AK (acanthamoeba keratitis)  158q, 179a anterior uveitis syndrome  88q, 111a
Alagille syndrome  59a acute 116q, 126a
albinism 52q, 76a anti-​acetylcholine receptor (AChR) antibody  200a
α-​2 adrenergic agonists  197t antibiotics 127t
side effects  10q, 28t anti-​neutrophil cytoplasmic antibodies (ANCA)  112a, 159q
206 Index

anti-​nuclear antibody (ANA)  195a Benedikt syndrome  70t


anti-​treponemal antibodies, syphilis treatment  196a β-​blockers  197t
anti-​tumour necrosis factor agent  114a contraindications 125a
anti-​vascular endothelial growth factor (VEGF) side effects  10q, 28t
treatment 128a betaxolol 197t
Anton syndrome  52q, 76a BETT (Birmingham Eye Trauma terminology)  83q,
Apert syndrome  191a 104a, 104f
APMPPE (acute posterior multifocal pigment placoid bevacizumab, dexamethasone vs. 61a
epitheliopathy) 111a, 173q, 198a, 199t BEVORDEX study  61a
aponeurotic dehiscence  83q, 105a BHL see bilateral hilar lymphadenopathy (BHL)
aponeurotic ptosis  105t BIGH3 gene
apraclonidine 43q, 197t Meesmann dystrophy  18a
apricots, zinc  202a Meretoia’s syndrome  178a
aqueous misdirection (malignant glaucoma)  10q, 28a bilateral capillary abnormalities  40q, 62a
AREDS (Age-​Related Eye Disease Study)  151a, 202a bilateral granulomatous panuveitis  184a
AREDS2 (Age-​Related Eye Disease Study  2) 128a bilateral hilar lymphadenopathy (BHL)  189a
arginine:glycine amidinotransferase (AGAT)  201a new-​onset polyarthralgia  166q
Argyll Robertson pupil  187a bilateral inferonasal lens subluxation  12q, 31a
arm-​eye time, FFA  120q, 130a bilateral periorbital ecchymosis (raccoon eyes)  105a
ARN (acute retinal necrosis)  72a bilateral red eye  168q, 191–​192a
Arnold–​Chiari syndrome, Budd–​Chiari syndrome vs.  73–​74a bilateral retinal striae  116q, 127a
A-​scan  122q, 132a bilateral sequential optic neuropathy, anterior ischaemic optic
ASCRS (American Society of Cataract and Refractive neuropathy vs. 68a
Surgery) 185a bilateral superior oblique palsies  49q, 71a
AS-​OCT (anterior segment optical coherence bimatoprost 197t
tomography) 121q, 132a biometry repetition  161q, 182a
ASRS (American Society of Retina Specialists)  185a biomicroscopic classification  55a, 55t
autosomal dominant familial microspherophakia  182a birdshot chorioretinopathy  111a
Avellino cortical dystrophy  1a,  15–​16q HLA 88q, 112a
avocado, zinc  202a HLA-​A29  112a
Axenfeld–​Rieger anomaly  7q, 25a Birmingham Eye Trauma terminology (BETT)  83q, 104a, 104f
Axenfeld–​Rieger syndrome see anterior segment dysgenesis bitemporal hemianopia  123q, 134q
(Axenfeld–​Rieger syndrome) blepharophimosis, ptosis and epicanthus inversus syndrome
axial proptosis, unilateral  86q, 109a (BPES) 106a
azathioprine (AZA)  125a blepharophimosis syndrome type 2  84q, 106a
blindness
B cortical blindness  52q, 76a
bacterial endophthalmitis  143q, 153a see also vision loss
bacterial keratitis  5q,  21–​22a blood agar  131t
Bagolini glasses, torsion measurement  133a Blue Mountain Eye Study, age-​related macular
Bardet–​Biedl syndrome  186a degeneration 140q, 148a
Bartonella henselae infection blunt trauma, double vision  169q, 193a
cat-​scratch disease  73a blurred vision  166q, 171q, 189a, 196a
Parinaud’s oculo-​glandular syndrome  6q, 23a Borrelia burgdorferi infection, erythema chronicum
basal cell carcinoma (BCC) migrans 73a
squamous cell carcinoma vs. 170q, 194a botulinum toxin  172q, 197–​198a
upper lid  86q, 109a Bowen’s disease (conjunctival epithelial neoplasia)  16a
vismodegib 117q, 128a Bowman’s layer, curly fibres  4q, 20a
basic exotropia (BE)  98a BPES (blepharophimosis, ptosis and epicanthus inversus
Bassen–​Kornzweig syndrome (abetalipoproteinaemia)  186a syndrome) 106a
Bazex syndrome  85q, 107a brain heart infusion  131t
BCC see basal cell carcinoma (BCC) branch retinal artery occlusion (BRAO)  189a
BE (basic exotropia)  98a brimonidine 197t
Beauchamp, Tom  152a side effects  125a
Behçet’s disease  88q, 112a brinzolamide 197t
Crohn’s disease vs. 113a British National Formulary (BNF), acetazolamide
HLA associations  112a contraindications 125a
Index 207

Brown syndrome chemodenervation, acute onset of concomitant esotropia  97a


acquired 80q, 100a chest X-​ray, thymoma  67a
congenital 100a children
Bruch’s membrane  137q, 145a parental worries about vision  167q, 190a
Budd–​Chiari syndrome  51q,  73–​74a vision impairment  77q, 92a
Arnold–​Chiari syndrome vs.  73–​74a Childress, James  152a
bulbar conjunctiva, primary acquired melanoma  22a Chi-​squared test  148a, 201a
chlorpromazine 183a
C chocolate agar  131t
calcification, retinoblastoma  146a cholinergic medications  197t
calcineurin inhibitors  114a chorioretinopathy, birdshot see birdshot chorioretinopathy
calcofluor white stain  198t choroidal melanoma  201a
CAMP mnemonic  191a metastases 170q, 194a
cancer-​associated retinopathy (CAR)  43q, 64a, 196a prognosis 174q, 201a
canthaxanthin 64a choroidal naevus  87q, 110–​111a
capacity 176q, 202a choroidal neovascularization (CNV)  42q, 63a
capillary abnormalities, bilateral  40q, 62a choroidal neovascular membrane  130a
capillary haemangioma  43q, 64a choroidal neovascular membrane (CNVM)  118q, 128a
carbonic anhydrase inhibitors  197t CT 37q, 38f, 58a
side effects  10q, 28t chromosomal analysis, anterior segment
carotene 151a dysgenesis 159q, 180a
carotid endarterectomy (CEA)  141q, 151a chronic anterior uveitis  171q, 195–​196a
carteolol 197t chronic conjunctivitis  118q, 129a
cataracts Churg–​Strauss syndrome (CSS) eosinophilic granulomatosis
congenital cataracts  78q, 95a, 161q, 183a with polyangiitis (EGPA)
hypermature cataracts  11q, 29a CI (convergent insufficiency)  98a
posterior subcapsular cataracts see posterior subcapsular cicatricial ectropion  169q, 194a
cataract ciclosporin 197a
sunflower cataracts  183a mechanism of action  128a
unilateral, amblyopia and  183a pregnancy and  125a
unilateral cataracts  183a CJD (Creutzfeldt–​Jakob disease)  176q, 203a
cataract surgery Claude syndrome  70t
femtosecond laser-​assisted cataract surgery  14q, 33a clinically detectable retinal thickness  60a
intraocular lenses  160q, 182a clinically significant diabetic maculopathy  60a
posterior capsule rupture  11q,  29–​30a closed globe injury, gun wounds  104a
postoperative bacterial endophthalmitis  162, 184a cluster headache (CH)  190a
problems after  13q, 32a CMO (cystoid macular oedema)  111–​112a
problems in  13q, 32a CNTGS (Collaborative Normal Tension Glaucoma
CAT-​FANGS mnemonic  179a Study) 8q, 25a
cat-​scratch disease  73a CNV see choroidal neovascularization (CNV)
cavernous haemangioma  64a, 109a CNVM see choroidal neovascular membrane (CNVM)
CCF (cortico-​cavernous fistula)  82q, 103a coagulase-​negative staphylococci, bacterial endophthalmitis  54a
CCTS (Collaborative Corneal Transplant Studies)  7q, 24a Cochrane collaboration, pigment dispersal syndrome  182a
central nervous system (CNS), anterior segment Collaborative Corneal Transplant Studies (CCTS)  7q, 24a
dysgenesis 7q, 25a Collaborative Normal Tension Glaucoma Study
central retinal vein occlusion  37q, 57a (CNTGS) 8q, 25a
central serous chorioretinopathy (CSCR)  42q, 62–​63a, 118q, Collaborative Ocular Melanoma Study (COMS)  140q,
119f, 129a 149a, 149t
central vision deterioration  43q, 64a compressive optic neuropathy, acute  85q, 107a
CFEOM (congenital fibrosis of extraocular muscles)  81q, 101a computed tomographic angiography (CTA), Horner’s
CHALeLS mnemonic  17a syndrome 134a
chaperon 142q, 152a computed tomography (CT)
Charles Bonnet syndrome  52q, 76a choroidal neovascular membrane  37q, 38f, 58a
Chédiak–​Higashi syndrome  52q, 76a Horner’s syndrome  134a
chemical eye injury  2q, 17a, 157q, 177a, 177t infective orbital cellulitis  104–​105a
classification 16a, 16t intracranial aneurysm  133a
prognosis 1q, 16a neurofibromatosis type 1  106a
208 Index

COMS (Collaborative Ocular Melanoma Study)  140q, cortico-​cavernous fistula (CCF)  82q, 103a
149a, 149t corticosteroids 114a
congenital Brown syndrome  100a central serous chorioretinopathy  62–​63a
congenital cataracts  78q, 95a, 161q, 183a Jarisch–​Herxheimer reaction  172q, 196a
congenital (infantile) esotropia  77q, 94a see also steroids
congenital fibrosis of extraocular muscles cosmetic squints  141q, 151–​152a
(CFEOM) 81q, 101a cost-​benefit analysis  153a
congenital glaucoma  115q, 125a cost-​effectiveness  153a
congenital megalocornea  168q, 192a cost-​minimization  153a
congenital optic pit maculopathy  57a cost-​utility  153a
congenital ptosis  105t co-​trimoxazole  125a
congenital upper eyelid ptosis  169q, 193a craniofacial abnormalities  7q, 25a
congenital Zika syndrome (CZS)  75a craniosynostosis 191a
tyrosinase 75a CRAVE mnemonic  71a
Congo red stain  15q Creutzfeldt–​Jakob disease (CJD)  176q, 203a
conjunctiva Crohn’s disease, Behçet’s disease vs. 113a
bulbar, primary acquired melanoma  22a Crouzon syndrome  191a
fibrovascular lesions  159q cryotherapy
melanosis 21a retinal detachment treatment  54a
naevus 21a retinoblastoma 99a
papilloma 21a Cryotherapy for Retinopathy of Prematurity (CRYO-​ROP)
pigmented lesions  4q, 21a study 93a
conjunctival epithelial neoplasia (Bowen’s disease)  16a CSCR (central serous chorioretinopathy)  42q, 62–​63a, 118q,
conjunctivitis 119f, 129a
chronic conjunctivitis  118q, 129a CT see computed tomography (CT)
mydricaine 126a CTA (computed tomographic angiography), Horner’s
viral infection see viral conjunctivitis syndrome 134a
consent culture media  131t
age issues  141q, 151–​152a corneal scraping  120q, 131a, 131t
foreign language  142q, 152a curly fibres, Bowman’s layer  4q, 20a
contralateral eye, muscle supply  145a CXL (corneal collagen cross-​linking)  157q, 177–​178a
contralateral hemiparesis  188t cyclophosphamide 197a
contralateral oculomotor nucleus  137q, 145a cyclosporine
convergence retraction nystagmus  71a mechanism of action  117q, 127a
convergent insufficiency (CI)  98a side effects  117q, 128a
convergent squint, sudden onset  79q, 97a CYP1B1 gene  159q
cooked meat broth  131t cystoid macular oedema (CMO)  111–​112a
copper 151a cytomegalovirus infection  50q, 72a
cornea CZS see congenital Zika syndrome (CZS)
congenital megalocornea  168q, 192a
dystrophies, gene mutations  2q, 18a D
epithelium penetrating infections  2q, 17a dacryocystocele 81q, 102a
hazy cornea  168q, 191–​192a dacryoscintigram (DSG)  124q, 135a
leukaemia, anterior segment dysgenesis  7q, 25a DALK (deep anterior lamellar keratoplasty)  6q,  23–​24a
scraping, culture media  120q, 131a, 131t Dandy–​Walker malformation  191a
corneal arcus  3q, 19a dapsone 129a
corneal collagen cross-​linking (CXL)  157q, 177–​178a data analysis, glaucoma  139q, 147a
corneal grafts DCCT (Diabetic Control and Complications Trial)  60a
disease recurrence  148q, 178a decompensated (partially accommodative) exotropia  101a
rejection of  19t decontamination of instruments  144q, 154a
subepithelial/​stromal corneal graft rejection  3q, 19a deep anterior lamellar keratoplasty (DALK)  6q,  23–​24a
corneal impression cytology  2q,  17–​18a delayed visual maturation  190a
cornea verticillata (vortex keratopathy)  158q, 179–​180a dendritic keratitis management  1a, 15q
coronal magnetic resonance imaging, septo-​optic depression, topiramate  127a
dysplasia 123q, 124f, 134a dexamethasone
corpus callosum lesions  166q, 189a bevacizumab vs. 61a
cortical blindness  52q, 76a intravitreal implant  61–​62a
Index 209

macular oedema management  185a E


retinal implants  185a Early Manifest Glaucoma Study (EMGS)  8q, 25a
DEXA (dual energy X-​ray absorptiometry) scan  50q,  Early Treatment for Diabetic Retinopathy Study
72–​73a (ETDRS) 39q, 60–​61a, 60a
diabetes mellitus type 1 Early Treatment of Retinopathy of Prematurity (ETROP)
diabetic retinopathy  39q, 60a study 93a
proliferative retinopathy  56a ear pain  165q, 187a
vitreous haemorrhage  9q, 27a economic evaluation  153a, 153t
diabetes mellitus type 2 Effectiveness of early lens extraction for the treatment of
cholesterol management  140q, 148a primary angle closure glaucoma (EAGLE)  25a
diabetic retinopathy  50q, 72a EGFR (epidermal growth factor receptor)
diagonal diplopia  47q, 68a inhibitors 119f, 129a
macular oedema  40q,  61–​62a EGPA (eosinophilic granulomatosis with polyangiitis)  84q,
Diabetic Control and Complications Trial (DCCT)  60a 105–​106a
diabetic macular oedema Ehlers–​Danlos syndrome, keratoconus and  20a
dexamethasone vs. bevacizumab  61a electrodiagnostic tests  58a
diabetes mellitus type 2  40q,  61–​62a electronegative electroretinogram (ERG)  38q,  58–​59a
fluocinolone acetonide  40q, 61a embryonal rhabdomyosarcoma  100a
management 144q, 154a EMGS (Early Manifest Glaucoma Study)  8q, 25a
diabetic maculopathy  60a endogenous endophthalmitis  90q, 114a
diabetic retinopathy endophthalmitis
diabetes mellitus type 1  39q, 60a bacterial 143q, 153a
diabetes mellitus type 2  50q, 72a endogenous 90q, 114a
three port pars plana vitrectomy  36q,  55–​56a management 153a
Diabetic Retinopathy Vitrectomy Study (DRVS)  55–​56a Endophthalmitis Vitrectomy Study (EVS)  32a, 184a
diagonal diplopia  47q, 68a endotamponade 161q, 183a
DIF test  15q endothelial corneal graft rejection  19t
diplopia enophthalmos, causes  194a
acquired Brown syndrome  100a enzyme deficiency  52q, 75a
diagonal diplopia  47q, 68a EON (ethambutol optic neuropathy)  69a
new-​onset  164q, 186a eosinophilic granulomatosis with polyangiitis (EGPA)  84q,
vertical diplopia  45q, 45t, 67a, 168q, 192–​193a 105–​106a
disease-​modifying antirheumatic drug (DMARD) epidermal growth factor receptor (EGFR)
guidelines  125–​126a inhibitors 119f, 129a
distance exotropia (DE)  98a epilepsy, topiramate  127a
DON (dysthyroid optic neuropathy) management  84q, 106a epithelial corneal graft rejection  19t
dorzolamide 197t epithelial disturbance, retinal pigment  42q, 63a
double elevator palsy (monocular elevation eplerenone  62–​63a
deficiency) 79q,  97–​98a ERG (electronegative electroretinogram)  38q,  58–​59a
double Maddox rod, torsion measurement  133a erythema chronicum migrans  50q, 73t
double vision, blunt trauma  169q, 193a ESCRS see European Society of Cataract & Refractive
Driver and vehicle Licensing Agency (DVLA)  142q, 152a, Surgeons (ESCRS)
176q, 202a esotropia
driving accommodative esotropia  81q, 101–​102a
transient vision loss  143q, 152a infantile (congenital) esotropia  77q, 94a
vision requirements  176q, 202a ETDRS (Early Treatment for Diabetic Retinopathy
visual field loss  142q, 152a Study) 39q, 60–​61a, 60a
droopy eyelids  43q, 43t, 65a ethambutol optic neuropathy (EON)  69a
DRS study  60–​61a ETROP (Early Treatment of Retinopathy of Prematurity)
dry age-​related macular degeneration treatment  42q,  63–​64a study 93a
dry eyes  157q, 178a European Society of Cataract & Refractive Surgeons
DSG (dacryoscintigram)  124q, 135a (ESCRS)
dual energy X-​ray absorptiometry (DEXA) Guidelines for Prevention and Treatment of
scan 50q,  72–​73a Endophthalmitis Following Cataract Surgery  55t, 184a
Duane syndrome type II  78q,  95–​96a postoperative endophthalmitis prophylaxis  36q, 55a
DULL-​PC mnemonic  70a euryblepharon 167q, 190a
dysthyroid optic neuropathy (DON) management  84q, 106a EVS (Endophthalmitis Vitrectomy Study)  32a, 184a
210 Index

exotropia foreign language, consent and  142q, 152a


basic exotropia  98a forniceal conjunctiva, primary acquired melanoma  22a
distance exotropia  98a Förster–​Fuchs’ spot  63a
non-​refractive exotropia  101a FOXC1 gene  21a
partially accommodative (decompensated) exotropia  101a Francisella tularensis infection  6q, 23a
refractive exotropia  101a Fraser competence  151a
extraocular muscle surgery  97a FRAX® 73a
exudative retinal artery macroaneurysm  184a Friedman criteria, idiopathic intracranial hypertension  66a
eyelids Frisby test  200a
congenital upper eyelid ptosis  169q, 193a Fuchs heterochromatic cyclitis (FHC)  9q, 26–​27a, 89q, 113a
droopy eyelids  43q, 43t, 65a fujimycin see tacrolimus (fujimycin/​FK506)
full-​thickness defect reconstruction  86q, 108a, 108t full blood count (FBC), infective orbital cellulitis  104a
upper eyelid basal cell carcinoma  86q, 109a full-​thickness lid defect, reconstruction  86q, 108a, 108t
eye pain fundus autofluorescence (FAF)  121f, 121q, 131a
headache 167q, 190a long-​term with hydroxychloroquine  203a
hypermature cataract  11q, 29a fundus fluorescein angiography (FFA)  173q, 198a
Eylea see aflibercept (Eylea/​VEGF Trap) arm-​eye time  120q, 130a
CNVM 119q, 130a
F fusidic acid  117q, 127a, 127t
Fabry’s disease  179a
alpha-​galactosidase deficiency  75a G
posterior subcapsular cataract  33a galactosaemia 75a
facial asymmetry  169q, 194a galactose-​1-​phosphate uridyltransferase deficiency  75a
facial injection sites  169q, 193a gamma-​interferon assay (G-​IFN/​Quantiferon®) 130a
FAF see fundus autofluorescence (FAF) tuberculosis diagnosis  113a
FAME trials  61a Gardner syndrome  85q, 107a
familial exudative vitreoretinopathy (FEVR)  162q, 184a GCA see giant-​cell arteritis (GCA)
familial microspherophakia, autosomal dominant  182a gefitinib 118q, 129a
Fasanella Servat  193–​194a gene mutations, corneal dystrophies  2q, 18a
fascicular third nerve syndromes  48q, 70a, 70t Gerstmann syndrome  69a
FBC (full blood count), infective orbital cellulitis  104a gestational diabetes  74a
femtosecond laser-​assisted cataract surgery (FLACS)  14q, 33a giant-​cell arteritis (GCA)  44q,  66–​67a
Fenofibrate Intervention and Event Lowering in Diabetes tocilizumab 126a
Study (FIELD)  149a tonic pupils  65a
FEVR (familial exudative vitreoretinopathy)  162q, 184a giant retinal tears, retinal detachment  39q, 59a
FFA see fundus fluorescein angiography (FFA) Giemsa staining  198t
FHC (Fuchs heterochromatic cyclitis)  9q, 26–​27a, 89q, 113a G-​IFN see gamma-​interferon assay (G-​IFN/​Quantiferon®)
FIELD (Fenofibrate Intervention and Event Lowering in Gillick competence  151a
Diabetes Study)  149a glaucoma
Fisher’s exact test  201a acute iatrogenic secondary angle glaucoma  10q, 28a
FK506 see tacrolimus (fujimycin/​FK506) congenital glaucoma  115q, 125a
FLACS (femtosecond laser-​assisted cataract data analysis  139q, 147a
surgery) 14q, 33a laser trabeculoplasty  10q, 29a
Flexner–​Wintersteiner rosettes, retinoblastoma  146a latanoprost therapy  160q, 181a
floaters 171q, 196a lens-​induced glaucoma  11q, 29a
fluocinolone acetonide lens particle glaucoma  29a
diabetic macular oedema  40q, 61a malignant glaucoma  10q, 28a
macular oedema management  185a neovascular glaucoma  160q, 181a
fluorescein angiography open angle glaucoma  9q, 27a
acute posterior multifocal pigment placoid phacoanaphylactic glaucoma  29a
epitheliopathy 198a phacolytic glaucoma  29a
anaphylactic shock  51q, 74a phacomorphic glaucoma  29a
fluoroquinolone 197a primary angle closure glaucoma  8q, 25a
flying saucer sign  130–​131a primary congenital glaucoma see primary congenital
Food and Drug Administration (FDA, US), glaucoma glaucoma (PCG)
medication 181a primary open angle glaucoma see primary open angle
Forbes–​Albright syndrome  75a glaucoma
Index 211

red cell glaucoma  9q, 27a herpes simplex virus iridocyclitis  150a


treatment 159q, 181a herpes zoster-​associated acute retinal necrosis  196a
glucocorticoids herpes zoster ophthalmicus (HZO)  148q, 178a
dysthyroid optic neuropathy management  106a herpes zoster ophthalmoplegia  3q,  18–​19a
induced osteoporosis  50q,  72–​73a tonic pupils  65a
ptosis 84q, 107a herpes zoster vaccination  6q, 24a
glucose-​6-​phosphate dehydrogenase (G6PD) Herpetic Eye Disease Study (HEDS)  15q, 141q, 149–​150a
deficiency 118q, 129a herpetic simplex keratitis (HSK)  15q
glucose receptor, adipose tissue/​striated muscle  138q, 146a Hess chart, torsion measurement  133a
GMS (Gomori methenamine silver) stains  198t Hess tests  132a
gold, lens opacities  183a Histoplasma capsulatum 128a
Goldman–​Favre syndrome  63a HIV infection, ocular surface squamous neoplasia  16a
Gomori methenamine silver (GMS) stains  198t HLA
gonioscopy, peripheral anterior synechiae  5q, 22a birdshot chorioretinopathy  88q
Gorlin–​Golz syndrome (naevoid BCG syndrome)  85q, 107a disease associations  112a
gradient accommodative convergence/​accommodation Holmes–​Adie syndrome  67a
(AC/​A) ratio  133a Homer–​Wright pseudo-​rosettes, retinoblastoma  146a
Gram staining  198t homocystinuria 12q, 31a, 183a
granulomatosis with polyangiitis (GPA) hookworm, Mooren’s ulcer  22a
(Wegener’s granulomatosis) Horner’s syndrome  124q
peripheral alternative keratitis  159q MRI FLAIR  134a
scleritis 112a unilateral ptosis and anisocoria  187a
granulomatous panuveitis, bilateral  184a HORV (haemorrhage occlusive retinal vasculitis)  163f,
Grave’s disease see thyroid eye disease (TED) 163q, 185a
(Grave’s disease) HSK (herpetic simplex keratitis)  15q
greenstick floor fracture (white eye blow-​out fracture)  102–​103a Hudson–​Stahl line  159q
greenstick orbital fractures  193a Humira® (adalimumab)  171q, 196a
Griscelli syndrome  52q, 76a hydroxychloroquine  176, 203a
Guillain–​Barré syndrome, Zika virus infection  75a hydroxychloroquine maculopathy, OCT  120f, 120q,
gun wounds  83q, 104a 130–​131a
gyrate atrophy  175q, 201a hypercholesterolaemia, corneal arcus  19a
hyperlysinaemia 183a
H hypermature cataract  11q, 29a
haemangioma hypermetropia 10q, 28a
cavernous haemangioma  64a, 109a hyper-​reflective deposits, optic disc drusen  38q, 59a
racemose haemangioma  64a hypertension
segmental capillary haemangioma  167q, 190a ocular  see ocular hypertension (OHT)
haemorrhage, vitreous in diabetes type 1  9q, 27a RAPD 46q, 68a
haemorrhage occlusive retinal vasculitis (HORV)  163f, hypoplasia, midfacial  168q, 191a
163q, 185a HZO (herpes zoster ophthalmicus)  148q, 178a
haemorrhagic retinal artery macroaneurysm  184a
hazy cornea  168q, 191–​192a I
HbA1C levels  50q, 72a iatrogenic secondary angle glaucoma, acute  10q, 28a
headache ice pack test, ocular myasthenia  67a
cluster headache  190a ICE (iridocorneal) syndrome  5q, 22a
ocular pain  167q, 190a ICGA see indocyanine green angiography (ICGA)
temporal headache  66a IDEX 133a
hearing loss  166q, 189a idiopathic intracranial hypertension (IIH)  44q, 66a
Hedgehog pathway, vismodegib  128a IFIS (intraoperative floppy iris syndrome)  32a, 197a
HEDS (Herpetic Eye Disease Study)  15q, 141q, 149–​150a IIH (idiopathic intracranial hypertension)  44q, 66a
hemianopia, bitemporal  123q, 134q illuvien 154a
hemifacial spasm  51q, 74a imiquimod 129a
hemiparesis, contralateral  188t immunosuppression
hepatitis C infection  22a cytomegalovirus infection  50q, 72a
Hermansky–​Pudlak syndrome  52q, 76a pregnancy contraindications  115q, 125a
herpes simplex keratitis (HSK)  149–​150a impotence 9q, 26a
acanthamoeba keratitis vs. 179a incorrect rejection, null hypothesis  140q, 148a
212 Index

indocyanine green angiography (ICGA)  96a, 121q, 131–​132a keratoconus 4q, 20a


acute posterior multifocal pigment placoid penetrating keratoplasty  3q, 18a
epitheliopathy 198a keratoplasty, deep anterior lamellar keratoplasty  6q,  23–​24a
infantile (congenital) esotropia  77q, 94a Kifs syndrome  186a
infective orbital cellulitis  104–​105a kiwi fruit, zinc  202a
inferior proliferative vitreoretinopathy  161q, 183a Klebsiella pneumonia infection, endogenous
inferior rectus muscle  145a endophthalmitis 90q, 114a
inferonasal lens subluxation, bilateral  12q, 31a Knudson’s two-​hit hypothesis  110a
inflammatory diseases, tonic pupils  65a Kruskal–​Wallis test  149a
inheritance patterns  138q, 147a
instrument decontamination  144q, 154a L
interferon-​alpha  64a LAMA1 gene  147a
intermittent esotropia (IXT)  98a Lambert–​Eaton syndrome  197–​198a
International Society for Clinical Electrophysiology of Vision lampalizumab  63–​64a
(ISCEV) 58a Lang test  200a
intra-​arteria embolus of filler  193a LARGE mnemonic  18a
intracranial aneurysm  123q, 133a laser photocoagulation, retinoblastoma  99a
intraocular lenses (IOLs) laser trabeculoplasty, glaucoma  10q, 29a
cataract surgery  160q, 182a latanoprost 197t
posterior capsule opacification (PCO)  160q, 182a glaucoma 160q, 181a
intraocular pressure (IOP) mechanism of action  172q, 196–​197a
elevation of  160q, 181–​182a pregnancy 181a
medications for  10q, 27–​28a, 28t Lawrence–​Moon syndrome  186a
primary congenital glaucoma  96a Leber’s hereditary optic neuropathy (LHON)  46q, 49q,
treatments 115q, 125a 67–​68a, 71a
Urrets–​Zavalia syndrome (UZS)  3q, 18a acute phase  81q, 101a
intraoperative floppy iris syndrome (IFIS)  32a, 197a Lees test, ocular deviation  122q, 132–​133a
intravitreal injection lenses 137q, 146a
lampalizumab 42q,  63–​64a intraocular  see intraocular lenses (IOLs)
silicone oil for pseudophakic eye  13q, 33a opacity 161q, 183a
IOLs see intraocular lenses (IOLs) lens-​induced glaucoma  11q, 29a
IOP see intraocular pressure (IOP) lens particle glaucoma  29a
ipsilateral eye, muscle supply  145a Leptospira interrogans 73a
ipsilateral sixth and seventh nerve palsies  165q, 187a, 188t LHON see Leber’s hereditary optic neuropathy (LHON)
iridocorneal (ICE) syndrome  5q, 22a Lifitegrast 7q,  24–​25a
ISCEV (International Society for Clinical Electrophysiology of LIGHT trial  29a
Vision) 58a limbal conjunctiva  22a
ischaemia, third nerve palsy  46q, 68a limbal ischaemia  1q, 16q
IXT (intermittent esotropia)  98a limbal stem cell deficiency (LSCD)  2q, 17a
corneal impression cytology  2q,  17–​18a
J lipofuscinosis, neuronal ceroid  186a
Jansky–​Bielschowsky syndrome  186a Löfgren’s syndrome  189a
Jarisch–​Herxheimer reaction, corticosteroids  172q, 196a Lowe’s syndrome  179a
jaw claudication, giant-​cell arteritis  66a anterior lenticonus  33a
juvenile idiopathic arthritis (JIA)  171q, 195–​196a LSCD see limbal stem cell deficiency (LSCD)
juvenile X-​linked retinoschisis (XLR)  63a, 78q, 96a Lyme disease
erythema chronicum migrans  73a
K tonic pupils  65a
Kay Picture  169q, 193–​194a lymphoma, orbital inflammatory disease vs. 86q, 109a
Kayser–​Fleischer rings  183a
Wilson’s disease  20a M
Kearns–​Sayre syndrome (KSS)  80q, 100a macular cystic changes, vitamins  42q, 63a
keratitis 117q, 128a macular degeneration, dry age-​related
bacterial 5q,  21–​22a treatment 42q,  63–​64a
herpetic simplex keratitis  15q macular holes, sttage2 full-​thickness  36q, 55a
microbial keratitis  17a macular oedema, diabetic see diabetic macular oedema
keratoconjunctivitis, vernal  7q, 24a macular pucker  41f, 41q, 62a
Index 213

macular telangiectasia  40q, 62a microvascular sixth nerve palsy  186–​187a


macular telangiectasia type 2  63a midfacial hypoplasia  168q, 191a
MacuShield 117q, 128a migraine 53q, 76a
magnetic resonance angiography (MRA), intracranial topiramate 127a
aneurysm 133a mitomycin C  128a
magnetic resonance imaging (MRI) MIVI-​TRUST (Microplasmin for Intravitreal Injection–​Traction
coronal in septo-​optic dysplasia  123q, 124f, 134a Release without Surgical Treatment)  56a
coronal, septo-​optic dysplasia  123q, 124f, 134a MMAGHMLACSLO mnemonic  15–​16q
FLAIR, Horner’s syndrome  134a mode 139q, 148a
infective orbital cellulitis  104–​105a modified Dandy criteria, idiopathic intracranial hypertension  66a
neurofibromatosis type 1  106a modified Hughes classification  157q
T2-​weighted scan with FLAIR sequence  174f, 200a monocular elevation deficiency (MED/​double elevator
Wilson’s disease  20a palsy) 79q,  97–​98a
malignant glaucoma (aqueous misdirection)  10q, 28a Mooren’s ulcer  5q, 22a
malnutrition 47q,  68–​69a MPO (myeloperoxidase)  159q
MALT (mucosa-​associated lymphoid tissue) lymphoma  194a MRA (magnetic resonance angiography), intracranial
Mann–​Whitney tests  148a, 201a aneurysm 133a
Mantoux skin testing  130a MRI see magnetic resonance imaging (MRI)
MAR (melanoma-​associated retinopathy)  43q, 64a MRSE (mean refractive spherical equivalent)  147a
Marcus–​Gunn jaw-​winking syndrome (MGJWS)  164q, 187a mucopolysaccharidoses 186a
marfanoid stature  12q, 31a mucosa-​associated lymphoid tissue (MALT) lymphoma  194a
Marfan syndrome  182–​183a Muenke syndrome, craniosynostosis  191a
keratoconus and  20a Muir–​Torre syndrome  85q, 107a
MD (myotonic dystrophy)  166q, 188a multinucleate evanescent white dot syndrome
mean 139q, 148a (MEWDS) 87q, 111a, 199t
comparison of  175q, 201q multiple sclerosis  47q, 69a
mean refractive spherical equivalent (MRSE)  147a musculoskeletal (MSK) disorders  197a
MED (monocular elevation deficiency/​double elevator myasthenia gravis  197–​198a
palsy) 79q,  97–​98a autoantibodies 174q, 200a
medial canthus, swelling below  81q, 102a diagnostic tests  67a
medial rectus muscle  145a myasthenic ptosis  105t
median 139q, 148a Mycobacterium tuberculosis infection  119q, 130a
medical ethics  142q, 152a mycophenolate mofetil  125a
Meesmann dystrophy  2q, 4q, 18a, 20a mydricaine 116q, 126a
megalocornea, congenital  168q, 192a myeloperoxidase (MPO)  159q
melanoma MYOC gene  159q
choroidal  see choroidal melanoma myogenic ptosis  105t
ocular melanoma  87q, 110a myopia, congenital megalocornea  192a
primary acquired melanoma  5q, 22a myopic refractive surprise  12q, 30–​31a, 31t
uveal melanoma  194a myotonic dystrophy (MD)  166q, 188a
melanoma-​associated retinopathy (MAR)  43q, 64a
Meretoja’s syndrome  178a N
mesoderm 146a NA-​AION (non-​arteritic anterior ischaemic optic
meso-​xanthine  117q, 128a neuropathy) 44q, 65–​66a, 68a, 165q, 187–​188a
methotrexate 125a naevoid BCG syndrome (Gorlin–​Golz syndrome)  85q, 107a
MEWDS (multinucleate evanescent white dot nanophthalmia 32a
syndrome) 87q, 111a, 199t NASCET (North America Symptomatic Carotid
MFC 199t Endarterectomy) study  141q, 151a
MGJWS (Marcus–​Gunn jaw-​winking syndrome)  164q, 187a National Osteoporosis Guideline Group (NOG)  73a, 73t
microadenoma 75a NDSEP (NHS Diabetic Eye Screening Programme)  143q,
microbial keratitis  17a 153–​154a
microbiological staining  172q, 198a, 198t neovascular glaucoma  160q, 181a
Microplasmin for Intravitreal Injection–​Traction Release nerve palsies  188t
without Surgical Treatment (MIVI-​TRUST)  56a acquired third nerve palsies  48q, 70t
microspherophakia 161q, 182–​183a ipsilateral sixth and seventh nerve palsies  165q, 187a, 188t
microspherophakia, autosomal dominant familial  182a microvascular sixth nerve palsy  186–​187a
microtropia 96a third nerve palsies see third nerve palsy
214 Index

neural crest cells  138q, 146a ocular fundus, abusive head trauma (shaken baby
embryologic origins  146a syndrome) 104a
neuroectoderm 146a ocular hypertension (OHT)  10q, 28a
neurofibromatosis type 1  80q, 100–​101a reviews 28t
investigations pre-​surgery  84q, 106–​107a Ocular Hypertension Study (OHTS)  8q, 25a
neuroimaging 173q, 174f, 200a ocular melanoma  87q, 110a
intracranial aneurysm  123q, 133a ocular motility problems  164q, 186a
neuromyelitis optica (NMO), bilateral sequential optic ocular myasthenia  45q, 67a
neuropathy vs. 68a ocular pain see eye pain
neuronal ceroid lipofuscinosis  186a ocular surface squamous neoplasia (OSSN)  2q,  16–​17a
NFAT (nuclear factor of activated T cells)  127t ocular trauma score (OCS)  82q, 103a, 103t
NHS Diabetic Eye Screening Programme (NDSEP)  143q, oculomotor nucleus, contralateral  137q, 145a
153–​154a oedema
niacin (vitamin B3/​nicotinic acid)  42q, 63a cystoid macular oedema  111–​112a
NICE (National Institute for Health and Care Excellence) diabetic macular see diabetic macular oedema
choroidal neovascular membrane guidelines  58a post-​cataract surgery macular oedema  11q, 30a
corneal collagen cross-​linking  177a post-​phacoemulsification cystoid macular
dexamethasone intravitreal implant  61–​62a oedema 11q, 30a
diabetic control guidelines  72a OHT see ocular hypertension (OHT)
glaucoma treatment  181a OHTS (Ocular Hypertension Study)  8q, 25a
macular oedema management  163q, 185a OID (orbital inflammatory disease), lymphoma vs. 86q, 109a
ocriplasmin 36q, 56a omega-​3 fatty acids  151a
tuberculosis treatment  130a one-​way analysis of variance  148a
nicotinic acid (vitamin B3/​niacin)  42q, 63a ONTT (Optic Neuritis Treatment Trials)  141q, 150a, 150t
NMO (neuromyelitis optica), bilateral sequential optic open angle glaucoma  9q, 27a
neuropathy vs. 68a ophthalmic artery embolism  169q, 193a
NOG (National Osteoporosis Guideline Group)  73a, 73t ophthalmic ultrasonography  120q, 131a
non-​arteritic anterior ischaemic optic neuropathy optical coherence tomography (OCT)  40f, 40q, 41f, 62a,
(NA-​AION)  44q, 65–​66a, 68a, 165q, 187–​188a 116q, 127a, 133a, 134a
non-​Hodgkin’s lymphoma  194a anterior segment optical coherence tomography
non-​nutrient agar with E coli overlay  131t (AS-​OCT)  121q, 132a
non-​parametric tests  148a conditions 118q, 119f, 129a
non-​purulent discharge  168q, 191–​192a hydroxychloroquine maculopathy  120f, 120q, 130–​131a
non-​refractive exotropia  101a long-​term with hydroxychloroquine  203a
North America Symptomatic Carotid Endarterectomy retinal dearrangement  171q, 196a
(NASCET) study  141q, 151a optic disc drusen  38q, 59a
Nothnagel syndrome  70t optic disc maculopathy  36q, 57a
nuclear factor of activated T cells (NFAT)  127t optic neuritis  187a
null hypothesis, incorrect rejection  140q, 148a imaging 123q, 134a
nutritional supplements, AMD  141q, 151a multiple sclerosis  47q, 69a
nystagmus Optic Neuritis Treatment Trials (ONTT)  141q, 150a, 150t
convergence retraction  71a optic neuropathy
see-​saw nystagmus  48q,  69–​70a acute compressive  85q, 107a
bilateral sequential vs. anterior ischaemic  68a
O optic pit maculopathy, congenital  57a
OAT (ornithine aminotransferase) deficiency  201a optic sulci  201a
occult membrane classification  130a optic vesicle  174q, 201a
OCP (ocular cicatricial pemphigoid)  1a, 15q orbital cellulitis  168q, 191a
ocriplasmin 36q, 56a, 56t infective  104–​105a
Ocriplasmin for treatment for Symptomatic Vitreomacular orbital floor fracture  82q, 102–​103a
Adhesion Including Macular Hole (OASIS)  56a orbital inflammatory disease (OID), lymphoma vs. 86q, 109a
OCS (ocular trauma score)  82q, 103a, 103t orbital lesions  86q, 109a
OCT see optical coherence tomography (OCT) orbital malignancy diagnosis  170q, 194–​195a, 195t
ocular albinism, X-​linked  81q, 102a orbital metastases  109a
ocular cicatricial pemphigoid (OCP)  1a, 15q ornithine aminotransferase (OAT) deficiency  201a
ocular deviation, Lees test  122q, 132–​133a orthoptic tests  173q, 200a
ocular embryolog6y  201a OSSN (ocular surface squamous neoplasia)  2q,  16–​17a
Index 215

osteogenesis imperfecta, keratoconus and  20a Phacoanaphylactic glaucoma  29a


osteoporosis  72–​73a phacoemulsifications 11q, 30a
Ozurdex 154a phacolytic glaucoma  29a
phacomorphic glaucoma  29a
P phakomatoses 138q, 146a
PACK (photoactivated chromophore for keratitis)-​CXL  177a pheochromocytoma 195a
Paediatric Eye Disease Investigator Group, congenital PHN (post-​herpetic neuralgia)  178a
(infantile) esotropia  94a photoactivated chromophore for keratitis (PACK)-​CXL  177a
pain photodynamic therapy (PDT)
ear pain  165q, 187a vascular abnormalities  63a
eyes see eye pain verteporfin and  42q, 63a
palinopsia 71a photophobia 2q, 17a, 157q, 188a
PAM (primary acquired melanoma)  5q, 22a phytanic acid alpha hydrolase deficiency  75a
panretinal photocoagulation (PRP) PIC 199t
Early Treatment for Diabetic Retinopathy Study pigment dispersal syndrome (PDS)  160q, 181–​182a
(ETDRS) 39q, 60a pigmented lesions, conjunctiva  4q, 21a
proliferative diabetic retinopathy  39q, 60a pigment epithelium derived factor (PEDF)  181a
panuveitis pilocarpine 197t
bilateral granulomatous  184a pituitary adenoma  123q, 134q
severe active bilateral  115q, 125a pituitary tumours
PAPER-​CLIP mnemonic  58a adenoma 123q, 134q
papilloedema 51q,  73–​74a prolactin production  52q, 75a
parametric tests  148a PITX2 gene  21a
paretic extraocular muscles  168q, 192–​193a, 193f PITZ3 gene  96a
parietal lobe infarction  48q, 69a PIV (punctate inner choroidopathy)  111a
Parinaud dorsal midbrain syndrome  48q,  70–​71a PKP see penetrating keratoplasty (PKP)
Parinaud’s oculo-​glandular syndrome (POGS)  6q, 23a plaque radiotherapy, retinoblastoma  99a
Parks–​Bielschowsky test  192a pleomorphic adenomas  109a
Park’s three-​step test  192a plexiform neurofibroma  106a
partially accommodative (decompensated) exotropia  101a PMD (pellucid marginal degeneration)  5q, 23a
partial third nerve palsy  47q, 68a pneumatic retinopexy  54a
PAT (prism adaptation test)  122q, 133a POGS (Parinaud’s oculo-​glandular syndrome)  6q, 23a
pattern electroretinogram (pERG)  173q, 199a POHS (presumed optical histoplasmosis
PAX6 gene  21a syndrome) 118q, 128a
PCG see primary congenital glaucoma (PCG) polyarthralgia, new-​onset  166q, 189a
PCMO (post-​cataract surgery macular oedema)  11q, 30a Poretti–​Boltshauser syndrome  147a
PCO see posterior capsule opacification (PCO) Posner–​Schlossman syndrome  9q, 27a, 195a
PDR see proliferative diabetic retinopathy (PDR) post-​cataract surgery macular oedema (PCMO)  11q, 30a
PDS (pigment dispersal syndrome)  160q, 181–​182a posterior capsule opacification (PCO)  13q, 33a
PDT see photodynamic therapy (PDT) intraocular lenses (IOLs) type  160q, 182a
PEDF (pigment epithelium derived factor)  181a posterior capsule rupture, cataract surgery  11q,  29–​30a
pellucid marginal degeneration (PMD)  5q, 23a posterior lenticonus  96a
penetrating infections, corneal epithelium  2q, 17a posterior reversible encephalopathy syndrome  74a
penetrating keratoplasty (PKP)  3q, 6q, 19a,  23–​24a posterior segment ischaemia  160q, 181a
keratoconus 3q, 18a posterior subcapsular cataract  96a
Pentacam machine  122q, 132a associated diseases  14q, 33a
perfluorocarbon 183a post-​herpetic neuralgia (PHN)  178a
pERG (pattern electroretinogram)  173q, 199a postoperative endophthalmitis (POE), acute  11q, 30a
periocular swelling, unilateral  93q, 104–​105a post-​phacoemulsification bacterial endophthalmitis  36q,
periodic acid-​Schiff stain  198t 54–​55a, 55t
periorbital ecchymosis, bilateral (raccoon eyes)  105a post-​phacoemulsification cystoid macular oedema (CMO/​
peripheral alternative keratitis (PUK)  159q, 180a Irvine–​Gass syndrome)  11q, 30a
peripheral anterior synechiae, gonioscopy  5q, 22a PR see proliferative retinopathy (PR)
peripheral neurofibromatosis  see neurofibromatosis type 1 prednisolone
Peter’s anomaly  4q, 21a, 183a peripheral alternative keratitis  159q
Pfeiffer syndrome, craniosynostosis  191a syphilis treatment  196a
PHACES syndrome  190–​191a pre-​eclamptic toxaemia  51q, 74a
216 Index

pregnancy size difference between eyes  44q, 65a


drug contraindications  115q, 125a tonic pupils see tonic pupils
latanoprost 181a Purtscher-​like retinopathy  74a
preoperative prism correction  133a Purtscher retinopathy  74a
presumed optical histoplasmosis syndrome PVR see proliferative vitreoretinopathy (PVR)
(POHS) 118q, 128a PXE (pseudoexfoliation)  182a
primary acquired melanoma (PAM)  5q, 22a PXS (pseudoexfoliation syndrome)  12q, 32a
primary angle closure glaucoma  8q, 25a
primary angle closures, terminology  10q, 27a, 27t Q
primary biliary cirrhosis  179a quality-​adjusted-​life-​year (QALY)  176q, 203a
primary congenital glaucoma (PCG)  7q, 25a, 79q, 96a Quantiferon® see gamma-​interferon assay (G-​IFN/​
genetics 159q Quantiferon®)
primary open angle glaucoma  138q, 147a quetiapine 172q, 197a
topical β-​blockers  9q, 26a
primary Sjögren’s syndrome (PSS)  158q, 179a R
Primary Tube Versus Trabeculectomy (PTVT) study  202a raccoon eyes (bilateral periorbital ecchymosis)  105a
Principles of Biomedical Ethics (Beauchamp and racemose haemangioma  64a
Childress) 152a RADIANCE trials  63a
prism adaptation test (PAT)  122q, 133a RAM (retinal artery macroaneurysm)  162q, 184–​185a
prolactin, pituitary tumours  52q, 75a ranibizumab 154a
prolactinoma 75a RAPD see relative afferent pupillary defect (RAPD)
proliferative diabetic retinopathy (PDR) RCOphth see Royal College of Ophthalmologists (RCOphth)
high-​risk definition  39q,  60–​61a red cell glaucoma  9q, 27a
panretinal photocoagulation  39q, 60a red eye, bilateral  168q, 191–​192a
proliferative retinopathy (PR) Reese–​Ellsworth group, unilateral retinoblastoma  99a
classification 54a refractive exotropia  101a
retinal detachment  36q, 54a Refsum disease/​syndrome  185a
proliferative vitreoretinopathy (PVR)  183a phytanic acid alpha hydrolase deficiency  75a
retinoschisis-​related retinal detachment  183–​184a Reis–​Buckler dystrophy  20a
proptosis relative afferent pupillary defect (RAPD)  87q, 111a,
steroid therapy  85q, 107a 165q, 187a
unilateral axial  86q, 109a hypertension 46q, 68a
prostaglandin analogues  197t REPAIR trials  63a
side effects  10q, 28t retina
proteinase  3 (PR3) 159q astrocytoma 170q, 195a
PRP see panretinal photocoagulation (PRP) bilateral striae  116q, 127a
psammoma bodies, retinoblastoma  146a breaks 57a
pseudoexfoliation (PXE)  182a optical coherence tomography  171q, 196a
pseudoexfoliation syndrome (PXS)  12q, 32a tears 37q, 57a
pseudophakic eye, intravitreal silicone oil  13q, 33a unilateral lesions  80q, 99a
PSS (primary Sjogren’s syndrome)  158q, 179a vascular tumours  43q, 64a
pterygium 159q, 180a retinal artery macroaneurysm (RAM)  162q, 184–​185a
ptosis 165q, 187a retinal detachment  119q, 129–​130a, 161q, 183a
aponeurotic dehiscence  83q, 105a acute inferior macular  36q, 54a
congenital 105t differential diagnoses  129a
congenital upper eyelid  169q, 193a giant retinal tears  39q, 59a
features 105a, 105t proliferative retinopathy  36q, 54a
glucocorticoids 84q, 107a treatment 36q, 54a, 90q, 91t, 114a
unilateral 187a retinal pigment epithelium (RPE)  145a
PTVT (Primary Tube Versus Trabeculectomy) study  202a disturbance 42q, 63a
PUK (peripheral alternative keratitis)  159q, 180a retinal vein occlusion (RVO)  37q, 57a, 163q, 185a
Pulfrich phenomenon  52q, 76a Retina Society, proliferative retinopathy classification  54a
punctate inner choroidopathy (PIV)  111a retinitis 50q, 71a
pupils retinitis pigmentosa (RP)  63a
Adie’s tonic pupil  44q, 65a associated conditions  164q, 185–​186a
Argyll Robertson pupil  187a retinoblastoma 99a
reverse pupil block  32a genetics 110a
Index 217

pathology 137q, 146a shaken baby syndrome (abusive head trauma)  82q, 104a


unilateral 99a shingles vaccination  6q, 24a
retinoblastoma protein (pRb)  138q, 146–​147a SICCANOVE study  128a
retinopathy silicon oil tamponade  183a
diabetic  see diabetic retinopathy simulated IDEX  133a
of prematurity  77q,  93–​94a SINA_​LFTs mnemonic  145a
proliferative see proliferative retinopathy (PR) skew 139q, 148a
retinoschisis-​related retinal detachment (RSRD)  162q, slit-​lamp fundus lenses  175q, 201q
183–​184a SO (sympathetic ophthalmia)  90q, 91t, 114a, 162, 184a
reverse pupil block  32a SOD (septo-​optic dysplasia)  123q, 124f, 134a
rhabdomyosarcoma (RMS)  80q, 87q, 99–​100a, 109–​110a SOF (superior orbital fissure)  137q, 145a
alveolar  99–​100a soft contact lenses, biometry repetition  182a
embryonal 100a SOM (superior oblique myokymia)  47q, 69a
rheumatoid arthritis  189a Sorsby fundus dystrophy  21a
HLA associations  112a spherical equivalents  139q, 147a
scleritis 112a spiramycin 113a
tocilizumab 126a spironolactone  62–​63a
tonic pupils  65a Sporotrichum schenckii infection  6q, 23a
riboflavin 177a squamous cell carcinoma (SCC)  194a
Riddoch phenomenon  52q, 76a basal cell carcinoma vs. 170q, 194a
rifampicin  62–​63a squints 81q, 101–​102a
right divergent squints  79q,  98–​99a cosmetic squints  141q, 151–​152a
right Horner’s syndrome  43q right divergent  79q,  98–​99a
right inferior rectus weakness  45q, 67a sudden onset convergent squint  79q, 97a
Roper–​Hall classification, chemical eye injury  16a, 157q SSTT mnemonic  183a
Royal College of Ophthalmologists (RCOphth) STAIB-​LONG mnemonic  66a
diabetic maculopathy guidelines  60a Standardization of Uveitis Nomenclature (SUN)  88q,
diabetic retinopathy risk  60a 111–​112a
instrument decontamination  144q, 154a Staphylococcus infections
retinal vein occlusion guidelines  57a acute postoperative endophthalmitis  30a
RP see retinitis pigmentosa (RP) bacterial endophthalmitis  54a
RPE see retinal pigment epithelium (RPE) statistical tests  140q, 148–​149a
RSRD (retinoschisis-​related retinal detachment)  162q, steatorrhoea 164q, 185–​186a
183–​184a stereopsis 200a
Rude Little Green man mnemonic  178a steroids
RVO (retinal vein occlusion)  37q, 57a, 163q, 185a macular oedema management  163q, 185a
proptosis 85q, 107a
S see also corticosteroids
Sabouraud dextrose agar  131t Sticker’s syndrome  78q, 94a
Sabril see vigabatrin (Sabril) Stickler syndrome  59t
saddle nose deformity  52q,  74–​75a retinal detachment  59a
sample T-​tests  148a Stocker’s iron line  159q
SANSIKA study  128a strabismus 78q, 95a
sarcoidosis, tonic pupils  65a acute onset of concomitant esotropia  97a
SC see squamous cell carcinoma (SCC) sudden-​onset  80q, 99a
Schirmer’s test  179a thyroid eye disease  79q, 98a
Schwalbe’s line  22a Streptococcus infections  30a
Schwartz–​Matsuo syndrome  9q, 27a, 195a striated muscle, glucose receptors  138q, 146a
scleral buckle, retinal detachment treatment  54a stromal corneal graft rejection  19t
scleritis 89q Sturge–​Weber syndrome (SWS)  77q, 92a, 146a
diagnosis 89q, 112–​113a subepithelial/​stromal corneal graft rejection  3q, 19a
see-​saw nystagmus48q,  69–​70a subperiosteal abscess  168q, 191a
segmental capillary haemangioma  167q, 190a sulphonamide 115q, 125a
Seidel test, trabeculectomy  25–​26a SUN (Standardization of Uveitis Nomenclature)  88q,
septo-​optic dysplasia (SOD)  123q, 124f, 134a 111–​112a
severe active bilateral panuveitis  115q, 125a sunflower cataracts, Wilson’s disease  183a
sexual harassment  142q, 152a superficial basal cell carcinoma  118q, 129a
218 Index

superior oblique myokymia (SOM)  47q, 69a torsion, measurement of   122q, 133a


superior oblique palsies, bilateral  49q, 71a total retinal detachment  171q, 195a
superior orbital fissure (SOF)  137q, 145a toxic anterior segment syndrome (TASS)  13q, 33a
superotemporal quadrant swelling  85q, 108a toxoplasma uveitis, management  89q, 113a
suprachoroidal buckling  54a TPMT (thiopurine methyltransferase)  115q, 125–​126a
supranuclear pathway  186a trabeculectomy 8q, 25–​26a, 26a
surface ectoderm  146a trabeculotomy
Susac’s syndrome  189a Fuchs heterochromatic cystitis (FHC)  26–​27a
sutures 146a pigment dispersal syndrome (PDS)  181–​182a
SWS (Sturge–​Weber syndrome)  77q, 92a, 146a trachoma, trichiasis  105t
sympathetic ophthalmia (SO)  90q, 91t, 114a, 162, 184a transient vision loss, driving  143q, 152a
synoptophore, torsion measurement  133a transpupillary diode laser  93–​94a, 93t
syphilis (Treponema pallidum infection)  73a traumatic optic neuropathy (TON)  82q, 103a
Parinaud’s oculo-​glandular syndrome  6q, 23a treponemal antibody testing  166q, 188a, 189t
reactivation 196a Treponema pallidum infection see syphilis (Treponema pallidum
testing for  166q, 188a, 189t infection)
treatment 172q, 196a triamcinolone
choroidal neovascular membrane  58a
T presumed optical histoplasmosis syndrome  128a
tacrolimus (fujimycin/​FK506) trichiasis 83q, 105a
mechanism of action  117q, 127a, 128a trigeminal neuralgia  127a
pregnancy and  125a t-​tests  201a
tamoxifen 43q, 64a tuberculosis 196a
TED see thyroid eye disease (TED) (Grave’s disease) diagnosis 113a
temporal artery biopsy, giant-​cell arteritis  66–​67a Tube Versus Trabeculectomy (TVT) study  175q, 202a
temporal headache  66a TVT (Tube Versus Trabeculectomy) study  175q, 202a
TFSOMUHHD mnemonic  110–​111a tyrosinase 75a
TGFB1 gene  178a
thermotherapy, retinoblastoma  99a U
thiopurine methyltransferase (TPMT)  115q, 125–​126a UK eye retrieval and eye bank services  144q, 155a
third nerve palsy ultrasonography, ophthalmic ultrasonography  120q, 131a
acquired 48q, 70t ultrasound biomicroscopy (UBM)  131t
ischaemia 46q, 68a unilateral axial proptosis  86q, 109a
partial 47q, 68a unilateral cataracts, amblyopia  183a
three port pars plana vitrectomy  36q,  55–​56a unilateral periocular swelling  93q, 104–​105a
thymoma 67a unilateral ptosis  187a
thyroidectomy 43q, 43t, 65a unilateral retinal lesion  80q, 99a
thyroid eye disease (TED) (Grave’s disease)  85q, 86q, unilateral retinoblastoma  99a
107–​108a, 109a, 169q, 193a unit of analysis issue  147a
strabismus 79q, 98a upper eyelid basal cell carcinoma  86q, 109a
thyroiditis 179a upper eyelid ptosis, congenital  169q, 193a
timolol 197t Urrets–​Zavalia syndrome (UZS)  3q, 18a
TIMP3 gene  21a Usher syndrome  185a
Titmus test  200a uveal melanoma  194a
TNO 200a uveitis
tocilizumab 116q, 126a acute anterior  116q, 126a
TON (traumatic optic neuropathy)  82q, 103a adalimumab 171q, 196a
tonic pupils chronic anterior  171q, 195–​196a
associated diseases  65a classification 88q, 111–​112a
pharmacological tests  45q, 67a differential diagnosis  89q, 113a
topical β-​blockers, side effects  9q, 26a
topical ciclosporin (Ikervis)  128a V
topical intraocular pressure-​lowering variant Creutzfeldt–​Jakob disease (vCJD)  203a
medication  196–​197a, 197t vasoproliferative tumours  41f, 41q, 62a, 64a
topical steroids, bacterial keratitis  5q,  21–​22a VAVFC (visual field constriction attributable to
topiramate, bilateral retinal striae  127a vigabatrin) 153a
torsional nystagmoid movement  47q, 69a VEGF (vascular endothelial growth factor)  181a
Index 219

VEGF Trap see aflibercept (Eylea/​VEGF Trap) VIVID trial  61a


VEIN mnemonic, thyroid eye disease  109a VJC (vernal keratoconjunctivitis)  7q, 24a
VEP (visual evoked potential)  122q, 132a, 190a Vogt–​Koyanagi–​Harada (VKH) disease  119q, 129a
vernal keratoconjunctivitis (VKC)  7q, 24a Vogt–​Spielmeyer–​Batten disease  186a
verteporfin 42q, 63a Von-​Hippel–​Lindsay syndrome  195a
vertical diplopia  45q, 45t, 67a, 168q, 192–​193a von Recklinghausen neurofibromatosis  see
vigabatrin (Sabril)  143q, 153a neurofibromatosis type 1
mechanism of action  116q, 126a vortex keratopathy (cornea verticillata)  158q, 179–​180a
side effects  64a Vossius ring  183a
viral conjunctivitis  191–​192a V-​RRR mnemonic  150a
treatment 192t
vision W
blurred 166q, 171q, 189a, 196a Waardenburg syndrome  52q, 76a, 186a
central vision deterioration  43q, 64a anterior lenticonus  33a
delayed visual maturation  190a Weber syndrome  70t
double, blunt trauma  169q, 193a Wegener’s granulomatosis see granulomatosis with
vision loss  67–​68a polyangiitis (GPA) (Wegener’s granulomatosis)
sudden 44q, 49q, 66–​67a, 72a Weill–​Marchesani syndrome  182–​183a
transient vision loss in driving  143q, 152a wide-​field fluorescence angiography, familial exudative
see also blindness vitreoretinopathy 184a
vismodegib 117q, 128a Wilcoxon rank-​sum tests  201a
VISTA trial  61a Wilcoxon signed-​rank tests  148a
visual acuity reduction  47q,  68–​69a Wilson’s disease  4q, 20a
visual disturbances, intermittent  47q, 49q, 69a, 71a sunflower cataracts  183a
visual evoked potential (VEP)  122q, 132a, 190a World Health Organization (WHO), cataract surgery  12q, 31a
visual field constriction attributable to vigabatrin Worth four-​dot (W4D) test  173q, 199–​200a
(VAVFC) 153a WT1 tumour suppressor gene  159q
visual field loss, driving license  142q, 152a Wyburn–​Mason syndrome  64a, 146a
vitamin B3 (niacin/​nicotinic acid)  42q, 63a
vitamin(s), macular cystic changes  42q, 63a X
vitamin B2 177a X-​linked ocular albinism  81q, 102a
vitamin C  151a X-​linked recessive juvenile retinoschisis (XLRS)  96a
vitamin E  151a
vitrectomy Z
tamponade with  54a Ziehl–​Nielsen staining  198t
three port pars plana vitrectomy  36q,  55–​56a zinc 151a
vitreoretinal clinics, biomicroscopic classification  55a, 55t supplements 175q, 202a
vitreous haemorrhage, diabetes type 1  9q, 27a zoster-​associated acute retinal necrosis  196a

You might also like